You are on page 1of 901
VOT Solved Engineering Fundamentals Problems Third Edition Wiltauee ste) ify (ets elds ed : Solved Engineering Fundamentals Problems Third Edition Michael R. Lindeburg, PE — Professional Publications, Inc. * Belmont, California How to Locate and Report Errata for This Book AL PPI, we do our best to bring you errarfree books. But when errors do occur, we ‘want 10 make sure you can view corrections and report any potential errors you find, so the errors caute as litle confusion as posible, ‘A current ist of known errata and other updates for this book ls avalable on the PPI ‘wobaite at www:pplapass.comverrata. We update the errata page as often as necessary, so checkin regulary. You wil also ind instructions for submlting suspected cerrara, We are grateful to every reader who takes the time to help us improve the quality ‘of our books by pointing out an error. 1001 SOLVED ENGINEERING FUNDAMENTALS PROBLEMS ‘Third Raition ‘Current printing of this edition: 2 Printing History umber number update 2 ‘3 Minor corrections 3 1 Now edition. Copyright update. All units converted to SI 3 2 Minor corrections Copyright © 2006 by Profeasional Publications, Inc. (PPI). Allright reserved. No part of thia publication may be reproduc, stored in a rotrieval aystem, oF transmitted, in any form or by any means, electronic, mechanical, photocopying, recording, or otherw, without the prior written permission of the publisher. Printed in the United States of America PPL 1250 Fifth Avenue, Belmont, CA 94002 (650) 593-0119 ‘www pplzpass.com ISBN: 978-150126-002-8 ‘The CIP data is pending Carton rhowone Pee eee Oanenbas TABLE OF CONTENTS PREFACE TO THE THIRD EDITION... . . ACKNOWLEDGMENTS ........... HOW TO USE THISBOOK... 2... 0... MATHEMATICS Economics SYSTEMS OF UNITS ....... FLUID STATICS AND DYNAMICS... . ‘THERMODYNAMICS POWER CYCLES CHEMISTRY STATICS MATERIALS SCIENCE MECHANICS OF MATERIALS DYNAMICS . DC ELECTRICITY AC ELECTRICITY PHYSICS . . ao ee SYSTEMS MODELING. ....... COMPUTER SCIENCE . PROFESSIONAL PUBLICATIONS, INC. a1 BL a1 a1 on + 101 . el . 131 Mt 11 7 PREFACE TO THE THIRD EDITION In the good old days, when examinations for engineering licensing were in their infancy, most review books were mainly compilations of problems with little supporting theory—much like this book. Such books placed the burden on the examinees to accumulate and become familiar with numerous textbooks and references. ‘The National Council of Examiners for Engineering and Surveying (NCEES) changed all thet when it limited you to a single reference booklet that it provides. While this eliminated the “shopping cart syndrome” (wherein somo examinees literally brought shopping carts of books to the exam), it also changed the na- ture of how most examinees conduct their review. The scope of the exam has narrowed significantly, and the scope of the examinees’ review has shrunk ac- cordingly, placing more emphasis on the ability to quickly work certain standard types of problems. Since you cannot use your own reference books in the Fundamentals of Eng: neering (FE) exam, working countless practice problems has become the review ‘method of choice for many examinees. This book contains the grist for the problem review mill. Parallel to the actual exam, this edition features problems that all use the SI systom of units. Approximately half of the problems have been revised or re- placed in order to use these units. New problems have been added to cover high-probability subjects that had no representation in previous editions. The ‘book has been edited to incorporate PI's rigorous style and quality stan- Thope you will find this book particularly useful and valuable. Michael R. Lindoburg, PE PROFESSIONAL. PUBLICATIONS, INC. ACKNOWLEDGMENTS ‘The FE examination is a secure examination, which means problems are not made public after the exam is given. Therefore, all of the problems in this book were developed from scratch, based on our knowledge of the examination. Working from detailed outlines that I prepared, a team of more than 20 engineers and engineering students fleshed out the problems in the first edition. ‘This third edition shares a similar heritage with the first edition in that many people contributed to it. Timothy W. Zeigler, PE, revised, wrote, and rewrote approximately 750 problems for the Mathematics, Economics, Systems of Units, Fluid Statics and Dynamics, Thermodynamics, Chemistry, Statics Materials Seience, Mechanics of Materials, Dynamics, and Physics chapters. Similarly, ‘Yanging Du, PhD, PB, revised, wrote, and rewrote almost 300 problems for the Power Cycles, DC Electricity, AC Electricity, Systems Modeling, Computer Sei- ence, and Atomic Theory chapters. Without their authorship, this new edition ‘would still be sitting in the in-basket on my desk, Christopher Lew, a master’s program candidate at Stanford University and for- ‘mer engineering intern in the PPT editorial department, performed the first round of calculation and unit checks. You can thank him for the accuracy of the cal- culations. PPI editorial and production staff who world diligently on this book include: Sarah Hubbard, editorial director; Cathy Schrott, production director; Amy Schwertman, illustrator; Kate Hayes, typesetter; Heather Kinser, project editor; and Sean Sullivan, project editor. These people gave the book a consistency and personality of its own, The styles and conventions you see in this book are the result of their work, Many of the thousands of readers of the previous two editions sent in their com- ments and suggestions. Though they are nameless, this book is better because of their help. Michae! R. Lindeburg, PE PROFESSIONAL PUBLICATIONS, INC. HOW TO USE THIS BOOK Ifyou never read the material at the front of your books anyway, and if you are xe in a hurry to begin and you only want to read one paragraph, here it is. All chapters in this book are independent. Start with any one. Solve all the problems for which you have time. Don't peck at the answers. Work problems in your weak areas as well as in your strong areas. Keep studying until the exam, Good luck. However, if you want: a thorough review, you will probably want to know a little more about reviewing for the exam. The rest of this introduction is for you. ‘This book doesn’t contain any supporting theory and is not really meant to be used as a stand-alone exam review. It was meant to be used in conjunction with two other references: a FE exam review book (e.g, either the FE Review ‘Manual or Engineer-In-Training Reference Manual) and the NCEES Handbook. All three of the books listed are available from PPI Depending on your preference, you might decide to first review a subject and then work practice problems. Or, you might docide to jump right in and try to work the problems, reviewing only those subjects that are rusty or unfamiliar. ‘This book can be used either way. While working through the problems, you should try to use the NCEES Hand- book as your sole reference source. Refer to the FE Review Manual or Engineer- In-Training Reference Manual when you need to refresh your memory about subjects that have become dim. However, use the NCEES Handbook when you simply need a formula or data. That way, you will become intimately familiar with the only reference that is permitted in the examination room. PROFESSIONAL PUBLICATIONS, INC. 1 MATHEMATICS MATHEMATICS-1 The set A consists of elements {1,3,6}, and the set B consists of elements {1,2,6,7}. Both sets come from the universe of {1,2,3,4,5,6,7,8}- What is the intersection, APB? (A) 7) (B) 23,7} (©) {2,4,5,7,8} (D) {4,5,8) ‘The set of “not A” consists of all universe elements not in sot A: {2,4,5,7,8} ‘The intersection of {2,4,5, 7,8} and {1,2,6,7} is the set of all ele- ments appearing in both. ‘Thus ANB is {2,7}. ‘The answer is (A). MATHEMATICS-2 For a given function, f(t) = f(—t). What type of symmetry does f(t) have? (A) odd symmetry (B) even symmetry (C) rotational symmetry (D) quarter-wave symmetry When f(t) = f(t), the function is “mirrored” on either side of the vertical axis. This is known as even symmetry. The answer is (B). PROFESSIONAL PUBLICATIONS, INC 12 1001 SOLVED ENGINEERING FUNDAMENTALS PROBLEMS MATHEMATICS-3 ‘What is the value of each interior angle of a regular pentagon? © 2x x an As QBy Oy OF For a regular polygon, the value of each interior angle, 8, is (number of sides ~ 2) =~ jumber of sides For a regular pentagon, ‘The answer is (D). MATHEMATICS-4 A cubical container that measures 2 m on a side is tightly packed with eight balls and is filled with water. All eight balls aro in contact with the walls of the container and the adjacent balls. All of the balls aro the same size. What is the volume of water in the container? (A) 0.38 m? (B) 25 m® (©) 38m? (D) 42m? Since the balls are tightly packed, ray = 0.5 m. Veatar = Voox — 8Voait = (2 m)$ ~ (8) ($(0.5 m)*) =3.8 m> ‘The answer is (C). MATHEMATICS-5 Which number has four significant figures? (A) 0.0014 (B) oo14i4 (© oa (D) 14140 PROFESSIONAL PUBLICATIONS, INC. MATHEMATICS 13 ‘The number of significant figures, or digits, for each choice is (A) 2, (B) 4, (C) 3, and (D) 5. Only option (B) has four significant figures, ‘The answer is (B) MATHEMATICS-6 What is the solution of the equation 502% + 5(x — 2)? = —1, where x is a real-valved variable? (A) -6.12 and -3.88 (B) -0.52 and 0.700 (©) 75 (D) no solution For real-valued 7, the left-hand side of the equation must always be greater than or equal to zero, since all terms containing x are squared. ‘There is no solution to this equation for real values of z. The answer is (D). MATHEMATICS-7 What: are the roots of the cubic equation 2° ~ 82 ~3 (A) 2 = -7.90, -3, -0.38 (B) z=-3, -2,2 (C) c= -3, -0.38, 2 (D) 2 = 262, -0.38, 3 By inspection, +3 is a root, and (2~8) is a factor. Factor out (z—3). S88 yar 1 PROFESSIONAL PUBLICATIONS, INC. 14 1001 SOLVED ENGINEERING FUNDAMENTALS PROBLEMS Use the quadratic equation to solve 2? + 3z+1 3 yon4 3a 2.62, 0.38, 3 ‘The answer is (D) MATHEMATICS-8 Naperian logarithms have a base closest to which number? (A) 2u7 (B) 2.72 (©) 314 () 100 ‘Tho base of Naperian logarithms is the number ¢ F: 2.7183. Of the choices given, 2.72 is the closest to e. ‘The answer is (B). MATHEMATICS-9 ‘What is the radius of the circle defined by x* + y? — 4x + 81 (A) V3 (B) 2v5 (C) 3v5 (D) 4v3 Since the general equation for a circle is (x — a)? + (y — 6)? = r?, rearrange the equation given to fit the general equation dot y+ By =7 Complete the binomial forms. wade + dt y? + 8y tl (2-28 + (y+4y? = 27 aor avi ‘The answer is (C). T+4 +16 PROFESSIONAL PUBLICATIONS, INC. MATHEMATICS, 15 MATHEMATICS-10 ‘What is the natural logarithm of e*¥7 2.78 (A) = (B) zy (© 2.7180y ©) By definition, the natural logarithin of a number is Ine? =G Ine = ay The answer is (B MATHEMATICS-11 ‘What is the value of (0.001)?/#? (A) antilog (3 10g00.001) (B) Gantilog (lox 0.001) ‘i 0.001 (©) antiog | log 2S 3 (D) antilog (j log 0.001) log2* = aloge 1og(0.001)?/8 = 3 1og.0.001 (0.001)? ‘The answer is (D). antilog (3 1og0.001) MATHEMATICS-12 ‘The salary of an employee's job has five levels, cach one 5% greater than the one below it. Due to circumstances, the salary of the employce must be reduced from the top (fifth) level to the second level, which results in a reduction of $122.00 per month. What is the employee's present salary per month? (A) $440/mo (B) $570/mo ——(C) $680/mo ——_(D) $900/mo PROFESSIONAL PUBLICATIONS, INC. 16 1001 SOLVED ENGINEERING FUNDAMENTALS PROBLEMS ‘The salary levels represent a geometric sequence. Let $; be the salary at level i. Sy = 1.0552 S4= 1.0555 Ss = 1.0554 = (1.05)8S S So = (1.05) (s 122 ~ = 896/mo ($900/mo) Ss 122 MATHEMATICS-13, Which of the following statements regarding matrices is FALSE? (a) aNP=a (B) A(B+C) = AB+ AC (i 6) (2)-() (D) (AB)! = AB ‘The inverse of a product of two matrices is the product of the inverses, in reverse order. (AB)! =BOA"! ‘The answer is (D). MATHEMATICS-14 ‘What is the determinant of the following 2 x 2 matrix’ 59 7 6 (A) -33, (8) -27 (© 2 (D) 33 PROFESSIONAL PUBLICATIONS, INC. MATHEMATICS LT ‘The determinant, D, is calculated as follows. 59 m=h gl (5)6) — (90) 3 ‘Tho answer is (A). MATHEMATICS-15 ‘What is the determinant of the following matrix? 111 2-101 1 2-1 (ayo (8) 1 (5 (7 To find the determinant, expand by minors across the top row. 2a i ale] 7 al (1) = (2)()) = (2-1) = GAD) + (2) - AD) ‘The answer is (D). MATHEMATICS-16 What is the inverse of the matrix A? a-(24 88) PROFESSIONAL PUBLICATIONS, INC. 18 (Se ie) 2) (ereen cee) © (8 4) Oy (ae sind) a( 1001 SOLVED ENGINEERING FUNDAMENTALS PROBLEMS te) D is the determinant of X. For matrix A, D = cos? — (sind)(~sind) = cos" 0+ sin? = -1_ (cod ate (ge ‘The answer is (C). MATHEMATICS-17 ‘What is the rank of the matrix A? =( (ayo (B)1 sind cos (D) 3 PROFESSIONAL PUBLICATIONS, INC. ‘MATHEMATICS 19 ‘The rank of a matrix is the number of independent vectors (rows) ‘The rank can be found by row-redueing (diagonalizing) the matri ‘and counting the number of pivots in the row-reduced form of the matrix. Row 2 = (—2)(Row 2) + (3)(Row 4) Row 4 = Row 4 —Row 2 Row 3 = (2)(Row 3) + Row 2 The row-reduced form of A is 4) o 0 0 0 ‘The matrix cannot be further row-reduced. There are three pivots and, therefore, three independent rows. The rank of matrix A is 3. ‘The answer is (D). MATHEMATICS-18 Determine the values of x; and rp that satisfy the following, ()6)-0) a() (2) ©) 0) ‘The linear equations represented by this system are year system. 304 +72, =2 20; +622 =4 PROFESSIONAL PUBLICATIONS, INC. 1.10 1001 SOLVED ENGINEERING FUNDAMENTALS PROBLEMS Use Cramer's rule to solve the system of equations, 27 46 agi] 26 ‘The answer is (D). MATHEMATICS~19 Isina = 2, what is seca? (a) vi=a (B) © Sinco sina is the side facing angle a divided by the hypotenuse, the hypotenuse = 1, Therefore, side adjacent to angle a= 1-23 hypotenuse B80 Side adjacent to angle a The answer is (C). ‘PROFESSIONAL PUBLICATIONS, INC. MATHEMATICS aan MATHEMATICS-20 ‘Experimental data show that: a body’s temperature declines exponentially in time according to the expression T(#) = 50e~° (where 60 is a constant expressed in °C, 0.04 is the cooling rate in min=!, and ¢ is the cooling time expressed in minutes). How long would it take the body to reach 25°C? (A) 24min (B) 15.6min = (C) 16.5 min = (D) 17.3 min T(t) = 502-0 25°C = (60°C) e-204 mine 0.08 min~*t _ 25°C _ € 506 785 ‘Take the natural logarithm of both sides. ‘The answer is (D). MATHEMATICS-21 If tho sine of angle A is given as K, what is the tangent of angle A? w wo o% wm PROFESSIONAL PUBLICATIONS, INC. 12 1001 SOLVED ENGINEERING FUNDAMENTALS PROBLEMS The answer MATHEMATICS-22 ‘Which is true regarding the signs of the natural functions for angles between 90° and 180°? (A) The tangent is positiv (B) The cotangent is positive. (C) The cosine is n (D) The sine is negative. In the second quadrant, the natural functions and their signs are as follows, The answer is (C). MATHEMATICS-23 cot positive negative negative negative negative positive What is the inverse natural function of the cosecant? (A) secant (B) sine (©) cosine (D) tangent PROFESSIONAL PUBLICATIONS, INC. MATHEMATICS 143 In a tight triangle, the cosecant is the hypotenuse divided by the ‘opposite side, The sine is the opposite side divided by the hypotenuse. The answer is (B). MATHEMATICS-24 ‘What is the sum of the squares of the sine and cosine of ust angle? (A) 0 B) 1 © v8 (2 For any angle, cos? + sin? 2 = ‘The answer is (B) MATHEMATICS-25 ‘What is an equivalent expression for sin 22? (A) Ysinzcosz (B) 2sinzeos}x (C) -2sinzcosz (D) 2822 ‘The double angle formula for the sine function is, sin 2x = 2sinz cose _ 2sinz © seer ‘The answer is (D). PROFESSIONAL PUBLICATIONS, INC 14 1001 SOLVED ENGINEERING FUNDAMENTALS PROBLEMS MATHEMATICS-26 ‘The Taylor series expansion for cos: contains which powers of 2? (A) 0,2,4,6,8... ‘The Taylor series expansion for cos. is as follows. 2,8 ata ot ‘Only the positive even powers of 2 are contained in the expansion of ‘The answer is (A) MATHEMATICS-27 A transit set up 40 m from the base of a vertical chimney reads 32°90/ with the crosshairs set on the top of the chimney. With the telescope level, the vertical rod at the base of the chimney is 2.1 m. Approximately how tall is the chimney? (A) 15m (B) 26m (©) 28m (D) 38m ‘To find the height, H, of the chimney, refer to the following figure. x= 400m PROFESSIONAL PUBLICATIONS, ING. ‘MATHEMATICS 115 aie tang y= (40.0 m) tan 32.5° = 25.5 m H=210m+y 10 m+25.5 m ‘The answer is (C). 27.60 m (28 m) MATHEMATICS-28 At opproximately what time between the hours of 12:00 noon and 1:00 p.m. would the angle between the hour hand and the minute hand of a continuously driven clock be exactly 180°? (A) 12:28 p.m. (B) 1230 pm. (C) 12:33pm, (D) 12:87 pm. 7 @ ‘The change in the angle of the minute hand between 12:00 p.m. and 1:00 p.m., an, is = (6) ‘The change in the angle of the hour hand between 12:00 noon and 1.00 p.m., ax, is, : We miny = (0.5t)" PROFESSIONAL PUBLICATIONS, INC. 116 1001 SOLVED ENGINEERING FUNDAMENTALS PROBLEMS In tho precoding equations, ¢ is in minutes past 12:00 noon. The angle between the two hands is a1 — a2. ‘The time is approximately 12:93 pam, ‘The answer is (C). MATHEMATICS-29 In finding the distance, d, between two points, which equation is the appropriate one to use? (A) d= V@i— mF Ga—w (B) d= V@i—n)? + —mP (d=/@- mt G-w (D) d= Mi — 1? + (a -mP ‘The distance formula is defined as follows. d= V(t. — a1) +(-n)? ‘The answer is (D). MATHEMATICS-30 The equation y = a1 +020 is an algebraic expression for which of the following? (A) a cosine expansion series (B) projectile motion (C) a circle in polar form (D) a straight line PROFESSIONAL PUBLICATIONS, INC. MATHEMATICS 17 ma +b is the slope-intercept form of the equation of a straight line. Thus, y= a1 +agu describes a straight line. The answer is (D).. MATHEMATICS-81 Find the slope of the line defined by y = 5. (A) 5+2 (B) -1/2 (c) 1/4 (D) 1 ‘The slope-intercept form of the equation of a straight line is y = ‘me 4b, where m is the slope and b is the y-intercept. y-2 yarts ‘Tho coefficient of 2, m, is ‘The answer is (D). MATHEMATICS-32 Find the equation of a line with slope = 2 and y-intercept ‘The slope-intercept form of the given equation is, y=22-3 PROFESSIONAL PUBLICATIONS, INC. 118 1001 SOLVED ENGINEERING FUNDAMENTALS PROBLEMS MATHEMATICS-33 Find the equation of the line that passes through the points (0,0) and (2, ~2). (A) v=o (B) y= 2242 (C) y= —2e @) v= Since the line passes through the origin, the y-intercept is 0. Thus, the equation simplifies to y = msz. Substituting for the known points, 2-0 2-0) 7 2 ‘The answer is (D). MATHEMATICS-34 ‘What is the name for a vector that represents the sum of two vectors? (A) scalar (B) resultant (©) tensor (D) moment By definition, the sum of two vectors is known as the resultant. ‘The answer is (B). MATHEMATICS-35 ‘What is the resultant, R,, of the vectors F), Fa, and Fs? Fy = 4i+ 746k FP, = 9425+ 11k Fy = 51-3} - 8k (A) R= 181+ 6j+9k (B) R= ~185 - 6j- 9k (©) R= 181 + 12}-+ 25k (D) R=21 PROFESSIONAL PUBLICATIONS, INC. MATHEMATICS 1g ‘The resultant of vectors given in unit-vector form is the sum of the components. R AF OF E+ (7+2—-B))+ (6+ 11-8) = 18i +6) + 9k ‘The answer is (A). MATHEMATICS-36 Simplify the expression (A x B) - C, given Anst+2) Bo%+S)+k C= 5142 (©) G0i+24k — (D) 5i+2ke i +2) x (21+ 35+) k| 0 1 (2 0) ~ (3-0) + k(9— 4) = 4-3) +5k (24 — 8} + 5ke)- (Bi + 0) + 2k) = (2)(6) + (-8)(0) + (5)(2) 20 (AxB)-C MATHEMATICS-37 What type of curve is generated by a point that moves in uniform circular motion about an axis, while travelling with a constant speed, v, parallel to the axis? (A) acycloid —(B) anepieycloid (C) ahypocyeloid (D) a helix PROFESSIONAL PUBLICATIONS, INC. 1.20 1001 SOLVED ENGINEERING FUNDAMENTALS PROBLEMS A curve generated by the method described is called a helix and is, illustrated in the following figure. Tho answor is (D).. MATHEMATICS-38 ‘What is the term that describes a possible outcome of an experiment? (A) asample space (B) a random point (C) an event (D) a finite set By definition, an event is a possible outcome of a trial or experiment. ‘The answer is (C). MATHEMATICS-39 In probability theory, what is the term that describes the set of all possible ‘outcomes of an experiment? (A) a set of random events (B) a fuzzy set {C) a cumulative distribution {D) a sample space PROFESSIONAL PUBLICATIONS, INC. MATHEMATICS, Lat By definition, the sample space is the set of all possible outcomes of an experiment. ‘The answer is (D). MATHEMATICS~40 How can the values of a random variable defined over a sample space be described? (A) always continuous (B) always numerical (C) strictly nonzero (D) defined only over a finite horizon ‘The values of a random variable can be continuous or discrete over ‘a finite or infinite domain. ‘The values in the sample space can be shared by other sample spaces. However, the values of a random variable must be numerical. the answer is (B) MATHEMATICS~41 If two random variables are independently distributed, what is their relationship? (A) They are not identically distributed. (B) They are uncorrelated. (C) They are mutually exclusive. (D) Bither option (A) or option (B) is true. By definition, two independently distributed random variables are un- correlated. Any two random variables may or may not be identically distributed, Independent events cannot be mutually exclusive. The answer is (B PROFESSIONAL PUBLICATIONS, INC. 122 1001 SOLVED ENGINEERING FUNDAMENTALS PROBLEMS MATHEMATICS~42 Which of the following properties of probability is NOT valid? (A) The probability of an event is always positive and less than or equal to one. (B) If Zp is an event which cannot occur in the sample space, the probability of Bp Is zero. (©) Hevents B, and 2 are mutually exclusive, then the probability of both events occurring is zero. (D) events E; and By are events from the same sample space, then P(E) + Ey) = P(E) + P(E) — PELE). ‘The probability law given in option (D) is valid for events from two sample spaces, not events from a single sample space. ‘The correct, rule for events from a single sample space is P(E, + Ea) = P(E1) + P(E) The answer is (D). MATHEMATICS-43 ‘Which one of the following functions cannot be a probability density funetion for the variable 2? a x) 0 ®) Ax) PROFESSIONAL PUBLICATIONS, INC. MATHEMATICS 1.23 © fix) @) ‘To be a probability density function, the area under the curve must ‘qual 1. That is, the cumulative density function must sum to 1. The aren under the curve for option (A) is /s. Therefore, it cannot be probability density function. The answer is (A). MATHEMATICS~44 IE mis the number of trials, and m is the number of successes, what is the frequency based interpretation of the probability of event E? (A) PCE) = im *™ (B) P(E) = tim 2 (© P(e) = jim (D) P(E) = Jim. The probability of an event can be interpreted as the fraction of suc- cessful outcomes when the experiment is performed an infinite number of times. Thus, P(E) = im ™ The answer is (D). PROFESSIONAL PUBLICATIONS, INC. 124 1001 SOLVED ENGINEERING FUNDAMENTALS PROBLEMS MATHEMATICS-~45 For a continuous random variable X with probability density funetion f(x), what is the expected value of X? (A) BOX) = fp? afl) de (B) BX) = [,ef(e) de (©) BX) = fe Na) ae (D) BX) = f° ads The expected value or average of X can be defined mathematically as follows. Bx) = [" aeyde ‘The answer is (B). MATHEMATICS-46 If P(B) #1, and A and B are not independent events, what is P(A|B)? Pua (Payee) @) (ecaiay (P'4) (© Pla) ) (Pears (2) ‘Te probaly of erent A oxcrrng, given thatthe dependent event hae occured, predited by the onion probably law om monly known sa Bayes theores, (BIA) = - Similarly, Pais) = Fa) ‘Therefore, P(AIB) = (reeiap (rt) ‘The answer is (B). PROFESSIONAL PUBLICATIONS, INC. MATHEMATICS 1.25 MATHEMATICS-47 Ifthe discrete random variable X has a geometric distribution parameter P and smallest mass point 0, what is the expected value of X? (AP BP (© PrP ©) > ‘The geometric distribution is a special case of the negative binomial distribution. ‘The mean is (1 —P)/P, and the variance is (1 ~ P)/P*. ‘Note: Some authors define the geometric distribution with the small- ‘eat mase point being 1 (instead of 0). In that case, the mean is 1/P and the variance is the same as before. ‘The answer is (D). MATHEMATICS-~48 If the variable X has a Poisson distribution with parameter 1, what is the expected value of X? (A) 2 (8) AG-a) (©) x" @)A For the Poisson distribution, both the mean and variance are equal tod. The answer is (D) MATHEMATICS-~49 If X is a binomial random variable with parameters n and p, what is the expected value of X? (A) nl-p)—(B) mwt-») (OP (D) np For a binomial distribution, the mean is np, and the variance is np( —p). ‘The answer is (D). PROFESSIONAL PUBLICATIONS, INC. 1-26 1001 SOLVED ENGINEERING FUNDAMENTALS PROBLEMS MATHEMATICS-50 For a discrete random variable X with probability mass function P(X), what is the expected value of X? (A) EX) = Yo aP@) Fy (B) E(X) = DO a}P(a) a (© Bx) = YPC) (D) BCX) = Ye -2)Ple) The expected value of a discrete function is given by the following, E(X) = 2: P(z1) + 22P(a2) + =D aPe@) ae ‘The answer is (A) MATHEMATICS-51 An item's cost distribution is given. What is the approximate expected cost? cost (8) probability i a 0.23 3 0.46, 4 0.17 5 0.04 6 0.03 (A) 82.5 (B) 82.9 (©) $3.0 (D) $3.1 ‘The expected value is the sum of the products of the individual values and their respective probabilities. PROFESSIONAL PUBLICATIONS, INC. ‘MATHEMATICS 1.27 (cost) = ($1)(0.07) + (82)(0.23) + ($3)(0.46) + (84)(0.17) + (85)(0.04) + (86)(0.03) 82.97 ($3.0) The wer is (C) MATHEMATICS-52 One fair die is used in a dice game, The player wins $10 if he rolls either a 1 or 2.6. He loses $5 if he turns up any other face. What is the expected winning for ‘one roll of the die? (A) $0.00 (B) 83.33 (©) $5.00 (D) $6.67 For a fair die, the probability of any face turning up is "/o. ‘Therefore, ‘the expected value is Bain = (10) (@ (2) - 8) (40 (3)) = 8000 MATHEMATICS-53 ‘An urn contains four black balls and six white balls. What is the probability of getting one black ball and one white ball in two consecutive draws from the urn without replacement? (A) 0.040 (®) 024 (© 027 (D) 0.53 P(black and white) = P(black then white) + P(white then black) (a) 6) Ge) @) = 0.53 hhe answer is (D) PROFESSIONAL PUBLICATIONS, INC. 1.28, 1001 SOLVED ENGINEERING FUNDAMENTALS PROBLEMS MATHEMATICS-54 The probability that both stages of a two-stage rocket, will function correctly 1s 0.95. The reliability of the first stage is 0.98. What is the reliability of the second stage? (A) 0.95 (B) 0.96 (©) 0.97 (D) 0.98 In a serial system consisting of two units, R= RR In the preceding question, Ra is the reliability of stage 2, and Ry is the total reliability of all stages. For the second stage, ‘The answer is (C) MATHEMATICS-55 ‘What is the exponential form of the complex number 3+ 4? (A) esa" (B) Be" (©) Bet760” (D) 70188 Any complex number a + bi can be converted to its equivalent expo- unential form as follows, at bi = Val + Betarctan b/a Therefore, BH di = (Fp Aeterna arctan 4 = 53.1° 844i = 5c ‘The answer is (B) PROFESSIONAL PUBLICATIONS, INC. MATHEMATICS 1-29 MATHEMATICS-56 What is the product of the complex numbers 3+ 4i and 7 ~ 24? (A) 10-428 (B) 13-422 = (©) 13 + 344 (D) 29+ 224 (8+ 4i)(7 — 21) = 21 — 81? + 28% — 64 =21+8428i—6i 29-4 22% The answer is (D} MATHEMATICS-57 What is the rectangular form of the complex number 7.2e""/""? (A) 715 40.874 (B) 7.150.874 (©) 0.87 + 7.15% (D) -0.87 ~ 7.18% A complex number of the form ce" can be converted to rectangular form as follows. ce! = coosd + (csind)i eilT/19) ra (gin zat = 04) (02S + (cn'2).) 087 +7151 ‘The answer is (C). MATHEMATICS-58 ‘What is the product of the complex numbers 2—2i and y32e*/47 (A) 16 B) 16: (©) 16¢'*/4 (D) 1601-4) 2-2) = Vy Bet arctan -2/2 = Viet PROFESSIONAL. PUBLICATIONS, INC. 1-30 1001 SOLVED ENGINEERING FUNDAMENTALS PROBLEMS ‘Therefore, (= 21) VIB"! = Yhe!*/4 VFM = ViVBel/t-2) =16 ‘The answer is (A). MATHEMATICS-59 What is the rationalized value of the following complex quotient? 6 +251 344i (A) -0.32+0.66 (B) 0.320.668 (C) 11-0.66% — (D) -1.7411 In order to rationalize a complex number, multiply the numerator and denominator by the complox conjugate of the denominator. 642.51 _ (6 +2.5i)(3 ~ di) Sea ~ GFa)G—a) 16.54 aaa = 11-0661 The answer is (C). MATHEMATICS-60 ‘What is the first derivative with respect to 2 of the function g(2) = 4/5? (Ayo @) 4/9 «4 () 2 ‘The derivative of a constant is zero. Therefore o/() ‘The answer is (A). PROFESSIONAL PUBLICATIONS, INC. MATHEMATICS 131 MATHEMATICS-61 Ifa is a simple constant, what is the derivative of y = 2%? (A) ax (B) a"! (©) axe? @) (@-1)e ‘The answer is (C). MATHEMATICS-62 What is the derivative of f(2) = (2° — (2-19)? {A) 32? - 3(@ - 1)? (B) 3(¢8- @-1)9)* (C) 9 (29 — @—1)9) (@? -(@- 1) (D) 9(2* - (@—1)°)" (@ - @-1)") f(a) = (2 = (@-1)°)° F@) =8 (2 (2D Z (@ -(e-1)9) = 3 (2 — (= 1)°)° (82? — 3(2 - 1)%()) =9 (29 —(2—1)9)" (2? - @-1)) ‘The answer is (D). PROFESSIONAL PUBLICATIONS, INC. 1.92 1001 SOLVED ENGINEERING FUNDAMENTALS PROBLEMS MATHEMATICS-63 Differentiate f(z) = V2? + 4241. (A) 242 @) ive (0 ett? Viet dest () #4 Vaated tt Sle) = Vie ead = (20 +4041)? Wa) = 5 (a? + 40-41) S (20? +d +1) 1 (4 +4) MATHEMATICS-64 Find the second derivative of y = ViT-+ 2 (A)1-22-9 — (B)1-e-~* (3 os ya VE +e Note: 2/Va? = +1 because by defini ‘The answer is (D). PROFESSIONAL PUBLICATIONS, INC. MATHEMATICS 1.33 MATHEMATICS-65 Find dy/dt given the following two simultaneous differential equations. (a) 5 (cost-+ $v 34) @) j (ont + bev — $8) © int + bx-+y?— 3) 2 (0) 5 (cost + $e — Su 34) Solve both equations for dz/dt. a e #23 (eey-asatl) we 4 #1 (atte-2) a3 Combine and solve for dy/dt. 1 e a 5 (e+y-2+0%t) =} (wot re -2%) a) 3 at dy dy 9M 5 gM ap a 9G + AGE = Bk — By + 80+ 2eost +2: dy _ yy 13} = 2eost + Sa — Sy — 3k dy _ 2 Se =F (cost + fo - Bu - 38) ‘The answer PROFESSIONAL. PUBLICATIONS, INC. 134 1001 SOLVED ENGINEERING FUNDAMENTALS PROBLEMS MATHEMATICS-66 If y = cosa, what is dy/de? (A) see (B) - seer (©) ese (D) ~sinz qpome a nein ‘The answer is (D). MATHEMATICS-87 If the second derivative of the equation of a. curve is proportional to the negative of the equation of the same curve, what is that curve? (A) a hyperbola (B) asquarewave (C) asinusoid — (D) a cycloid The only type of function that fits the description is a sinusoidal one. a Apsing = © cos ae ae sing The answer is (C). MATHEMATICS-68 Given P= 2299717? + RV9S sin 27, what is OP/OT? (A) RES*T9/2 4 2R1 cos 27 (ons 2 ar (©) 28897"? + RYSS cos 27 (D) RES°T-¥2 4 2R'85 cos2T All variables other than 7" are treated as constants. OP < opess (Lp?) 4 pV8S(c0s27)(2) oF 2 = RES7-V7 4 ORS cos TP ‘The answer is (D). PROFESSIONAL PUBLICATIONS, INC. MATHEMATICS 1.35 MATHEMATICS-69 Which of the following describes the first derivative at point A of the function shown in the figure? y A (A) positive only (B) negative only (©) zet0 (D) positive oF negative y ‘ A, ‘The first derivative corresponds to the slope of a tangent line at the point. The slope of this tangent line is positive. Therefore, the first derivative of the function at point A is also positive. ‘The answer is (A). PROFESSIONAL PUBLICATIONS, INC. 1-36 1001 SOLVED ENGINEERING FUNDAMENTALS PROBLEMS MATHEMATICS-70 Which of the following describes the second derivative at point A of the function shown? (A) positive only (B) negative only (© zero (D) positive or negative ‘The second derivative corresponds to the concavity of the function, Since the curvature at this point is concave down, the second deriva- tive is negative. The second derivative also indicates what is happen- ing to the first derivative, the slope. Since the slope is decreasing at point A, the second derivative must be negative. ‘The answer is (B).. MATHEMATICS-71 ‘What is the slope of the graph y = ~2? at x = 2? (A) -4 B® (1 () 3 ‘The slope of a curve is given by the first derivative. uz) v@) Ate 2) = (~2)@) 4 ‘The answer is (A). PROFESSIONAL PUBLICATIONS, INC. ‘MATHEMATICS 137 MATHEMATICS-72 Given the function f(z) atc= (A) 2 (B) 327-5 —52+2, find J"(2), the value of the first derivative @) 8 FQ) = @QP=5 ‘The ancwer is (C) MATHEMATICS~73 Find the slope of the tangent to a parabola, y where x = */, (A) -1/2 (B) 0 (©) 1/4 (D)1 fat a point on the curve MATHEMATICS~74 ‘What is the slope of the curve — 4 at the origin’? (A) -4 (B) -3 (c) 0 (@)4 PROFESSIONAL PUBLICATIONS, INC. 1-38 1001 SOLVED ENGINEERING FUNDAMENTALS PROBLEMS ‘Tho answer is (A). MATHEMATICS-~75 Find the slope of the line tangent to the curve y= 23 ~ 22-41 at r= (ay ya @ 1 (2) 1/2 1 yor? -2rt1 y= 827-2 2 ¥Q)= =1 The answer is (D). MATHEMATICS~-76 Determine the equation of the line tangent to the graph y = 2r*-+1 at the point (1,3). Ay (B) y=4e-1 (C) ys2e—1 First, determine the slope of the graph at x = 1 PROFESSIONAL PUBLICATIONS, INC. MATHEMATICS 1.39 Since the tangent line intersects the graph at (1,3), the equation of ‘the tangent line is y=4e+b ayy +o ‘The answer is (B). MATHEMATICS-77 Given yy = 42-+3 and yy = 2? + G, find C such that yp is tangent to ys. (ay2 (B)4 5 @) 7 The slope of y; = 4x +3 is 4 everywhere, Therefore, yz has a slope of 4 at the tangent point. x =2at the tangent point. Find y: = yz at the tangent point and substitute in to find C. ‘The answer is (D). PROFESSIONAL PUBLICATIONS, INC. 1.40 1001 SOLVED ENGINEERING FUNDAMENTALS PROBLEMS MATHEMATICS~78 Given What is the value of k such that y: is perpendicular to the curve yz = 2x at =i? (a2 @s sé (07 For two lines to be perpendicular, my = —1, where my isthe slope of line n : te _ ae? ‘Therefore, at (1,1), ‘The answer is (D). MATHEMATICS~79 ‘The location of a body as a function of time is 2(¢) = 18t +912, velocity at ¢ = 2. the body's (A) 20 (B) 24 (©) 36 (D) 54 ‘PROFESSIONAL PUBLICATIONS, INC. MATHEMATICS La Velocity is the first time derivative of the position function. a(t) = 18e +90 v(t) =2/() 18+ 18 -v(2) = 18 + (18)(2) LE The answer is (D). MATHEMATICS-80 [A particle moves according to the following functions of time. ‘What is the resultant velocity at t = 1? (ayo (B) 3 (4 (D) 9 (&) +(@) dy zt y(t) = V9cos* t + 16si va) = VOI FCN OF ‘The answer is (B) [PROFESSIONAL PUBLICATIONS, INC. 142 1001 SOLVED ENGINEERING FUNDAMENTALS PROBLEMS MATHEMATICS-81 ‘Water is pouring at a varying rate into a swimming pool that is initially empty. After t hours, there are ¢ + vi liters in the pool. At what rate is the water pouring into the pool when f = 9 h? (A) 1/6L/h (BY /2Lh_— (OAL (D) 7/6L/h Let V = volume of water in the tank in liters and Q = flow rate in fers per hour. Ventsvi aw. 1 Oa le Aet= ob, Qe1¢ 35 = 7/6 L/h The answer is (D). MATHEMATICS-82 If x increases uniformly at the rate of 0.001 per unit time, at what rate is the expression (1 + 2)° increasing when becomes 9? (A) 0.001 (B) 0.003 (C) 0.3 @t = 0.003(1 +2)? g| Si, = (00an +9)? =03 ‘The answer is (C). ‘PROFESSIONAL PUBLICATIONS, INC MATHEMATICS 143 MATHEMATICS-83 A spherical balloon is filled with air at a rate of 1 m?/s. Compute the time rate of change ofthe surface area ofthe balloon at the instant the balloon’s volume is 13.1 mi (A) 067 m/s (B) LT m¥/s (CO) 3.1 m/s (D) 3.7 m/s -O4 Solve for the radius of the balloon when the volume is 113.1 m°, and substitute into the equation for the rato of change of the surface area. (=)" - (aus 1 zy" ar r=3m 2-05 )09) 0.67 m?/a The answer is (A). PROFESSIONAL. PUBLICATIONS, INC. 14d 1001 SOLVED ENGINEERING FUNDAMENTALS PROBLEMS MATHEMATICS-84 Consider a strictly concave up function of one variable, «, with lower and upper bounds on 2. At what value(s) of z will the function be minimized? (A) at the upper bound of 2 (B) at the lower bound of = (C) strictly between the upper and lower bounds of (D) at any of the above 1B UB=x x LB (lower bound) (upper bound) ‘The answer is (D). MATHEMATICS-85 Consider a strictly concave down function in one variable, x, with lower and fx \ fox 7 tu = ux NY UB ‘The examples given demonstrate that, for concave up function, the minimum could occur at the lower bound, the upper bound, or somewhere between. Option (A), (B), or (C) could be correct. upper bounds on . At what value(s) of x will the function be minimized? (A) at the upper bound of z (B) at the lower bound of 2 (C) strictly between the upper and lower bounds of = (D) at either the upper ot lower bound of z to toe a _~ wor ux iB ua=" x (lower bound) (upper bound) PROFESSIONAL PUBLICATIONS, INC. oT) MATHEMATICS 145 The illustrations demonstrate that for a concave down curve, the min- imum could occur at either the lower or the upper bound. Therefore, option (D) is correct. ‘The answer is (D). MATHEMATICS-86 ‘What is the maximum of the funetion y= ~2° + 82, for 2 > -1? (A) 2 (@®) -1 ©@o @)2 and y/’ <0 or at an endpoint. ‘The maximum oceurs where y/ y=—2°+3r yf = 327 +3 yf! = 62 yv O= 827 +3 Pal = 1 [+1 is also an endpoint) (1) = -(-1) + @)(-1) =-2 (1) = -()* + (8)Q) a Vana 2 MATHEMATICS-87 ‘The cost, C, of an item is a function of the quantity, x, of the item: C(z) = 2? — 4000s + 50. Find the quantity for which the cost is minimum (A) 1000 (B) 1500 (©) 2000 (D) 3000 PROFESSIONAL PUBLICATIONS, INC. 1.46 1001 SOLVED ENGINEERING FUNDAMENTALS PROBLEMS: ‘Thus, cost is a minimum when x = 2000. ‘The answer is (C). MATHEMATICS-88 Compute the following limit. ee oa s3 (ayo @1 (2 (0) 0 Divide both the numerator and denominator by z, and allow x to approach infinity. 2 at " ite oe z lio ~1=0 el ‘The answer is (B). ‘PROFESSIONAL PUBLICATIONS, INC. MATHEMATICS a7 MATHEMATICS-89 Simplify the following expression. (Ayo (B) 8 © (D) 16 sor the numerator, and simplify tho fraction before taking the time Z im AME +4) wT eS = lige + =8 MATHEMATICS-90 ‘Compute the following limit. (ayo @ iA (© 12 1 Since both the numerator and denominator approach zero, use LHepital's rule. L'Bopitals tule states that the derivative of the numerator divided by the derivative of the denominator has the same ts the original fraction, provided that both the numerator and denominator of the original fraction approach zero 1 cose Since the numerator and denominator both approach zero, apply ‘The answer is (C). ‘PROFESSIONAL PUBLICATIONS, INC. 148, 1001 SOLVED ENGINEERING FUNDAMENTALS PROBLEMS MATHEMATICS-91 ‘The existence of the two equations, ¥/ = J( of the following equations is true? (A) 62) = ff(@)de +e and y = $(2), implies that which (B) o(z) = f(z) (C) (2) = J f(z)dz +0 (D) ¢@) =a y= (2) &@ ra, (2) = [ve +o Since v= f(z) = [tease MATHEMATICS-92 Fill in the blank in the following statement. ‘The integral of a function betwoon certain limits divided by the difference in abscissas between those limits gives the ____ of the function. (A) average (B) middle (©) intercept __(D) asymptote [x ‘The answer is (A). = the average value of the function PROFESSIONAL PUBLICATIONS, INC. ‘MATHEMATICS 1.49 MATHEMATICS-93 Find the area under the curve y = 1/x between the limits y = 2 and y = 10. (A) 1.61 (B) 2.39 (c) 371 (D) 897 The area under the curve f(2) between 21 and <2, A, is given by A= £ " f(a)ae ‘The @ limits corresponding to the y limits are x = 1/2 and x = 1/10. 1/2 An [har hji0 12 = Ing 1/30 =161 (A). MATHEMATICS-94 Find the area of the shaded region between y = 6r~1 and y = '/«r + 3, bounded by «= 0 and the intersection point. ye 6x-4 (A) 32/529 (B) 16/23 (©) 32/23 (D) 1440/529 PROFESSIONAL PUBLICATIONS, INC. 1.50 1001 SOLVED ENGINEERING FUNDAMENTALS PROBLEMS ‘The area between curve 1 and curve 2 is equal to the area under curve 1 minus the area under curve 2. The intersection point of the ‘two curves is found by equating both functions. Ge-1= 4243 ‘The area, A, is, ‘The answer is (C). MATHEMATICS-05 If it is known that y = 1 when x = 1, what is the constant of integration for the following integral? ue / (e* -22) az (A) C=2-2 (B) C=3-e2 (C) Cn4-e ©) c=} 4-2) PROFESSIONAL PUBLICATIONS, INC. MATHEMATICS: 151 ule) = [eree2 f nae ea +O Lege =F 2) 40 However, v(l 1=}(2-@ay)+e 1 =5 (8-240 Cal41- he =1U-e) 3 MATHEMATICS-96 It is known that y(z) passes through the points (0,2) and (1,4). Solve for y(z) if the second derivative is. (A) y= (2? +32) 42 (8) v= F (+32) 42 (v= 5 (ea) -2 (D) y= 5 (@? +82) -2 Tategrate twice to get the general form of the equation @y = dy = fae =t4+0Q, v= fere) =? 4024+ PROFESSIONAL PUBLICATIONS, INC. 182. 1001 SOLVED ENGINEERING FUNDAMENTALS PROBLEMS Now solve for C; and Cz using the given conditions. S@+C0)+e 2 Cy =2 el 4= FY + O(a) 42 C=3/2 yaks feve ‘The answer is (B). MATHEMATICS-97 ‘What is a solution of the first-order difference equation y(k +1) = y(hk) +5? (a) way aa-F (B) y(k) = C —k, where C is a constant “ye (©) oy = LE (D) 9(k) = 204 5k Asus the yolution has the form u{k) = 20+ 5k Substitute the assumed solution into the difference equation. uk +) 10 + 5(k + 1) = 2045-45 y(k) +5. The answer is (D). ‘PROFESSIONAL PUBLICATIONS, INC. ‘MATHEMATICS 1.53 MATHEMATICS-98 What is the solution of the linear difference equation y(& + 1) = 15y(k)? (A) w= see @) 9) = R (C) y(k) + 15*, where C is a constant (D) y(k) = 15" Assume the solution has the form y(k) = 15" Substitute into the difference equation. (k-+1) = 158+! 15) (154) = 15y(k) Note: If y(k) = C+ 154, then y(k +1) =C +19" 4 15y(k). The answer is (D). MATHEMATICS-99 ‘What is the solution of the linear difference equation (k-+1)y(k-+1)—ky(k) = 1? (A) a) =28-E (B) y(k) (C) y(k) = 12+ 3k ©) vie) = 342 ‘Assume the solution has the form uk) PROFESSIONAL PUBLICATIONS, INC. 154 1001 SOLVED ENGINEERING FUNDAMENTALS PROBLEMS Substitute the solution into the difference equation, (&+1)(U( +1) — ACU 2 (+1) 1-23)-#0-2 ween (H5¥) (4? RFT F k+1-2-k+ = 1 ‘Thus, y() —12/k solves the difference equation. ‘The answer is (B). MATHEMATICS-100 ‘Which of the following is « differential equation of the first order? (A) (y' +29 =-3 AA first-order differential equation contains only first derivatives and does not have partial derivatives. The only choice that fulfills this requirement is option (C). ‘The answer is (C). PROFESSIONAL PUBLICATIONS, INC. MATHEMATICS: 185 MATHEMATICS~101 How can the following differential equation best be described? Px de oF + BOG +C= De) (A) linear, homogeneous, and first order (B) homogeneous and first order (C) linear, second order, and nonhomogeneous (D) linear, homogeneous, and second order ‘The differential equation has a second derivative, so it is of second ondet. ‘The forcing fanction is nonzero, 60 the equation io nonhomo- geneous. All of the terms on the left-hand side only have coefficients that are either constant or a function of the independent variable. ‘Therefore the equation is also linear. The answer is (C) MATHEMATICS-102 ‘The differential equation given is correctly described by which of the following choices? (A) linear, second order, homogeneous (B) nonlinear, second order, homogeneous (C) linear, second order, nonhomogeneous (D) nonlinear, second order, nonhomogeneous Since there is a second derivative, the differential equation is of sec- ‘ond order. Since the cocfficient of one of the terms contains the dependent variable, y, the equation is nonlinear. Since the forcing function, f(z), is implied to be nonzero, the differential equation is also nonhomogeneous. the answer (D), PROFESSIONAL. PUBLICATIONS, INC. 186 1001 SOLVED ENGINEERING FUNDAMENTALS PROBLEMS MATHEMATICS-103 Determine the solution of the following differential equation. yitby=0 (A) y=52+0 (B)y=Ce (0) y=Ce* —(D) (A) or (B) ‘This is a first-order linear equation with characteristic equation 7+5=0. Therefore, the form of the solution is y=Ce-™ In the preceding equation, the constant, C, could be determined from ‘additional information, the answer is (B) MATHEMATICS-104 ‘What is the general solution of the following differential equation? FU say =o (A) y=sinz + 2tanz+C (B) y=e"-2e-" +0 (C) y=2x?-2+C (D) y=sin2r + cos2r+C Examination of the ifferential equation shows that a multiple of the function and its second derivative must sum to zero. Sines and cosines have the property that their second derivatives are the negatives of the original natural function. If y = sin 2x + 00s 2z, then xy = 2cos2r — 2sin2e yf’ = —4sin 2x — 400822 ay! +4y = —4sin 2x — 4eos2x + 4(sin 2z + cos2z) = ‘The function in option (D) solves the differential equation. ‘The answer is (D). PROFESSIONAL PUBLICATIONS, INC. MATHEMATICS 1.57 MATHEMATICS-105 In the following differential equation with the is the value of x(2)? al condition 2(0) = 12, what de Gta=0 (A) 335 10-4 (B) 403 10-3 (C) 3.35 (D) 6.04 ‘This is a first-order, linear, homogeneous differential equation with characteristic equation r +4 = 0, a +de= 2(0) = aoe MATHEMATICS-106 ‘A curve passes through the point (1,1). Determine the absolute value of the slope of the curve at x = 25 if the differential equation of the curve is the exact equation y#dr + 2eydy = 0. (A) 1/250 B) 1/135 (©) 1/50v8_—(D) 1/V TB wda + 2aydy = 0 Daydy = vide PROFESSIONAL PUBLICATIONS, INC. 1.58 1001 SOLVED ENGINEERING FUNDAMENTALS PROBLEMS Integr both sides, 2iny = —Inz+InC Iny? +Inz = nC Inzy? = Inc np =o Use the fact that the curve passes through the point (1,1) to solve for , then determine the slope at x = 25. ‘The answer is (A). MATHEMATICS-~107 Determine the constant of integration for the separable differen de + 6y*dy = 0. It is known that x = 0 when y= 2. (a) 2 (B) 16 (©) 24 (D) 64 ‘equation Since this differential equation is alrendy separated, integrate to find the solution. [eset [oven fo ¢ det tyP= PROFESSIONAL PUBLICATIONS, INC. MATHEMATICS 1.59 ‘The answer is (D} MATHEMATICS-~108 ‘What is the Laplace transform of e“"*? Ooh ® Oc oe ‘The Laplace transform of a function, £(f), can be calculated for the definition of a transform. However, it is easier to refer to a table of transforms, ‘The answer is (A). ‘PROFESSIONAL PUBLICATIONS, INC. 2 ECONOMICS ECONOMICS-1. How is the capital recovery factor (A/P, i, n) related to the uniform series sinking fund factor (A/F, i, n)? 4 is the effective annual rate of return, and 1 is, the number of periods (A) (A/Prisn) = (A/Pign) +6 (B) (A/Pign) = (A/Fyign) =i — A/F) (A/F, i,n) +4 7m (C) (A/P,i,n) (D) (A/Pyign) = By definition, ‘The answer is (A). ECONOMICS-2 ‘What is an annuity? (A) the future worth of a present amount (B) an annual repayment of a loan (C) a series of uniform amounts over a period of time (D) a lump sum at the end of the year ‘The answer is (C). PROFESSIONAL. PUBLICATIONS. INC. 22 1001 SOLVED ENGINEERING FUNDAMENTALS PROBLEMS ECONOMICS-3 Which of the following expressions is INCORRECT? (A) Tho future worth of a present amount, (F/P,i,n), oF oa (B) Tho fata worth of an annuity, (F/A,t.n) = Cp AT (C) The present worth of an annuity, (P/A,i,n), TPT (D) (A/F,in) (A/P,i,n) (A/F yin) +i = (A/P,in) Therefore, option (D) is false. ‘The answer is (D). ECONOMICS~4 ‘When using net present worth calculations to compare two projects, which of the following could invalidate the calculation? (A) differences in the magnitudes of the projects (B) evaluating over different time periods (C) mutually exclusive projects (D) nonconventional eash flows Options (A), (©), and (D) are all problems with internal rate of return calculations that net present worth handles nicely. Howover, the net present: worth of two projects must be calculated for the same time period. ‘The answer is (B). ECONOMICS-5 ‘What is most nearly the present worth of a $100 annuity over a 10 yr period if the interest rate is 896? (A) 8450 (B) $530 (©) $670 () $700 PROFESSIONAL. PUBLICATIONS, INC. ECONOMICS 23 P= A(P/A,i,n) = ($100)(P/A,8%, 10) = (8100)(6.71) =8671 ($670) The answer is (C). ECONOMICS-6 With a 12% interest rate, approximately how much money must be invested today in order to withdraw $1000 per year at the end of each year for 10 yr? (A) $4300 (B) $5650 (©) $5800 (D) $6150 P=A(P/A,i,n) = ($1000)(P/4, 12% 10) = ($1000)(5.650) = $5650 ECONOMICS-7 ‘A machine is under consideration for purchase. ‘The cost of the machine is $25,000. Each year it operates, the machine will generate a savings of $15,000. Given an effective annual interest rate of 18%, what is the discounted payback period on the purchase in the machine? (A) 1.67 yr (B) 1.75 yr (©) 215 yr (B) 3.7 yr P A(P/A,i,n) =4(Sa) PROFESSIONAL PUBLICATIONS. INC. 24 1001 SOLVED ENGINEERING FUNDAMENTALS PROBLEMS Substituting, (L+0.18)" =1 Cone Tap) $825,000 = ($15,000) ( (0.3)(1.18)" = 1.18" -1 (0.7)(1.18)" =1 The answer is (C). ECONOMICS-8 What is the present worth of two $100 payments at the end of the third and fourth years if the annual interest rato is 896? (A) $122 (B) 8153 (©) 8160 (D) 8162 P=A(P/A,in) = ($100) ((P/A, 8% 4) ~ (P/A,8%, 2)) = (8100)(3.31 — 1.78) = $153 answer is (B). ECONOMICS-9 Consider @ project that involves the investment of $100,000 now and $100,000 at the end of year 1. Revenues of $150,000 will be generated at the end of years 1 and 2. What is most nearly the net present value of this project if the effective annual interest rate is 10%? (A) $43,300 (B) $50,900 (©) $69,500 (D) $78,500 PROFESSIONAL PUBLICATIONS, INC. ECONOMICS 25 {$100,000 + ($180,000 — $100,000)(P/F, 10%, 1) + ($150,000)(P/F, 10%, 2) $100,000 + (850,000)(0.9091) + ($150,000)(0.8264) = $69,415 (869,500) ‘The answer is (C). ECONOMICS~10 At an annual rate of return of 8%, what is the future worth of $100 at the end of year 4? (A) $130 (B) $132 (C) $135 (D) $136 P= AE/A,i,n) = (8100)(F/P,8%, 4) ($100) (1.3608) ‘= 136,05 ($136) The answer is (D) ECONOMICS-~11 |A person invests $450 to be collected in 8 yr. Given that the interest rate on the investment is 14.5%/yr, compounded annually, most nearly what sum will be collected 8 yr from now? (A) $1050 (B) $1130 (©) 81240 (D) 81330 P= A(E/P,i,n) = ($450)(F/P, 14.5%, 8) = ($450)(2.954) $1329 ($1330) ‘The answer is (D). ‘PROFESSIONAL PUBLICATIONS, INC 26 1001 SOLVED ENGINEERING FUNDAMENTALS PROBLEMS ECONOMICS-12 An investment of « dollars is made at the end of each year for 3 yr, at an interest rate of 9% per year compounded annually. What will the dollar value of the total investment be, most nearly, upon the deposit of the third payment? (A) 80.772 (B) $1.32 (©) $232 (D) 83.32 P= A(F/A,isn) = a(n =2(2 +0.09)° 0.09 = 83.2782 (83.32) ‘The answer is (D) ECONOMICS-13 If $500 is invested at the end of each year for 6 yr at an effective annual interest rate of 7%, what is most nearly the total dollar amount available upon the deposit of the sixth payment? (A) $3000 (B) $3210 (©) $3580 (D) $4200 P= A(F/A,i,n) = (8500)(F/A,7%,6) ($500)(7.153) ‘The answer is (C). ECONOMICS-14 ‘Assuming i = annual rate of return, n = number of years, F = future worth, and P = present worth, what is the future worth of a present amount P? (A) P+)" (B) PAF)! (POF) — (D) PO+ny ‘PROFESSIONAL PUBLICATIONS, INC. ECONOMICS 27 This situation corresponds to a single payment compound amount. ‘Therefore, F=Pa+i The answer is (A) ECONOMICS-15 $1000 is deposited into a 9% account today. At the end of 2 yr, another $3000 will be deposited. In 5 yr, a $4000 purchase will be made. Approximately how much will be left in the aeconnt: yr after the purchase? (A) $1230 (B) $1420 (©) 1540 (D) $1690 year___ cash flow (8) 1000 0 3000 0 0 4000 0 F = A(F/A,isn) = (81000)(/P, 9%, 6) + (83000)(F/P, 9%, 4) — ($4000)(F/P, 9%, 1) = ($1000)(1.6671) + (88000)(1.4116) — (84000) (1.0900) = $1542 ($1540) ‘The answer is (C)- ECONOMICS-16 ‘A student needs $4000/yr for 4 yr to attend college. Her father invested $5000 in a 7% account for her education when she was born. If the student withdraws $4000 at the end of her 17th, 18th, 19th, and 20th years, how much money will be left in the account at the end of her 2ist year? (A) $1700 (B) $2500 (©) s3400 (D) $4000 PROFESSIONAL. PUBLICATIONS, INC. 28 1001 SOLVED ENGINEERING FUNDAMENTALS PROBLEMS P= A(F/A,i,n) = (85000)(F/P, 7%, 21) x (—84000)(F/P, 7%, 4) — (84000)(F/P,7%,3) x (84000) (7/P, 7%, 2) ~ (84000)(F/P, 7%, 1) = ($5000)(4.1406) — (84000)(1.3108) x (—84000)(1.2250) ~ ($4000)(1.1449) x (—84000)(1.0700) = 81700 ‘The answer ECONOMICS-17 ‘The following schedule of funds is available to form a sinking fund. yr $5000 =1yr $4000 2yr $3000 3yr $2000 At the end of the fourth year, equipment costing $25,000 will have to be pur- chased as a replacement for old equipment. Money is valued at 20% by the company. At the time of purchase, how much money will be needed? t (A) $820 (B) $1000 (C) $2000 (D) $8200 First, find the future worth of the available funds. F = P(F/P.,n) = (85000)(F/P, 20%, 4) + (84000)(F/P, 20%,3) + (83000)(F/P, 20%, 2) + (82000)(F/P,20%,1) = (85000)(2.074) + ($4000)(1.728) +- (83000)(1.44) + ($2000)(1.20) = $24,000 PROFESSIONAL PUBLICATIONS, INC. ECONOMICS 29 ‘The additional funds needed are equipment cost — F = $25,000 — $24,000 = $1000 ‘The answer is (B). ECONOMICS-18 {$10,000 is invested at the beginning of a year in a 15% security and held for 5 yr. ‘During that time, the average annual intiation is 6%. Approximately how much, in terms of year zero dollars, will be in the account: at maturity ? (A) $11,700 (B) $13,400 (C) $15,000 (D) 815,400 First, find the future worth, /, without accounting for inflation. P=P(F/P,i,n) (810,000)(F/P, 15%, 8) ($10,000)(2.0114) = $20,114 Next, figure in inflation and express F in terms of real dollars. Freat = ($20,114)(P/F, 6%, 5) = ($20,114)(0.7473) = $15,030 (815,000) The answer is (C) ‘PROFESSIONAL PUBLICATIONS, INC. 210 1001 SOLVED ENGINEERING FUNDAMENTALS PROBLEMS ECONOMICS-19 A firm borrows $2000 for 6 yr at 8%, to be repaid in a lump sum at the end of G yr. At the end of 6 yr, the firm renews the loan for the amount: due plus $2000 more for 2 yr at 8%. What is most nearly the amount of the Joan renewal? (A) $5280 (B) $5750 (©) $5510 (D) $6140 ($2000)(/P, 8%, 6) + (82000)(F/P, 8%, 2) = (82000)(1.587) + ($2000)(1.166) = 35506 (85510) ‘The answer is (C). ECONOMICS-20 A company invests $10,000 today to be repaid in 5 yr in one lump sum at 12% compounded annually. If the rate of inflation is 3% compounded annually, approximately how much profit, in present day dollars, is realized over the 8 yr? (A) $3200 (B) $5200 (©) $5630 (D) $7620 First, find the future worth of the investment without accounting for inflation, F = P(F/P,i,n) = ($10,000)(F/P, 12%, 5) = ($10,000)(1.7623) = 817,623 Next, find the present worth accounting for inflation Freat = (817,623)(P/F,3%,5) = (817,628)(0.8626) = $15,202 profit = $15,202 — $10,000 = 85202 ($5200) ‘The answer is (B). PROFESSIONAL PUBLICATIONS, INC. ECONOMICS aL ECONOMICS-21 ‘What must two investments with the same present worth and unequal lives have? (A) identical salvage values (B) different salvage values (C) identical equivalent uniform annual eash flows (D) different equivalent uniform annual cash flows ‘Two investments with the same present worth and unequal lives must have different equivalent uniform annual cash flows. The answer is (D) ECONOMICS-22 "The following cash-flow dingram represents an investment of $400 and a revenue of z at the end of years one and two. Given a discount rate of 15% compounded annually, what: must © approximately be for this set of cash flows to have a net present worth of zero? $400 (A) $246 (B) $255, (©) 8287 (D) $281 P=$0= A(P/A,i,n) ~ ~$400 + 2(P/A, 15%, 2) 400 + (1.6287) Rearranging to solve for <, 400 = 1.62572 2 = $246 ‘The answer is (A). PROFESSIONAL. PUBLICATIONS, INC. 242 1001 SOLVED ENGINEERING FUNDAMENTALS PROBLEMS ECONOMICS-23 A replacement electrie motor is being considered for purchase. It is eapable of providing 200 hp. ‘The pertinent data are as follows. cost $3200 cloctrical efficiency 0.85 maintenance cost per year $50 life expectancy Myr salvage value after 1d yr 80 ‘The motor is used for 400 h/yr and the cost of electricity is $0.04/kW-b. (1 hp = 0.746 kW.) What is most nearly the effective annual cost using an interest rate of 10%? (A) 82920 (B) $3250 (©) $3290 (D) $3610 ‘The capital recovery factor for 10% and 14 yr is 0.13857. A= P(A/P,i,n) + annual maintenance cost + annual operating cost annual cost = ($3200)(0.1357) + $50 h ) 8 400 4 ee iw. = 83293 ($3290) ‘The answer is (C). ECONOMICS-24 What annuity over a 10 yr period at 8% interest is most nearly equivalent to a present worth of $100? (A) 812.50 (B) $13.80 (©) 14.10 (D) $14.90 A=P(A/P.iyn) = (8100)(A/P,8%, 10) $100)(0.149) = $14.90 ‘The answer is (D). PROFESSIONAL PUBLICATIONS, INC. ECONOMICS 213 ECONOMICS-25 Mr. Richardson borrowed $15,000 two years ago. The repayment terms of the Joan are 10% interest for 10 yr and uniform annual payments. He just made his second payment. How much principal, most neatly, does he still owe? (A) $10,100 (B) $11,700 (©) 812,000 (D) $13,000 ‘The annual payments, A, are A=P(A/P,in) = ($15,000)(4/P, 10%, 10) = ($15,000)(0.1627) = 82441 amount interest yeur_owed (8) ___owed (8) _payment (8) balance (8) T 15,000 + 1500 ‘Badd $14,059 2 14,059 + 1406, - 2441 $13,024 ‘Thus, Mr. Richardson still owes $13,024 (813,000) on the principal The answer is D) | ECONOMICS-26 Given that the discount rate is 15%, what is the equivalent uniform annual cash flow for the following stream of cash flows? year 0 — —$100,000 year 1 —$200,000 year2 850,000 year 3 ~$75,000 (A) -8158,100 (8) -8124,200 (CG) -$106,250 (D) -$90,260 PROFESSIONAL PUBLICATIONS, INC. 214 1001 SOLVED ENGINEERING FUNDAMENTALS PROBLEMS BUAC = (A/P,i,n)P = (4/P.isn) (SL F(P/Risn)) = (A/P, 15%, 3)( — $100,000 - $200,000)(P/F, 15%, 1) ~~ (850,000( P/F, 15%, 2) — (875,000)(P/F, 15%, 3)) = 0.4380(-~ $100,000 ~ ($200,000)(0.8696) ~ (850,000)(0.7561) ~ ($75,000)(0.6575)) = (0.4380)(—$361,000) = ~$158,100 ‘The answer is (A). ECONOMICS-27 A company must relocate one of its factories in 3 yr. Equipment for the loading dock is being considered for purchase. ‘The original cost is $20,000, and the salvage value after 3 yr is $8000. The company’s rate of return on money invested is 10%. The capital recovery rate per year is most nearly (A) 84810/yr —(B) $4950/yr (©) $5120/yr_ _—(D) $5630/yr CR = P(A/P,i,n) — F(A/F,i,n) = ($20,000)(A/P, 10%, 3) ~ (88000)(A/F, 10%, 3) (820,000)(0.4021) — ($8000)(0.3021) = $5625/yr ‘The answer is (D). ECONOMICS-28 In 5 yr, $18,000 will be needed to pay for « building renovation. In order to gen- erate this sum, @ sinking fund consisting of three annual payments is established now. No further payments will be made after the third year. What payments are most nearly necessary if money is worth 15% per year? (A) $2670 (B) $2870 (©) $3920 (D) $6100 PROFESSIONAL PUBLICATIONS, INC. ECONOMICS 215 ‘The present worth of $18,000 at the end of the third year is Ps = F(P/Fisn) = ($18,000)(P/F, 15%, 2) ($18,000)(0.7561) $13,610 ‘The sinking fund must generate $13,610 in 3 yr. The payments that are necessary are A=P(A/P,isn) = (813,610)(4/F, 15%, 3) 813,610) (0.2880) $3920 The answer is (C). ECONOMICS-29 Me. Johinson borrows $100,000 at 10% effective annual interest. He must pay back the loan over 80 yr with uniform monthly payments due on the first day of each month, Approximately what amount does Mr. Johnson pay each month? (A) $840 (B) $850 (© $870 (D) $880 An effective annual interest rate of 10% is equivalent to an effective monthly rate of (l+a?-1=08 = (11)? =1= 0.007974 (0.7974%/mo) ‘The number of months that Mr. Johnson has to pay off his loan is mo) n= (oy (2 22) = 360 mo end of month payment = P(A/P, i,n) {8100,000)(4/ , 0.7974%, 360) (0.007974)(1 +-0.007974)°2 eA ) ( a+ rrrapa 1 = $846 PROFESSIONAL PUBLICATIONS, INC. 216 1001 SOLVED ENGINEERING FUNDAMENTALS PROBLEMS beginning of month payment = F(P/F, i,n) = (8846)(P/F,0.7974%, 1) = ($846)(1 +0.007974)"* = $839 ($840) ‘The answer is (A). ECONOMICS-30 What isthe formula fora straight-line deprecation rate? (ay 00%= 9 et sarge vane ‘estimated service life (py 200 seo value dstimated wervice life vy, 100% net salvage value (©) ~Srimated eorvice life average net salvage value ©) ~ catimated sorvice life 100% — % net salvage value straight-line depreciation rate = ana estimated servi The answer is (A). ECONOMICS-31 ‘What is the book value of equipment purchased 3 yr ago for $15,000 if it is depreciated using the sum of years’ digits (SOYD) method? ‘The expected life is b yr (A) 83000 (B) 84000 (©) 86000 () 89000 In the SOYD method, the digits corresponding to n, the number of ‘years of estimated life, are added. ‘The total sum of years’ digits, t, is n(n+1) 2 (5 yt)(5 yr + 1) 2 = 1b yr PROFESSIONAL. PUBLICATIONS, INC. ECONOMICS 247 ‘The depreciation charge for the first year is n= ()F = (#) (815,000) = $5000 ‘The depreciation charge for year two is n= (*F)P = (=) (815,000) = $4000 For year 3, the depreciation charge is Ds= ¢ > *) P (2) ($15,000) 15 yr, = $3000 The total depreciation is Dioiat = Di + Da + Ds $5000 + $4000 + $3000 = 812,000 ‘The book value is BV = Drotat $15,000 — $12,000 3000 ‘The answer is (A). PROFESSIONAL PUBLICATIONS, INC. 218 1001 SOLVED ENGINEERING FUNDAMENTALS PROBLEMS ECONOMICS-32 ‘The purchase of a motor for $6000 and a generator for $4000 will allow a company to produce its own energy. The configuration can be assembled for $500. The service will operate for 1600 h/yr for 10 yr. ‘The maintenance cost is $300/yr, and the cost to operate is $0.85/h for fuel and related costs. There is $400 in salvage value for the system at the end of 10 yr. Using straight-line depreciation, what is the anmual cost for the operation?’ (A) $2480/yr (B) $2630/yr (C) $2670/yr (D) $2710/yr total initial cost 36000 + $4000 + $500 = $10,500 total initial cost ~ salvage vale 7 ___ $10,500 ~ $400 Sy = $1010/yr $ bh crea cot = (a0 8) (1000) = $1360/yr straight-line depreciation ‘annual cost = straight-line depreciation + maintenance cost + operation cost = $1010 + $300 + $1360 = $2670/yr ‘The answer is (C). ECONOMICS-33 Company A purchases $200,000 of equipment in year 0. It decides to use straight- line depreciation over the expected 20 yr life of the equipment. ‘The interest rate is 14%. If its overall tax rate is 40%, what is the present worth of the after-tax depreciation recovery? (A) $23,500 (B) $24,000 (C) $26,500 (D) $39,700 PROFESSIONAL PUBLICATIONS, INC. ECONOMICS 219 equipment: cost: equipment life expectancy $200,000 20 yr = $10,000/yr write off P= A(P/A,i,n)(tax rate) = ($10,000)(P/A, 14%, 20) tax rate) = (810,000)(6.623)(0.40) = $26,500 straight-line depreciation = ‘The answer is (C ECONOMICS-34 Which of the following is true regarding the minimum attractive rate of return used in judging proposed investments? (A) tis the same for every organi (B) It is larger than the interest rate used to discount: expected cash flow from investments. (©) It is frequently « policy decision made by an organization's management. (D) It is not relevant in engineering economy studies. ‘The answor is (C) ECONOMICS-35 What is most nearly the effective annual interest rate on a loan if the nominal interest rate is 12%/ye compounded quarterly? (A) 11.8% (B) 12.0% (©) 12.3% (D) 12.6% The effective interest rate, i, is i= (04Z)"- o2)* = (+22) - = 0.1255 (12.55%) PROFESSIONAL PUBLICATIONS, INC. 2.20 1001 SOLVED ENGINEERING FUNDAMENTALS PROBLEMS In the preceding equation, ris the nominal interest rate and mis the number of times the nominal interest rate is compounded per year. The answer is (D).. ECONOMICS-36 ‘A person pays interest on a loan semiannually at a nominal annual interest rate of 16%, What is most nearly the effective annual interest rate? (A) 15.5% (B) 15.7% (©) 16.4% (D) 16.6% ‘The effective interest rato, i, is (+Zy"-1 = 0.1664 (16.64%) In the preceding equation, r is the nominal interest rate and m is the number of times the nominal interest rate is compounded per year. ‘The answer is (D). ECONOMICS-37 Which of the following statements is NOT correct? (A) A nominal rate of 12% per annum compounded quarterly is the same as 1296/4 = 39%/quarter. {B) $1 compounded quarterly at 3% for n yr hus a future value of (1.03). (C) Compounding quarterly at: a nominal rate of 12%6/yr is equivalent to com- pounding annually at a rate of 12.55%. (D) Effctive rate of return in options (A), (B), and (C) is the difference between 12.56% and 12%. ‘The statements given in options (A), (B), and (C) are correct. Only option (D) is false. The answer is (D). PROFESSIONAL PUBLICATIONS, INC. ECONOMICS 221 ECONOMICS-38 A bank is advertising 9.5% accounts that yield 9.84% annually. How often is the interest compounded? (A) daily (B) monthly (©) bimonthly (D) quarterly ‘The formula for effective interest rate, i, is i=(42)"-1 ( + a) In the preceding equation, r is the nominal interest rate and m is the ‘number of times interest is compounded per year. Substituting for i, 0.0984 Solve for m algebraically, or by trial and error using the five choices. ‘The solution ism ‘The answer is (D). ECONOMICS-39 A firm is considering renting a trailer at $300/mo. The unit is needed for 6 yr. ‘The leasing company offers a lump sum payment of $24,000 at tho end of 5 yr 1s an alternative payment plan, but is willing to discount this figure. The firm places a value of 10% (effective annual rate) on invested capital. How large should the discount be in order to be acceptable as an equivalent? (A) 8750 (B) $820 (©) $080 (D) $1030 For a 10% effective rate per year, the effective monthly rate, i, is o1=(1+s)—-1 .007974/mo ‘At $300/mo for 5 yr, F = A(F/A,i,n) :300)(F/A, 0.7974%, 60) = ($300)(76.561) 22,970 PROFESSIONAL PUBLICATIONS, INC. 2-92 1001 SOLVED ENGINEERING FUNDAMENTALS PROBLEMS discount = $24,000 — F = $24,000 ~ $22,970 1030 ‘The answer is (D). ECONOMICS-40 Consider a deposit of $1000, to be paid back in 1 yr by $975. What are tho conditions on the rate of interest, i, in %/yr compounded annually, such that the net present worth of the investment is positive? Assume f > 0%. (A) 0% <1 < 50% (B) 0% 0%, and P > 0. ‘The answer is (D). ECONOMICS-41 Consider a deposit of $600, to be paid back in 1 yr by $700. What are the conditions on the rate of interest, i, in 9/yr compounded annually, such that the net present worth of the investment is positive? Assume i > 0%. (A) 12.5% 0> F(P/F,i,n) > (~8600 + $700)(P/F, 4,1) > (8600 + $700) (4) 1 oiti i< 0.167 (16.7%) Thus, for P > $0, 0% —ee 2h (A) 2103 (B) 3203 (©) 4805 (D) 5403 ‘The work done by the force is W = Freosd (ass m = (200 Ibf)(2 ft) cos 28° (14s i) (0.008 = 470.04 3 (480 J) ‘The answer is (C). PROFESSIONAL PUBLICATIONS, INC. SYSTEMS OF UNITS aah ‘UNITS-19 ‘Two particles moving in the same direction collide, stick together, and continue their motion together. Each particle has a mass of 10 g, and their respective velocities before the collision were 10 m/s and 100 m/s. The energy of tho system after the collision is most nearly (A) 223 (B) 30J (C) 485 (D) 7d By = drmoav7 ‘ictal = +2 vy is the velocity of the two masses after the collision. Use the prin- ciple of conservation of momentum. meet ene (20 «) (10 4-100 %) = (20 ery vy = 55 m/s ) (0.02 ke) (85 @)” = 80.25 kg-m?/s? (30 J) Ey ‘The answer is (B). ‘UNITS-20 ‘Two protons, each of charge 1.6 x 10-9 C, are 3.4 ym apart. What is most nearly the change in the potential energy of the protons if they are brought 63 nm closer together? (A) 64x 10°? J (B) 7.2x10- J (C) 1.3x10-* J (D) 3.6x10-8 J ‘The potential energy of a system of two charges is given by the fol lowing. vaKhe T k 3.99 x 10° Nem?/C? PROFESSIONAL PUBLICATIONS. INC. a2 1001 SOLVED ENGINEERING FUNDAMENTALS PROBLEMS 4 and qz are the charges, and r is the distance between the charges. : oreo oy 1 * (Gawra- arora) = 936% 10-" J ‘The answer is (D). UNITS-21 According to the Bohr model of the atom, the energy of the atom when the electron is at the first Bohr radius is given by mkt ont mm mass of the electron 9.1095 x 10-®) kg k Coulomb's constant 8.99 x 10 N-m?/C? e electron charge 1.602 x 10-99 © fh ee oe h Planck's constant, 6.626 x 10-4 Jog Using these values in metric units, calculate the value of Ep, (A) 2.18x10-° WB) 2.18x10-8 J (©) 2.18x10-8 J (D) 2:18%10-" W PROFESSIONAL PUBLICATIONS, INC SYSTEMS OF UNITS 313 (oitor230- te) (aoner0t SY (410 10-¥ 6) Bye 0 eo @) (= x10" ) =218x 10-8 5 ‘The answer is (C). UNITS-22 ‘A copper bar is 90 cin loug at. 86°F. What is most nearly the increase in its length when the bar is heated to 95°F? The linoar expansion coefficient for copper is LT x 1078 1/°C. (A) 2.1x10°% m (B) 32x10- m (©) 53x10-5 m (B) 7.7 x 10-7 m ‘The change in length of the bar is given by the following. AL=aloAT Convert the temperatures from °F to °C. “c= (Ser 3 OF. 39°) = (2)oe-2)- be () (96°F — 32°) = 30°C 32°) T AT=T-T = 35°C — 30°C =5°C Ab (ar x10 3 %) (0.9 m)(5°C) =1.7x 107% m ‘The answer is (D). PROFESSIONAL PUBLICATIONS, ud 1001 SOLVED ENGINEERING FUNDAMENTALS PROBLEMS ‘UNITS-23 A slab of iron with initial temperature Ti, = 48°C is used to heat a flat glass plate that has am initial temperature of Tj, = 18°C, No heat is lost to the environment. The masses are m, = 0.49 kg for the slab and my = 310 g for the plate. What is the amount of heat transferred when the two have reached the equilibrium temperatures? The specific heats are c; = 0.11 keal/kg:°C for iron, and ¢y = 0.20 keal/kg.°C for glass. (A) 860 cal (B) 82 keal (©) 53 keal (D) 320 keal ‘The heat transferred by an object is given by the following equation. Q=meAT Qis the heat transferred, m is the mass of the object, AT is the change in temperature of the object, and c is the specific heat capacity of the object. Since no heat is lost, Qi = Qy- meAT, = mycyAT, ‘The final temperature of ench object is the same. ~mici (Te — Ta) 6s (Ta ~ Ty) Solve the equation for T>, then substitute to find Q. miciTis + mgesTig mics + Mey (0.49 kg) (uo & ) (48°C) + (0.310 kg) (220 mss) (asc) (0.49 kg) (uo mo Ta) = (0.490 kg) (oat x 108 ) (92° — 48°C) kg °C, = 860 cal ‘Thus, 860 calories were transferred from the iron slab to the glass plate. ‘The answer is (A) PROFESSIONAL PUBLICATIONS, INC. SYSTEMS OF UNITS 315 ‘UNITS-24 “What is the average thermal conductivity for the composite material shown’? Jom bg 200 Brule y= 100 Brunet 50 Btufhe-fF (A) 2W/mK — (B) 75.5 W/mK (C) 115 W/mK — (D) 155 W/mK wg) [one] mK ° beoF, = 0.0050 m?-K/W Lot = 0.21 Z 0.21 m im Kk 0.0050 =~ = 42 W/mK ‘The answer is (A). ‘PROFESSIONAL PUBLICATIONS, INC. 316 1001 SOLVED ENGINEERING FUNDAMENTALS PROBLEMS UNITS-25 Approximate the energy transfer rate across a 6 in wall of firebrick with a tem- perature differonce across the wall of 50°C. The average thermal conductivity of firebrick is 0.65 Btu/hr-ft-°F. (A) 110 W/m? (B) 290 W/m? (C) 870 W/m? (D) 430 W/m? AT= (3) (50°C) = 90°F [for linear profile] 17 Beu/hr-fe? = (7 2) (02001 eR? Btu 369 W/m? (370 W/m?) The answer is (C) UNITS-26 ‘A house has brick walls 15 mm thick. On a cold winter day, the temperatures of the inner and outer surfaces of the walls are found to be 20°C and —12°C, respectively. There is 120 m? of exterior wall space. The thermal conductivity of brick is 0.711 J/ms°C. Most nearly how much heat is lost through the walls per hour? (A) 1803 (B) 13k (©) 660 kd (D) 660 MJ Caleulate the temperature difference, AT. AT = Tinnor ~ Tower 20°C — (-12°C) = 37°C PROFESSIONAL PUBLICATIONS, INC. SYSTEMS OF UNITS 317 ‘The thickness of the wall, «, is 0.015 m. kAAT. (0.1 4g) (20 92 (9000) (2°) 0015 mr = 655 x 10° J/h (660 MJ/h) ‘Thus, the heat transferred per hour is most nearly 660 MJ. [ ‘The answer is (D). ‘UNITS-27 Air has a specific heat, cp, of 1 kI/kg-K. If 2 Btu of enérgy are added to 100 g of air, what is most nearly the change in air temperature? (A) 10°C (B) 22°C (C) 4° (D) 88°C Q= meyAT are ney ky Q= (2 Btu) (008 =) 1 kd m= (100 g) (tats) = 0.10 kg oy = 1 kak K ST =— iJ (0.10 kg) (0 =21.1K (21°C) Note: The temperature differences of 1K and 1°C are equivalent, ‘The answer is (B). ‘PROFESSIONAL PUBLICATIONS, INC. 348 1001 SOLVED ENGINEERING FUNDAMENTALS PROBLEMS UNITS-28 Air has a specific heat, ¢p, of 1 kI/kg-K. 1100 g of air aro heated with a 1500 W heater, which of the following occurs? L. The air heats up at a rate of 15K /s. IL, ‘The air reaches a final temperature of 1500K. IIL. The air undergoes a nonisentropic process. (A) Tonly (B) Land 1 (©) Mand 1 (D) Tand IIT Qu meAT Q=meAt at=2 my 100 2 5 (or (108) =15 K/s ‘Thus, 1 is correct. It is not possible to predict the final temperature of the air without knowing the length of time it is being heated. Theroforo, II is false. ‘The addition of heat is a nonisentropic process (the entropy of the air changes). Therefore, III is correct ‘The answer is (D). UNITS-29 ‘The change in enthalpy of an incompressible liquid with constant specific heat is given by Ia—hy = (Tp ~ Ti) + (2 - mr) Zp is the temperature at a stato n, Py is the pressure at state n, and v is the specific volume of the liquid. Water, with cy = 4.18 kJ/kg-K and v= 1.00 x 10? m?/kg, has the following final stat mates state 1: T= 19°C py = 0.1013 MPa state 2: Ty =30°C pz = 0.113 MPa PROFESSIONAL PUBLICATIONS, INC. SYSTEMS OF UNITS 3:19 What is the change in enthalpy from state 1 to state 2? (A) 46 kJ/kg, (B) 46 KN/kg (©) 46 KPa/kg = (D) 56 kJ/kg 101300 Pa 111300 Pa P Pa ‘Temperature differences need not be in K, since a temperature dif- ference of 1°C equals a temperature difference of 1K. The specific volume, v, and specific heat, cy, are already in consistent units. tym = (10 gx) (wre- 190) xm 7 svem 9 (1am $2) + » (0.00 F = 46000 J/kg (46.0 kJ /ke) The answer (A) UNITS-30 Tn a constant-temperature, closed-system process, 100 Btu of heat are transferred to the working fluid at 100°. What is the change in entropy of the working fluid? (A) 018 1G/K —(B) 0.25 WK —(C) O34 KI/K —(D) 0.57 kI/IK For a closed system at a constant temperature, -@ B-=F 100 Btu ‘J = (sarreisr) (4 gen) (18 = 0.34 ks/K ‘The answer is (C) PROFESSIONAL. PUBLICATIONS, INC. 3-20 1001 SOLVED ENGINEERING FUNDAMENTALS PROBLEMS UNITS-31 In the illustration, pressure is plotted using a scale of 10000 Pa per unit, volume is plotted using a scale of 1 L per unit, and the area enclosed by the cycle is 3 units®, What is the work done during the cycle? A pvt A” YW Ve (A) 3x10-? Ws (B) 3x10" ergs (C) 307 (D) 30000 Nem ‘The area enclosed by the cycle, W is W=Feo =v N mi =() (10000 %) ay (oon) 30Nm (80) ‘The answer is (C). UNITS-32 Ifa %/ hp pump runs for 20 min, what is the energy used? (A) 0.060 ergs (B) 0.25 kW = (C) 0.30 MJ (D) 0.11 eWe »») (02407 ®) (20 min) (was) (3000 = = 0.298 MJ (0.30 MJ) W =P ‘The answer is (C). PROFESSIONAL. PUBLICATIONS, INC. SYSTEMS OF UNITS 321 UNITS-33 ‘A machine repeatedly accelerates a 1 kg mass at 1 m/s? for 1 min, ‘The machine rruns at 60 rpm. What is the power output of the machine? (A) Lerg (B) 1 cal (jis @)1w _ Ge) (15) 0m ate ‘UNITS-34 A power of 6 kW is supplied to the motor of a crane. ‘The motor has an efficiency of 90%. With most nearly what constant speed doce the crane lift an 800 Ibf woight? (A) 0.091 m/s (B) 0.32 m/s (C) 0.98 m/s (D) 1.5 m/s B22 1001 SOLVED ENGINEERING FUNDAMENTALS PROBLEMS Faq is the input power, and 1 is the efficiency. The useful power, P,, is Pe=nPn w = ane) (so) = 5400 W veh F mee ma _ (3400 W) (1 bt = a0 tor) \aa5N = 1.52W/N (15 m/s) ‘The answer is (D). UNITS-35 Given a heat exchanger with specified inlet and outlet enthalpies, what is most nearly the energy requirement for the heating coil? hg = 200 Btuflbmn rine stmt: ~ Lt a (A) 500 kW (B) 530 kW (©) 560 kw (D) 600 kW Q=th(h2 - hn) (22) (20 28-1002) (x0 = 527.6 kJ/s (630 kW) ‘The answer is (B PROFESSIONAL PUBLICATIONS, INC. SYSTEMS OF UNITS 323 UNITS-36 An engine has an efficiency of 26%. The engine uses 2 gal/hr of gasoline. Gasoline has a heating value of 20500 Btu/lbm and a specific gravity of 0.8. What is most nearly the power output of the engine? (A) 033 KW (B) 21 kW (C) 26 kW (D) 42 kW First, find Pq, the input power of the engine. Pa cy = V pep = VIS) PwnterCp _ (gal (18° Ibm’ Ibm -(°&) (ae va) (08) (aR) BH (seq 181) (18 « (oe00 BE (sce 2) (52%) = 80 kW Next, find Pau, the power output of the engine Powe = Pa (0.26)(80 kW) = 208 kW (21 KW) The answer is (B) UNITS-37 2 L of an ideal gas, at a temperature of T, = 25°C and a pressure of py = 0.101 MPa, are in a 10 cm diameter cylinder with a piston at one end. The piston is depressed, so that the cylinder is shortened by 10 em. ‘The temperature increases by 2°C. What is most nearly the change in pressure? 7 inital state Ps tom Te final stato Pe (A) 0.16MPa—(B) 0.17MPa = (C) 0.25 MPa (D) 0.33 MPa PROFESSIONAL PUBLICATIONS, INC. 324 1001 SOLVED ENGINEERING FUNDAMENTALS PROBLEMS Apply the ideal gas law to the gas in the cylinder. pi Ty m= BM on Ty = 270 +.273° = 300K AV =ALA= AL (ar) = (10 cm)r(6 crn)? 785 em? Note that AV <0 because the piston is depressed. Va=V+av = 2000 em® — 785 em* = 1215 em* (0.101 MPa)(2000 cm®) (300K) “Q58K)(T25 em?) = 0.167 MPa (0.17 MPa) Pa (8). UNITS-38 ‘The average power output of a cylinder in a combustion engine is given by P=pLAN ‘pis tho average prossure on the piston during the piston stroke, L is the length of tho piston stroke, A is the aren of the piston head, and N is the number of strokes per second. An eight-cylinder engine has the following specifications at optimum speed. p 283 kPa Lo item 4 diameter of piston head, 12 em N 1500 strokes/min PROFESSIONAL PUBLICATIONS, INC SYSTEMS OF UNITS B25 “What is the average power output of this engine? (A) 89.5 N/s (B) 89.5 kW (©) 89.5% 10 J-m/s (D) 89.5 kd LAN for one cylinder) Pasta = 8PLAN [for eight eylinders] a, (4) Asa =(3) 12. em\" CF) = 0.0113 m? Prorat = (8)(283000 Pa)(0.14 m)(0.0113 m*) (1500 a =) ¢: a) = 89500 J/s (89.5 KW) ‘The answer is (B). ‘UNITS-39 What is the power required to transfer 97000 C of charge through potential rise of 50 V in 1 hr? (A) 055KW (BONS KW (C) 1.3 KW (D) 28 kW Waw is the charge, and V is the potential rise w 7 wv oe (97.000 €)(50 V) ( a) 3600 s 1000 ay =134 kW (1.3 kW) ‘PROFESSIONAL PUBLICATIONS, INC. 3.26 1001 SOLVED ENGINEERING FUNDAMENTALS PROBLEMS | ‘UNITS~40 A current of 7 A passes through a 12 9 resistor. What is the power dissipated in the resistor? (a) 8 Ww (B) 0.59 hp (©) 0.79 bp (D) 7.5 hp P=PR Tis the current, and R is the resistance. P=(TA)*(12 m( =0.79 hp The answer is (C). UNITS-41 If the average energy in a nuclear reaction is 200 MeV /fission, what is the power output of a reactor experiencing 2.34 x 10" fissions/3? (A) 550 W (B) 120 kW (©) 35 MW (0) 750 Mw The power output of the reactor is the energy per fission times the number of fissions per second. P= (200 ue) (204 x10" Sen) (A) (1.010 «107 3) fission, MeV = 750x108 W (750 MW) The answer is (D). PROFESSIONAL PUBLICATIONS, INC. FLUID STATICS 4 AND DYNAMICS FLUIDS-1 ‘Which statement is true for a fluid? (A) It cannot sustain a shear force. (B) It cannot sustain a shear force at rest. (C) It is a liquid only. (D) It has a very regular molecular structure. [A fluid is defined as a substance that deforms continuously under the ‘application of a shear force. This means that it cannot sustain a shear force at rest. Therefore, option (B) is true. answer is (B). FLUIDS-2 Which of the following is NOT a basic component of motion of a fiuid element? (A) translation (8) rotation (©) angular distortion (D) twist ‘The motion of a fluid clement may be divided into three categories: translation, rotation, and distortion. Distortion can be further sub- divided into angular and volume distortion. The only choice that is rot: a basic component of fluid element motion is twist. ‘The answer is (D). PROFESSIONAL PUBLICATIONS, ING. 42 1001 SOLVED ENGINEERING FUNDAMENTALS PROBLEMS FLUIDS-3 Which of the following must be satisfied by the flow of any fluid, real or ideal? I. Newton’s second law of motion IL. the continuity equ IIL the requirement of a w IV. Newton’s law of viscosity ‘V. the principle of conservation of energy (A) LU, and (B) 1 HandIV (C) 1, I,andV (D) 1,11, U,andIV 1 velocity distribution ‘Newton's second law, the continuity equation, and the principle of ‘conservation of energy always apply for any fluid. ‘The answer is (C). FLUIDS-4 ‘What is the definition of pressure? area in 1280 force en ae A) oreo B) tg crew (O) lity row (P) Hil force ‘The mathematical definition of pressure is force ‘The answer is (C). FLUIDS-5 For a fluid, viscosity is defined as the constant of proportionality between shear stress and what other variable? (A) time derivative of pressure (B) time derivative of density (C) spatial derivative of velocity (D) spatial derivative of density By definition, PROFESSIONAL PUBLICATIONS, ING. FLUID STATICS AND DYNAMICS 43 Thus, viscosity, 1, is the constant of proportionality between the shear stress, 7, and the gradient (spatial derivative) of the velocity. ‘The answer is (C). FLUIDS-6 Surface tension has which of the following properties? I. It has units of force por unit length. IL, It exists whenever there is a density discontinuity. IIL. It is strongly affected by pressure. (A) Tonly (B) Monty (©) only (D) Land IIT is incorrect because pressure only slightly affects surface tension. Tand I are correct. ‘The answer is (D). FLUIDS-7 ‘A leak from a faticet: comes out in separate drops. Which of the following is the ‘main cause of this phenomenon? (A) gravity (B) air resistance (C) viscosity of the fluid (D) surface tension Surface tension is caused by the molecular cohesive forces in a fluid. It is the main cause of the formation of the drops of water. FLUIDS-8 ‘The surface tension of water in air is approximately 0.0756 N/m. If the atmo- spheric pressure is 101 kPa (abs), what is the pressure inside a droplet 0.254 mm_ in diameter? (A) 90.83 kPa (abs) (B) 101.0 kPa (abs) (©) 101.5 kPa (abs) (D) 102.2 kPa (abs) PROFESSIONAL PUBLICATIONS, INC. 44 1001 SOLVED ENGINEERING FUNDAMENTALS PROBLEMS For a spherical droplet, AP = Pin Pout 2 Pra = Poue + (a) (758 x 10-5 x Ax 10-7 m = 102.2 kPa (abs) = 101 kPa+ ‘The answer is (D). FLUIDS-9 Which of the following describes shear stress in a moving Newtonian fluid? (A) It is proportional to the absolute viscosity. (B) It is proportional to the velocity gradient at the point of interest (C) It is nonexistent. (D) both A and B aw ay Shear stress is proportional to the velocity gradient at a point, as well as the absolute viscosity. ‘The answer is (D). FLUIDS-10 If the shear stress in a fluid varies linearly with the velocity gradient, which of ‘the following describes the fluid? (A) It is inviscid, (B) It is a perfect gas. (C) It is a Newtonian fluid. (D) It is at a constant temperature, In order for shear stress to vary linearly with the velocity gradient, the fluid must be Newtonian, ‘The answer is (C). PROFESSIONAL PUBLICATIONS, INC. FLUID STATICS AND DYNAMICS 45 FLUIDS-11 How are lines of constant pressure in a fluid related to the force ficld? (A) They are parallel to the force field. (B) They are perpendicular to the force field. (©) They are at a 45° angle to the force field. (D) They are perpendicular only to the force of gravity. Lines of constant pressure are always perpendicular to the direction of the force field. The answer is (B). FLUIDS-12 Which of the following statements about a streamline is most accurate? (A) Tes a path of a fluid particle. (B) It sa line normal to the velocity vector everywhere. (C) Ibis fixed in space in steady flow. (D) It is defined for nonuniform flow only. Streamlines aro tangent to the velocity vectors at every point in the field. Thus, for a steady flow dv/dt = 0, a streamline is fixed in space. (©). ‘The answer FLUIDS-13 Which of the following describes a streamline? I. It is a mathematical concept. II, Tt cannot be crossed by the flow. IIL. Tt is a line of constant entropy. (A) Tonly (B) Monly (©) Tand It (D) Tand 1 A streamline is a mathematical concept that defines lines that are tan- ential to the velocity vector. Therefore, no flow can cross a stream- line. Entropy is not related to streamlines, ‘The answer is (C) ‘PROFESSIONAL PUBLICATIONS, INC. 46 1001 SOLVED ENGINEERING FUNDAMENTALS PROBLEMS FLUIDS-14 The following illustration shows several streamlines near the comer of two infinite plates. Which of the following could be the correct expression for the stream function, W, of this potential flow? y (A) ¥=a-y (B) ¥=22y (v= (D) W=y Streamlines are graphs of constant values for the stream function. ‘The graph shows hyperbolas that are of the form ary = b, where a and b are constants. ‘Thus, of the choices shown, the stream function, could only be ¥ = 2zy. ‘The answer is (B) FLUIDS-15, What is most nearly the gage pressure at point A in the tank of water if h = 2m? naam A (A) 12 kPa (B) 13 kPa (C) 16 kPa (D) 20 kPa P= poh = (1000 48) (oa $) 2m) = 19620 Pa (20 kPa) ‘The answer is (D). PROFESSIONAL PUBLICATIONS, ING. FLUID STATICS AND DYNAMICS 47 FLUIDS-16 A drinking glass filled with a fluid of density p is quickly inverted. The top of ‘the glass, which becomes the bottom after the glass is inverted, is open. What ig the pressure at the closed end at point A? x | — D (A) Pate ©) pam + egh —(C) Pam — gh — (D) poh ‘The pressure at point A, p, plus the pressure exerted by the fluid coquals the pressure outside the glass. P+ pgh= Porm ‘Therefore, ‘The answer is (C) PROFESSIONAL PUBLICATIONS, INC. 48 1001 SOLVED ENGINEERING FUNDAMENTALS PROBLEMS FLUIDS-17 Find the pressure in the tank from the manometer readings shown. A} Pym = 100 KPa pressurized tank 10 m| tbr om oN tI fluid A. pq = 500 kg/m? 03am =}. tuidB, pg = 750 kgim? fluid C, pg = 1000 kg? (A) 102kPa —(B) 108KPa (©) 112 KPa (D) 118 kPa Pa— Pa = pog(zi — 22) Ps — Po = pog(z2 23) ‘Pa — Ps = pag(2s — 24) P.—Pi = (D4 ~ Ba) + (Da Pa) + (D2 — Pn) Pa =P: + 9(cl2r — 22) + p22 — 23) + pales — za) = 100000 Pa-+ (01 3) ((1000 Mf) 1 mp + (70 3) (-0.3 m) + (00 3) (oa ™)) = 108100 Pa (108 kPa) ‘Tho answer is (B). PROFESSIONAL. PUBLICATIONS, INC. FLUID STATICS AND DYNAMICS 49 FLUIDS-18 In which fuid will e pressure of 700 kPa occur? = 90KPa alcoho : p-7makgind : ee ol 10m p= 8996 kgm? water z pe 97a0kgim | gs aa i ‘ p= ra8dign® We not to scale (A) ethyl alcohol (B) oil (C) water (D) glycerin Let p; be the maximum pressure that can be measured in fluid level i. If p; > 700 kPa, then a pressure of 700 kPa can be measured at that level. Po = 90 kPa P= pot pge (773 $8) (20 $) (60m) 90 kPa + = 545.16 kPa pi < 700 kPa Po = Pu + pagea 809.6 8) (o.81 3) om) = 545.56 kPa + + 1000 = TkPa = 683.81 kPa PROFESSIONAL. PUBLICATIONS, INC. 4.10 1001 SOLVED ENGINEERING FUNDAMENTALS PROBLEMS Pa < 700 kPa (Ps = p2+ psgzs (ov. 0 8) (81 3) 6m = cast uma SD BPE 1000 1 kPa = 081.88 kPa pa < 700 kPa Pa=Ps+ pug (126 38) (0. = 681.89 kPa+ a 1000 Tie = 142.46 kPa i> 700 kPa ‘Thus, a pressure of 700 kPa occurs in the glycerin level. ‘The answer is (D). FLUIDS-19 ‘The pressure drop across a turbine is 200 kPa. ‘The flow rate is 0.25 m®/min, What is most nearly the power output of the turbine? (A) O41 KW (B) 0.83KW —(C) 0.95KW = (D) 1.3 KW P = (pressure drop)(flow rate) = (200 kPa) (¢ a ) (a a) = 0.833 KW (0.83 kW) ‘The answer is (B). PROFESSIONAL PUBLICATIONS, INC. FLUID STATICS AND DYNAMICS 411 FLUIDS-20 A circular window with a radius of 0.25 m has its center 3 m below the water's surface. ‘The window is vertical. What is most nearly the force acting on the window? (A) 29 (B) 5.8 kN (©) 18 kN (D) 2918 F=pA p= (4) (o9sina) PROFESSIONAL. PUBLICATIONS, INC. 412 1001 SOLVED ENGINEERING FUNDAMENTALS PROBLEMS With y: = 2.75 m, yp = 3.25 m, the angle a between the surface of the water and the surface of the window = x/2, and A= xr?, P= (22) onena = 8 mt 208 ) (100 4) (2 a 2) (n(0.25 my =5T80.N (6.78 kN) FLUIDS-21 ‘What is most nearly the overturning moment per unit width due to water acting on the dam shown? (A) 15kNm —(B) 30KN-m = (C) 44ENm = (D) 72 KNm ‘The hydrostatic force per unit width of dam is = fogha A= (3 m)(1 m) = 3 m? per meter of width. Fe (3) (2000 #) (o81 3) @ mom) = 44145.N (44.145 kN-m/m) ‘This force acts one-third up from the base. PROFESSIONAL PUBLICATIONS, INC. FLUID STATICS AND DYNAMICS 413 ‘The overturning moment is Méam (css 2) (22 =44.1 kNm (44 KN) ‘Tho answer is (C). FLUIDS-22 What is most ncarly tho minimum required foree per unit width, Fasm, to prevent ‘the dam shown from sliding? (A) 15. kN (B) 30 kN (C) 44 kN (D) 72 kN F = hoghA A= (8:m)(1 m) = 3 m? per meter of width Pe (3) (2000 8) (281 3) @my(6.m) 44145 N (44 KN) ‘The answer is (C). ‘PROFESSIONAL PUBLICATIONS, INC. 414 1001 SOLVED ENGINEERING FUNDAMENTALS PROBLEMS FLUIDS-23 ‘Water is held in a tank by the sluice gate shown. What force per unit width of the dam must the latch supply to keep the gate closed? hinge latch force (A) 25KN/m (BY) 34 KN/m (©) 52KN/m_—(D) 74 KN/m Draw a free-body diagram of the gate. ite system in the diagram. For the gate to stay in place, the sum of the moments around the hinge must be zero. YT Mince 4m =(4 mR, - f awdy PROFESSIONAL PUBLICATIONS, INC. FLUID STATICS AND DYNAMICS 415 oO ~ cern = f°" rudy = f°” randy om hom ‘mn = [wa an (os 5) ¥ = 209.3 kN Rearranging to solve for Ri, 2.3 N/m (62 KN/m) ‘The answer FLUIDS-24 A tank with one hinged wall is filled with water. The tank wall is held at a 30° angle by a horizontal cable. What is most nearly the tension in the cable per mater of the tank? Fee (A) 19 kN (B) 23 KN (©) 25 kN (D) 40 kN PROFESSIONAL PUBLICATIONS, INC 416 1001 SOLVED ENGINEERING FUNDAMENTALS PROBLEMS: ‘The average pressuro is, ‘The length of the wetted inclined wall is, h 4m 0830" ~ cosa” = 462m ‘The wall area per foot of wall is A= lw = (4.62 m)(1 m) = 4.62 n ‘The resultant foree is R= PA = (196 Pa) (4.62 m*) = 90.6 kN ‘This resultant acts perpendicular to the wall at .67 [vertical distance measured from surface] ‘Taking moments about the hinge et the bottom, 90.6 KN)(4.m ~ 2.67 m) _ (Tear) (cow 30°)(6 mn) uM cos cos = 130.1 Wm (6 m)earne Rearranging to solve for Tessie, = 28.2 KN (23 kN) ‘The answer is (B), PROFESSIONAL PUBLICATIONS, INC. FLUID STATICS AND DYNAMICS 417 FLUIDS-25 ‘A tank of water has a rectangular panel at its lower left side, as shown. ‘The location of the center of pressure on the panel is at the point P. Deseribe the distance along the panel from the bottom of the tank to the center of pressure as PA, Determino the length of PA. ‘The distance along the surface of an object from the surface of the fluid to the center of pressure, h,, is given by hiyhy (m tha a.) PROFESSIONAL PUBLICATIONS, INC. 418 1001 SOLVED ENGINEERING FUNDAMENTALS PROBLEMS In the preceding equation, h is the distance along the surface of the object from the surface of the fluid to the object's upper edge, and ‘hz is the distance along the surface of the object from the surface of the fluid to the object's lower edge. ‘From the illustration, PA=hy—-h, The plane is inclined at 30° below horizontal, has its upper edge at 3m (vertically) below the surface of the fluid, and is 12 m long. Thus, ‘the following can be determined. lg=h+2m=6m+2m =18m 2 (6 mQ8 m m=(5) (om+180- Foes) =18m PA = hy = hy = 18 m= 18 m =5m (B). The answer FLUIDS-26 ‘What is most nearly the total force exerted on the curved surface described by the equation y = <*? ‘The width of the curved plate is 2 m, and the specific weight of water is 9.81 KN/m°, Pa = 0kPa gage (A) 1020 kN (B) 1070 KN (C) 1260kN ——(D) 1380 KN PROFESSIONAL PUBLICATIONS, IN. FLUID STATICS AND DYNAMICS 419 y n=10m 1w is the width of the tank. na Vt ‘The weight of the water in the portion of the tank above the curved surface, Fy, is Fy=W Vi = 10 m= y)dz wf com-y) 7 m aw [00 m=2\¢2 vive om = (oa ) @m) (w V5 m - 20.0v0 =) = 413.6 KN Fe=PA, ay (Oth Bar(> ‘The area of the tank perpendicular to the z-axis, As, is Az = wh R= (os 5) (=) (2 m)(10 m)) = 981.0 EN F; ((413.6 KN)? + (981.0 KN)? = 1065 KN (1070 kN) ‘The answer is (B), PROFESSIONAL PUBLICATIONS. INC. 4-20 1001 SOLVED ENGINEERING FUNDAMENTALS PROBLEMS FLUIDS-27 The stream potential, ®, of a flow is given by ® = 2zy—y. Determine the stream function, , for this potential. (A) Geat-40 @B) Var—-2 4740 (C) Verse? -y4o (D) Vast? + 4 By definition, Substituting w = 2y, aw 7 Rearranging, = 7 2ydy + f(2) P+ F(z) Substituting v = 2x —1, PROFESSIONAL PUBLICATIONS, INC. FLUID STATICS AND DYNAMICS A2L FLUIDS-28 Determine the average velocity through a circular section in which the velocity distribution is given ns V = Vax(1 ~(r/re)*). ‘The distribution is symmetric with respect to the longitudinal axis, r = 0. To is the outer radius and Vmax is the velocity along the longitudinal axis. (A) Venax/4 {B) Venex/3 (©) Vimax/2 (D) vmnax FLUIDS-29 Under what conditions is mass conserved in fluid flow? (A) The fluid is baratropic. (B) The flow is isentropic. (C) The flow is adiabatic. (D) Its always conserved. ‘Mass is always conserved in fluid flow. ‘The answer is (D). PROFESSIONAL PUBLICATIONS, INC. 422 1001 SOLVED ENGINEERING FUNDAMENTALS PROBLEMS FLUIDS-30 What is the absolute velocity of a real fluid at a surface? (A) the same as the bulk fluid velocity (B) the velocity of the surface (C) zero (D) proportional to the smoothness of the surface For a real (nonzero viscosity) fluid there is no slip at the boundaries. In other words, the velocity of the surface is the same as the velocity of the fluid at the surface. ‘Thus, option (B) is truc. Option (C) is true only if the velocity of the surface is zero. FLUIDS-31 Which of the statements is true concerning the following continuity equation? A , Hou) , Woe) , O(a) ot Or ay Oz pis density, u is velocity in the x direction, v is velocity in the y direction, and w is velocity in the 2 direction. (A) It is valid only for incompressible flow. (B) It is valid only for steady flow. (C) It is derived from the principle of conservation of mass. (D) It is derived from the principle of conservation of energy. In essence, the continuity equation states that the mass flux entering a control volume is equal to the mass flux leaving the control volume plus the rate of accumulation of mass within the control volume. ‘Thus, it is derived from the principle of conservation of mass. It is, valid for all real and ideal fluids, and for all types of fluid flow. ‘The answer is (C). PROFESSIONAL PUBLICATIONS, INC. FLUID STATICS AND DYNAMICS 4-23 FLUIDS-82 ‘Which of the following sets of dimensional flow equations satisfies the continuity equation? (u, v, and w are the components of velocity in the x, y, and z directions, respectively.) Luscty-t vette w=t-I+e I usy?-a? v= 2ey (A) Land IT (B) lend = (C) Mand t(D) 1, Il, and TIT ‘The continuity equation states that V-V = 0. Check to see if this is true for each of the given flows. du | Ov, dw 1 eg Et (a) $1 =0 du, ov , Ow _ UW Fe + Set Ge = tet tet t= TAO du Ov, dw UL Get By t Be eet (test) —t=0 ‘Thus, flows I and ITT both satisfy the continuity equation. FLUIDS-33 A pipe has a diameter of 100 mm at section AA and a diameter of 50 mm at section BB. The velocity of an incompressible fluid is 0.8 m/s at section AA. What is the flow velocity at section BB? (A) 0.95 m/s (B) 1.2 m/s (©) 21 m/s (D) 3.5 m/s Uso the continuity equation. mass through AA. pA = pAava ‘PROFESSIONAL PUBLICATIONS, INC. 424 1001 SOLVED ENGINEERING FUNDAMENTALS PROBLEMS ‘Rearranging to solve for v2, ALY Aa _ Ai =(4)" =12 m/s The answer is (B).. FLUIDS-34 Consider the following two flows of water. nom 20m 2 1 2 5 = 20mm ‘Py = 100 kPa 1 30" we same y= 20mm p= 100KPs ‘What is the relation between va(I) and vo(II)? (A) ve) = vat) (B) vag = "20D (C) va(1) = 2va(11) () va(l) =4 va(I) PROFESSIONAL PUBLICATIONS, INC. FLUID STATICS AND DYNAMICS 4.25 From the continuity equation, Ay = Aaya Rearranging to solve for va, AL vo= (1) vi Gndopendent of tilt angle} A va(D) = va(tl) ‘The answer is (A). FLUIDS-35 A constant-volume mixing tank mixes two inlet streams containing salt. ‘The alt concentration in stream 1 is 5% by weight, and in stream 2 it is 15% by weight, Stream 1 flows at 25 kg/s, and stream 2 flows at 10 ke/s. There is only cone exit stream. Find the salt concentration in the exit streatn, (A) 5.5% (B) 7.9% (C) 1% (D) 13% LY rire = DD tints some ooriet (00 (252) +019 (108) =2 (638) Rearranging to solve for , =03m eer (A) 52 kg/s (B) 61 kg/s (©) 65 ke/s (D) 79 kg/s From the continuity equation, Aava = Anve var (0.15 m)? = vpa(0.05 m)® = ( (0.05 m)? vas (CSE) v0 = 0.111 vp Use Bernoulli’s equation along the streamline in the center of the pipe. v 24+ PROFESSIONAL PUBLICATIONS, INC. FLUID STATICS AND DYNAMICS 439 Rearranging, v= v= 100 m/s? m vh—(Oniyy)? = 100 2 2) = 190 v&(0~ (0.11)%) = 100 vp = 10.06 m/s wade 7 (2000 8) (10.06 #) (0.05 mj? = 70 ke/s ‘The answer is (D) FLUIDS-52 ‘What is the volumetric discharge rate for the tank shown? The coefficient of contraction for the orifice is 0.61, and the coefficient of velocity is 0.98. (©) 0.41 m*/s — (D) 0.52 m°/s (A) 021 m?/s—(B) 0.33 m* Vactuat = CeAoutVout In the preceding equation, C, is the coefficient of contraction, and ‘Aout is the area of the outlet. Vout = Ov/2gh PROFESSIONAL PUBLICATIONS, INC, 4-40 1001 SOLVED ENGINEERING FUNDAMENTALS PROBLEMS In the preceding equation, Cy is the coofficiont of velocity, and h is the vertical distance from the exit to the fluid’s surface. Vactuat = CcCy Aout /2gh = (081009005 =) /@ (at 3) @5m) =0.21 m*/s ‘The answer is (A). FLUIDS-53 ‘The upper plate illustrated is fixed, while the lower plate moves in the positive z direction at 0.5 m/s. ‘Tho plate separation is 0.001 m, the fluid viscosity is 7x 104 Pass, and the velocity profile is linear. Calculate the shear stress, in the moving fluid. ie (A) 0050Pa —(B) O15 Pa (©) 0.25Pa —(D) 0.85 Pa Tay = — ay GE w=O.7eP (0.7 g/ms*) We _ Ave wy By 05 2 -—e 001m = 500s re» = (0.0007 Pas) (500 2) = 0.35 Pa The answer is (D) PROFESSIONAL PUBLICATIONS, INC. FLUID STATICS AND DYNAMICS 441 FLUIDS-54 ‘What are the units of Reynolds number for pipe flow? (A) m/s (B) m/s (©) ke/ms? (D) none of the above "The Reynolds number is dimensionless. The answer is (D). FLUIDS-55 ‘Which of the following ratios represents a physical interpretation of the Reynolds number? buoyant foroes (A) Re = 5 ertal forces viscous forces (B) Re= serial foroes drag forces (©) Rem sous forces (0) Ro = itil Sorces viscous forees Re ap By definition, the Reynolds number is the ratio of the inertial forces on an element of fluid to the viscous forces. Tre answer sD). | FLUIDS-56 Which of the following statements is FALSE? (A) ‘The Reynolds number is the ratio of the viscous foree to the inertial force. (B) Steady fows do not change with time at any point. (C)_ The Navier-Stokes equation is the equation of motion for a viscous Newton- iam fluid (D) Bernoulli's equation only holds on the same streamline. ‘The Reynolds aumber is the ratio of the inertial forves to the viscous forces. answer is (A) PROFESSIONAL. PUBLICATIONS, INC. 442 1001 SOLVED ENGINEERING FUNDAMENTALS PROBLEMS FLUIDS-57 Calculate the Reynolds number for water at 20°C flowing in an open chennel. ‘The water is flowing at o volumetric rate of 0.8 m*/s. The channel has a height of 1.2 m and a width of 2.5 m. At this temperature, water has a kinematic viscosity of 1.02 x 10° m?/s, (A) 65x 10% (B) BB X10" = (C) 9.2x10® —(D) 12x 10° _vDe cross-sectional area’ D, = 4 ( Tossseetional area * 4( ‘wetted perimeter ) -0( (1.2 m)(2.5 m) ) m+25m+12 = 245 m VevA Rearranging, v A 08 -— Wajesm = 0.27 m/s (0 or =) (2.45 m) a 1.02 x 10-6 & $ 6.5 x 10° ‘The answer is (A). PROFESSIONAL PUBLICATIONS, INC. FLUID STATICS AND DYNAMICS 443 FLUIDS-58 A fluid with a kinematic viscosity of 2.5 x 10~® m?/s is flowing at 0.03 m/s from tan orifice 75 mm in diameter. How can the fluid be described? (A) ‘The fluid is completely turbulent. (B) The fluid is in the transition zone. (C) The fluid is laminar. (D) The fluid’s turbulence cannot be calculated from the information given, wD _ (008 2) (075 mi 25 x 10 Re = 900 ‘A Reynolds number of 900 means that the flow is well within the laminar (Re < 2000) region. ‘The answer is (C). FLUIDS-59 ‘The Reynolds number of a sphere falling in air is 1 x 10°. If the sphere's radius is 0.5 m, what is most nearly its velocity? (pate = 1.225 keg/m®, pate = 1.789 x 10° Pas) (A) 25 m/s (B) 5.2 m/s (©) m/s (D) 15 m/s ‘The answer is (D) PROFESSIONAL PUBLICATIONS, INC. 44d 1001 SOLVED ENGINEERING FUNDAMENTALS PROBLEMS FLUIDS-60 ‘Which of the following is NOT true regarding the Blasius boundary layer solu- tion? (A) It is valid only for potential flow. (B) tis valid for laminar flow (C) Its an approximate solution. (D) It permits one to calculate the skin friction on a fat plate. ‘The Blasius solution is an approximate solution to the boundary layer equations and makes some simplifying assumptions. It is valid for laminar, viscous flow and permits the evaluation of shear stress and skin friction, ‘The Blaslus solution or any other boundary layer concept has no meaning for potential flow. ‘The answer is (A). FLUIDS-61 From the Blasius solution for laminar boundary Inyer flow, the average coefficient of ekin friction is Cy = 1.328/V/Re. If air (pie = 1.225 kg/m® and fiae = 1.789 x 10-® Pa-s) is flowing past a 10 m long fiat plate at a velocity of 30 m/s, what is most nearly the force per unit width on the plate? (A) 0.85 N (B) 10N ()13N (D) 16 PairVeirLpiate Hate ee) fy 6 225 8) (30 2) (10 m) > "T7189 x10 Pas = 2.054 x 107 1.328 Rez, Os VRer PROFESSIONAL PUBLICATIONS, INC. FLUID STATICS AND DYNAMICS 445 Rearranging to solve for F, F Se Vaielptate ag 7 ACs Pa Veirlos = () 98x 10-* (1.225 $8) (80 2)" tom 62.N/m_ (1.6 N per unit width of plate) FLUIDS-62 From what were the curves of the Moody friction factor diagram for pipe flow determined? (A) calculations based on potential flow (B) theoretical solutions of the Navier-Stokes equations (C) experimental results for inviscid fluids (D) experimental results for viscous fluids ‘The curves in the Moody diagram are experimental data plots. They are valid for viscous fluids. [the wer is (D). FLUIDS-63 What is most nearly the friction factor for fow in a circular pipe where the Reynolds number is 1000? (A) 0.008 (B) 0.06 (©) 0.08 (D) 01 For Re < 2000, the friction factor, f, is given by the following. 64 ot 1000 = 0.064 (0.06) ‘The answer is (B). PROFESSIONAL. PUBLICATIONS, INC. 446 1001 SOLVED ENGINEERING FUNDAMENTALS PROBLEMS FLUIDS-64 For pipe flow in the laminar flow region, how is the friction factor related to the Reynolds number? wra(&) Osan — sant In the laminar region, f = 64/Re. FLUIDS-65 Which of the following flow meters measure(s) the average fluid velocity rather than a point or local velocity in a. pipe? 1. venturi meter I, pitot tube TIL impact tube IV, orifice meter V. hot-wire anemometer (A) Lonly (B) IL only (©) TandIV— (D) Wand V Of the four choices given, only venturi and orifice meters measure average velocity. ‘The answer is (C). FLUIDS-66 ‘What is the hydraulic radius of the semicircular channel shown? (A) 2m 3m (4m (0) 6m PROFESSIONAL PUBLICATIONS, INC. FLUID STATICS AND DYNAMICS 447 The hydraulic radius, ra, is oss-section area ‘wetted perimeter dom?) m= ‘The answer is (A). PLUIDS-67 ‘What is the hydraulic radius of the channel shown’? / phade\ en LEX am (A) 0.33 m (B) 0.43 m (©) 0.49 m (D) 15m d= /@my—Cm = Vim Lm 3.0 yea = (2 m)(v3 m) 46 mt? cron sectional area = ( PROFESSIONAL. PUBLICATIONS, INC. 4-48 1001 SOLVED ENGINEERING FUNDAMENTALS PROBLEMS Substituting, ory = Srossesectional area hn “wetted perimeter 3.46 m? 2m+3m+2m =0.49 m ‘The answer is (C). FLUIDS-68 For fully developed laminar flow of flulds through circular pipes, the average velocity is what fraction of the maximum velocity? (ay 1/8 (Baa (12 (D) 3/4 For laminar flow in pipes, Varo = The answer is (C). FLUIDS-69 ‘Tho flow rate of water through a cast-iron pipe is 20 m/min, The diameter of the pipe is 0.3 m, and the coefficient of friction is f = 0.0173. What is most: nearly the pressure drop over a 30 m length of pipe? (A) 9.8 kPa, (B) 13 kPa (©) 17 kPa (D) 19 kPa 4.71 m/s PROFESSIONAL PUBLICATIONS, INC. FLUID STATICS AND DYNAMICS 449 The head loss, Ah, is an=(3) (&) (un 8) = cor (2%) aos 2) 1.96 m ‘Rearranging to solve for Ap, bn- 22 79 Ap = po (000 8) (est 3) (2.96 m) = 19200 Pa (19 kPa) The answer is (D) FLUIDS-70 ‘A completely full cast-iron pipe of equilateral triangular cross section (vertex up) and with side length of 0.5 m has water flowing through it. The flow rate is 22 m3/min, and the friction factor for the pipe is 0.017. What is most nearly the pressure drop in a 30 m length of pipe? (A) 68 kPa, (B) 98 kPa (©) 10 kPa (D) 15 kPa. 05m, 05m 05m PROFESSIONAL PUBLICATIONS, INC. 450 1001 SOLVED ENGINEERING FUNDAMENTALS PROBLEMS A= /(05 my = 0.488 m cross-sectional area = bh = (3) (0.5 m)(0.433 m) = 0.108 m? = (seeesoaama) wetted perimeter 0.108 m? = (sentare casa) = 0.288 m ~ 108 =3.40 m/s ‘The head loss, Ah, is an=s() (2) = 1.043 m PROFESSIONAL PUBLICATIONS, INC. FLUID STATICS AND DYNAMICS 451 Ap = pgAh= Ah = (1000 3) (21 3) 4.088 mp - (os 5) (1.043 m) = 10.2 KN/m? (10 kPa) ‘The answer (©). FLUIDS-71 ‘A circular cylinder 4m long and 3 m in diameter is in an air stream. The flow velocity is 5 m/s perpendicular to the longitudinal axis of the cylinder. Given that the coefficient of drag on the cylinder is 1.3, and the density of air is 1.225 kg/m, what is most nearly the drag force on the cylinder? (A) 0.090 kN (B) 0.1L KN (©) 0.24 KN (D) 0.91 KN Ais the frontal area of the cylinder. Fp = 3CopA = (5) 0. (1208 8) (6 2) eae m = 2089 (024 RN) FLUIDS-72 ‘Air flows past a 50 mm diameter sphere at 30 m/s. What is most nearly the drag, force experienced by the sphere? ‘The sphere has a coefficient of drag of 0.5. The density of the air is 1.225 kg/m®. (A) 0.26 N (B) 034N (©) 054 (D) 0.68 N PROFESSIONAL PUBLICATIONS, ING. 452 1001 SOLVED ENGINEERING FUNDAMENTALS PROBLEMS Ais the frontal area of the sphere. Fo = 3CppaiviyA 2 = }Copacvder (==) — Co paisViu® Ey rere = Spear Gomme (0) (1.295 $) (50 2)” n(0.05 =0.541N (0.54N) The answer is (Q), FLUIDS-73 A cylinder 10 m long and 2 m in diameter is suspended in air flowing at 8 m/s. The air flow is perpendicular to the longitudinal axis of the cylinder. The density of air is 1.225 kg/m®, and the coefficient of drag of the eylinder is 1.3. What is most nearly the drag force on the cylinder? (A) 0.31 kN (B) 0.85 kN (C) LOKN (D) 2.3 kN Fp = }CppvA = (5) 019) (1.295 $$) (8 2)* 00 m2 m = 1019.2N (1.02 kN) ‘The answer is (C). FLUIDS~74 What is most nearly the terminal velocity of a 50 mm diameter, solid aluminum sphere falling in air? ‘The sphere has a coeflicient of drag of 0.5, the density of aluminum, patum, is 2650 kg/m®, and the density of air, pac, is 1.225 kg/m®. (A) 25 m/s (B) 53 m/s (©) 88 m/s (D) 180 m/s Let v; be the terminal velocity. At terminal velocity, the drag force, Fp, equals the weight. PROFESSIONAL PUBLICATIONS, INC. FLUID STATICS AND DYNAMICS 453 Fo = $Cppairv?A = mg wo tr($) ame G (4)(0.05 m) (2080 #) (81 5) nm (3)(05) (125 EY) 3.2 m/s he answer @) FLUIDS-75 Ina flow of air (p = 1.225 kg/m*) around a cylinder, the circulation is calculated to be 3.97 m?/s, If the froe-stream velocity is 30 m/s, what is most nearly the lift generated per meter length of the cylinder? (A) 160N/m — (B) 16DN/m —(C) ON/m_—(D) 200 N/m ‘The Kutto-Joukowsky theorem states that, itt Tm Povo!” = (1228 8) («0 8) (a9 =) = 146 N/m (150.N/m) ‘The answer is (A). ‘PROFESSIONAL PUBLICATIONS, INC. 454 1001 SOLVED ENGINEERING FUNDAMENTALS PROBLEMS FLUIDS-76 A 65 mm radius cylinder rotates at 3600 rpm. Air is flowing past the cylinder at 30 m/s. The density of air is 1.225 ka/m®. Approximately how much lft is generated by the cylinder per unit length? 00 rom 65 mm, 30 mis (A) 190.N/m — (B) 220N/m —(C) 290N/m_ —(D) 370 N/m From the Kutta-Joukowsky theorem, Me mat re fr ae = [onan = arty = 2n(0.005 m)? ( (200 =10 m/s SE = m= (1.205 4) (20 2) (10 =) = 367.5 N/m ‘The answer is (D). FLUIDS-77 A pump produces a head of 10 m. The volumetric flow rate through the pump is 6.3 x 10-4 m°/s. The fluid pumped is oil with a specific gravity of 0.83, Approximately how much energy does the pump consume in one hour? (A) 8.7 (8) 7d (©) 180 ky (D) 200 Ks ‘PROFESSIONAL PUBLICATIONS, INC. FLUID STATICS AND DYNAMICS 455 P=Apv ‘The change in pressure, Ap, is Ap= poh = (0.83) (2000 3%) (oat 3) (10 m) = 81423 Pa E=Pt = Apve = (81423 Pa) 0-8) 9 = 184607 Nem (180 kd) ‘The answer is (C). FLUIDS-78 ‘A pump has an efficiency of 65%. It is driven by a 530 W motor. The pump produces a pressure rise of 120 Pa in water. What is the required flow rate? (A) 30m/s (B) 34m%/s—(C) 46m/s—(D) 48 m/s ‘The power supplied by the pump to the water, P,, is Pe=nP, In the preceding, equation, 17 is efficiency and P, is ideal power. P= dev In the preceding equation, Ap is pressure rise and V’ is the volumetric, flow rate. pv = nF ‘Rearranging, Therefore, (0.65)(550 W) 120 Pa 2.08 m/s (3.0 m, v ‘The answer is (A) PROFESSIONAL PUBLICATIONS, INC. 5 THERMODYNAMICS ‘THERMODYNAMICS-1 Which of the following are intensive properties? I. temperature TL. pressure IIL. composition TV. mass (A) Tonly (8) only (C) Land (D) 1, and OT ‘An intensive property does not depend on the amount of material present. This is true for temperature, pressure, and composition. THERMODYNAMICS-2 How many independent properties are required to completely fix the equilibrium state of a pure gaseous compound? (Ayo Br (c) 2 (D) 3 ‘The number of independently variable properties needed to fix the state of a gaseous compound, fis given by the Gibbs phase rule, fan-p42 In the preceding equation, n is the number of components and p is ‘the number of phases. PROFESSIONAL PUBLICATIONS, INC. 52 1001 SOLVED ENGINEERING FUNDAMENTALS PROBLEMS For a pure gas, THERMODYNAMICS-3 Which of the following thermodynamic relations is INCORRECT? (A) Tas = dU + av (B) TAS = dH —Vap (©) U=9-w (D) H=U+pv U=Q+W. Therefore, the relation in option (C) is incorrect. ‘The answer is (C) THERMODYNAMICS—4 1f air is at a pressure, p, of 135 Pa, and at a temperature, 7, of 440K, what is ‘most nearly the specific volume, u? (Air's specific gas constant is R= 88.81 3/kg:K, and air can be modeled as an ideal gas.) (A) 110 m9/kg —(B) 130 m°/kg_— (C) 290 m%/kg —(D) 300 m/kg pu=RT Rr yet > 88.81 -*-) (440k) - ig ~ 135 Pa = 289 m°/keg (290 m°/kg) ‘PROFESSIONAL PUBLICATIONS, INC. ‘THERMODYNAMICS THERMODYNAMICS-5 Which of the following relationships defines enthalpy? (A) haut (B) h=u+pu (0) hauth Enthalpy is given by h The answer is (B). ‘THERMODYNAMICS-6 +p. (D) h=pu+T Tn a certain constant mass gystem, the conditions change from point 1 to point 2. How does the change in enthalpy for path A differ from the enthalpy change (A) AH > Ap (B) AH, = AHp (©) AH, (AVA BB (oc () D Fiquid solid ae water ‘riple point Vaner ‘The triple point is the point at which the liquid, solid, and vapor states are all in equilibrium. Therefore, point C is the triple point. The answer is (C). ‘PROFESSIONAL PUBLICATIONS, INC. 56 1001 SOLVED ENGINEERING FUNDAMENTALS PROBLEMS ‘THERMODYNAMICS-10 ‘What does the dashed curve in the figure represent? (A) the solidus line (B) an isotherm (O) tho saturated liquid line (D) the saturated vapor line ‘The dashed line shown is the saturated vapor line. AB is the saturated liquid line. Bis the eritical point. BC is the saturated vapor line DE represents a possible path of the system when heated. ‘The answer is (D) PROFESSIONAL PUBLICATIONS, INC. ‘THERMODYNAMICS BT ‘THERMODYNAMICS-11 In an ideal gas mixture of constituents i and j, what is the mole fraction of component i, x¢, equal to? T zi i 5) ms OG ® 75g ©) stm ©) sae my For an ideal gas, the mole fraction is equal to the partial pressure fraction. pene B+Py ‘THERMODYNAMICS-12 1.36 kg of air are held at 6.89 kPa and 38°C. Given that Rac = 88.89 J/ke-K, what is most nearly the volume of the container? (A) 2.2 m> (B) 3.1 m* (C) 4.8 m* (D) 5.5 m* ‘Use the ideal gas tw. T = (38°C 4.278%) = 311K pV =mRT es P 1.36 ke) (58.89 _ 030) (8889 6890 Pa 5.46 m* (5.5 m°) (311k) The answer is (D). PROFESSIONAL PUBLICATIONS, INC. 58 1001 SOLVED ENGINEERING FUNDAMENTALS PROBLEMS THERMODYNAMICS-13 ‘Tho compresbilty factor, Z, is used for predicting the behavior of nonideal gases. How is the compressibility factor defined relative to an ideal gas? (Sub- script “c” refers to critical value.) PB eee fae =(2\(® @z-2 @z-R @z-z wen(Z)(% For ral gases, th comproiblity factor, Z, i &dinensloness con- stant given by pV = ZRT. Therefore, Z = pV/RT. ‘The answer is (B), THERMODYNAMICS-14 On what plane is the Mollier diagram plotted? (A) pv (B) pT (©) hes (D) ww ‘The axes for a Mollier diagram are enthalpy and entropy (f-s). ‘The answer is (C). THERMODYNAMICS-15 How is the quality, 2, ofa liquid-vapor mixture defined? (A) the fraction of the total volume that is saturated vapor (B) the fraction of the total volume that is saturated liquid (C) the fraction of the total mass that is saturated vapor (D) the fraction of the total mass that is saturated liquid ‘The quality of the liquid-vapor mixture is defined as the fraction of the total mass that is saturated vapor. ‘The answer is (C). ‘PROFESSIONAL PUBLICATIONS, INC. ‘THERMODYNAMICS 59 ‘THERMODYNAMICS-16 ‘What is the expression for the heat of vaporization? hg = enthalpy of the saturated vapor hy = enthalpy of the saturated liquid (A) hp hy (BY phy (©) y= By (D) HZ ‘The heat of vaporization, hy», is the difference between the enthalpy of the saturated vapor and the enthalpy of the saturated liquid. Thus, higg = hy — hy [Te ‘ancwer is (C). ‘THERMODYNAMICS-17 From the steam tables, de The average specific heat at constant pressure, (A) 1.79 kJ/kg"C (B) 10.3kJ/kg°C (C) 90.6 kI/kg:°C (D) 100 kJ/kg: Ah=oAT ny OAT From the steam tables, for 10 kPa, at 47.7°C: at 43.8°C: 2588.1 k5/keg — 2581.1 kJ/kg, FTC 43.8°C 79 kJ /eg-®C ‘The answer is (A). PROFESSIONAL PUBLICATIONS, INC. 5.10 1001 SOLVED ENGINEERING FUNDAMENTALS PROBLEMS THERMODYNAMICS-18 A 10 m® vessel initially contains 5 m® of liquid water and 5 m® of saturated ‘water vapor at 100 kPa. Calculate the internal energy of the system using the steam tables. (A) 5x10% IJ (B) 8x107 kJ (C) 1x10 (D) 2x10 From the steam tables, vy = 0.001043 m* kg, 1.6940 m3 /kg, ty = 417.3 K/h ty = 2506 kI/kg anal 0.001043 kg = 4794 ke UE UsMig + Ugtvap (az 8) (4794 kg) + (2505 1 2) (2.95 kg) .01 x 10° kd PROFESSIONAL. PUBLICATIONS, INC. ‘THERMODYNAMICS bal THERMODYNAMICS-19 ‘A vessel with a volume of 1 m* contains liquid water and water vapor in equilib- riuin at 600 kPa, ‘The liquid water has a mass of 1 kg. Using the steam tables, determine the approximate mass of the water vapor. (A) 0.99 ke (B) 1.6 kg (©) 19 ke (D) 3.2 kg From the steam tables at 600 kPa, vy = 0.001101 m?/kg Uy = 0.3157 m?/ke Viana = my + m9U%p Veotal = myvy U9 1 wt (4g (ono 2) osisr 16 ke (3.2 ke) e answer is (D). THERMODYNAMICS-20 What is most nearly the entropy of steam at 476 kPa with a quality of 0.6? (A) 24 kd eK (B) 38 kd/kgK (©) 48 kd /kgK (D) 5.7 ki/kgK From the steam tables at 476 kPa, 9 = 1.8418 kJ/kg-K 84g = 4.9961 KI /ke-K = sy +2879 PROFESSIONAL PUBLICATIONS, INC 512 1001 SOLVED ENGINEERING FUNDAMENTALS PROBLEMS In the preceding equation, is quality. J ; 8418 Ee + (0.6) (co = 4.839 kI/kg-K (4.8 kJ /kg-K) ‘THERMODYNAMICS-21 If 0.45 kg of steam at 101.3 kPa and 63% quality is heated isentropically, at approximately what prossure will it reach the saturated vapor state? (A) 15200 kPa (B) 16300 KPa (©) 17300 kPa (D) 17800 kPa Use the steam tables. pi = 101.3 kPa 4p: = 1.307 kid eg 048 ke kg a= 51 = 8p +0.638j91 a Py ~ 1207 EE + (069) (cos Z) = 5.117 Kafe k Now, find pa such that 92 = 5.117 kI/kg-K. Interpolating from the steam tables, pa ~ 17800 kPa, ‘The answer is (D) ‘THERMODYNAMICS-22 ‘The first law of thermodynamics is based on which of the following principles? (A) conservation of mass (B) the enthalpy-entropy relationship (©) action-reaction (D) conservation of energy PROFESSIONAL PUBLICATIONS, INC. ‘THERMODYNAMICS B13 ‘The first law of thermodynamics is based on the principle of conser- vation of energy. ‘The answer () ‘THERMODYNAMICS-23 ‘The general energy equation for an open system involves the following five terms. 1. accumulation of energy TI. not onorgy transfor by work (standard sign convention) IIL. net energy transfer by heat (standard sign convention) TV. transfer of energy in by mass flow \V. transfer of energy out by mass flow Using the standard sign conventions, what is the proper arrangement of these terms for the general energy equation satisfying the first law of thermodynamics? (A) I= + 1 +IV-V (B) I=0+Il+V+V {(C)1=4+M+IV-V (D) 1=0-M-IV+v ‘The first law of thermodynamics states that the total change in energy (0) is equal to the energy in (IV) minus the energy out (V) minus the svork done on the system (II) plus the heat transferred to the system (HD). Thus, I= - 11+ I+ IV - Vv. ‘The answer is (A). ‘PROFESSIONAL PUBLICATIONS, INC. 514 1001 SOLVED ENGINEERING FUNDAMENTALS PROBLEMS ‘THERMODYNAMICS-24 In a reversible process, the state of a system changes from state 1 to state 2 as shown on the p-V diagram. What does the shaded area on the diagram represent? (A) free-energy change (B) heat transfer (C) enthalpy change (D) work done by the system For a reversible process, the work done by the system is given by the following, Ww ff nv ‘Therefore, the shaded area represents the work done by the system. The answer is (D). 'THERMODYNAMICS-25 ‘What is the value of the work done for a closed, reversible, isometric system? (A) zero (B) positive (C) negative (D) positive or negative w= [oa PROFESSIONAL PUBLICATIONS, IN. ‘THERMODYNAMICS 515 An isometric system is a system which has a constant volume (dV =0). Therefore, the work done by the system is zero. 7) ‘THERMODYNAMICS-26 ‘The expansion of a gas through a plug at a high pressure results in a temperature rise, while at lower pressures a temperature drop occurs. The Joule-Thompson coefficient, j.y7, is defined as the ratio of the change in temperature to the change in pressure, ‘The temperature at which s.r changes from positive to negative is called the inversion temperature. When yr is negative, which of the following statements is true? (A) Gases may be liquified by pressurization. (B) No liquification is possible. (C) Only trace liquification is possible. (D) Liquification can be obtained only with a catalyst When jiyr <0, then O7/8p <0. Thus, a pressure rise is accom- panied by a temperature drop. Therefore, a gas may be liquified by pressurization, ‘The answer is (A). ‘THERMODYNAMICS-27 A5 m® vessel initially contains 50 kg of liquid water and saturated water vapor at a total internal energy of 27 300 kJ. Calculate the heat requirement to vaporize all of the liquid. (A) 100000 kJ —(B) 200000 kJ (C) 300000kJ_—(D) 400000 kJ ‘An expression for the first law of thermodynamics is Qa Uy = 27300 kd Find Uz in the steam tables at 100% vapor. vy = 5 m8/50 kg = 0.10 m*/kg. PROFESSIONAL. PUBLICATIONS, INC. 516 1001 SOLVED ENGINEERING FUNDAMENTALS PROBLEMS The final state is at p= .00 MPa and ty = 2600 kJ/kg. Us =r kJ = (2600 ©) (50 (210) come = 150000 kr Q = 130000 kJ — 27300 kJ = 103000 kJ (100000 ks) ‘Tho answer is (A) ‘THERMODYNAMICS-28 What is most nearly the change in internal energy of 2.27 kg of oxygen gas when the temperature changes from 38°C to 49°C? (c, = 0.658 kJ/kg K) (A) 16g (B) 420 xy (©) 470 ks (D) 680 kJ AU = me,AT AT = 49°C — 38°C =1°C AU = (2.27 kg) (0 658 (ure) ke = 164k) (16 kJ) PROFESSIONAL PUBLICATIONS, INC. THERMODYNAMICS 517 THERMODYNAMICS-29 Water (specific heat cy = 4.2 ki/ke-K) is being heated by a 1500 W heater. ‘What is most nearly the temperature rate of change for 1 kg of water? (A) 0.043K/s (B) 0.18K/s (©) 0.36K/s (D) 1.5K/s Q=moy(AT) ar=2 nes _ 1500 w 7 7 wa (oe Se) =0.357K/s (0.36K/s) ‘The answer is (C). ‘THERMODYNAMICS-30 1 kg of water (cy = 4.2 kJ/kg-K) is heated by 316 kJ of energy. What is most nearly the change in temperature? (a) 18K @) 71K (©) UK (D) 75K moyAT =Q ar = mes 316 kJ (1 kg) (4 ‘The answer is (D) PROFESSIONAL PUBLICATIONS, INC. 518 1001 SOLVED ENGINEERING FUNDAMENTALS PROBLEMS ‘THERMODYNAMICS-31 ‘What is most nearly the change in enthalpy per kg of nitrogen gas as its tem- perature changes from 260°C to 93°C (cp = 1.04 kJ /kg-K)? (A) -200k (B) 170k (©) 110k (D) 170 ky Ah=c,AT 5 (108 Ze) (93°C — 260°C) 173.7 kJ (—170 kd) ‘THERMODYNAMICS-32 Calculate the change in enthalpy as 1 kg of nitrogen is heated from 1000K to 1500K, assuming the nitrogen is an ideal gas at a constant pressure. The temperature-dependent specific heat of nitrogen is Cy = 39.06 — 512.71 (Gp is in kJ /kmol-K, and T is in K.) (A) 600 i (B) 70 ks (©) 800 kd (D) 900 kg oH o= (én), aH = Cor = [car 1800 -[ (39.06 - 512.797-" 4. 1072.77-? — 820.47-8) aT = | (39.067 + 1025.67-°% + 1072.77-} +. 410.20-7)| ad Py = 88617 (5 — 30004 = 19523 kJ /kmol PROFESSIONAL PUBLICATIONS, INC. ‘THERMODYNAMICS 519 AF = mAh =m (#7) - (Co +) (11g) CE) = 697.3 kJ (700 kd) ‘The answer is (B). THERMODYNAMICS-33 What is most nearly the resulting pressure when 400 g of air at 103.6 kPa and 98°C is heated at constant volume to 427°C? (A) 160kPa—(B) 200 KPa (©) 250kPa—(D) 480 kPa n_h DP pila q __ (208.6 kPa)(427°C + 273°) G°C + DP 08 kPa (200 kPa) 'THERMODYNAMICS-34 Approximately how much power is required to isothermally compress 23 m*/min of air from 101.5 kPa to 828.5 kPa? (A) 64 kW (B) 82 kW (©) 92 kW (©) 98 kw For an isothermal process, ; bm Wo-mvine mt Pa =p; In mn = (101.5 kPa)(23 m®) tn eo 2) 101.5 kPa, = 4901 Id PROFESSIONAL PUBLICATIONS, INC. 5-20 1001 SOLVED ENGINEERING FUNDAMENTALS PROBLEMS wy z doors ws ‘81.7kW (82 kW) ‘The answer is (B). THERMODYNAMICS-35 ‘What is most nearly the work done by a system in which 1 kmol of water com- pletely evaporates at 100°C and 1 atm constant pressure? (A) 1000} (B) 29004 (©) 250011) 5100s p= (atm) (1019 22) = 101.0 ure vom the steam ables, ‘uy = 0.001 044 m® /kg 673 m ka w= ‘The molecular weight of water is MWn,0 Vi 8.016 kg/kemol yMWingom (aonon 32) (sa tay 0.01881 m? gMWn,om mm kg ~ (102) (50 (oor 2) (so = 30,141 m* Ya (1 kamol) PROFESSIONAL PUBLICATIONS, INC. ‘THERMODYNAMICS 5-21 p(Va— Vi) W = (101300 Pa)(30.141 m* — 0.01881 m*) = 3.05 x 10° J (3100 ki) ‘The answer is (D) ‘THERMODYNAMICS-36 5 mol of water vapor et 100°C and 1 atm pressure are compressed isobarically to form liquid at 100°C. The process is reversible, and the ideal gas laws apply. ‘What is most nearly the initial volume of the vapor? (A) 1201 (B) 130L (© wou () 1501 Use the ideal gos law. pV =nR'T yank P (6 mon (ose 2 (100°C + 273°) Tatm -153L (150 L) The answer is (D) ‘THERMODYNAMICS-37 5 kinol of water vapor at 100°C and 1 atm pressure are compressed isobarically from an initial volume of 153 L to form liquid at 100°C. The process is reversible, and the ideal gas laws apply. What is most nearly the work done on the system? (A) 60k (B) 6245 (©) 60 MI (D) 6.2 Ma w -p(Va — Vi) PROFESSIONAL PUBLICATIONS, INC. 5-22 1001 SOLVED ENGINEERING FUNDAMENTALS PROBLEMS From the steam tables, wy = .001 044 m8 /kg MWno = 18.016 kg/kanol vi m? = (153 L) (0.001 = 1.153 m? Va =nMWu,0vy = (6kmoy (s8.016 : ) (oso #2) W = —pAV = —(101 325 Pa) (0.094 m? — 0.153 m®) = 50783 (6.0 kJ) ‘The answer is (A). ‘THERMODYNAMICS-38 = 0,094 m? 5 kmol of water vapor at 100°C and 1 atm pressure are compressed isobarically to form liquid at 100°C. The process is reversible, and the ideal gas laws apply. ‘The heat of vaporization is 2257 kJ/kg. Whnt is most nearly the heat required for condensation for the amount of water given? (A) -200M3—(B) -140 MJ (C) 200 MJ (D) 410 MI hnyg = 2957 kg Q=AH = m(=hya) = nM) (ton (1038 28,) (ar #2) 208.3 MJ. (-200 MJ) ‘The answer is (A) PROFESSIONAL PUBLICATIONS, INC. ‘THERMODYNAMICS 5-23 ‘THERMODYNAMICS-39 ‘What is the equation for the work done by a constant temperature system? (A) W = mRTIn(Ve- Vi) = Ye @) W= mR -T) Inge va (© W=mrrin (D) W = mRT In "px we [piv _ mar perv ? RD. we [ Sra = mRT wy | Va W = mRrin The answer is (C) ‘THERMODYNAMICS-40 20 g of oxygen gas (O2) are compressed at a constant: temperature of 30°C to 5% of their original volume, What work is done on the system? (A) 820 cal (B) 920 cal (C) 950 cal (D) 1120 cal Ra negative to get work Ww i. pv ‘done on system rin Yt meen cal p-® 1} nok MW = inal = 0.0619 cal/g-K ‘PROFESSIONAL PUBLICATIONS, INC. 524 1001 SOLVED ENGINEERING FUNDAMENTALS PROBLEMS 80°C +.273° = 303K W=-(20) (00610 &) (303K) In ( = 1124 cal (1120 eal) ‘he answer is (D) ‘THERMODYNAMICS-41 Helium (R* = 0.6403 kJ /kg-K) is compressed isothermally from 101.3 kPa and 20°C. ‘The compression ratio is 4. What is most nearly the work done by the gat (A) ~320 ki/kg (B) —260 kJ/kg (C) 170 ki /kg —(D) 180 ki /kee 9) 1 en = (ots 4.) cnn 260 ki /keg ‘The answer is (B) THERMODYNAMICS-42 Gas is enclosed in a cylinder with a weighted piston as the top boundary. The gas is heated and expands from a volume of 0.04 m* to 0.10 m® at a constant pressure of 200 kPa. Calculate the work done by the system. (A) 80 ks (B) 10k (©) 12 ks (D) aK constant F LA PROFESSIONAL PUBLICATIONS, INC. ‘THERMODYNAMICS 5.25 At constant pressure, we [nav =P(V2—Vi) = (200.000 Pa}(0.10 m* ~ 0.04 m®) = 120003 (12k3) ‘Phe answer is (C). ‘THERMODYNAMICS-43 ‘A piston-cylinder system contains a gas that expands under a constant pressure of 57 kPa. If the piston is displaced 0.3 m during the process, and the piston diameter is 0.6 tm, what is the work done by the gas on the piston? (A) 243 (B) 32kI (© 84kI (D) 4813 ‘The work is done at constant pressure. A we [vv =pAv AV = AAL = (0.3 m)?(0.3 m) 0.085 m* W = (57 kPa)(0.085 m*) =48 kd The answer is (D). ‘PROFESSIONAL PUBLICATIONS, INC. 5-26 1001 SOLVED ENGINEERING FUNDAMENTALS PROBLEMS THERMODYNAMICS—44 Gas is enclosed in a cylinder with a weighted piston as the top boundary, The gas is heated and expands from a volume of 0.04 m? to 0.10 m®, The pressure varies such that pV is constant, and the initial pressure is 200 kPa. What is ‘most nearly the work done by the system? (A) 6.8 kd (B) 73kd (©) 10K (©) 12K ‘The work done by the system on the piston is given as follows. ayy, (0-10 m? ~ nau 004) (22289) ee) [pon | THERMODYNAMICS 4s Steam flows into a turbine at a rate of 10 kg/s, and 10 kW of heat are lost from the turbine. Ignoring elevation and kinetic energy effects, what is most nearly the power output from the turbine? inlet conditions exit conditions pressure 2.0 MPa 0.1 MPa ‘temperature 350°C = quality = 100% (A) 4000 KW (B) 4400 kW (©) 4600KW ——(D) 5000 KW. PROFESSIONAL PUBLICATIONS, INC. ‘THERMODYNAMICS 5.27 Use the first Inw of thermodynamics. P=W = 1h(ly— he) +Q From the steam tables, hy = 3137.0 I /kg he = 2675.5 kI/kg, Fa) (vo * amr # w= (10 '8) (oxsra ers (00'8) (nso ass #2) = 4605 KW (4600 KW) 10 kW ‘The answer is (C). 'THERMODYNAMICS—46 atic process compare to an isentropic process? How does an (A) adiabatic: heat transfer = 0; isentropic: heat transfer # 0 (B) adiabatic: heat transfer # 0; isentropic: heat transfer = 0 (C) adiabatic: reversible; isentropic: not reversible (D) both: heat transfer = 0; isentropic: reversible ‘An adiabatic process is one in which there is no heat flow. It is not necessarily reversible. An isentropic process has no heat flow and is reversible. Th answer is (D), ‘THERMODYNAMICS-~47 ‘What is true about the polytropic exponent, n, for a perfect gas undergoing an isobaric process? (A) n>0 @) n0 @) as <0 PROFESSIONAL. PUBLICATIONS, INC. 5-34 1001 SOLVED ENGINEERING FUNDAMENTALS PROBLEMS For an irreversible process, ds = 'Seystom + ASeurroundings > 0 The answer is (C), ‘THERMODYNAMICS-58 For which type of process is the equation dQ = TdS valid? (A) irreversible (B) isothermal (CG) reversible (D) isobaric TaS = dH ~Vdp aH = dU + paV + Vp TdS = dU + pdV dv = aw TdS = dU +aWw ‘The first law of thermodynamics is dQ =a +4aw ‘Therefore, dQ ‘AS for a reversible process] ‘The answer is (C). ‘THERMODYNAMICS-59 Which of the following is true for any process? (A) ASwurroundings + ASaystem 2 0 B) ASsurroundings + ASoystem $0 (©) ASpurroundings + ASeyatem <0 (D) ASrurrounaings + ASeystem > 0 PROFESSIONAL PUBLICATIONS, INC. ‘THERMODYNAMICS 5-35 ‘The total entropy either increases or, for a reversible process, remains the same. Therefore, the total change in entropy is always greater than or equal to zero. answer (A), ‘THERMODYNAMICS-60 In a reversible process, the state of a system changes from state 1 to state 2, as shown on the T-S diagram. What does the shaded area of the dingram represent? ood 2 (A) free-energy change (B) heat transfer (©) enthalpy change (D) work For areversible process, @Q = J'TaS. Thus, the shacled area represents the heat transfer. ‘The answer is (B). THERMODYNAMICS-61 Helium is compressed isothermally from 14.7 psia and 68°F. The compression ratio is 4. What is most nearly the change in entropy of the gas, given that the specific gas constant is Rive = 0.6411 kJ/ke-K? (A) -097 kI/kgK — (B) -0.89 kd /kgK (©) 0.45 kd /ke-K (D) 0.89 ks/kgK PROFESSIONAL PUBLICATIONS, INC. 5-36 1001 SOLVED ENGINEERING FUNDAMENTALS PROBLEMS For an isothermal process, Ve As=Rueln ye = (oan: #2.) nd = (vom: 2) nd = 0.880 kd ke (~0.89 1S eK) ‘The answer is (B). ‘THERMODYNAMICS-62 For an ideal gas, what is most nearly the specific molar entropy change during ‘an isothermal process in which the pressure changes from 200 kPa to 150 kPa? (A) 2.0 3/moli (B) 24 J/molk (© 28 3/molK (D) 81 J/mokK For an ideal gas, 1, AS =n ont, For an isothermal process, =k Ta _ ng=0 AS=-R'In2 nm J 150 kPa’ = (-884 san)!" (ome) = 239 J/molK (24 J/molK) ‘The answer is (B). ‘PROFESSIONAL PUBLICATIONS, INC. ‘THERMODYNAMICS 537 THERMODYNAMICS-63 In the p-V diagram shown, heat addition occurs between points 1 and 2. Given that c, = 0.434 kJ/kg;K, what is most nearly the entropy produced during this step? 2 3178 Po, 1634K 4} 1248 Ps, 600K (A) O41 KI/kgK—— (B) —0.28 i kg (©) 0.23 kd /keK (D) 0.41 kI/kgK =eqlnZ “% Asm eying + Ringe Va=Vi (0 x) "oar ) = 0.407 ki/kg-K (0.41 kI/kg:K) ‘The answer is (D). ‘THERMODYNAMICS-64 200 g of water are heated from 5°C to 100°C and vaporized at a constant pressure. ‘The heat of vaporization of water at 100°C is 539.2 cal/g. ‘The heat capacity ft constant pressure, cp, is 1.0 cal/g:K. What is most nearly the total change in entropy? (A) 250 cal/K —(B) 300 cal/K — (C) 350 cal/K —(D) 400 eal PROFESSIONAL PUBLICATIONS, INC. 5-38 1001 SOLVED ENGINEERING FUNDAMENTALS PROBLEMS Ty = 5°C +273" = 278K Te = 100°C + 273° = 373K AS = AStoat + ASeaporization Aspens = 82-81 a = [tara In T PL pap Asma = [Bar _ ot =o cal ‘373K = (15%) (sr) 0.2940 cal/e.K AShent MASreat ~ ca (0200 2) = 58.8 cal/K Asyapor 1.446 cal/e-K AScaporiaation = MASyaporeation cal ~ (148 2) = 289.2 cal/K cal K = 348 cal/KK (350 cal/K) As = 585 S 4 2902 PROFESSIONAL PUBLICATIONS, INC. 6 POWER CYCLES POWER CYCLES-1 What kind of process occurs between points $ and 4? (A) isentropic (B) isobaric (©) isothermal (D) ‘There is insufficient information to determine the process type. "The process between points 3 and 4 is indeterminate, All that can be said about the process is that it is neither isobaric nor isothermal. ‘The answer is (D). ‘PROFESSIONAL PUBLICATIONS, INC. 62 1001 SOLVED ENGINEERING FUNDAMENTALS PROBLEMS POWER CYCLES-2 ‘Which of the following thermodynamic eycles is the most efficient? (C) Carnot (D) combined Brayton-Rankine No cycle is more efficent. than the Carnot cycle, because it is com- pletely reversible. The answer is (C). POWER CYCLES-3 For the reversible heat engine shown, which area on the corresponding T-S dia- ram represents the work done by the system? Tw 7 ; Te. eit. [OK es fou Te 5] 4 (A) work=0 (B)1-2-4-5 (@) 6-3-4-5 — (D) 1-2-3-6 ‘Use the first law of thermodynamics. W=Qu~Qc TyAS ~ToAS 1-2—-4—8)-(8-4-5-6) ~2-3-6 ‘The answer is (D). PROFESSIONAL PUBLICATIONS, INC. POWER CYCLES 63 POWER CYCLES~4 "The ideal reversible Carnot cycle involves several baste processes. What: type of processes are they? (A) all adiabatic (B) all isentropic (C) two adiabatic and two isentropic (D) two isothermal and two isentropic By definition, a Carnot cycle consists of two isothermal processes and to isentropic processes. The answer is (D) POWER CYCLES-5 ‘An ideal reversible Carnot cycle is represented on the T-S diagram shown. The efficiency of the cycle is represented by which of the following ratios of areas? Ty ‘PROFESSIONAL PUBLICATIONS, INC. 64 1001 SOLVED ENGINEERING FUNDAMENTALS PROBLEMS ‘The efficiency, 7, is defined as follows. Woes Qu ‘The answer is (A) POWER CYCLES-6 Which of the following T-S diagrams may be that of a Carnot cycle? a © 7 (7 (a) 7 ®) 7 A Carnot cycle hes two isothermal processes (horizontal lines on a T-S diagram) and two isentropie processes (vertical lines on a T-S diagram). The only diagram that has both of these properties is the diagram represented in option (D). ‘The answer is (D). PROFESSIONAL PUBLICATIONS, INC. POWER CYCLES 65 POWER CYCLES-7 ‘Which of the following is/are representations of a Carnot cycle? rT | 1 1. um. (A) Tonly {B) only (©) Land I (D) Mand 1 ‘A Camot cycle has two isentropic processes and two isothermal pro- cesses. Diagram I has two isentropic processes, but no isothermal processes. Diagram IT has two isentropic processes and two isother- mal processes, When h is constant, T' is constant. Diagram III also has two isentropic processes and two isothermal processes. Thus, II and III both represent a Carnot cycle. The answer is (D) POWER CYCLES-8 Consider the T-S diagram of a Carnot cycle in the figure. What is the amount of total work done in one cycle? ees W mL 2 => 3 h 7 Te nh = 4 8 % % % (A) (S2-S:)Ta~Th) (8) (Sa S1)Ta (©) (S1- 8) (©) (S2.- S)-T ‘PROFESSIONAL PUBLICATIONS, INC. 66 1001 SOLVED ENGINEERING FUNDAMENTALS PROBLEMS W=Qoe - AU =Qu-Qe-0 =TyASy -TeASe = Ti(S2 ~ Si) - Ti(S2 — 51) = (S-S)(-T) ‘The answer is (A). POWER CYCLES-9 Consider the T-$ diagram of a Carnot eycle shown. What amount of heat is rejected to the surroundings? Ty =; Te 5 & (A) Tu(Ss— $2) (B) Ty(S2 - $1) (©) To(Ss ~ $2) (D) To(S2- 51) TH Te| f— reject 5 % ‘The rejected heat is equal to the area under the lower branch of the re i Qnincea = Te(S2 ~ $1) ‘The answer is (D). [PROFESSIONAL PUBLICATIONS, INC. POWER CYCLES 67 POWER CYCLES-10 ‘What is the temperature difference of the cycle if the entropy difference is AS and the work done is W? (A) was Lis Oy oF (>) was) In the figure, the work done, W, is represented by the shaded area. Tw — Te| as Ww =(AT)(AS) Rearranging, w AT = 55 The answer is (B). POWER CYCLES-11 In the Carnot cycle shown, the net amount of heat put into the system is equal to the total amount of work done by the system. However, it cannot be stated that the heat: put into the system between states 1 and 2 is equal to the work done between states 1 and 2. What is the reason for this? PROFESSIONAL PUBLICATIONS, INC. 68 1001 SOLVED ENGINEERING FUNDAMENTALS PROBLEMS (A) The process is adiabatic. (B) The process is not adiabatic. (©) The second law states that the amount of energy put into the system is ‘equal to the amount taken out of the system. (D) The first law states that dQ = dU +dW, Since dU #0, dQ 4 aW. ‘The first law states that dQ = dU + dW. Between states 1 and 2, AU £0. Therefore, dQia # dW. ‘The answer is (D). POWER CYCLES-12 For the Carnot eycle shown, helium is the gas used with a specific heat ratio, k, of 5/3. Given that Va/Va = 2 and Ta/Tp = 1.9, calculate pc/pa. 7 aps o © — (A) 0.0633 (B) 0.100 (© 0.180 (D) 0.262 7a _ Th Rie" For a constant temperature, pa _ Va =heaip Pal ole PROFESSIONAL. PUBLICATIONS, INC. POWER CYCLES 69 For n Carnot cycle, stage B to C is isentropic. The relation for ideal gases undergoing a constant entropy process is, ey" @e) we = 0.100 Ele 318 answer is (B). POWER CYCLES-13 A Carnot engine operates between 444K and 555K. What is its thermal effi- ciency? (A) 20% (B) 30% (©) 40% () 50% 800K ~ 000K =02 (20%) ‘The answer is (A). POWER CYCLES-14 For a heat engine operating between two temperatures (73 > Ti), what is the maximum efficiency attainable? @i-B @i--k oF w)1- (2) PROFESSIONAL PUBLICATIONS, INC. 610 1001 SOLVED ENGINEERING FUNDAMENTALS PROBLEMS ‘The maximum efliciency attainable is the Carnot efficiency. POWER CYCLES-15 Which of the following is NOT an advantage of a superheated, closed Rankine eyclo over an open Rankine cycle? (A) increased efficiency (B) increased turbine work output (C) increased turbine life (D) increased boiler life Option (D) is not an advantage because a superheated Rankine cycle has higher boiler heat temperatures that decrease boiler life The answer is (D) POWER CYCLES-16 ‘Which of the following statements regarding Rankine cycles is FALSE? (A) Use of a condensible vapor in the cycle increases the efficiency of the cycle. (B) The temperatures at which energy is transferred to and from the working liquid are less separated than in a Camot cycl (C) Superheating increases the efficiency of a Rankine cycle. (D) In practical terms, the susceptibility of the engine materials to corrosion is not a key limitation on the operating efficiency. PROFESSIONAL PUBLICATIONS, INC. POWER CYCLES en Corrosion is a principal limitation on the use of higher temperatures for this type of engine. Thus, the susceptibility of engine materials to corrosion does limit operating efficiency. ‘The answer is (D). POWER CYCLES-17 “What type of power cycle does the following diagram illustrate? lines of constant (A) a Carnot cycle (B) an idealized Rankine cycle with superheat and reheat (C) am idealized diesel cycle (D) an idealized Stirling cycle ‘The diagram shows an idealized Rankine cycle with reheat. ‘The answer is (B) PROFESSIONAL. PUBLICATIONS, INC. ez 1001 SOLVED ENGINEERING FUNDAMENTALS PROBLEMS POWER CYCLES-18 For the steam Rankine cycle shown, determine the approximate enthalpy at state 3. 700.5 kPa 101.949 (A) 2000 ks /kg (B) 2500 kJ/kg (©) 2680 Ki /k (D) 2760 kiI/k State 3 is saturated steam at 700.5 kPa. From the steam tables, hig = hy 763.5 kJ/kg (2760 ki/kg) PROFESSIONAL PUBLICATIONS, INC. POWER CYCLES 613 POWER CYCLES-19 In a Rankine cycle, state 3 is saturated steam at 1.398 MPa, Assuming that the turbine is isentropic and py = 101.35 kPa, find the approximate enthalpy at state 4, h (A) 420 ki/kg —(B) 2260 ki/kg— (C) 2350 kJ/kg (D) 2680 J/kg, Py = 1.308 MPa, From the steam tables, 43 = 8 = 6.4698 kI/kgK 698 kJ/ke-K S455 +0849 kd = 1.8000 Rte ‘Rearrange to solve for quality 2. ry ki samy 04698 eg — 18000 GE Se pts a7 ae 60480 = 0.854 PROFESSIONAL PUBLICATIONS, INC. ei 1001 SOLVED ENGINEERING FUNDAMENTALS PROBLEMS ha up bahgy a) ky = 410.04 + (0.864) (22520 2) 340.5 J/kg (2350 kJ/kg) ‘The answer is (C) POWER CYCLES-20 In a steam Rankine cycle, state 1 is saturated liquid at 101.35 kPa. State 2 is high-pressure liquid at 0.7005 MPa. Approximately how much work is required to pump 0.45 kg of water from state 1 to state 27 > % (A) 125 kg (B) 1365 (C) 2803 (D) 19.5 vy = 0.001 044 m? /kg, W = m(hy— hi) =mvdp ~ 08536 (1040. 30-* 22 ners —s1.s8 wr (sa 5) = 281.5 J (280 J) Note: State 2 is not a saturated state, The answer is (C). PROFESSIONAL PUBLICATIONS, INC. POWER CYCLES 615 POWER CYCLES-21 In a steam Rankine cycle, saturated liquid at 101.35 kPa is pumped to 1.398 MPa, If the pump were isentropic, the enthalpy of state 2 would be 420.2 kJ/kg. ‘The isentropic efficiency of the pump is 60%. What is most nearly the enthalpy of state 2? 7 (A) 252.1 ki/ke (B) 417.5 kJ/kg (C) 418.9 kI/kg_ (D) 421.0 K/kg cA Ww _ mAh, mah Ah, * Bh (ha = bade fay 1) hgh = a= hy + Cate " From the stoam tables, hy = 419.04 kJ/kg yy na gnc hg = 419.04 igt 08 = 421.0 KI/kg The answer is (D). PROFESSIONAL PUBLICATIONS, INC. 6-16 1001 SOLVED ENGINEERING FUNDAMENTALS PROBLEMS POWER CYCLES-22 Steam in a Rankine cycle is expanded from # 1.398 MPa saturated vapor state to 0.1433 MPa. The turbine has an efficiency of 0.8. What is most nearly the enthalpy of state 4? a (A) 2233 kJ/kg (B) 2294 kJ/kg (C) 2393 KI /kg — (D) 2476 kl /kg. Call the isentropic state 4,. Ps = 1.398 MPa [saturated vapor] 85= 8pa0 + 28 4q.40 eee to ea Esse (saa 2) = 0.8679 hay = has Fah pase re wy 12. (oor) (2290.2 ig tes (202 =) 2396.89 13 kg hg — he "tig has fa = ha — (ha — has) 3 oo ka 2790.0 — — (0.8) | 2790.0 — — 2396.89 — 08 (m0 =) = 2476 kd kg = 461.30 ‘Phe answer is (D) PROFESSIONAL PUBLICATIONS, INC POWER CYCLES oir POWER CYCLES-23 ‘Which of the following sets of reversible processes describes an ideal Otto cycle? I. adiabatic compression, constant volume heat addition, adiabatic expan- sion, constant volume heat rejection IL isothermal compression, isobaric heat addition, isothermal expansion, isobaric heat rejection (A) T only (B) H only (C) I and I in succession (D) and I in succession An Otto cycle is defined by the set of reversible processes in I. The answer is (A) POWER CYCLES-24 In the power stroke (8 to 4) of the ideal Otto cycle shown, what is the entropy change? 3 Ty=16343K 18 MPa r= 9.3K Ty= 0.25 MPa (A) 0.536 ki/ke-K —(B) 0.00 kd/kgK (©) 0.536 15 /kg- (D) 0.749 1 kg PROFESSIONAL PUBLICATIONS, INC. 618 1001 SOLVED ENGINEERING FUNDAMENTALS PROBLEMS In the ideal Otto cycle, the expansion process is reversible and adia- atic (isentropic). For an isentropic process, AS = 0. ‘The answer is (B). POWER CYCLES-25 ‘The compression ratio of an ideal air Otto cyel is 20°C. What is the temperature at state 2? 1. py is 101.35 kPa, and Ty (A) 159.4°C (B) 332.6°C (©) 600.0°C (D) 600.0K ‘The process from state 1 to state 2 is an isentropic compression, with = 14 for air, e Be ) pi Va, ~ (6) P2= pi(6)** = (101.85 kPa)(6)"4 1245.19 kPa, (i) ” = (6) PROFESSIONAL PUBLICATIONS, INC. POWER CYCLES 6-19 %=T6" = (20°C + 273°)(6)"4 99.97K (600.0K) ‘The answer is (D). POWER CYCLES-26 What is the ideal efficiency of an Otto cycle with a compression ratio of 6:1? ‘The gas used ie air. (A) 0167 B) 0.91 (©) 0.488 (0) 0512 By definition, The answer is (D) POWER CYCLES-27 ‘The eyele shown in the dingram can be described as follows. The process from 1 to 2 is adiabatic, isentropic compression. ‘The process from 2 to 3 is isobaric heat addition. ‘The process from 8 to 4 is adiabatic, isentropic expansion, ‘The process from 4 to 1 is constant volume heat rejection. PROFESSIONAL PUBLICATIONS, INC. 6-20 1001 SOLVED ENGINEERING FUNDAMENTALS PROBLEMS 1 ‘Which of the following is the name of this cycle? (A) Otto (B) Carnot (©) diesel (D) Rankine The cycle described above is a diesel cycle. ‘The answer is (C). POWER CYCLES-28 A device produces 37.5 J per power stroke. There is one power stroke per revolution. Approximate the power output, Pout, if the device is run at 45 rpm. (A) 47 @) uw (©) 28 W (D) 280 W w Poot =F ~ (5 soma) (1 Set) (# "s)he = 28.125 W (28 W) The answer is (C) PROFESSIONAL PUBLICATIONS, INC. POWER CYCLES 621 POWER CYCLES-29 A steam generator produces saturated steam at 700.5 kPa from saturated liquid ‘at 101.35 kPa. If the heat source is a bath at 171°C that provides 1860.8 kJ/kg, which of the following is true? (A) The device violates the first law of thermodynamics only. (B) The device violntes the second law of thermodynamics only. (C) The device violates both the first and second laws of thermodynamics. (D) No thermodynamic laws are violated. For the heat output, AQ = hz kl iW = 2703.5 © — a19.0a 2 kg ie = 2344.5 Kk However, it is given that the heat input is 1860.8 kJ/kg. This would ‘mean that the heat output is greater than the heat input. Therefore, the first law of thermodynamics is violated. = Q As=s-- 7 J 1960.8 or Pa ig =( ets ke eae en) WeCy IRR J ki = 5.4009 tee ~ 41910 = 1.2099 kJ/kg K Since As > 0, the second law of thermodynamics is not violated. ‘The answe a) POWER CYCLES-30 A device that is meant to extract power from waste process steam starts with steam of 75% quality at 700.5 kPa. The exit conditions of the steam are 70% ‘quality at 101.35 kPa, Which of the following statements are true? (A) This device violates the first law of thermodynamics. (B) This device violates the second law of thermodynamics. (C) The device generates positive not power. (D) The device generates no net power. PROFESSIONAL PUBLICATIONS, ING. 6-22 1001 SOLVED ENGINEERING FUNDAMENTALS PROBLEMS W = ~(Ionat ~ hiatal) ha hy +ahyy hy and hyg can be found in the steam tables. KI ky nat = 418.04 + (0.7) (22870 2) = 1998.94 Id /kg i Pinas = 097.34 = + (0.75) (2066.2 kg 1246.99 ki /kg W = hints ~ Anat ki kJ = 2246.00 | — 1998.94 kg kg = 248.05 keg ‘Thus, the device generates positive net power, and the first law of thermodynamics is not violated. As = Steal ~ Snitiat s= sy +2875 ‘8s and sy can be found in the steam tables. 3 kJ kJ na = 1.3069 5 + (02) (S800 Ee) = 5.5405 ki /legK J ks Sat = 1.0025 ET + (0.5) (s 7153 zx) = 55200 kJ /ke-K a 3 as 55405 EE ~ss00 = 0.0115 kI/keK Since As > 0, the second law of thermodynamics is not violated. ‘Therefore, options (A), (B), and (D) are all false. Only option (C) is true, ‘The answer is (C). PROFESSIONAL PUBLICATIONS, INC. POWER CYCLES 6-23 POWER CYCLES-31 ‘An engineer devises a scheme for extracting some power from waste process steam. ‘The engineer claims that the steam enters the device at 700.5 kPa and quality 75%, and the steam exits at 101.35 kPa and 65% quality. Which of the following statements are true? I, The device produces 316 kJ/kg, of work. Il, The device violates tho second law of thermodynamics. IIL, The device violates the first Inw of thermodynamics. (A) only. (B) only (©) I only (D) Tand It hn hy tangy +hy and hy can be found in the steam tables. Ad kJ Ainitias = 697.34 —~ + (0.75) { 2066.2 — ig 11070 (20802) 246.99 keg hana = 419.04 F + (005) (22570 2) kg, = 1886.09 kJ/kg. kd i W = 2246.99 == — 1886.09 — kg ke = 360.9 kI/keg "Thus, the device produces 316 kJ/kg of work without violating the first Inw of thermodynami As = spinal — Sinitiod 857 +284 4 and syq can be found in the steam tables. nt (078) (17188 529 kI/kgK kd Stn = 1.060 gE + (088) (6.0480 be) 2381 kJ/kg-K s kg As = 5.2381 5.529 igK = 0.2909 kJ /kg-K PROFESSIONAL PUBLICATIONS, INC. 6-24 1001 SOLVED ENGINEERING FUNDAMENTALS PROBLEMS Since As <0, the device violates the second law of thermodynamics. ‘Thus, I and II are true, but IIT is false. ‘Phe answer is (D). POWER CYCLES-32 An engine burns a liter of fuel each 12 min. The fuel has a specific gravity of 0.8 and a heating value of 45 MJ/kg. The engine has an efficiency of 25%. What is most nearly the brake horsepower of the engine? (A) 12.5 hp (B) 15.6 hp (C) 16.8 hp (D) 21.0 hp ‘The fuel flow rate is @= (za) (naz) (ie) 39 x 107° m/s For the power input to the engine, ‘n(heating value) = p@q(heating value) 0.8) (2000 3) (2.28010 6 =) ( = 0.0500 MJ/s (50.0 KW) For the power output (actual power), Paw = nPi (0.25)(60.0 sv) ( me ) O76 KW, 16.8 hp PROFESSIONAL PUBLICATIONS, INC. POWER CYCLES 6.25 POWER CYCLES-33 A Carnot refrigerator operates between two reservoirs. One reservoir is at a higher temperature, Ty, and the other is at a cooler temperature, To. What is the coefficient of performance, COP, of the refrigerator? Ta To Te. “To O) gee @m- @i-z © sarroundinos retngeraxor |-—w cold reservoir Te ‘The answer is (D). POWER CYCLES-34 A refrigeration system produces 348.9 kJ/kg of cooling. In order to have a rating of 1 ton of refrigeration, what must be the mass flow rate of the vapor? (1 ton of refrigeration = 12.66 MJ/hr, approximately the rate required to freeze 2000 Ibm of ice in a day-) (A) 0.998 kg/hr (B) 6.800 kg/hr (C) 36.29 kg/hr (D) 163.3 kg/hr PROFESSIONAL PUBLICATIONS, INC. 6-26 1001 SOLVED ENGINEERING FUNDAMENTALS PROBLEMS reftigeration rate = ra(cooling capacity) Therefore, the mass flow rate, rit, is = 36.29 kg/he ‘The answer is (C). POWER CYCLES-35 What is the ideal compression ratio of an Otto cycle that uses air as the gas and has an efficiency of 50%? (A) 43: (B) 5.01 (©) 5.74 6 By defi * k Tore ris the compression ratio. huis = 14 r= (1=nou0)!/-* = (1—05)'/-04 ‘Therefore, the ratio is 5.7:1. ‘The answer is (C). PROFESSIONAL PUBLICATIONS, INC. 7 CHEMISTRY CHEMISTRY-1 ‘The mole is a basic unit of measurement in chemistry. Which of the following is NOT equal to or the same as 1 mol of the substance indicated? (A) 22.4 L of nitrogen (Na) gas at STP (B) 6.02 x 10° oxygen (2) molecules (©) 12 gof carbon atoms (D) 16 ¢ of oxygen (02) molecules ‘Oxygen has a molar mass of 16 g/mol. Therefore, 1 mol of Oz has a mass of 32 g. ‘The answer is (D). CHEMISTRY-2 Which one of the following is standard temperature and pressure (STP)? (A) OK and one atmosphere pressure (B) 0°F and zero pressure (C) 82°F and zero pressure (D) 0°C and one atmosphere pressure By definition, standard temperature and pressure is 0°C and 1 atm, pressure. ‘The answer is (D). PROFESSIONAL PUBLICATIONS, ING. 12 1001 SOLVED ENGINEERING FUNDAMENTALS PROBLEMS CHEMISTRY-3 ‘An ideal gas at 0.60 atm and 87°C occupies 0.450 L. The gas constant is R* = 0.0821 L-atm/mol-K, How many moles are in the samplo? (A) 0.00020 mol (B) 0.0091 mol (C) 0.0120 mol (D) 0.038 mol Use the ideal gas law. pV =nR'T Bw ‘atm (coos Ea ) (src + ara) 0.0091 mol The answer is (B). CHEMISTRY-4 A gas occupies 0.213 L nt STP. How many moles are there in this sample of gos? (A) 0.0089 mol (B) 0.0095 mol (C) 0.089 mol (D) 0.095 mol pV =nRT wv RF At STP, p=tatm T= 273K (2.0 atm)(0.213 atmL) 0.0821 =) (273K) 0.009 50 mol The answer is (B). PROFESSIONAL PUBLICATIONS, INC. CHEMISTRY 18 CHEMISTRY-5 Am ideal gas is contained in a vessel of unknown volume at a pressure of 1 atmo- sphere. The gas is released and allowed to expand into a previously evacuated ‘vessel whose volume is 0.500 L, Once equilibrium has been reached, the temper- ature remains the same while the pressure is recorded as 500 mm of mercury. ‘What is the unknown volume, V, of the first vessel? (A) 0.853 L ®) 0.962 L (©) 1071 (D) 1384 For an ideal gas at a constant temperature, piVi = paVa py = 1.0 atm = 760 mm Hg, (760 mm Hig)V; = (500 mm Hg)(0.5 1 + Vi) Vj = 0.962 L CHEMISTRY-6 What is most nearly the combined volume of 1.0 g of hydrogen gas (Ha) and 10.0 g of helium gas (He) when confined at 20°C and 5 atm? (A) OL (B) 2b (ub (D) 161 Use the ideal gas law. pV =nk°T vy = Beata Pp Mrotal = Nn, + Mate 1 mol He 1 mol He ~coo( 2g ) +000 ( Tg He 3.0 mol (3.0 mol) (0 0821 ant) (293K) atm ‘PROFESSIONAL PUBLICATIONS, INC. 4 1001 SOLVED ENGINEERING FUNDAMENTALS PROBLEMS CHEMISTRY-7 ‘The valve between a 9 L tank containing gas at 5 atm and a 6 L tank containing gas at 10 atm is opened. What is the equilibrium pressure obtained in the two tanks at constant temperature? Assume ideal gas behavior. (A) 5 atm (B) 6 atm (©) 7 atm (D) atm Protsl = Pit D2 Pn = partial pressure of gas For am ideal gas at a constant temperature, PM = PyVe _ Ye n= n(Z r= (Guim (24) = 308m ou p= cam (SE) = Aatm Protat = 3 atm +4 atm ‘The answer is (C). CHEMISTRY-8 A bieycle tire has @ volume of 600 cm’. It is inflated with CO» to a pressure of 5.4 atm at 20°C. Approximately how many grams of CO» are contained in the tire? (A) 38¢ (B) 48¢ (C) 60g (D) 6.45 pV =nR'T W “RT V = (600 em) (wars) =06L PROFESSIONAL PUBLICATIONS, INC. CHEMISTRY 15 293K 20°C + 279° = __ (64 atm)(0.6 1) (ose ae ) (293K) mor = 0.135 mol ‘The molecular weight of CO2, MWco,, is =12-£'+(16 2) @ = 44 g/mol MWe The mae of 003 in het, m= n(htWeo,) = (0385 mob (14 85 =5.94g (6.0g) CHEMISTRY-9 On a hot day, the temperature rises from 13°C early in the morning to 37°C in the afternoon, What is the ratio of the concentration (in mol/L) of helium in a spherical balloon in the afternoon to the concentration of helium in the balloon in the morning? (A) 051 (B) 0.69 (©) 0.92 (p) 14 pV =nRT ‘The concentration, C, is PROFESSIONAL PUBLICATIONS, IN. 76 1001 SOLVED ENGINEERING FUNDAMENTALS PROBLEMS Determine the ratio by dividing the concentration of helium in the balloon in the afternoon, C2, by the concentration of helium in the balloon in the morning, Ci. On CQ Ty _ e427 ~ 37°C + 273" =092 The answer is (C). CHEMISTRY-10 ‘When 0.5 g of a liquid is completely evaporated and collected in a 1 L manome- ter, the pressure is 0.25 atm and the temperature is 27°C. Assuming ideal gos bchavior, what is most nearly the molecular weight? ‘The universal gas constant is RY = 0.0821 L-atm/mol:K. (A) 10 g/mol (B) 12 g/mol (C) 30 g/mol _—(D) 49 g/mol pV =nR'T maT Mw= "> (058) (ose =) (300K) (035 atmy(1.0 Ly = 49.3 g/mol In the preceding equation, MW is molecular weight. The answer is (D). PROFESSIONAL PUBLICATIONS, INC. CHEMISTRY 1 CHEMISTRY-11 200 ml of oxygen gas (Oz) are collected over water at 23°C and a pressure of 1 ‘atm. What volume would the oxygen oceupy dry at 273K and 1 atm? (A) 179 mL (B) 184 mL (©) 190 mL (D) 104 mL ‘At 28°C, the vapor pressure of water is 0.0277 atm. Find the pressure of the oxygen assuming ideal gas behavior. Pratt = past pressures = Po + Paar por Po, = 1.000 atm — 0.0277 atm = 0.9723 atm =P piVi _ poe i Te Ga 0.9723 atm’) (273K ~ (Star) (ix) == = 179 mL ‘The answer is (A) CHEMISTRY-12 8 g of AgaO (solid) are heated to produce oxygen gas (Oz) as follows 2AgiO — 4Ag + Oz ‘The oxygen gas is collected at 35°C over water. The water vapor pressure at 35°C is 0.0555 atm. Given that the barometric prossure is 1 atm, what (wet) volume of O2 is collected? (A) 415 mL (8) 425 mL, (©) 434 mL (D) 455 mb ‘PROFESSIONAL PUBLICATIONS, INC. 78 1001 SOLVED ENGINEERING FUNDAMENTALS PROBLEMS ‘The number of moles of AgzO, ragao, is n weight of substance ino = 1 mol *e ( woo (108 =) + (6) a =a) = 0.084 mot Since 2 mol of Ag,O produce 1 mol of O2, 1 no, = (5) (0.084 mol) = 0017 mol T= 35°C 4.273" 308K Protal ~ Pig 1 atm ~ 0.0555 atm = 0.945 atm pV = nT no RT Yo, = 1 a PO: (0.017 mol) (00801 ood inal O45 atm 0.455 L (455 mL) (308K) ‘The answer is (D). PROFESSIONAL PUBLICATIONS, INC. CHEMISTRY 79 CHEMISTRY-13 AA total of 0.1 g of water is held in a closed container at 40°C. The container holds 500 cm®. ‘The pressure in the container is atmospheric pressure, and the vapor pressure of water at 40°C is 55.3 torr. Most nearly how much water is in liquid form at equilibrium? (A) There is no liquid present. (B) "/2 of the water is liquid. (C) %Js of the water is liquid. (D) 9/4 of the water is liquid. Use the ideal gas law to determine how much of the water is vapor. ‘MW is the molecular weight of the water. py(Mw) T MW =2g416g =18¢ 55.3 torr ftom 700 = 0.073 atm — 500 mL 1000 =051 T = 40°C +273° = 313K (18 2)(0.073 atm)(0.5 L) (0 0821 Ee) (313K) = 0.026 g PROFESSIONAL. PUBLICATIONS, INC. 710 1001 SOLVED ENGINEERING FUNDAMENTALS PROBLEMS ‘Tho romainder of the HzO is liquid. Miquid = Mota ~ Mater = 0.1 8 — 0.026 g = 0074 g fraction that is liquid = “a =074 (3) ‘The answer is (D) CHEMISTRY~14 Which of the following statements is FALSE for an ideal gas? (A) The molecules behave as solid spheres of finite radius. (B) WV=nRT (C) Collisions between gas molecules are perfectly elastic and result in no decrease in kinetic energy. (D) No attractive forces exist between the molecules. ‘The volume of molecules in an ideal gas is not considered. Real gases consist of molecules of finite volume. ‘The answer is (A). PROFESSIONAL PUBLICATIONS, INC CHEMISTRY, TL CHEMISTRY-15 ‘The following statements are made with regard to the boiling point of a liquid. ‘Which statement is FALSE? (A) () ‘A nonvolatile substance having zero vapor pressure in solution (e.g., sugars ot salts) has no true boiling point. ‘The boiling point is the temperature at which the vapor pressure of a liquid ‘equals the applied pressure on the liquid. Combinations of liquids having different boiling points can be separated by slowly raising the temperature to draw off each fraction (.e., by fractional distillation). [At high elevations, water boils at a lower temperature because of a reduc- tion in the surface tension of the water. A liquid boils when its vapor pressure is equal to the pressure of the surroundings. The lower boiling temperature at high elevations is due ‘to the reduced atmospheric pressure, not to a change in the surface tension of a liquid. the answer is (D). CHEMISTRY-16 ‘The critical point for a mixture occurs for which of the following cases? (A) The vapor and liquid exist in a single form, (B) The liquid has no absorbed gas. (C) The vapor phase is stable. (D) The liquid is completely vaporized. ‘The critical point for a mixture occurs when the vapor and t hhave a form that is stable for a “critical temperature and critical pressure.” Tt is both a liquid and a vapor with no boundaries and a uniform composition (a single form). A few substances have a triple point at which a solid, a liquid, and a gas are in equilibrium. The answer is (A). ‘PROFESSIONAL. PUBLICATIONS, INC. 712 1001 SOLVED ENGINEERING FUNDAMENTALS PROBLEMS CHEMISTRY-17 How is “molality” defined? (A) the number of moles of solute in 1000 g of solvent (B) the number of moles of solute in 1 L of solution (C) the number of gram-formula weights of solute per liter (D) the number of gram-equivalent weights of solute in 1 L of solution Molality is defined as the number of moles of solute per 1000 g of solvent. Option (B) is the definition of molarity, option (C) is the definition of formality, and option (D) is the definition of normality. The answer is (A) CHEMISTRY-18 L is a nonvolatile, nonelectrolytic liquid. A solid, 8, is added to L to form a solution that just boils at 1 atm pressure. The vapor pressure of pure L is 850 torr. What is the mole fraction of liquid L in the solution? (A) 64.3% (B) 79.4% (©) 85.7% (D) 89.4% A liquid boils when its vapor pressure equals the pressure of its sur- roundings. Thus, the vapor pressure of the solution is 760 torr. From Raoult’s law, Peotution ™ Protvons (m0l% of solvent) 760 torr = (850 torr)(mol% of L) 760 torr 350 torr = 0.894 (89.4%) mol% of L= PROFESSIONAL PUBLICATIONS, INC. CHEMISTRY 713 CHEMISTRY-19 Which of the following postulates does Bohr’s model of the hydrogen atom in- volve? (A) @B) ©) (D) "The electron in an atom has an infinite range of motion allowed to it When an atom changes from a low energy state to a high energy state, it emits a quantum of radiation whose energy is equal to the differenco in energy betwoon the two states, In any of its encrgy states, the electron moves in a circular orbit about the nucleus. ‘The states of allowed electron motion are those in which the angular momentum of the electron is an integral multiple of f/m. Bohr’s model of the hydrogen atom involves the following postulates. 1, Bach atom has only certain definite stationary states of motion allowed to it. 2. A quantum of energy is emitted when an atom changes from a higher energy state to a lower energy state. 3, The states of allowed electron motion are those in which the an- gular momentum of the electron is an integral multiple of 1/2. ‘Thus, the only choice that is correct is option (C). The answers ©) | CHEMISTRY-20 Which of the following diagrams best depicts the electron configuration of 0 0 TT ” 10 GL ° fi GT ” 10 0 GL PROFESSIONAL PUBLICATIONS, INC. 74 1001 SOLVED ENGINEERING FUNDAMENTALS PROBLEMS Carbon has a total of six electrons. Electrons position themselves in orbitals according to the following rules. 1, There is a maximum of two electrons per orbital. 2, Electrons in the same orbital have different spins (:4#/). 3. Electrons usually fill up empty orbitals before moving into the same orbital as another electron. ‘Thus, option (D) gives the correct electron configuration of carbon. CHEMISTRY-21 ‘Which of the following elements and compounds is reactive in its pure form? (A) sodium (Na) (B) helium (He) (C) carbon dioxide (COs) (D) hydrochloric acid (HCI) Helium is an inert gas and, therefore, is not very reactive. Hydro- chloric acid and carbon dioxide have all of their valence orbitals filled. Thus, they are also not very reactive. Sodium has only one valence lectron that is easly ionizable. Therefore, it is very reactive ‘The answer is (A). CHEMISTRY-22 ‘Two major types of chemical bonds are observed in chemical bonding: ionic and covalent, Which of the following has a bond that is the least ionic in character? (A) Nach (B) CHy (©) (D) 20 ‘The electronegativity difference between two similar atoms is zero. Therefore, the Hz bond is completely covalent. It: has no ionic bond characteristics. ‘The answer ©). PROFESSIONAL PUBLICATIONS, ING. CHEMISTRY 715 CHEMISTRY-23 ‘Which of the following statements is FALSE? (A) It is not possible for bonds between a pair of atoms to be different: (@.g., different bond lengths or bond energies) in different compounds. (B) The bond length for a pair of atoms is the point of lowest energy. (C) The electrostatic repulsion between two nuclei increases as the atoms are brought together. (D) The repulsion between two nuclei increases as their charge increases. It is possible for bonds between a pair of atoms to be different in different compounds. For example, there is more than one type of carbon-carbon bond. [_ The anewer i @. | CHEMISTRY-24 Which of the following statements is FALSE? (A) For a diatomic molecule, the bond dissociation energy is the change in the enthalpy of the reaction when the diatomic molecule is separated into atoms. (B) The average bond energy is the approximate energy required to break a bond in any compound in which it occurs. (C) The energy released when a gascous molecule is formed from its gaseous fatoms can be estimated using average bond energies (D) The change in enthalpy is negative when energy is absorbed in the formar tion of a compound from its elements. ‘The change in enthalpy is negative for the formation of a compound from elements when energy is released in the process. ‘The answer is (D). | CHEMISTRY-25 ‘Which of the following is the correct Lewis structure for sulfur dioxide? 3 3 (A) 4S, 8) AN WAS vs PROFESSIONAL PUBLICATIONS, INC. 716 1001 SOLVED ENGINEERING FUNDAMENTALS PROBLEMS Sulfur and oxygen each have six valence electrons. ‘Thus, there are a total of 18 valence electrons in $Oz. Therefore, there is one single S-O bond and one double $=0 bond. The Lewis structure of sulfur dioxide ia as follows. ‘The answer is (B). CHEMISTRY-26 ‘The molecule methane, CH, is often represented by the followi formula. structural What is the actual geometric shape of the molecule? (A) linear (B) square planar (C) planar, but not 90° bond angles (D) tetrahedral The structure of methane is as follows. 4 x In the tetrahedral structure, bond angles are maximized and repul- sions minimized, with bond angles of 109°. ‘The answer is (D). PROFESSIONAL PUBLICATIONS, ING. CHEMISTRY war CHEMISTRY-27 $0, has a structural formula represented as a resonance hybrid. ‘Which of the following is a true statement: about the meaning of such a structure? (A) One-third of the SOs molecules exists as each of the three structures shown, © bond (B) The true structure is a combination of the three with each $ identical to another. (©). The molecule fluctuates between the three structures. (D) The arrows indicate oq place. brium where an actual chemical reaction is taking. ‘The true structure is a combination with each bond identical, some- ‘where between a single and a double bond. ‘The answer is (B) CHEMISTRY-28 Which of the following chemical equations is incorrect? (A) S+ Fe Fes. (B) Zn$O4 + NagS + Zn$ + NazSOx (C) Hy804 + ZnS + ZnSO, + HaS (D) Zn + O2 + $02 + Zn0 ‘The equation in option (D) does not balance. It needs */202 on the left side. The equation ZnS + 3/:09 ~» $02 + ZnO would be correct. The ans (D). PROFESSIONAL. PUBLICATIONS, INC. 718 1001 SOLVED ENGINEERING FUNDAMENTALS PROBLEMS: CHEMISTRY-29 ‘NazOOs reacts with HCl, but not by the stoichiometry implied in the following unbalanced chemical equation ‘NaOOs + HC] —+ NaCl + H,0 + CO. ‘What is the smallest possible whole-number coefficient for Nays in the bal- anced equation? (aya 2 4 (0) 5 ‘The simplest balanced equation is NazCOs + HCI —+ 2NaCl + #20 + CO2 ‘Tho smallest whole-number coefficient for NagCO, The answer is (A). CHEMISTRY-30 Which of the following is the result of the reaction given’ 5802 +2KMnO, + 2H20 ~—+ ? (A) 2MnS0, + K2801 +2H,S0+ {B) 2MnSO4 + Ky802 +1804 + H20 (C) 2MnSO4 + K2801 + 12804 (D) Mn$O« + 2K2SO« + 21804 Only the products listed in option (A) would balance the elements on the right and left sides of the equation, ‘The answer is (A). PROFESSIONAL PUBLICATIONS, INC. CHEMISTRY 719 CHEMISTRY-31 What is the balanced form of the equation given? 1,0 +P, + OH — PHy + H2POy (A) 40H™ + 4P, +120 — 6H:PO} + 2PHe (B) Py+H20 —+ HzPO; + 3PHs (©) 80H +2P, +210 — HPO; + PH (D) 30H + Py + 3H20 — SHPO, +PHs "The two half reactions are BOHM 4+ Py —+ 4HyPOs + 4B 2B 4 12H,0 + Py —+ APH + 1208- Multiplying the top equation by 3 and adding the two equations together yields 120H™ + 4P4 + 12H20 —+ 12H2PO3 + 4PHs In order to reduce the equation to the lowest whole number coeffi cients, divide by 4 30H- +P, +3H20 —+ 3H»POs + PHy ‘The answre ©). CHEMISTRY~32 Which of the following chemical reactions relates to the softening procedure in ‘water purification? (A) COz + Ca(OH) — CaCOs + #20 (B) Ca(HCOs) + Ca(OH) —+ 2CaCOs + 2420 (C) 21,0 + Oz —> 24202 (D) NaOH + HCl — NaCl + 1,0 Option (B) gives the chemical reaction for adding lime to hard water in order to remove calcium salts. ‘The resulting calcium carbonate precipitate can be removed by sedimentation. PROFESSIONAL PUBLICATIONS, INC. 7-20 1001 SOLVED ENGINS PEERING FUNDAMENTALS PROBLEMS CHEMISTRY-33 ‘A substance is oxidized when which of the following occurs? (A) It turns red, (B) It becomes more negative. (C) It loses electrons. (D) It gives off heat. By definition, a substance is oxidized when it loses electrons. The answer is (C). CHEMISTRY-34 Inorder to assign oxidation states in polyatomic molecules, which of the following rules is followed? (A) The oxidation of all elements in any allotropic form is zero. (B) The oxidation state of oxygen is always ~2. (C) The oxidation state of hydrogen is always +1. (D) All other oxidation states are chosen such that the algebraic sum of the oxidation states for the ion or molecule is zero. Option (B) is false because it does not take into account the peroxides in which the oxidation state of O is ~1. Option (C) is false because it does not account for hydrogen combined with metals, where its oxidation state is ~1. Option (D) is wrong because the sum of the ‘oxidation states should equal the not charge on the ion or molecule Thus, only option (A) is correct. ‘The answer is (A). PROFESSIONAL PUBLICATIONS, INC. CHEMISTRY TAL CHEMISTRY-35 ‘What is the oxidation state of nitrogen in NO§" wt @) +1 © +8 D) 45 ‘The oxidation state of O is ~2, and the net charge on the ion is —1. ‘The oxidation state of nitrogen is given as follows. 3(oxidation state of 0) + (oxidation state of N) = -1 (3)(~2) + (oxidation state of N) = -1 ‘oxidation state of N= +5 "The answer is (D CHEMISTRY-36 ‘What is the oxidation number of Cr in the dichromate ion (Crz07)-?? (ay -1 (B) 0 3 (p) 6 ‘The oxidation number of O is ~2. Therefore, the oxidation number of Or is —14. The charge on the ion is ~2, so the charge on Cra is, 12, Thus, the oxidation number of Cr is 6. ‘The answer is (D). CHEMISTRY-37 Given the following information, determine the oxidation state of nitrogen in nitric acid, HNOs. oxidation state _formula name T HCIO — bypochlorous acid 3 CIO, chlorous acid 5 HCO; chlorie acid 1 HCIO, perchloric acid 3 HINO, nitrous acid (A) 2 3 4 @) 5 PROFESSIONAL PUBLICATIONS, INC. 7-92 1001 SOLVED ENGINEERING FUNDAMENTALS PROBLEMS ‘The outer shell of oxygen is 2 electrons short of being full (inert gases have a full shell). Thus, the oxidation number of axygen is 2 for both ions, By adding another oxygen atom to nitrous acid, the oxidation level is increased by 2. This situation compares directly with that of HCIO2 and HCIO3. ‘Thus, the oxidation state of nitric acid is 5. ‘The answer is (D). CHEMISTRY-38 ‘Which are the oxidizing and reducing agents in the following reaction? 2CCly + KaCrOy —+ 2ClgCO + CrO2Cla + 2KCL reducing agent: chlorine 1& agent: oxygen; reducing agent: chlorine (C) oxidizing agent: chromium; reducing agent: oxygen (D) There are no oxidizing or reducing agents in this reaction. ‘The oxidation state of chromium is 6 in each compound. Carbon remains with a +4 oxidation state throughout the reaction. ‘The oxi- dation states of both chlorine and oxygen remain the same throughout, this reaction. Thus, nothing is axidized or reduced in the reaction ‘The answer is (D). CHEMISTRY-39 A volumettic analysis of a gaseous mixture is as follows. CO, 12% O 4% Nz 82% CO 2% ‘What is the percentage of CO on a mass basis? (A) 0.5% (B) 0.8% (©) 1% (D) 2% PROFESSIONAL PUBLICATIONS, INC. CHEMISTRY 728, name vol. (%6) mole frac. (mol%) _ mol. mass (g) Co, «2 0.12 x 5, O: 4 0.04 x 2 1 Na 82 os x 28 23 co 2 00 20 x38 0.8 30.0 ‘The total mass of the mixturo is 30.08 kg. ‘Thus, the mass percentage of CO is given as follows. mass % of CO = (2%) ‘The answer is (D) CHEMISTRY~40 ‘What is the empirical formula for a compound that has the following composition by mass? element mass % Si 30. ° 8.59 F 612 (A) SIOFs (B) SiO (©) Sin0Fs (D) SisOFs clement mass (9%) mass (g, based on 100 g) moles _ mole (mol%) —s 02 #2 10% m2 ° 8.59 8.59 0537 a F 612 612 3.221 66.6, Find the smallest whole-nunaber ratio of the mole percentage of each element to that of oxygen. 226 _ lite =? 65.5 6 ie lig ig a ole olm ole ‘Therefore, the simplest formula is SizOF 5. ‘The answer PROFESSIONAL PUBLICATIONS, INC. T24 1001 SOLVED ENGINEERING FUNDAMENTALS PROBLEMS CHEMISTRY-41 ‘Phe following equation describes the decomposition of potassium chlorate to produce oxygen gas. 2KCIOs —+ 2KCI (solid) + 302 (gas) Approximately how many grams of KCIOs must be used to produce 4.00 L of Op (gas) measured at 7400 torr and 30°C? (A) 110g (B) 120g (© 1808 (D) 140g HOO to 74 tn 760 BEE atm V=4L T = 30°C +273" = 303K no. of moles KCIO3 “Do. of moles of Op = 2 MOV mol xno, of moles KCIO3 needed = # ) (1.57 mol) = 1.05 mol =990.1 £4955 2. ayeee MWxcio, = 39.1 285 +35.5 + (6 a) (3 mol) = 123 g/mol no. of grams KCIO3 = (1.05 mol) (123 nol. =129g (130g) The answer is (C) PROFESSIONAL PUBLICATIONS, ING. CHEMISTRY 725 CHEMISTRY-42 Determine which of the statements is true, given the following facts. 1. A401 sample of Hz (gas) at 10°C and 740 torr is added to a 75 L sample of Oz (gas) at 20°C and 730 torr. 2. The mixture is ignited to produce water. (A) There is an excess of O2 greater than 0.2 mol. (B) There is an excess of Hz greater than 0.2 mol. (C) There is H,0 only. (D) There is an excess of Hy less than 0.2 mol. ( 0 « a ‘The stoi metric equation is H+ 402 + #20 ‘The number of moles of each gas initially present is pv mm = Be 10.0.) 491) 2 169 OE (10°c + 273°) (00801 = mol =1.7 mot wv 20. = FF 1e0 tar) 51) = 700 sem 0 mol For each mole of H,O formed, 0.5 mol of Og and 1 mol of Hy are required. The oxygen necessary to completely react with 1.7 mol of Ha is given by L7 mol ‘PROFESSIONAL PUBLICATIONS, INC. 7-26 1001 SOLVED ENGINEERING FUNDAMENTALS PROBLEMS Therefore, there is an excess of O2. ‘The amount of O2 extra is 3.0 mol — 0.85 mol = 2.15 mol ‘The answer is (A). CHEMISTRY-43 1f 2.25 g of pure calcium metal are converted to 3.13 g of pure CaO, what is the atomic weight of calcium? The atomic weight of oxygen is 16 g/mol. (A) 8 g/mol (B) 33 g/mol (C) 87 g/mol (D) 41 g/mol ‘The stoichiometric equation is Ca +0 —+ Cad One mol of oxygen and 1 mol of calcium are required to make 1 mol of Ca. 3.13. g~ 2.25 g z 6S ‘mol = 0.055 mol Rex = 0.055 mol . 2.25 & atomic weight of Ca 2.25 g 0.055 mol =41 g/mol atomic weight of Ca Tho answer is (D). PROFESSIONAL PUBLICATIONS, INC. CHEMISTRY TOT CHEMISTRY-44 Methane, CHs, burns to form CO, and H,0 according to the equation CHy +202 —+ COn +2020 How many grams of CO» will theoretically be formed when a mixture of 50 g of, CH, and 100 g of Op is ignited? (A) 34g. (B) 68g (C) 698 @) 2g nw MW In the preceding equation, m is the mass of compound and MW is the molecular weight of compound now, = Since 1 mol of CH, and 2 mol of Oz are needed for each mole of CO2 formed, Oo is the limiting reactant. no. of moles COz formed n0. of moles Oe ignited neoy = (8:18 wa (428) mol/2 mol = 1.563 mol meo, = co,MWeo, = (1.509 mot (48 a nol 9g Tho answer is (C). PROFESSIONAL PUBLICATIONS, INC. 7-28 1001 SOLVED ENGINEERING FUNDAMENTALS PROBLEMS CHEMISTRY-45 Determine the mole percent of CO, in the produets of combustion of CsHis ‘when 200% theoretical air is used. (A) 5.5% (B) 6.5% (©) 75% (D) 8.5% ‘The formula for theoretical air is O2 +3.76N2. For 200% theoretical air, the stoichiometric equation is CeHig + 25(Op + 3.76N2) —> 8COg + 9Hg0 + 12.502 +94Np ‘The mole percent of COz is given by the ratio of the number of moles of COs formed to the total number of moles formed. 8 mal Tinol +9 mol +12.5 mol +94 mol = 6.5% WOO2 The answer is (B). CHEMISTRY-46 Approximately what volume of O2 at 298K and 1 atm is required for complete combustion of 10 L of CaH (gas) at 500K and 1 atm? The combustion equa- tion is 702 + 2C2H5 —+ 6120 + 4CO2 (A) 16L (B) 19 (© aL (D) 2 Assume ideal gas behavior. ame Note = Pep (1.0 atm)(10 1) (0 san at) (600K) mol K = 0.24 mol = 7 mol/2 mol Tose i 7 mol Nos ( 5 ae) (0.24 mol) = 0.84 mol PROFESSIONAL PUBLICATIONS, INC. CHEMISTRY 729 ‘The volume of 1 mol of ideal gas at STP (standard temperature and pressure) is 22.4 L. Therefore, the volume of O required at 208K is, vs Yox= no, (Fatt) Yorr Vaosx _ Taos Ver ~ Tore 208K ~ 3K = 109 Vou = (0.84 mol) (1.09) (wa +) inal 206% CHEMISTRY~47 ‘One gram of gas made up of carbon and hydrogen is ignited in excess oxygen to produce 3.30 g of COp and 1.125 g of H20. What is the empirical formula of the ‘compound (a) CH (B) CHy (C) CoH (D) CoH ‘The stoichiometric equation is Cally + (2+ 4) 0p — 2602 + $20 MWoo, = 44 g/mol MWy,0 = 18 g/mol = moles of C = moles of COs 335 aes 4 ol = 0.0750 mot y= moles of H = 2(moles of HyO) 730 1001 SOLVED ENGINEERING FUNDAMENTALS PROBLEMS 2 _ Catoms 7 Watoms inoles of © ~~ moles of H 0.075 mol ~ 0.125 mol = 3/5 ‘Thus, the empirical formula of the gas is C3Hs. ‘The answer is (D). CHEMISTRY-48 Complete combustion of 13.02 g of a compound (CzHy) produces 40.94 g CO and 16.72 g of HgO. Determine the empirical formula of the compound. (A) CH (B) CHy (©) CHy (D) CHO ‘The stoichiometric equation for combustion is Cally + (0+ 2) 02 — C02 + $1120 "aw In the preceding equation, m is the mass of compound and MW is, the molecular weight. 40.94 NCO. = Ee aa mol = 0.93 mol = ne Therefore, PROFESSIONAL PUBLICATIONS, INC. CHEMISTRY 731 ny = 1.86 mol ‘Therefore, the empirical formula for the compound is CH CHEMISTRY-49 ‘When 0.01 mol of a substance consisting of O, H, and C is burned, the following products are obtained. 1. 896 cm* of CO» et standard temperature and pressure (STP) 2. 0.72 g of water It is also found that the ratio of oxygen mass to the mass of H plus C in the substance is “/r. What is the chemical formula of the substance? 1 mol of CO has a volume of 22400 em? at STP. (A) CHO, (B) CyHeO2 (©) CH20, (D) Cytig02 ‘The stoichiometric equation is yz y CeH1,0, + (2+ 4 ~ 5) O2 + 2002 + FH20 woV i (ore noe (im) ») (_10 mol ~(10¢ en) (Sapo) = 0.04 mol amo 2 TiO 10 = Se _ ong a ae we = 0.04 mol = z ny = 0.08 mol ‘PROFESSIONAL PUBLICATIONS, INC. 7-32 1001 SOLVED ENGINEERING FUNDAMENTALS PROBLEMS ‘Thus, there are 0.04 mol C and 0.08 mol Hin 0.01 mol of the substance CxH03. For 1 mol of CpH,Ox, there are x = 0.04/0.01 = 4 mol of Cand y = 0.08/0.01 = 8 mol of H. 5 massofO (16 8) ( mon mass of H+ mass fC ~ man (i Z) + am (aE (6 mo) (1 5) + (4 mon (12 2) =4/7 16 Baan 2=2mol ‘Thus, the formula is CalHyOa. Ee CHEMISTRY-50 (D). ‘What is most nearly the melting point of sodium chloride, given that the heat of melting is 30 kJ/mol, and the associated entropy change is 28 J/mol-K? (A) 870K (B) 880k (©) 990K (D) 1100K For the phase change, AG = AH -TpAS =0 AH T= BS so000 2 Sl 8 noFK = 1071K (1100K) ‘The answer is (D). PROFESSIONAL PUBLICATIONS, INC. CHEMISTRY 7-33 CHEMISTRY-51 ‘The temperature of 100 g of liquid water at 0°C is raised by 1°C. The number of calories consumed is most nearly (A) 12 cal (B) 4.2 cal (C) 99 cal (D) 100 cal By definition, 1 cal is the energy needed to heat 1 g of liquid water by 1°C. Therefore, the heat needed to heat: 100 g of water by 1°C is a= me,AT = (100 ¢) (1 ) (°C - 0°C) = 100 cal CHEMISTRY-52 Ice with a volume of 50 em? and at @ temperature of 0°C is added to 100 g of water at 20°C. Assume that there is no spurious heat loss. The density of ice is 0.92 g/em®, and the heat of fusion of ice is 144 keal/mol at 0°C. Approximately hhow much ice is left uamelted after the mixture reaches thermal equilibrium? (A) 13 em? (B) 19 em? (©) 23 em> (D) 39 em? ‘The ice will melt until the temperature of the water reaches 0°C. The heat necessary to lower the water temperature from 20°C to 0°C is q=me,AT = (100 «) (0 =) (20°C) = 2000 eal ‘The heat necessary to melt all the ice is iy MW a= Voy (0.99 85) (0 ont) (1440 £ = z we 3680 cal PROFESSIONAL PUBLICATIONS, INC. 34 1001 SOLVED ENGINEERING FUNDAMENTALS PROBLEMS The amount; of unmelted ice is 2am c= a8 ol) ¥ = 60 on ( Saar = 22.83 cm? (23 em®) The answer is (C). CHEMISTRY-53 ‘What is most nearly the final temperature when 10 g of copper and 20 g of lead fat -100°C are added to 50 g of HzO at 50°C? Disregard spurious hent losses. ‘The atomic weight of copper is 63.55 g/mol, and the specific heat of lead is 0.032 cal/g°C (0.134 J/g:°C). (A) 38°C (B) 38°C (©) 38° (D) 45°C ‘The law of Dulong and Petit is (atomic weight) [in g/mol x (specific heat) fin eal/g-*C} = (« 4 cool a) (« oa a) = 26.8 J/mol°C Since there are no spurious heat losses, the heat; loss by the water equals the heat gained by the copper and lead, meyAT PROFESSIONAL PUBLICATIONS, INC. CHEMISTRY 7-35 Tn the preceding equation, m is the mass (in grams), cp is the specific heat capacity, and AT is the change in temperature. ooa( x (Ty — (-100°C)) Ty = 45.21°C (45°C) ‘The answer is (D). CHEMISTRY-54 A bomb calorimeter is used to determine thermal properties. What is most nearly the enthalpy of reaction (in kcal/mol) of the combustion of glucose (molecular ‘weight = 180 g/mol) when 2.22 g of glucose are ignited, and the water in the ‘well-insulated calorimeter rises in temperature from 18.00°C to 23.19°C? Assume that the water absorbs all of the heat given off ineulation 2229 lucos 1200 g water ‘bomb calorimeter (A) 100 keal/mol (B) 820 keal/mol (C) 510 keal/mol (D) 730 keal/mol PROFESSIONAL PUBLICATIONS, INC 7-36 1001 SOLVED ENGINEERING FUNDAMENTALS PROBLEMS me, AP a0 = (1200 g) ( =) (23.19°C — 18.00°C) = 6228 cal Adtacne = 6228 cal nw 2.22 g 180 = inal 0123 mol 6228 cal = 0125 mol = 506.34 kcal/mol (610 keal/mol ‘The answer is (C). CHEMISTRY-55 ‘What is most nearly the standard heat of reaction, AH”, per mole of CgHg for the following reaction? Metucose molar enthalpy CcHe(g) — 3C2TI2(9) ‘The enthalpy of reaction for CaHz is 226757 J/emot; for CgHy, it is 82923 JYemol. (A) -650k —(B) -000k~——(C) 600 KJ (D) 650 kg Nogtte = 1 mol catty = 3 mol = onde 7 = 82023 J/mol (82.9 kJ) p= Dnt | “antes = 680300 J (680.3 ki) PROFESSIONAL PUBLICATIONS, INC. CHEMISTRY 7-37 ARH, 680.3 kJ — 82.9 kJ 507.4 kJ (600 kJ) The answer is (C). CHEMISTRY-56 The heats of reaction for three equations are as follows. 1, ~20gHz ~ 502 +4002 + 2820 620000 cal 0. = 02+ CO2 -¢ — 96960 cal Mm. — 02 +2Hg0-2Hz0 — 2H = -136800 cal What is the heat of formation of C2H,? (A) 4.14 kcal/mol (B) 45.7 keal/mol (©) 47-7 keal/mol (D) 95.7 eal /mot ‘Adding (Eq. 1) + 4(Eq. It) + (Eq. Ill) gives the formation of 2 mol of Cai 20 H+502~4C02—2H,0 = 620000 cal 40344002 4c —387 840 cal =O 42,0 ~2Hz = ~136800 cal ‘Therefore, 202Ha — AC — 212 = 95360 cal Because Hz and C are at the standard reference state, fy hay, =0. Therefore, 0, and host, = 95360 cal hieaty = 47680 cal/mol (47.7 keal/mol) ‘The answer is (C) ‘PROFESSIONAL PUBLICATIONS, INC. 7-38 1001 SOLVED ENGINEERING FUNDAMENTALS PROBLEMS CHEMISTRY-57 A chemical reaction involving the collision of two molecules of A and B goes ‘through the following energy profile, Ag +By— 2AB potential ‘energy | Toaction coordinate ‘The energy, E, shown on the diagram represents which of the following? (A) entropy of reaction (B) enthalpy of reaction (©) forward activation energy (D) reverse activation energy forward activation energy PROFESSIONAL. PUBLICATIONS, INC, CHEMISTRY 7-39 Bis the energy required for the reverse reaction (A~A + B-B — A-B + A-B) to proceed. Thus, £ is the reverse activation energy. ‘The answer is (D). CHEMISTRY-58 Reactions generally proceed faster at higher temperatures because of which of the following? (A) ‘The molecules collide more frequently. (B) The activation energy is less. (C) The molecules aro loss energetic. (D) Both options (A) and (B). At higher temperatures, the molecules travel faster and, therefore, hhave a higher kinetic energy. This means that the molecules will collide more frequently and that the activation energy for « chemical reaction will be smaller. The answer is (D). CHEMISTRY-59 Which of the following statements is FALSE? (A) In general, as reaction products are formed, they react with each other and re-form reactants (B) ‘The net rate at which a reaction proceeds from left to right is equal to the forward rate minus the reverse rate. (©) At equilibrium, the net reaction rate is zero. (D) The differential rate law is the mathematical expression that shows how the rate of a reaction depends on volume. ‘The differential rate law is the mathematical expression that shows how the rate of a reaction depends on coucentration, not, volume. ‘The answer is (D) PROFESSIONAL PUBLICATIONS, INC. 7-40 1001 SOLVED ENGINEERING FUNDAMENTALS PROBLEMS CHEMISTRY-60 For the reaction 3A +2B —+ C + D, the differential rate law is (3) #-G=saren Which of the following statements is FALSE? (A) The order of the reaction with respect to A is called n. (B) The sum of n-+m is callod the overall order of the reaction. (C) The exponents of [A] and BJ, n and m, are not necessarily equal to the stoichiometric coefficients of A and B in the net reaction. (D) The overall order for the reaction can be predicted by or deduced from the ‘equation for the reaction ‘The order for the reaction must. be found experimentally and cannot, be determined from the equation for the reaction. ‘The answer is (D). CHEMISTRY-61 Which of the following statements is FALSE? (A) When the temperature is raised, the rate of any reaction is always increased. (B) In general, when any two compounds are unmixed, a large number of re- actions may be possible, but those which proceed the fastest are the ones observed. (©) tis possible to influence the products of a chemical change by controlling the factors which affect reaction rates. (D) Heterogeneous reactions are the reactions that take place at the boundary surface between two faces When temperature is increased, the rates of most reactions increase. However, the rates of some reactions do decrease. ‘The answer is (A). PROFESSIONAL PUBLICATIONS, INC. CHEMISTRY 741 CHEMISTRY-62 ‘The following rate expression was found to accurately represent the kinetics of ‘a chemical reaction. kORCp IEC represents concentration in units of mol/L, what are the units of the rate constant, k? (A) unitless @B)s (©) L/mols ——(D) 1?/mol?.s ‘The reaction rate always has units of mol/L-s. The units of & may be found as follows. mo(e) (2) = (T) a) B The answer CHEMISTRY-63 Let C represent the concentration of a reagent. For a first-order reaction, what would a plot of In@ versus ¢ yield? (A) a straight line whose slope is —f (B) a straight line whose slope is (C) a logarithmic curve approaching a valuo of k (D) an exponential curve approaching a value of & For a first-order reaction, PROFESSIONAL PUBLICATIONS, INC. 7-42 1001 SOLVED ENGINEERING FUNDAMENTALS PROBLEMS: In the preceding equation, k is the rate constant and c is the concen- ‘en L8-f+ ie kt ine = it C InC = -kt+InCp ‘This is of the form y = ax + yo. Therefore, the graph is a straight line with a slope of —k. ‘The answer is (A) CHEMISTRY-64 ‘The following kinetic data were collected for a specific chemical reaction. What is the rate constant for the reaction? reaction: A +B— experiment Cx (mol/L) Cp (mol/L) _ initial rate (mol A/L-s) T 0.10 0.10 ‘0.0010 2 0.20 0.10 0.0020 3 0.30 0.10 0.0030 4 0.10 0.20 0.0010 5 0.10 0.30 0.0010 (A) 001s (B) 0.02 =? (©) 0.02 L/mol-s (D) 0.03 L/mots First, determine the rate lnw. Experiments 4 and 5 show that the rate is not a function of cp. Experiments 1, 2, and 3 show that the rate is directly proportional to ca. Therefore, PROFESSIONAL. PUBLICATIONS, INC. CHEMISTRY 7-43 ‘Use the data from experiment 1 to determine k. ‘mol 0.0010 i 0.10 =001 4 ‘The answer is (A). CHEMISTRY-65 ‘A certain temperature-dependent reaction proceeds 10 times faster at 500K than it does at 300K, Approximately how much faster will it react at 1000K than it does at 300K? (a) 10 (@) 20 © 99 () 60 r=kf(C) In the preceding equation, Ey is the activation energy. ‘Therefore, "0 oxo (are (se ~ zane) a0 OP 300K ~ T000K, 56 (60 ‘The answer is (D) | PROFESSIONAL PUBLICATIONS, INC. Ad 1001 SOLVED ENGINEERING FUNDAMENTALS PROBLEMS CHEMISTRY-66 A reaction rate is observed to triple as the result of raising the temperature from 0°C to 20°C. What is most nearly the activation energy of the reaction? (R* is ‘the universal gas constant.) (A) 3900R* (B) 4400R* (©) 470k" (D) s100R* In the preceding equation, C is a constant, and E'4 is the activation energy. A Ba)\ (2-2 Bo ((#)) (eon 1 Ba 1 1 37 (8? ae )) crane ~ aC are. inh -—Z4 (1 1 "3 ~~ Re (273K ~ 208K, R ind {i PK ~ 203K 4394R" (4400R*) ‘The answer is (B). CHEMISTRY-67 In the troposphere, ozone is produced during the day and consumed during the night. Determine the half-life of ozone if itis depleted to 10% of its initial value after 10 h of darkness. (A) 30h (B) 35h (©) 40n (0) 45h C= Oe Cin = 0.10 Ot = en 10* Indt 10 = 0.2303 b>! PROFESSIONAL PUBLICATIONS, INC. CHEMISTRY 745; In the preceding equation, t12 is the half-life. Caja = 0.500 05 = 70280882 m0. a= Sp.3308 B= =30h ‘The answer is (A). CHEMISTRY-68 ‘Which of the following statements is FALSE? (A) In considering chemical equilibrium, the relative stabilities of products and reactants are important. (B) In considering chemical equil the final state is important. (C) In treating reaction rates, the rate at which reactants are converted to products is important, (D) In treating reaction rates, the sequence of physical processes by which re- actants are converted to products is important, jum, the pathway from the initial state to Considerations of chemical equilibrium do not take into account the pathway from initial to final states The anover is) | CHEMISTRY-69 Consider the following reaction at equilibrium. ‘3H (gas) + Ng (gas) = 2NHs (gas) AH = -11.0 kcal/mol Which single change in conditions will cause a shift in equilibrium toward an increase in production of NH? (A) removal of hydrogen gas (B) increase in temperature (C) increase in volume of the system (D) increase in pressure on the system PROFESSIONAL PUBLICATIONS, INC. 7-46 1001 SOLVED ENGINEERING FUNDAMENTALS PROBLEMS According to Le Chételier’s principle, the effects of each change in condition are as follows. removal of hydrogen _shifts equilibrium to the reactants increase in temperature shifts equilibruim to the reactants increase in volumne shifts equilibrium to the reactants increase in pressure shifts equilibrium to the products CHEMISTRY-70 Consider the following reaction at equilibrium 4No+ $f SNHy AM = -11.0 kcal/mol ‘What would be the expected effect on the amount of NH3 produced under each of the following conditions? I. raise the temperature IL, compress the mixture TI, add additional Hp i decrease, II: increase, III: increase increase, II: increase, III: decrease increase, I: decrease, III: decrease : decrease, II: increase, III: decrease SSE According to Le Chatelier’s principle, each change has the following, effects. L. raise the temperature: shifts equilibri reaction is exothermic IL. compress the mixture: shifts equilibrium to the products because prod- ucts contain a smaller number of moles IIL. add hydrogen gas: shifts equilibrium to the products because adding addtional reactants will force the formation of more products m to the reactants, because the ‘Tho answor is (A). PROFESSIONAL PUBLICATIONS, INC. CHEMISTRY 747 CHEMISTRY-71 Consider the following reaction. Mg804 (solid) + MgO (solid) + 80s (gas) ‘What isthe equilibrium constant for th given reaction? [M803 ___[MeS04) 7Migs0,) ©) *™ Igo| S03] (©) E=MgO|803]@) Solids have a concentration of 1. ‘Therefore, k = [SOs] (A) k= = [80s] The answer (D). CHEMISTRY-72 ‘The solubility of barium sulfate, BaSO,, is 0.0001 g/L at 25°C. The molecular weight of barium sulfate is 233 g/smol. What is the value of the solubility product constant: kyp for BaSO,? (A) 1.52 x 10-9 mol?/L? (B) 4.24 x 10-* mol?/L? (©) 8.63 x 10-* mol?/L? (D) 2.98 x 10°® mol*/L? BaSO«(s) * Ba* (ag) + SOF (ag) yp = [Ba*|[S03-] 18980, = (0.0001 8) (=) .9 x 10° mol/L {Ba804) : ‘3.9 x 10-8 mol’ (sono 2) = 1.52 x 10°? mol/L? [pa (so}- Kap The answer (A). ‘PROFESSIONAL PUBLICATIONS, INC. 7-48 1001 SOLVED ENGINEERING FUNDAMENTALS PROBLEMS CHEMISTRY-73 Consider the reaction shown, Ha (gas) +z (gas) = 2H (gas) With keg = 25, determine the number of moles of H remaining when 1 mol each of Hg and Ip reach equilibrium in a 1 L vessel (A) 1/6 mot (B) 2/7 mot (© 5/7 mol (D) 5/6 mot Ha (gas) + Ia (gas) = _ HIT (gas) 0 2 initial moles 1 final moles = 1x 4x? m (25)(1 = 22 + 2%) 21a? — 502 +25 =0 _ +50 VGO = HAL) - (@)@1) = 5/8 mol or 5/7 mol Only the second value for x makes sense, because the first value is ‘eater than the initial amounts of Hy and Iz. ‘Thus, the remaining umber of moles of H at equilibrium is 1 ~ °/+ = %/ mol. The answer is (B). PROFESSIONAL PUBLICATIONS, INC. CHEMISTRY 749 CHEMISTRY-74 Consider the reaction. A+BS20 With keg = 50, what is most nearly the final concentration of C when 1 mol of both A’and B are added to a 1 L container containing 0.1 mol of C? (A) 0.77 mol (B) 0.95 mol (C) 1.5 mol (D) 1.6 mol A + Bos 2% (mol) (mol) (mol) initial moles 1 1 01 final moles 1-z ar t04 mol]? ae al ~ ‘mol ‘mol [a =] fp ay (2240.1 mol)? ~ Wimol = 2)(7 mol 4x? + 0.4 + 0.01 = 502? — 1002 + 50 462? ~ 100.42 + 49.99 = 0 0.7685 mol or 1.4141 mol However, x cannot be 1.4141 mol because this is greater than the initial amounts of A and B, and this value would make 1 —= negative. Therefore, x = 0.7685 mol. The final number of moles of C is 22 = (2)(0.7686 mol) = 1.537 mol. 20 +0. (2)(0.7685 mol) +0.1 = 1.637 mol (1.6 mol) ‘The answer is (D). PROFESSIONAL PUBLICATIONS, INC. 7-50 1001 SOLVED ENGINEERING FUNDAMENTALS PROBLEMS CHEMISTRY-75 ‘The voltage of a galvanic cell does NOT depend on which of the following pa rameters? (A) concentration of solutions (B) temperature of solutions (C) pressure of sohutions (D) volume of solutions ‘The cell potential is dependent on all of the above except volume. ‘The answer is (D). CHEMISTRY-76 Given the electrochemical cell shown, what is the reaction at the anode? he (A) Cu —+ Cu* + 2e~ (B) Cu? +26 — Cu (©) Zn — Zn + 20° (D) Zn%* + 2e- — Zn Zine has a higher potential and will, therefore, act as the anode, By definition, the anode is where electrons are lost. Thus, the reaction at the anode of the electrochemical cell is Zn —+ Zn?* + 20. ‘Tho answer is (C). PROFESSIONAL. PUBLICATIONS, INC. (CHEMISTRY 751 CHEMISTRY-77 Which of the following statements regarding a galvanic cell is FALSE? (a) (8) © (D) ‘A negative value of cell voltege AE means that the reaction in the cell proceeds spontaneously from right to left. If the standard potential of a coll is zero, a concentration difference alone sufficient to generate a voltage. When a current I flows through the voltage difference AE for a time, ¢, the ‘electrical work performed is (AE)It. ‘The cell voltage, AE, is totally independent of the number of electrons transferred in a given reaction. For tho reaction aA + 0B —+ cC + dD, the Nernst equation states, (ce Ae = AE? - ix ‘Here, n is the number of moles of electrons transferred in the reaction. Therefore, the cell voltage does depend on the number of electrons transferred in a given reaction. The answer is(D). | CHEMISTRY-78 Consider the Nernst equation. _ 0.059, (C}*(DJ* Ae = Ae? bos gs Which of the following statements is FALSE? (A) nis the number of moles of electrons transferred in the reaction. (B) The cell must be operating at a temperature of 25°C. (C) The equation holds for the reaction aA + 9B —+ cC +4D. (D) The factor of 0.059 is common to all cells, regardless of temperature. ‘The factor of 0.059 applies only to cells with an operating temperature of 25°C. ‘The answer is (D). PROFESSIONAL PUBLICATIONS, INC. 7-52 1001 SOLVED ENGINEERING FUNDAMENTALS PROBLEMS CHEMISTRY-79 A zinc-copper standard cell with AE® = 1.10 V is connected to an independent variable voltage supply such that the variable voltage opposes the cell voltage. Given the following reaction, what happens? %n+Cur Cut Zn AE? =1.10V (A) When the variable voltage is below 1.10 V, the cell reaction Cu + Zn?* —+ Cu + Zn predominates. (B) When the variable voltage is above 1.10 V, the cell reaction Cu + Zn? + Cu* + Zn predominates (C) When the variable voltage is above 1.10 V, the cell reaction Zn + Cue — Cu + Zn?* predominates, (D) When the variable voltage is equal to 1.10 V, the cell reaction Cu + ‘Zn?* — Cu? + Zn predominates. When the variable voltage is below 1.10 V, the reaction Zn + Qu?*—+ Cu + 2n?* predominates. When it is equal to 1.10 V, no net reac- tion occurs. When the variable voltage is above 1.10 V, the reverse reaction, Cu + Zn?* —+ Zn + Cu predominates. ‘The answer is (B). CHEMISTRY-80 Given that A€° = 0.08 V, [Ni] = 1 M, [Co] = 0.1 M, and T = 25°C, calculate the cell voltage for the following equation. Cot Ni — CoM + Ni (A) 001 V (B) 0.03 V (© 0.06 v (0) 0.09 v Use the Nernst equation. AE = Ae? — 2052 jog (Com) nS NBT ne2 059 V |, Oa los 5 = 0.08 V +0.03 V 0.06 v PROFESSIONAL PUBLICATIONS, INC. CHEMISTRY 753 CHEMISTRY-81 In organic chemistry, which compound families are associated with the following bonds? L a m1. (A) E:allene, TI: alkyne, 11 allane (B) I: alkyne, TI: alkane, IT: alkene (C) I: alkane, II: alkene, 111: alkyne (D) E: alkane, II: alkyne, IT: alkene ‘An allane is a saturated organic compound. Thus, the carbons may ‘only have single bonds. In an alkene, the carbon atoms may have double bonds. In alkynes, the carbon atoms may have triple bonds. ‘The answer is (C). CHEMISTRY-82 ‘Which one of the following statements regarding organic substances is FALSE? (A) Organic matter is generally stable at very high temperatures. (B) All organic matter contains carbon, (©) Organic substances generally do not dissolve in water. (D) Organic substances generally dissolve in high-concentration eci Organic matter contains carbon, is generally insoluble in water, solu- ble in high-concentration acids, not easily ionizable, and unstable at high temperatures PROFESSIONAL. PUBLICATIONS, INC. 154 1001 SOLVED ENGINEERING FUNDAMENTALS PROBLEMS CHEMISTRY-83 ‘Which one of the following is most likely to prove that a substance is inorganic? (A) @B) ) () ‘The substance is heated together with copper oxide and the resulting gases are found to have no effect on limestone. ‘The substance evaporates in room temperature and pressure. Analysis shows that the substance contains hydrogen. ‘The substance floats in water. ‘The carbon from organic matter generally reacts with copper oxide to produce carbon dixoide. This gas darkens limestone. CHEMISTRY-84 Which of the following organic chemicals is most soluble in water? (A) (B) (©) @) CHsCHs CH,OH CCl (CHg-(CHa)n-CHs Water is a polar molecule. Thus, a polar substance is more likely to dissolve in water than a nonpolar substance. Methanol (CHyOH) is polar and, therefore, very miscible in water. All ofthe other molecules are nonpolar. ‘The answer is (B). PROFESSIONAL PUBLICATIONS, INC. CHEMISTRY 7-55 CHEMISTRY-85 Which statement describes all of the following three chemical structural formulas? HOH OW 1otit 4—c—c—c—cHo a) HoH OH HoH OH ae H—E—F—E— eH HOW H iit Ot Fe boo HoH (A) They are isotopes of a certain substance. (B) They are the only possible forms of C_H202. (C) They are incorrectly written. (D) ‘They are isomers. ‘When a compound has one chemical formula, but different possible physical structures, the different structures are called isomers. The three formulas are all possible structures of CaHsOz. Therefore, they are isomers. CHEMISTRY-86 What structures do both aldehydes and ketones contai (A) the carboxyl group (B) the carbony! group (C) the hydroxyl group —oH Noe” (D) the carbon-carbon double bond °C. PROFESSIONAL. PUBLICATIONS, INC. 7-56 1001 SOLVED ENGINEERING FUNDAMENTALS PROBLEMS Aldehydes and ketones both contain the carbonyl group. ‘The answer is (B). CHEMISTRY-87 Identify the following acid structures. " ated TN i L ia Mu, (A) I: formic acid, Il: oxalic acid, IIT: acetic acid (B) E oxalic acid, II: acetic acid, II: formic acid (C) E: acetic acid, I: formic acid, IIT: oxalic acid (D) I: formic acid, Il: acetic acid, TI: oxalie acid HCOOH is formic acid. CHyCOOH is acetic acid. CzH,04 is oxalic acid. The answer is (D).. CHEMISTRY-88 According to the Bohr model of the hydrogen atom, which of the following statements are true? I. As the electron orbits the proton, it constantly radiates light with a frequency equal to its frequency of revolution. IL. The electron orbits the proton in cortain orbits that can be found by assuming that its angular momentum is quantized. TI. Because of the quantization of angular momentum, calculations using the Bohr model and those based on classical physics can never give the same results. IV. When an electron orbiting a proton changes states to a lower energy level, the frequency of the radiation given off is proportional to the change in energy. This accounts for the hydrogen spectrum. (A) Tend It (B) WandTV — (C) LU, and HT (D) U1, 111, and IV PROFESSIONAL PUBLICATIONS, INC. CHEMISTRY 787 ‘The Bobr postulates are 1. The electron moves in a certain orbit without radiating, 2, The frequency of the emitted photon is proportional to the change in energy of the electron. 3. The correspondence principle states that, in the limit as energies, and orbits become large, quantum calculations must agree with classical calculations. ‘Thus, only I and IV are true. ‘The answer is (B). CHEMISTRY-89 In a three-level laser, electrons in atoms are excited into an energy state, Fs, then decay spontancously to an energy Ea, which is a metastable state. The atoms are struck by photons of a specific frequency, and make stimulated emissions to the ground state, By. If the photons cause all of the emissions to be of the same froquency, the light will be amplified. What frequency must the photons be for this to occur? (a) f= @ 1-3 © = ee () p= Fe ‘The tronsition that must be amplified is the Ey to E; transition. ‘Thus, the frequency of the radiated light is (E> — E1)/R. Photons of this frequency will be more likely to cause this transition, Therefore, if photons of this frequency are used, more transitions will take place, and the light will be amplified. The anwer (A). | PROFESSIONAL PUBLICATIONS, INC. 7-58 1001 SOLVED ENGINEERING FUNDAMENTALS PROBLEMS CHEMISTRY-90 I In an atom such as sodium, there is one electron in the outermost shell (in this ase, n= 3). Which of the following statements is true regarding the energy required to excite an electron in the n = 1 shell compared to that required to excite an electron in the n = 2 shell? G ‘A) It is greater because the electron is closer to the proton, and thus the Coulomb attractive force is much stronger. (B) It is greater because the shell next to it is full. Thus, by the Pauli exclusion principle, it must jump to the first shell that is not full, in this case, the n=3 shell. (C) It is greater because an electron must first jump to the n = 3 shell from the n= 2 shell. Then the electron from the n = 1 shell can jump to the (D) It is equal to the energy required to excite an electron in the n = 2 shell because in both cases the electron makes a jump to the next shell. In an atom, an excited electron will jump to the next highest unfilled shell (in this case, the n = 3 shell). So the electrons in both the n = 1 shell and the n= 2 shell will jump to the n = 3 shell. However, the energy difference betweon the n = 1 and the n = 3 shell is greater than the energy difforence between the n = 2 shell and the n = 3 shell. Thus, option (B) is correct. ‘The answer is (B), CHEMISTRY-91 A state of energy By with a lifetime of t, decays into the state of energy Ep. ‘The state of By then decays with a lifetime of fg into the state of B3. The decay constants are related by 1 2rq. Initially all of the atoms (quantity No) are in the Ey state. Calculate the number of atoms Ng that are in the state 2 at any instant in time, t. (A) No (e-#/1 = e-/n) a ( a (B) No (e+ e-B/n) ©) 20% (e-#/" = e-#/") (D) 2g (e“!/ +e 2/72) ‘The number of atoms that decay from By to By is Ny = Moen PROFESSIONAL. PUBLICATIONS, INC. CHEMISTRY 7-59 ‘The number of atoms in state Hp equals the number of atoms coming from EB, to Ep minus the number of atoms decaying from E3 to Es. ‘Thus, the number of atoms in state E2 at a given time is Ny = Noe!" — Nage-!™ = Ngen/"' = Noge724/™ Since there are no atoms in state Ez at t = 0, the tial conditions CHEMISTRY-92 A source of radiation has a mean nucleus life of r = 36.8 s. There are initially ‘No = 5.37 x 101 nuclei in the source. Which of the following statements are true? 1. The decay constant is A = 0.0279 5? IL. The half-life is th» = 24.8 s. IIL. Ifa rate counter with an 80% efficiency is placed near the source, it will show a rate of 4.2 x 107 after 2 min. IV. The sample will essentially have all decayed (0.01% remaining) in 5.5 (A) Tand IIL (B) Wand It (©) 1,1, and IV (D) 1,11, 1H, and IV First, find the decay constant. ‘Therefore, statement I is true. PROFESSIONAL. PUBLICATIONS, INC. 7-60 1001 SOLVED ENGINEERING FUNDAMENTALS PROBLEMS: Next, find the half-life LL uate a t= rind = (95.8 5) In2 = 2488 ‘Thus, statement [Tis true. Next, find the count rate after 2 min. The count rate at time seconds) is R= Roo ‘The initial count rate, Ra, is Ro =ANo Since the detector is only 80% efficent, tho rate shown by the detector after 2 min, Ra, is Ra =080R = 0.8 ANe™ 08) (0zam 2) ar x10" =(08) (130.8 2) 0mm vam = 4.2 x 107 decays e! ‘Therefore, statement II is true. ‘Tho time it takes for the sample to decay to an amount V is nX® x "No N=1x 10-'No N 4 Fe 71x10 z } In x 10-4) 0.0279 © =5.5 min PROFESSIONAL PUBLICATIONS, INC. CHEMISTRY 761 ‘Thus, IV is also true, Statements I, Il, Il, and IV are all true. ‘The answer is (D). CHEMISTRY-93 ‘A fossil fern containing 49.9 g of carbon is carbon dated to determine its age. ‘The decay rato of C™ in the fossil is 191 decays/min. How old is the fern? (The half-life of C'* is 5730 years, and the rate of decay of Cin a living organism per g of carbon is 15.0 decays/min-g.) (A) 7290 yr (B) 11300 yr (C) 14100 yr (D) 23800 yr The rate of decay of C4 in a dead organism is given by R= Reo™ In the preceding equation, ) is the decay constant, Ro is the initial decay rate, and t is the time elapsed, ‘Ro is simply the decay rate of carbon-14 in a living organism, because up to the point it dies, it replenishes its carbon. Thus, until the organism's death, the decay rate is fairly constant. decays’ t= (180 222) ca 9 mz tip Di ‘The answer is (B). PROFESSIONAL PUBLICATIONS, INC 7-62 1001 SOLVED ENGINEERING FUNDAMENTALS PROBLEMS CHEMISTRY-04 What is the total relativistic energy of a particle if ts mass is equal to 1 kg when it is traveling at a speed of V2c? (A) o/2 @®) 2-1 oe (D) @41 Regardless of the particle's speed, the total relativistic energy is Ero = te = (1 kg)(e) ‘The answer is (C PROFESSIONAL PUBLICATIONS, INC. 8 STATICS STATICS-1 What is the length of the vector A+B4+C, the sum of three orthogonal vectors? (A) 35m (8) 43m (© tim (D) 10m |A+B4C|= VP FBO Bmp + Guys Ome =707m (7.1m) PROFESSIONAL PUBLICATIONS, INC. 82 1001 SOLVED ENGINEERING FUNDAMENTALS PROBLEMS STATICS-2 Determine the magnitude of the moment of the force F about the corner A. F=100N 2am amsanepacemndn sce aed (A) 120Nm —(B) 40N-m_ (©) 320m (D) 640 Nem 5 fs) a Fe = (100 N) cos 60° = 50.0.N Fy = (100 N) sin 60° = 86.6 N Taking counterclockwise moments as positive, SMa = vk +2, = ~(8 m)(50.0 N) + (12 m)(86.6 N) = 640 Nm ‘The answer is (D). PROFESSIONAL. PUBLICATIONS, INC. STATICS 83 STATICS-3 A cube of side length a is acted upon by a force F as shown. Determine the magnitude of the moment of F about the diagonal AB. oF A (oa) PROFESSIONAL PUBLICATIONS, INC. 84 1001 SOLVED ENGINEERING FUNDAMENTALS PROBLEMS Usn = e453 +) as Map = Usp ~My i+w) -( o43+8) SI a aF ‘The answer is (B) STATICS—4 ‘The boom shown has negligible weight, but it has sufficient strength to support the 1000 KN load without buckling. What is most nearly the tension in the supporting cable between points A and B? A B 1000 xn (A) 200 kN (B) 430 EN (©) 580 KN (D) 870 kN ; lon L2\ PROFESSIONAL PUBLICATIONS, INC. STATICS 85 rium, Fe =0 and 5 Fy UA- Fp = 1154.7 WN Deno Fycos60° -T =0 = Frysin 60° — 1000 kN ‘The tension in the cable, T, is T = Fycos60? = (1154.7 kN) eos 60° = 5774 KN (580 kN) ‘The answer is (C} STATICS-5 Find the tensions, T; and Ts, in the ropes shown so that the system is in equi: librium, son (A) T, = 50.0 N, Tp (B) 7 =500N, 75 (C) T, =70.7N, T (D) T, = 70.7 N, T2 PROFESSIONAL PUBLICATIONS, ING. 86 1001 SOLVED ENGINEERING FUNDAMENTALS PROBLEMS DOR =0=Tsinas? —50N=0 Ty sins? = 50 N Ty =70.7N Dro T, cos 45° — T, = 0 Ty = Tos 5° =50N The answer is (C). STATICS-6 For the system illustrated, determine the value of fh that puts the system in equilibrium. fitiontess re volier ‘ietioniess pulley TOON (A) 0.50.em (B) 10cm (C) 1.5 em (D) 21cm PROFESSIONAL PUBLICATIONS, INC. STATICS 87 YA =0 0=-P+ Wind +Tsine (P+ W)sind =P Rao 0 = Weosd ~ T cos T-w =100N 2Wsind = P sind = Pe ewe Vitra ~ 2W = BOON) =01 101(h? + d?) .o1d PROFESSIONAL PUBLICATIONS, INC. 88 1001 SOLVED ENGINEERING FUNDAMENTALS PROBLEMS STATICS~7 Hinges A and B support a 10000 kN bank vault door as shown. Determine the horizontal force in the hinge pin at A. hinge B lam ninge 4 J Ly 15m 15m (A) 1500 KN (B) 2500 KN (GC) 5500 kN (D) 7500 Sum the moments around j Sm =0 0 = (—10000 KN)(1.5 m) + (2. m)Ras Ras = 7500 KN ‘The answer is (D). PROFESSIONAL PUBLICATIONS, INC. STATICS 89 STATICS-8 ‘A cylindrical tank is at rest as shown. ‘The tank has a weight of 100 kN. Ap- proximately what horizontal force should be applied horizontally at point C to raise the tank? (A) 25 kN (D) 10 kN In order to raise the tank, > Ma > 0. Therefore, the minimum force ‘that must be applied at point C can be found as follows. DMa=0 W(8 m)sing = 0 _2Wesing ae F(2 m) 5 008 = 5 o= 60° po a 60° (2)(200 KN) sin 60° 3 = 57.7 KN (68 KN) ‘The answer @B). PROFESSIONAL PUBLICATIONS, INC. 8-10 1001 SOLVED ENGINEERING FUNDAMENTALS PROBLEMS STATICS-9 A weight is attached to a lover as shown. Determine the expression for @ when ‘the system is at oquilibrium. The spring constant is f, the length of the lever is L, the radius of the wheel is r, and the magnitude of the weight is W. weight, | = spring, k _ Whsind Whsind (A) = @) 0= Wr WLcosé (o= (©) o= "D PROFESSIONAL PUBLICATIONS, INC. STATICS SL At equilibrium, the sum of the moments about the center of the wheel (point ©) must equal zero, Mo =0 = rFepring — Win 8 1Fepeing = WLsind Fepring = kr rko =WLsin8 WLsind mF Successive iterations are necessary to solve for 0. ‘The answer is (B). STATICS-10 ‘Two identical spheres weighing 100 N each are placed as shown. The line con- necting the two centers of the spheres makes an angle of 30° to the horizontal surface. All walls are smooth and frictionless. What is most nearly the reaction force at A? (A) 33 (B) o7N (©) 5N (D) 100N PROFESSIONAL PUBLICATIONS, ING. ez 1001 SOLVED ENGINEERING FUNDAMENTALS PROBLEMS a0" Fe W=100N dR Ra cos0” — Kp cos 80° Ra= Ro “A —W + Rasin30° + Rp sin30° 2Ra = wi RaW =100N ‘The answer is (D). PROFESSIONAL PUBLICATIONS, INC. STATICS 813 STATICS-11 What is most nearly the force, P, that must be exerted on the handles of the bolt cutter shown if the force on the bolt is 1000 N? Pp y bott - wom oom 3am (A) TSN (B) 30N (©) 53N (D) 260 N First, consider the upper jaw: upper jaw Foy: Fey F= 1000 —(Ray)(10 em) + F(B em) =0 (is)? . (#2) (1000 N) =300N Roy PROFESSIONAL PUBLICATIONS, INC. 814 1001 SOLVED ENGINEERING FUNDAMENTALS PROBLEMS ‘Now, consider the upper handle. - upper handlo a DMa=0 —Rey(1 cm) + P(40 cm) = 0 P= GS ) Ro, = (am) oo™ =75N ‘The answer is (A). STATICS-12 Determine the force, J”, required to keep the package from sliding down the plane shown. (A) 15 ev (B) 35 eN (C) 65 N (D) 120 kN PROFESSIONAL. PUBLICATIONS, INC. STATICS 815 = 200 kN Py= un = (0.15)(200 kN) = 30 kN r=0 F-W.+Fy=0 F=W.-Fy We = 3 = 150 KN P= 150 KN 30 kN = 120 KN ‘Tho answer is (D} PROFESSIONAL PUBLICATIONS, INC. 816 1001 SOLVED ENGINEERING FUNDAMENTALS PROBLEMS STATICS-13 Determine the minimum coefficient of friction at point B required for a person to use the ladder shown. Assume there is no friction at point A. am (@) 018 ©) 028 (0) 0.56 rm ‘The total reaction force at B (Vp + Fyn) must point along the ladder. ‘Therefore, Fin _ wNa Re = Ae = eto p= cod ‘The answer is (C). PROFESSIONAL PUBLICATIONS, INC. STATICS B17 STATICS-14 ‘A4m ladder weighing 200 N is placed as shown. When an 800 N person climbs 3m above the lower end, the ladder is just about to slip. Determine the friction coefficient between the ladder and the floor. The coefficient of friction between the ladder and the wall is 0.20. (A) 019 (B) 0.29 (©) 0.39 (D) 0.49 ‘PROFESSIONAL. PUBLICATIONS, INC. 818 1001 SOLVED ENGINEERING FUNDAMENTALS PROBLEMS Ma =0 0 = (—200 N)(2 m) sin 30° — (800 N)(3 m) sin 30° + Np(4 m) cos 30° +0.2Np(4 m) sin 30° Np = 362.N DA-0 0 = Na + Hwan ~ 800 N — 200 N = Na + (0.20)(362 N) — 800 N - 200 N Ny = 928 N Dr=0 0 = poor Va — Na HrioorNa = Na. Hao ‘The answer is (C).. STATICS-15 A 2 kg block is released at point A on an inclined plane that is tangent to a circular arc. ‘The plane is tilted 15° from horizontal, and the coefficient. of friction is 0.4. Which point is the equilibrium position of the block? wa04 ‘curve radius aya (B)B joc (D) D PROFESSIONAL PUBLICATIONS, INC. STATICS 819 Weosd W= mg o=18 Weind yy = 04 —mgsind + Fy = —masind + uma cos For the block to slide downhill, Fs <0. =mgsind + wmgcos0 <0 mg cos 0 < mgsind w—@ |am = 651 Therefore, Ry + Ry =65 kN The answer is (B). STATICS-27 A beam is subjected to a distributed load as shown, Determine the reactions at the right support, A. 3m 1Nim 3m {ree-body diagram (A) 22eN (B) 43.aN (©) 5.5 kN () 70 N PROFESSIONAL PUBLICATIONS, INC. STATICS 831 = I 5 0 a wo = Since Rae = 0, Ray = Ra. Thus, Ra [_ Me anower sD) STATICS-28 ‘A beam is hinged at a wall and loaded as shown. What is the tension in the cable? frictionless hinge 100kN F hinge Hy om mn free-body diagram (A) 200 KN (B) 250 kN (©) 430 ka (D) 500 kN PROFESSIONAL, PUBLICATIONS, INC. 8-32 1001 SOLVED ENGINEERING FUNDAMENTALS PROBLEMS Ditasnge = (6 m) + (100 KN)(10 m) — T’sin 30°(10 m) F=Lh=(10m) (x *) 100 KN Substituting for F and rearranging to solve for T, (300 KN)(6 m) + (100 KN)(10 m) (0m) (sin 30") = 500.0 kN ‘The answer is (D). STATICS-29 Determine the position of maximum moment in the beam ABC. SiN 2kNim (A) at point A (B) at point B (©) at point C (D) 2m left of point ¢ Ms =0 5 0 = (5 kN\(1 u)- f 2rdz + (Re,)(5 m) =5-m—2"| + Ro, (5 m) lo = 5 kN-m — (25-0) + Ro, (5 m) kN Ro, ‘PROFESSIONAL PUBLICATIONS, INC. STATICS 8.33 5 noua fade +4 + Ry f —BKN— (10-0) +4 KN+ Rp BEN -10 KN44 KN + Rp Ry= 11 kN At point A, Ma=0 At point B, ‘Mp = (5 kN)(1.m) =5kNm From C to B, Me = Foye [ ede Bs -(«-%) = (e-2) a “eg 71-3 =O. [where My isa max) o= an) (2 S): oe 7 ™ =2m Menus = (4 KN)(2.m) —4 Nm =4kNm< Ma ‘Thus, the maximum moment is 5 kN-m, and it occurs at point B. ‘The answer is (B). PROFESSIONAL PUBLICATIONS, INC. 834 1001 SOLVED ENGINEERING FUNDAMENTALS PROBLEMS STATICS-30 For the cantilever beam with the distributed load shown, what is the moment at tho built-in end? 10KNm TO THT) builtin end ae 3m (A) 5 kNsm (B) 10kNm (©) 20kNm——(D) 45 KN ‘The answer is (D). = (0 KN)(1.5 m) 45 kN-m PROFESSIONAL PUBLICATIONS, INC. STATICS 835 STATICS-81 For the cantilever beam shown, what is the moment acting at the built-in end? ©b titi [outein ena| 2m 6m (A) 270KN-m — (B) S10KNm_— (©) S40 Kem (D) 720 kN Sum the moments around the support, A. mM Rearranging to solve for Ma, = Ma ~ (00 HN)(6 m) ~ (» Mg = 720 kN-m ‘Tho answer is (D). STATICS-32 Determine the resultant moment at the built-in end, A, for the cantilever beam shown. 25 KNim A_15 10m am om (A) 660 kNm —(B) 990 kN-m = (C) 1100 kN-m = (D) 1200 kNem [PROFESSIONAL PUBLICATIONS, INC. 1001 SOLVED ENGINEERING FUNDAMENTALS PROBLEMS 15 8.36 Divide the beam into two sections: from 0m to 4 m where Fj KN-m, and from 4 m to 10 m where F, = 5(¢-+5)/3 kN/m. Sum the ‘moments around point A. HITT EMa=o A Ma [' R(ezie— [° Fo(ayede in~ [ AG) (2) = fis 5(c +8) y= ff isoas+ [ tO ade 15e2|* 52 2522/1 “FT),t ote, ° ay! 2 ay? = ony) + R=), BOO = 4) = 990 KN-m ‘The answer is (B). STATICS-s3 Determine the moment atthe built-in end, A, for the beam shown. w= oy sin ie 3 = amt? a 2 (a) SE @) mt © o) 32 PROFESSIONAL PUBLICATIONS, INC. STATICS 8.37 : + : : Pw ‘wol? xa!" - (EP oon-0) + (SF) e¥[ STATICS-34 ‘The beam shown is statically indeterminate, but the reaction force at B is known to be 200 KN. What is the bending moment at point P? 104m imply supported end 8 a ? 2 uten [ Fg = 200 kN —__—_-| 3m (A) 90kNm —(B) 150 kN'm (©) 240 Kem (D) 350 KN PROFESSIONAL PUBLICATIONS, INC. 8-38 1001 SOLVED ENGINEERING FUNDAMENTALS PROBLEMS ‘Measure x from point B. ‘The moment at point P is 200 kN Y= ‘Rearranging to solve for Mp, Mp ~ F(1.5 m) + (200 kN)(3 m) My (200 KN)(3 m) — (m S) (8 m)(1.5 m) = 150 km ‘The answer is (B). STATICS-35 Determine the height of the centroid, 9, of the semicircle with radius r shown. “ Be 2nr 4r ar @ = og oF PROFESSIONAL PUBLICATIONS, INC. STATICS 8.39 A= nr? {for the semicircle] z=reosé sind dy = d(rsind) = reos0d0 dA = ady = rc0s0r cos 6d8 Since the area is symmetrical, area for 0 < 0 < 90° is half the total a fa [ (rsin0)(r-0080)(rcos )ad /2 [OP sin dco ea 4 =253 ong 9 |"? By definition, jA= f yd Is fia ‘The answer is (C). PROFESSIONAL. PUBLICATIONS, INC. 8-40 1001 SOLVED ENGINEERING FUNDAMENTALS PROBLEMS STATICS-36 ‘What is the height of the center of mass of the cono-sphere system shown? eter ™ephere = 89 00" Mone = 29 (A) 2em (B) 3m (© 4em (D) Sem By symmetry, the center of mass of the system is on the y-axis. Find the height of the center of mass for each part of the system, First, find the height of the sphere’s center of mass, OO 00° height of cone + rsphere 3.em =sen+5 45cm Next, find the height of the cone’s center of mass, he: he (height of cone) ) = con) (2) =20m PROFESSIONAL PUBLICATIONS, INC. STATICS 841 Finally, find the height of the cone-sphere system’s center of mass. j= Maher OO! + meonehe ‘Maphere + Meone (8 )(4.5 cm) + (2 w)(2 em) Set2e =4em STATICS-37 Which statoment about area moments of inertia is FALSE? (A) I= fea (B) The parallel axis theorem is used to calculate moments of inertia about a parallel displaced axis (©) The moment of inertia of a large aren is equal to the summation of the inertia of the smaller areas within th large area. (D)_ The areas closest to the axis of interest contribute most to the moment of inertia. ‘Area moment of inertia is defined as I = fd’dA, where d is the distance from the axis to the aren clement. Thus, the areas farthest from the axis have the largest contributions. 1e answer is (D) PROFESSIONAL PUBLICATIONS, INC. 842 1001 SOLVED ENGINEERING FUNDAMENTALS PROBLEMS STATICS-38 Determine the moment of inertia around the centroidal axis of the following i" - tem em Bom ceniroidal ons, 250m, 2em |__| Bem (A) 420em* —(B) 650em* (©) 730em* —(D) 950 em* ‘The moment of inertia of the beam is equivalent to the moment of inertia of a solid beam with the same dimensions (a height of 12 em ‘and a width of 8 cm) minus the moments of inertia of the missing sections. ‘The moment of inertia about the centroidal axis of rectangular sec- tions is ond pemeiiag PROFESSIONAL. PUBLICATIONS, INC. STATICS 843 L—_-l This equation is applied as follows for the solid section (8 cmx 12 cm), the two removed sections (2.5 em x 10 em each), and two other re- moved sections (1 em x 8 cm each). fe (3) (8 em)(12 em)* — (2) (8) (25 em)(t0cm)*) ~ 2) ((%) em¢s omy) 650 cm* ‘PROFESSIONAL PUBLICATIONS, INC. add 1001 SOLVED ENGINEERING FUNDAMENTALS PROBLEMS STATICS-39 Tee is the moment of inertia of the plane area. about its centroidal « axis. How can Tex be expressed? y > 7 ‘The answer is (C) PROFESSIONAL PUBLICATIONS, ING. MATERIALS 9 SCIENCE MATERIALS SCIENCE-1 Which of the following affects most of the electrical and thermal properties of ‘materiale? (A) the atomic weight expressed in grams per gram-atom (B) the electrons, particularly the outermost ones (C) the magnitude of electrical charge of the protons (D) the weight of the atoms ‘The outermost electrons are responsible for determining most of the ‘material’s properties. ‘The answer is (B) MATERIALS SCIENCE-2 ‘The atomic weight of hydrogen is 1 g/mol. What is most nearly the mass of a hydrogen atom? (A) 7x10“ g/atom (B) 6.0%10-* g/atom (©) 10x10" g/atom — (D) 1.0 g/atom PROFESSIONAL PUBLICATIONS, INC. 92 1001 SOLVED ENGINEERING FUNDAMENTALS PROBLEMS By definition, the mass of an atom weight divided by Avogadro's number, it W = mol 5 = 1.66 x 10™™ g/atom (1.7 x 10™™ g/atom) 6.02 x 19% Sm answer is (A) MATERIALS SCIENCE-3 What are valence electrons? (A) the outer-shell electrons (B) electrons with positive charge (C) the electrons of complete quantum shells (D) the K-quantum shell electrons By definition, the outermost electrons are the valence electrons. The answer is (A). MATERIALS SCIENCE-4 ‘What is the strong bond between hydrogen atoms called? (A) the ionic bond (B) the metallic bond (©) ionic and metallic bonds (D) the covalent bond Covalent bonds provide the strongest attractive forces between atoms. ‘The answer is (D). PROFESSIONAL PUBLICATIONS, INC. MATERIALS SCIENCE 93 MATERIALS SCIENCE-5 ‘What are van der Waals forces? (A) woak secondary bonds between atoms (B) primary bonds between atoms (C) forces between electrons and protons (D) forces not present in liquids By definition, van der Waals forces are weak attractive forces between atoms or molecules. ‘The answer is (A). MATERIALS SCIENCE-6 Which of the following curves best illustrates the relationship between inter- atomic forces and interatomic spacing? attraction (a) intoratomic fores spacing repulsion extraction 8) ,, intoratomie force |" spacing repulsion ‘PROFESSIONAL PUBLICATIONS, INC. a4 1001 SOLVED ENGINEERING FUNDAMENTALS PROBLEMS stuection (©) gg |e trator ng ropulson ettection ©) nteretarle ‘tora ; ‘spacing repulsion ‘The interatomic force changes from repulsion to attraction as spacing between atoms increases, ‘The answer is (A). MATERIALS SCIENCE-7 Compare the metallic iron atom Fe and the ferrous and ferric ons Fe and Fe** at the same temperature. Which has the smallest atomic radius? (A) Fe (B) Fee (C) Fett (D) They have the same radii. Tonizing removes valence electrons, causing the remaining electrons ‘to be pullod in closer to the nucleus. Further reduction in spacing occurs with the removal of more electrons. PROFESSIONAL PUBLICATIONS, INC. MATERIALS SCIENCE 95 MATERIALS SCIENCE-8 Cesium (Cs) and sodium (Na) both have the same valence (+1), yet with chlo- (C2), cesium has a coordination number of 8 in CsCl, while sodium has a coordination number of only 6 in NaCl. What is the main reason for this difference? (A) The atomic weight of Cs is larger than the weight of Na (B) Gs forms covalent bonds in CsCl. (©) Cs contains more eloctrons than Na. (D) Ces too large to be coordinated by only 6 chloride ions. Since the C1 atoms are of constant size, the larger coordination num- ber for Cs means that more Cl atoms are needed to fit around a C3 atom than around a Na atom. Therefore, the Cs atom is larger than the Na atom. The answer is (D} MATERIALS SCIENCE-9 Which of the following statements is FALSE? (A) Ceramics are inorganic, nonmetallic solids that are processed or used at high temperatures. (B) Metals are chemical elements that form substances that are opaque, Ius- trous, and good conductors of heat and electricity. (©) Oxides, carbides, and nitrides are considered to be within the class of materials known as glasses, (D) Most metals are strong, ductile, and malleable. In general, they are heavier than most other substances. ‘The classes of materials are ceramics, metals, and polymers. Oxides, carbides, nitrides, and glasses are all ceramics. ‘The answer is (C). [PROFESSIONAL PUBLICATIONS, INC. 96 1001 SOLVED ENGINEERING FUNDAMENTALS PROBLEMS MATERIALS SCIENCE-10 Which of the following materials is NOT a viscoclastic material? (A) plastic (B) metal (C) rubber (D) glass ‘A material that is viscoelastic exhibits time-dependent elastic strain. Of the choices, only metal does not fit this description. Metal is considered to be an clastoplastic material. ‘The answer is (B). MATERIALS SCIENCE-11 In molecules of the same composition, what are variations of atomic arrange- ments known as? (A) polymers (B) nonerystalline structures (C) monomers (D) isomers Isomers are molecules that have the same composition but different atomic arrangements. ‘The answer is (D). MATERIALS SCIENCE-12 Which of the following accurately doscribes differences between crystalline poly- ‘mers and simple crystals? 1. Crystalline polymers, of atoms. TL, Crystal size can be increasod by raising the crystallization temperature only in polymers. TIL, While a simple crystal may be totally crystallized, a polymer ean reach only partial erystallization, (A) Tonly (8) Monty (©) Monly ——(D) Tand inlike simple erystals, are made of folded chains PROFESSIONAL PUBLICATIONS, INC. oT Only crystalline polymers are composed of folded chains and, at best, exhibit partial crystallization. The crystal size of both simple crystals and polymers ean be increased by raising temperature. ‘The answer is (D)- MATERIALS SCIENCE-13 Polymers that favor crystallization are least likely to have which of the following? (A) an atactic configuration of side groups (B) small side groups (C) only one repeating unit: (D) small chain lengths In order for crystallization to be favored, the molecules must be able to arrange themselves into an orderly structure. “Atactic” refers to a random configuration of side groups in the polymer; such a configu- ration would hinder crystallization, The answer is (A) MATERIALS SCIENCE-14 What is the atomic packing factor (APF) for a simple cubic crystal? (A) 0.48 (B) 0.52 (10 (Daa PROFESSIONAL. PUBLICATIONS, INC. (G FUNDAMENTALS PROBLEMS 98 1001 SOLVED ENGINEERIN For a simple cubic crystal, there is one complete atom of radius r per coll. The cell has edges of length 2r. By dofinition, volume of atoms APE = Solume of unit cell ‘Phe answer is (B). MATERIALS SCIENCE-15 How many atoms are in the unit cell of a body-centered cubic structure? (A) one (B) tro (©) three (D) four ‘There is one atom at the conter position and ¥/s of an atom at each of the corners of the cube, since the atom present at each corner is, shared by the adjoining unit cells. Therefore, total no. of atoms = no. of atoms at conter + (no. of atoms at each corner)(no, of comers) =14()@)=2 ‘The answer is (B).. PROFESSIONAL PUBLICATIONS, INC. ‘MATERIALS SCIENCE 99 MATERIALS SCIENCE-16 How many atoms are there per unit cell for a faco-centered cubic structure? (A) one (B) two (C) three (D) four “Like a body-centered cubie structure, there is Ys of an atom at each comer of the cube. There is also "a of an atom at the center of each of the six faces, since each atom here is shared by the neighboring. unit coll. total no, of atoms = (no. of atoms at each eomer)(no. of corners) + (no. of atoms at center of each face)(no. of faces) =(j)@+()o=4 ‘The answer is (D). MATERIALS SCIENCE-17 ‘What is the first coordination number of a body-centered cubic structure? (aya Os () 10 (B)6 ie 7 [PROFESSIONAL PUBLICATIONS, INC. 9-10 1001 SOLVED ENGINEERING FUNDAMENTALS PROBLEMS The first coordination number is the number of nearest neighbor atoms. In a body-centered cubic cell of edge length J, the minimum distance between atoms is ( V/2)1. By inspection of the figure, there are eight neighboring atoms at this distance. ‘The answer is (C). MATERIALS SCIENCE-18 What is the first coordination number of a faco-contered eubie structure? we @4 os (p) 12 ‘The closest: atoms in a faco-centered cubie cell of edge length | are 1/V3 apart. Each atom in the center of a face has 12 such neighboring, atoms, The atoms are: (-(MYa)l, (¥/)l, 0), (Ya), 0, (/2)2), and (0, £0), £(*/2))). ‘The answer is (D). MATERIALS SCIENCE-19 ‘Which of the following statements is FALSE? (A) (B) (©) (D) Both copper and aluminum have a face-centered cubie crystal structure. Both magnesium and zinc have « hexagonal close-packed crystal structure. Iron can have either a face-centered or a body-centered cubic erystal struc- ‘ture. Both lead and cadmium have a hexagonal close-packed crystal structure. PROFESSIONAL PUBLICATIONS, INC. MATERIALS SCIENCE oan Lead does not have a hexagonal close-packed structure. Its structure is face-centered cubic. ‘The answer is (D). MATERIALS SCIENCE-20 Which of the following statements is FALSE? (A) The coordinates of the unique lattice points for a body-centered cubic unit cell are: (0.0 0) and (J $ })- (B) The coordinates of the unique lattice points for a face-centered cubic unit cell are: (0.0 0); (§ $ 0); (F 0 $)s and (0 $ 4). (C)_ The coordinates of the unique lattice points for a simple cubic unit cell are: (000). (D) The coordinates of the unique lattice points for a rhombohedral unit cell are: (5 3 3)- ‘The rhombohedral Bravais lattice is a primitive cell and has only the point (0 0 0). ‘The answer is (D). MATERIALS SCIENCE-21 How are the close-packed planes in a faco-centered cubic metal designated? (G0 =) G0 = AI =— MAI PROFESSIONAL PUBLICATIONS, INC. gaz 1001 SOLVED ENGINEERING FUNDAMENTALS PROBLEMS ‘The close-packed planes are as shown, ‘The answer is (D). MATERIALS SCIENCE-22 ‘Which crystal structure possesses the highest number of close-packed planes and close-packed directions? (A) simple cubic (B) body-centered cubic (C) face-contered cubic (D) close-packed hexagonal ‘The face-conterod cubie structure has four close-packed planes: (11 1), (111), (112), and (11 1). Each plane has three close-packed direc- tions. The answer is (C). MATERIALS SCIENCE-23 What are the most common slip planes for face-centered cubic and body-centered cubic structures, respectively? (A) face-centered: (1 1 1); body-centered: (1 1 0) (B) face-centered: (1 0 0); body-centered: (1 1 0) (C) face-centered: (1 1 0); body-centered: (1 1 1) (D) face-centered: (1 1 1); body-centered: (1 0 0) Slip planes are usually the most closely packed planes, since they have ‘the largest spacing. The close-packed planes are (1 1 1) and (1 1 0) for the respective crystal structures. ‘The answer is (A). PROFESSIONAL PUBLICATIONS, INC. MATERIALS SCIENCE, 9.13 MATERIALS SCIENCE-24 Comparing the face-centered cubic lattice with the hexagonal close-packed lat- tice, which of the following features describes the hexagonal close-packed structure only’? (A) It has the closest packed lattice structure (B) Its coordination number is 12. (C) Its deformation properties are more directional. (D) Its stacking order is ABCABC. Options (A) and (B) are true for both face-centered cubic and hexag- onal close-packed structures, while option (D) is true for the face- centered cubie lattice. Option (C) applies to the hexagonal close- packed lattice only. The answer is (C). MATERIALS SCIENCE-25 In the following unit cell, what direction is indicated by the arrow? (A) (012) (B) 013) (©) 210) (2) @ $4) PROFESSIONAL. PUBLICATIONS, INC. 914 1001 SOLVED ENGINEERING FUNDAMENTALS PROBLEMS Direction is given by the intercepts, as ratios of the lattice dimension. MATERIALS SCIENCE-26 ‘What are the Miller indices of the given plane? A = % (A) (444) (®) 11) arta (D) (222) ‘The z, y, and z intercepts are aj, ag, and ag, respectively, since ay = ay = as = 4. The Miller indices are (AYO)-OW)-- ‘Tho answer is (B). [PROFESSIONAL PUBLICATIONS, INC. MATERIALS SCIENCE 915 MATERIALS SCIENCE-27 ‘What are the Miller indices of the given plane? (A) (821) ®) Ga) © Gee (D) 236) ‘The intercepts for this plane are a1, 2a2/3, and ay/3. The Miller indices are ey'(2) @ = (13) =@36) MATERIALS SCIENCE-28 ‘A plane intercepts the coordinate axis at z= 1, y= 3, and = 2. What are the Miller indices of the plano? (A) (132) B) (123) (©) (623) (D) (826) ‘The Miller indices are computed by taking the reciprocal of each intercept and converting to whole numbers of the same ratio, (1 § 9) = 623) ‘The answer is (C). PROFESSIONAL PUBLICATIONS, INC. 916 1001 SOLVED ENGINEERING FUNDAMENTALS PROBLEMS MATERIALS SCIENCE-29 Which of the following gives the correct designations for the planes shown? 7 1. m™, (A)E(110, TH: (@200), IE (10) (B) E100), TH (110), m1) (CE (00), Te (11), ME (102) (D) E00), M11), HE (10) 1. Since the plane passes through the origin, move the plane to x y= 00, 2 = 00. Take reciprocals of the intercepts. 1_, 1,1 -h==0,5=0 (100 I a 7 (100) TI Since the plane passes through the origin, move the plane to 2 = =1, y= 1, z= 1. Take reciprocals of the intercepts. 1 1 =1,2= ga2 (G11y) a ‘y (-111), but (-1 1 1) is identical to (111). UL Since the plane passes through the origin, move the plane to 2 = 1, y=1, 7 = 00. Take reciprocals of the intercepts. (110) (-1 10) is identical to (1 1 0). ‘The answer is (D). PROFESSIONAL PUBLICATIONS, ING. MATERIALS SCIENCE 917 MATERIALS SCIRNCE-30 Using the four-index scheme for a hexagonal erystal system, how would the directions F and G shown be defined? 210) [0 300) (1320) [l2To) 2m RY 6 The F vector is the sum of one unit in the positive a; direction and one unit in the negative ag direction. The G vector is the sum of one unit in the negative a; direction, two units in the positive ay direction, and one unit in the negative ag direction, ‘The answer is (D). PROFESSIONAL JBUICATIONS, INC. 9-18 1001 SOLVED ENGINEERING FUNDAMENTALS PROBLEMS MATERIALS SCIENCE-31 Given that a is a lattice constant and that h, k, and J are the Miller indices, which of the following equations describes the interplanar distance d in a cubic crystal?” (8) d= ots (i) +) +) @) d=a(j +747) © d= (3) vBrRre ©) d= ere Geometrically, 1/d? = (h? +k? + 2)/a2. Therefore, a on VTE EE ‘The answer is (D). MATERIALS SCIENCE-32 ‘The atomic weight of copper is 63.5 g/mol. Calculate the theoretical density of copper given that the unt cell feee-cantared cuble and the lattice parameter is 3.61 A. (A) 45 g/cm? (B) 7.9 g/em’ —(C) 88 g/em* —_(D) 9.0 g/em* ‘There are four atoms per unit cell for a face-centered cubic structure. By definition, v (+airea) (6 a) (5 (ear (==) = 8.97 g/cm? (9.0 g/cm’) ‘The answer is (D). PROFESSIONAL PUBLICATIONS, INC. MATERIALS SCIENCE 9.19 MATERIALS SCIENCE-33 Determine the planar density of copper atoms in a (1.0 0) plane given that the unit cell is face-centered cubie nnd the lattice parameter is 3.61 A. (A) 7.68 x 108 atoms/m? (B) 1.53 x 10° atoms/m? (C) 2.80 x 102° atoms /m? (D) 3.84 x 10" atoms/m? There are two atoms total in the (1.00) plane. The planar density is, therefore, . no. atoms per face Planar denstty = ee of face 2 atoms Aare ~ Caxton 11.53 x 10" atoms/m? Che answer is (B) MATERIALS SCIENCE-34 Which of the following statements is FALSE regarding X-ray diffraction? (A) ‘The geometrical structure factor F(hkl) is the ratio of the amplitude of the X-ray reflected from a plane in a crystal to the amplitude of the X-ray scattered from a single electron. (B) X-ray diffraction is only useful for studying simpler erystals such as the body-centered cubie structure, rather than more complex crystals like the hexagonal close-packed structure. (C) X-ray diffraction can be used to determine the grain size of a specimen. (D) Bragg’s law states that nA/2d = sind (n is an integer, \ is the wavelength of the X-ray, d is the interplanar spacing, and @ is the scattering angle). PROFESSIONAL PUBLICATIONS, INC. 9-20 1001 SOLVED ENGINEERING FUNDAMENTALS PROBLEMS ‘X-ray diffraction is used to study all types of erystals. It is not limited to simple crystals. ‘The answor is (B). MATERIALS SCIENCE-35 A sample of faco-centered cubic nickel (Ni) was placed in an X-ray beam of wavelength A= 0.154 nm, If the lattice parameter for Ni is ao = 0.352 nm, what is the first-order angle of diffraction most nearly? (A) 57° (B) 7.0" (C) 13° (D) 19° Using Braga’s law, with n= 1, A= 0.154 nm, and d = 0.352 am, A= 2dsind geen 0.154 om 2d (C352 nm) = 126° MATERIALS SCIENCE-36 In a crystal structure, what is an interstitial atom? (A) an extra atom sitting at a nonlattice point (B) an atom missing at a lattice point (C) a different clement: at a lattice point (D) a line defect An interstitial atom is an extra atom lodged within the erystal struc- ture; itis point defect. ‘The answer is (A). PROFESSIONAL PUBLICATIONS, INC. ‘MATERIALS SCIENCE 921 MATERIALS SCIENCE-37 Which of the following is a line defect in a lattice crystal structure? (A) tilt boundary B) screw dislocation {C) vacancy {D) Schottky imperfection ‘The most common type of line defect is a dislocation. The answer is (B) MATERIALS SCIENCE-38 It is often desired to know the mumber of atoms, n, in a erystal structure that possess more than a specified amount of energy, E. Which of the following equations gives n, given that NV is the total number of atoms present, M is a constant, kis the Boltzmann constant, and T is the temperature of the specimen? M esr EM xr (A) n Bn MNe-#/Kr (D) n= MNerMTIE On ‘The equation in option (C) is the correct relationship for thermal energy distribution within a specimen. ‘The answer is (C) MATERIALS SCIENCE-39 Which of the following statements regarding diffusion in a crystal structure is true? (A) Solid interstitial atoms cannot diffuse through structures that lack vacancies. (B) It occurs only in alloys, never in pure erystals. (C) It often uses an exchange or vacancy mechanism. (D) It occurs primarily as a result of mechanical work. Diffusion is the movement of a defect from one point to another. The answer is (C). PROFESSIONAL PUBLICATIONS, INC. 9.22 1001 SOLVED ENGINEERING FUNDAMENTALS PROBLEMS: MATERIALS SCIENCE~40 ‘What is Ficl’s first law for ono-dimensional, steady-state diffusion? C is the ‘volume concentration of atoms, 2 is the distance along which diffusion occurs, Dis the diffusion coefficient, and J is the flux or current density. cass oe 1 ac wo a=(-)% ey sano ‘The flux is proportional to the diffusion constant and the concentra- tion gradient 0C/2z. (A) J=-D' @) = 082 ‘The answer is (A). MATERIALS SCIENCE-41 Which of the following are true about Fick’s first law for diffusion? . It is only applicable to gases and liquids, not solids. IL, The law states that the flux moves from high to low concentration. TIL. J, the flux, may be units of em?/ern?-s, (A) Tonly (B) Tonly (C) Monly (D) Wand mt Fick’s law says that the flux of diffusion is proportional to the negative volume concentration gradient; the negative sign indicates that the flux is in the down-gradient direction. It applies to diffusion in a crystal. Flux, by definition, is the amount of volume moving across a unit surface area in unit time. The answer is (D). MATERIALS SCIENCE~42 ‘What is the Arrhenius equation for the rate of a thermally activated process? A is the reaction constant, T is the absolute temperature, R is the gas constant, and Q is the activation energy. (A) rato = Ae-@/®T—(B) rate = Ae“ORT (©) rate = AeA? (D) rate AcQuT PROFESSIONAL PUBLICATIONS, INC. MATERIALS SCIENCE 9.23 The rate increnses as the thermal energy increases. ‘The answer is (A). MATERIALS SCIENCE-43 ‘Which of the following statements is FALSE? (A) The surface energy of a liquid tends toward « minimum. (B) The surface energy is the work required to create a unit area of additional space. (C) The energy of an interior atom is greater than the energy of an atom on the surface of a liquid. (D) Total surface energy is directly proportional to the surface area. In a liquid, the energy of a surface atom is greater than the surface ‘nergy of an interior atom. Note: Although surface energy and surface tension have the same numerical value, they have different units. The answer is (C). MATERIALS SCIENCE—14 Which point on the stress-strain curve shown gives the ultimate stress? ¢ (wa (B) B oc (D)D PROFESSIONAL PUBLICATIONS, INC. 9-24 1001 SOLVED ENGINEERING FUNDAMENTALS PROBLEMS ‘The ultimate stress corresponds to the point of maximum load, beyond which further strain is accompanied by a reduction in load. ‘The answer is (0). MATERIALS SCIENCE~45 A stress-strain diagram for a polymer is shown, Identify items A, B, and C. o-—_—_____4—__,, n> 4 (A) A= lower yield point; B = plastic deformation; C (B) A= lower yield point; B = proportional limit; C (©) A= yield point; B = elastic deformation; (D) A= yield point; B = elongation at fracturs ‘upper yield point upper yield point elastic limi C= fracture point Beginning at the yield point, considerable elongation occurs with no noticeable increase in tensile stress. Eventually, fracture occurs. The total strain at fracture is known as the elongation, ‘The answer is (D) PROFESSIONAL. PUBLICATIONS, INC. ‘MATERIALS SCIENCE 9-25 MATERIALS SCIENCE~46 Which statement is true for the stress-strain relationship for the metal shown? (A) Point A is the lower yield point. (B) Point D is the fracture stress point. (©) Point B is the upper yield point. (D) The range from point C to point D is known as the elastic range. Point A is the elastic limit, and point D is the ultimate stress point. ‘The region between points C and D is not the elastic region but the plastic region. Only option (C) is true. ‘The answer is (C} MATERIALS SCIENCE-47 [dentify the properties of the materials whose stress-strain diagrams are shown, Lee (A) I: soft and weak; Il soft and tough; III: hard and brittle (B) I: hard and brittle; Il: soft. and weak; II: hard and tough (©) (D) I: soft and tough; II: hard and brittle; III: hard and strong I: hard and strong; I: soft and brittle; III: soft and tough PROFESSIONAL PUBLICATIONS, INC. 9-26 1001 SOLVED ENGINEERING FUNDAMENTALS PROBLEMS ‘The properties and their relationships to the stress-strain diagrams are given in the following table. clastic yield elongation —ultimate modulus point _at fracture _strength hard and brittle high undefined low moderate to high soft and weak low low moderate low hard and tough high high high high ‘The answer is (B). MATERIALS SCIENCE-48 Which statement is most accurate regarding the two materials represented in the given stress-strain diagrams? (B) Material B would require more total energy to fracture than material A. (C) Material A will withstand more stress before plastically deforming than material B. (D) Material B will withstand e higher load than material A but is more likely to fracture suddenly. PROFESSIONAL PUBLICATIONS, INC. MATERIALS SCIENCE 9.27 From the graphs, the modulus of elasticity of material B is greater ‘than that of material A. This means that material A is more duc- tile, that is, it can undergo more strain before fracturing. However, material B can withstand higher loads than material A. Only option (D) is correct. ‘The answer is (D). MATERIALS SCIENCE-49 If the diagram below represents deformation of rigid bodies, what do, yy m, and n refer to? plastic deformation, n plastic deformation, = elastic deformation, n = plastic deformation lnstic deformation (C) 2 = stress, y = strain, m (D) 2 = strain, y = stress, m = elastic deformation, n. Option (C) is the only choice that fits the graph. The answer is (C). PROFESSIONAL PUBLICATIONS, INC. 9.28 1001 SOLVED ENGINEERING FUNDAMENTALS PROBLEMS MATERIALS SCIENCE-50 Which of the following best describes the 0.2% offset yield stress? (A) It is the elastic limit after which a measurable plastic strain has occurred. (B) It is the stress at which the material plastically strains 0.2%. (©) It is the stress at which the material elastically strains 0.2% (D) It is 0.2% below the fracture point of the material By definition, the offset yield stress is where the material undergoes 0.2% plastic strain. The answer is (B). MATERIALS SCIENCE-51 Which of the following is true regarding the ductile-to-brittle transition temper- ature? I. It is important for structures used in cold environments. IL It is the point at which the size of the shear lip or tearing rim goes to UL Its the temperature at which 20 J of energy causes failure in a Charpy v-notch specimen of standard dimensions. (A) Lonty (B) Land I (C)1and Ul (D) Mand II Lis the only choice that is false. A test piece broken at 20 J of energy usually has a small shear lip. ‘The answer is (C). MATERIALS SCIENCE-52 Which of the following are true regarding creep? 1. Itis caused by the diffusion of vacancies to edge dislocations, permitting dislocation climb. IL It involves the plastic deformation of materials at loads below the yield stress, IIL It may involve whole grain sliding. (A) Tonly (B) Monty (© Land (D) I, and PROFESSIONAL PUBLICATIONS, ING. MATERIALS SCIENCE 9.29 MATERIALS SCIENCE-53 Under conditions of very slow deformation and high temperature, it is possible to have plastic flow in a crystal at shear stresses lower than the critical shear stress, What is this phenomenon called? (A) atip (B) twinning (C) ereep (D) bending Creep involves the flow of material. The answer is (C). MATERIALS SCIENCE-54 ‘What does the Charpy impact test measure? L. the energy roquired to break a test sample TL, the strength of a test sample IML. the ductile to brittle transition temperature of metals (A) Tonly (B) Tonly (C) Ml only (D) T and Ut ‘The Charpy test measures toughness, the energy required to break a sample. By condueting the test at different temperatures, the brittle tran- sition temperature can be determined. ‘The answer is (D). PROFESSIONAL PUBLICATIONS, INC. 9-30 1001 SOLVED ENGINEERING FUNDAMENTALS PROBLEMS MATERIALS SCIENCE-55 A shaft made of good quality steo! breaks in half due to fatigue. What would ‘the surface of the fracture site look like? (A) like a cup and cone (B) quite smooth to the unaided eye, with ripples apparent under low-power magnification (C) smooth over most of the surface, with tearing at the location of fracture (D) very jagged and rough ‘Typically, the surface is mostly smooth. Where final fracture took place however, the surface is torn. ‘The answor ie (C). MATERIALS SCIENCE-56 To which of the following can the large discrepancy between the actual and theoretical strengths of metals mainly be attributed? (A) heat (B) dislocations (C) low density _(D) stress direction Although point defects do contribute to the discrepancy in strengths, ‘the major reason for the difference is the presence of dislocations. ‘The answer is (B). MATERIALS SCIENCE-57 ‘The ease with which dislocations are able to move through a crystal under stress accounts for which of the following? 1. ductility Tl. lower yield strength TH. hardness (A) Tonly {B) Monly (©) Monty (D) Land It ‘The ease with which dislocations move through a crystal accounts for its duetility and lower yield strength. ‘The answer is (D). PROFESSIONAL PUBLICATIONS, INC. MATERIALS SCIENCE 931 MATERIALS SCIENCE-58 ‘As the amount of slip increases, additional deformation becomes more difficult ‘and decreases until the plastic flow finally stops. Slip may begin again only if a larger stress Is applied. What is this phenomenon known as? (A) cooling (B) crowding (C) strain hardening (D) twinning ‘This is known as strain hardening =< MATERIALS SCIENCE-59 Which word combination best completes the following sentence? “Plastic deformation of a single crystal occurs either by by but is the more common method.” (A) bending; compression; bending (B) shearing; compression; compression (C) slips twinning; slip (D) twinning; stip; twinning or Bending, compression, and shear are clastic phenomena. Slip is a more common method of plastic deformation than twinning. The answer is (C). MATERIALS SCIENCE-60 ‘Which one of these statements is true for twinning? (A) It occurs at lower shear stresses than (B) It is the most significant form of plastic deformation. (©) Tt cannot be caused by impact or thermal treatment. (D) It frequently occurs in hexagonal close-packed structures. Options (A), (B), and (C) are false, Twinning requires a relatively high shear stress, is much less common than slip, and can be caused by impact or thermal treatment. It occurs in hexagonal close-packed crystal structures. ‘The answer is (D). PROFESSIONAL PUBLICATIONS, INC. 9-32 1001 SOLVED ENGINEERING FUNDAMENTALS PROBLEMS MATERIALS SCIENCE-61 ‘Which of the following does NOT produce vacancies, interstitial defects, or im- purity defects in a material? (A) plastic deformation (B) slow equilibrium cooling (C) quenching (D) increasing the temperature (which increases atomic energy) Slow equilibrium cooling is used to reduce variations in the material. ‘The answer is (B). MATERIALS SCIENCE-62 ‘Which of the following are true statements about the modulus of elasticity, E? (A) It is the same as the rupture modulus. (B) Its the slope of the stress-strain diagram in the linearly elastic region. (C) Itis the ratio of stress to volumetric strain. (D) Its value depends only on the temperature of the material. ‘The modulus of clasticity is equal to the ratio of stross to strain for a particular material. It is the slope of the stress-strain diagram in the linearly elastic region. The answer is (B PROFESSIONAL PUBLICATIONS, INC. MATERIALS SCIENCE 9.33 MATERIALS SCIENCE-63 What is the modulus of elasticity, E, for a composite material in which the fibers ‘take up 20% of the total volume and the load is applied parallel to the fibers as shown? 38 x 10" Pa 45 x 108 Pa Nm (A) 2.7610" Pa (B) 2.95%10! Pa (C) 1.88%10!" Pa (D) 8.45x10" Pa ‘The matrix and fibers experionce the same strain, ¢. The total stress, «@, is the sum of the stresses carried by the fibers and the matrix. EyeVy + Enel -V)) Vy is the fraction of the total volume taken up by the fibers. ‘Thus, 2 = ByV5 + E(t ~ Vy) (1.38 x 10" Pa)(0.2) + (3.45 x 10° Pa)(1 ~ 0.2) = 2.76 x 10! Pa ‘The answer is (A). MATERIALS SCIENCE-64 ‘What isthe proper relationship between the modulus of elasticity, B, the Poisson ratio, v, and the bulk modulus of elasticity, K? (A) B=KQ-2) — (B) B=K(1-») 3K OF (0) B=3K0-%) PROFESSIONAL PUBLICATIONS, INC. 9-34 1001 SOLVED ENGINEERING FUNDAMENTALS PROBLEMS For an clement in triaxial stress, the unit volume change can be ob- tained from Hooke's law. ‘The resultant equation is given by option @). ‘The answer is (D). MATERIALS SCIENCE-65 A crystal is subjected to a tensile lond acting along its axis. a is the angle between the tensile axis and the slip plane as shown, At what value of a will the shear stress in the stip plane be a maximum? F eae slip plano (A) 0° (B) 30° (©) 45° (D) 60° ‘The component of force along the shear surface is equal to Feosc ‘The area of the shear surface, A,, is related to the cross-sectional area, A, by Ay = A/sina. ‘PROFESSIONAL PUBLICATIONS, INC. MATERIALS SCIENCE, 9-35 ‘Taking the first derivative and setting it equal to zero, Be~ (§) estos Pa=0 a= 45° the answer is (C) MATERIALS SCIENCE.-66 An axial stress oz = F/A is applied as shown. Calculate the resolved shear stress, ry, along the slip plane. formal tin 10 slip plane sip plane ty = (©) sin 60° cos 30° (GG) DE) ‘The answer is (D). PROFESSIONAL. PUBLICATIONS, INC. 9.36 1001 SOLVED ENGINEERING FUNDAMENTALS PROBLEMS MATERIALS SCIENCE-67 If G is the shear modulus, b is the magnitude of the Burgers vector, and r is half the distance between particles, what is the local stress, 7, required to bend dislocations around a particle? (a) 2 (B) Gtr © z © & Line tension is given by r= 27/bl. T = Gb? and = 2r. Therefore, br. The answer is (A). MATERIALS SCIENCE-68 Given that d is the distance between dislocations and 6 is the magnitude of the Burgers vector, what is the expression for the misorientation angle @ of a tilt boundary? b b wee-$ aim ont a 6 By definition, tan ‘The answer is (B). MATERIALS SCIENCE-69 Tn general, what are the effects of cold working a metal? (A) increased strength and ductility (B) increased strength, decreased du (C) decreased strength and ductility (D) decreased strength, increased ductility vy ‘The strength of the metal will increase at the expense of a loss in ductility. ‘The answer is (B) PROFESSIONAL PUBLICATIONS, INC. MATERIALS SCIENCE 9.37 MATERIALS SCIENCE-70 Which of the following does cold working a metal cause? a) (B) © (@) ‘The answer is (A). longation of grains in the flow direction, an increase in dislocation density, ‘and an overall increase in energy of the metal clongation of grains in the flow direction, 2 decrease in dislocation density, and an overall decrease in energy of the metal elongation of grains in the Row direction, a decrease in dislocation density, and an overall increase in energy of the metal shortening of grains in the flow direction, a decrease in dislocation density, ‘and an overall decrease in the energy of the metal Cold working a metal produces clongations of grains coupled with increases in both dislocation density and energy. MATERIALS SCIENCE~71 Which of the following statements is FALSE? (A) @) © @) ‘The amount or percentage of cold work cannot be obtained from informa- tion about change in the area or thickness of a metal. ‘The process of applying force to a metal at temperatures below the tem- perature of crystallization in order to plastically deform the metal is called cold working Annealing eliminates most of the defects caused by the cold working of metal. Annealing reduces the hardness of the metal. The percentage of cold work can be caleulated directly from the re- duction in thickness or area of the metal. “The answer is (A). PROFESSIONAL PUBLICATIONS, INC. 9-38 1001 SOLVED ENGINEERING FUNDAMENTALS PROBLEMS MATERIALS SCIENCE-72 Which of the following statements is FALSE? (A) There is a considerable increase in the hardness and the strength of a cold- worked metal. (B) Cold working a metal significantly reduces its ductility. (©) Cold working causes a slight decrease in the density and electrical condue- tivity of a metal. (D) Cold work decreases the yield point of metal. Cold working increases the yield point as well as the strength and hardness of metal. The answer is (D). MATERIALS SCIENCE-73 Which of the following statements is FALSE? (A) Hot working can be regarded as the simultaneous combination of cold work ing and annealing. (B) Hot working increases the density of the metal. (©) One of the primary goals of hot working is to produce a fine-grained prod- uct (D) Hot working causes much strain hardening of the metal. In hot working, the high temperature immediately releases any strain hardening that could occur in the deformation of the metal The answer is (D) PROFESSIONAL PUBLICATIONS, INC. ‘MATERIALS SCIENCE 9:39 MATERIALS SCIENCE-74 Which of the following is FALSE? (A). Grain size is of minor importance in considering the properties of poly- crystalline materials (B) Fine-grained materials usually exhibit greater yield stresses than coarse- rained materials at low temperatures. (C) At high temperatures, grain boundaries become weak, and sliding occurs. (D) Grain boundary sliding is the relative movement of two grains by a shear ‘movement parallel to the grain boundary between them, Grain size is an important factor to consider in understanding the properties of polyerystalline materials because it affects the area and length of the grain boundaries. ‘Tho answer is (A). MATERIALS SCIENCE-75 Which of the following correctly describes atoms located at grain boundaries? (A) ‘They are subjected to the same type of interatomic forces that are present in the interior atoms of the crystal {B) They are located primarily in highly strained and distorted positions. (C) They have a higher free energy than atoms in the undisturbed part of the crystal lattice. (D) Allof the above are correct. All are correct statements regarding atoms at the grain boundary. ‘The answer is (D). MATERIALS SCIENCE-76 ‘What causes the vinyl interiors of automobiles to crack when subjected to pro- longed direct sunlight? (A) the volatilization (evaporation) of plasticizers (B) repetitive expansion and contraction of the plastic (C) oxidation of the plastic by sunlight and axygen (D) all of the above All of the statements are true. The answer is (D) PROFESSIONAL PUBLICATIONS, INC. 9-40 1001 SOLVED ENGINEERING FUNDAMENTALS PROBLEMS | MATERIALS SCIENCE-77 Low-density polyethylene undergoes extensive (over 100%) elongation prior to rupture, while polystyrene undergoes only 1-2% elongation, What is the main reason for this difference? (A) ‘The polyethylene is lews dense. (B) The large styrene groups in the polystyrene prevent slippage. (©) More cross-linking, occurs in the polystyrene, (D) Polyetiylene is loss crystalline. Polystyrene has large styrene groups on the side of its carbon chain. ‘These prevent slippage, making the polystyrene brittle. ‘The answer is (B). MATERIALS SCIENCE-78 Which of the following describe(s) the modulus of elasticity of an elastomer? L Its directly proportional to the number of cross links in the elastomer. IL, Its value increases with temperature. IIL, It is directly proportional to the number of double bonds in the chemical structure. (A) Tonly (B) Monly (©) Monly ——(D) Tand 1 Choice IIT is false, since a double bond prevents rotation along the ond, inhibiting elasticity. The answer is (D). MATERIALS SCIENCE-79 Which statement(s) describe(s) the glass transition temperature? I. It is the temperature at which the rate of volume contraction decreases abruptly. TL It is the temperature at which residual stresses in the glass can bo relieved. TIL It is the point where the material behaves more like a solid than a viscous liquid. (A) Tonly, (Bandit = (©) Tand MT (D) Mand mT PROFESSIONAL PUBLICATIONS, INC. MATERIALS SCIENCE 41 ‘The glass transition temperature isthe point at which the free move- iment: of the glass molecules past each other becomes difficult. The glass begins to act like a solid, increasing in specific volume. ‘The answer is (C).. MATERIALS SCIENCE-80 Ifthe following diagram represents the sintering of the ceramic MgO, what could the curves z.and y refer to? inerease in property ooo 1200" 1400"~—~1600——1800 sintering temperature (°C) (A) 2 = grain size; y = porosity {B) 2 = grain size; y = strength (©) = = porosity; y = grain size {D) © = strength; y = grain size As the sintering temperature increases, the strength of a ceramic will increase first and then drop abruptly. The grain size will increase linearly with rising temperature. ‘The answer is (D). PROFESSIONAL PUBLICATIONS, INC. 942 1001 SOLVED ENGINEERING FUNDAMENTALS PROBLEMS MATERIALS SCIENCE-81 Of the following inorganic glasses, which have tetrahedral lattice structures? Si02, BOs, BeF2, GeOx (A) $i0z and BoOs (B) Si0y and BeF2 (©) Si02, BaOs, and BeF2 (D) 8i02, BeF2, and GeOz SiO2, BeF2, and GeOz have tetrahedral structures. BOs hes an almost triangular structure. ‘The answer is (D). MATERIALS SCIENCE-82 Which of the following is NOT an important criterion for forming a complete inary solid solution? (A) The difference in radii should be less than 15% (B) The constituent elements must have the same crystal structure. (C) The atoms should be close to one another in the periodic table. (D) ‘The difference in atomic numbers should be sina. lll choices except option (D) are criteria for a binary solid solution. ‘The answer is (D). MATERIALS SCIENCE-83 How can an ordered solid solution be distinguished from a compound? (A) In an ordered solid solution, the solute atoms occupy interstitial positions within the lattice. (B) The solute atoms in an ordered solid solution substitute for atoms in the parent lattice, (©). The atoms in an ordered solid solution form layers in the lattice structure. (D) When heated, an ordered solid solution becomes disordered before melting. Unlike a compound, an ordered solid solution becomes disordered when heated. ‘The answer is (D). PROFESSIONAL PUBLICATIONS, INC. MATERIALS SCIENCE 9.43, MATERIALS SCIENCE-84 ‘What is transformed in a eutectoid reaction? (A) One liquid is transformed into two solids of different: composition. (B) A solid becomes a liquid at the eutectic temperature. (C) A liquid becomes a solid at the solidus temperature. (D) A solid becomes a liquid at the liquidus temperature. In a eutectoid reaction, one liquid is transformed into two different solids, The answer is (A) MATERIALS SCIENCE-85 Which of the following is the correct representation of a eutectic cooling reaction? (The subseripts denote different: compositions.) (A) (liquid) — (solid), + (solid) (B) (solid); + (liquid) — (solid)a (C) (solid): —+ (solid)2 + (Solids (D) (solid): + (solids — (solid) A cutectie reaction is the transformation from one liquid phase to two solid phases. ‘The answer is (A) MATERIALS SCIENCE-86 ‘Two pieces of copper are brazed together using a eutectic alloy of copper and silver. The braze material melts at 780°C. If a second braze is attempted in order to attach another piece of copper, which of the following is true? (A) The first: braze will mett if the braze temperature is again 780°C. (B) The braze temperature must be lowered below 780°C. (C) The first braze will partially melt, causing the parts to slide. (D) The first braze will not melt at 780°C, but the second braze will. ‘All compositions of copper and silver other than the eutectic will have ‘a melting point higher than the eutectic temperature. The alloy of the first braze will dissolve somewhat into the copper pieces, changing its composition. It will not melt again at the second braze temperature of 780°C, ‘The answer is (D). PROFESSIONAL PUBLICATIONS, INC. 944 1001 SOLVED ENGINEERING FUNDAMENTALS PROBLEMS MATERIALS SCIENCE-87 On an alloy phase diagram, what is the solidus temperature? (A) The point at which all solids completely reach the liquid stage. (B) The temperature of the liquid phase at which the first solid forms for a given overall composition, (©) The temperature of the solid phase at which the first liquid forms for a given overall composition. (D) The temperature at which the solid is at equilibrium. ‘The solidus temperature is the temperature at which liquid first forms. ‘The answer is (C). MATERIALS SCIENCE-88 In this phase diagram, what can be said about the phases present in regions I, 1, and II? temperature a ‘composition 8 (A) a, 8, and are present in region I. (B) 8 and 7 are present in region II. (C) a, 8, and are prosent in region Ill (D) @ and ¥ are present in region III. 8 and 7 are present in region II. -y is not present in region III, nor is, B present in region I. ‘The answer is (B). ‘| PROFESSIONAL PUBLICATIONS, INC. ‘MATERIALS SCIENCE 945 MATERIALS SCIENCE-89 Given the following phase diagram, determine the percentage of liquid remaining at 600°C that results from the equilibrium cooling of an alloy containing 5% silicon and 95% aluminum, Fiqui temperature (°C) 811%) (A) 0.0% (B) 47% (C) 53% (D) 67% Use the lever rule. At point A there is 1.4% Si and no liquid, while at point B there is 9.1% Si and all liquid. ‘Therefore, A% ereont liquid = i x 100% = 47% Ds ‘The answer is (B). PROFESSIONAL PUBLICATIONS, INC. 9-46 1001 SOLVED ENGINEERING FUNDAMEN ‘TALS PROBLEMS MATERIALS SCIENCE-90 Consider the Ag-Cu phase diagram given. Calculate the equilibrium amount of 6 in an alloy of 30% Ag, 70% Cu at 850°C. ee 1083" 1000 © 200 § 00 & 100 we i * 600 | \ I t 00 ' ' i nm 40040-2030 40°50 60 70 80 90 100 6 cu cuywe se (A) 0.0% (B) 22% (C) 49% (D) 52% At 70% Cu, A = 49% Cu and B = 92% Cu. percent 8 % Cu in alloy — % Cu at point A x 100% % Cu at point B— % Cu at point A 70% — 49% ~ D% = 49% [Ties x 100% = 49% PROFESSIONAL PUBLICATIONS, INC. ‘MATERIALS SCIENCE 9.47 MATERIALS SCIENCE-91 Using the given phase diagram, what are the relative weights of phases a and a for an alloy of 70% B at temperature T;? temperature 0 3 2 3 40 60 60 70 @0 90 100 8 Biwn%) (A) 10% ax, 90% as (B) 30% a1, 70% a2 (C) 50% a, 50% a2 (D) 70% a1, 30% os Let Wa; denote the weight fraction of a and Wa, denote the weight fraction of ag. From the diagram, Cq, = 25% and Ca, = 75%. Then, Woy + Was Wa,Cay +WaxCaz = Co Solving the two equations using Cp = 70%, Coz Co _ TH%— 10% Caz Ca TH — 28% = 10% Wa, PROFESSIONAL PUBLICATIONS, ING. 9.48 1001 SOLVED ENGINEERING FUNDAMENTALS PROBLEMS MATERIALS SCIENCE-92 For 50% B at 1275°C as shown, what is the relative amount of each phase present? 1600 1400 300 1275 1200 temperature °C) 1100 rt fe { 1 t la aa 1 1} { 1 1 t 1 i a L 002030408080 70809000 A 8 B wt 9) (A) 40% liquid, 60% solid (B) 45% liquid, 55% solid (C) 50% liquid, 50% solid (D) 60% liquid, 40% solid From the phase diagram, Ca = 65% and Cz, = 40%. With Cy given as 50%, and denoting the weight fraction of liquid and solid by Wr and Wa, respectively, We+We=1 WiC + Wala = Co Ca~Co _ 65% — 50% We = CXC, ~ 85% = 40% = 60% ‘The answer is (D). PROFESSIONAL PUBLICATIONS, INC. ‘MATERIALS SCIENCE 9-49 MATERIALS SCIENCE-93 ‘Which of the following is NOT a structural class of stecls? (A) carbon (B) high-strength, low-alloy (©) low-alloy (D) tool and die “Tool and dic” steel is an application class, not a structural class. ‘The answer is (D). MATERIALS SCIENCE-94 Which of the following phases of steel has a face-centered cubic structure? (A) ferrite (B) cementite (C) pearlite (D) austenite Only austenite has a face-centered cubic structure. ‘The answer is (D). MATERIALS SCIENCE-95 Low-carbon steels are generally used in the “as rolled” or “as fabricated” state. ‘What is the reason for this? (A) They come in many different shapes and thicknesses. (B) Their strength generally cannot be increased by heat treatment, (C) They degrade severely under heat treatment. (D) Their chromium content is s0 low. Since their strength cannot be increased by heat treatment, low- carbon steels are used as fabricated. ‘The answer is (B) ‘PROFESSIONAL PUBLICATIONS, INC. 9-50 1001 SOLVED ENGINEERING FUNDAMENTALS PROBLEMS MATERIALS SCIENCE-96 ‘The equilibrium cooling of a steel containing 0.8% carbon results in a product with little use because it is extremely brittle. Which of the following is the primary reason for this poor characteristic? (A) ‘The material has not been cold worked. (B) The austenite grains are too small, and the carbide grains are too large. (C) Thick layers of iron carbide surround the coarse ferrite grains. (D) The carbide forms thin plates that are brittle. ‘When hyperoutectoid steels are slow cooled, brittle carbide plates are formed. ‘The answer is (D). MATERIALS SCIENCE-97 Ductile cast iron and gray cast iron both contain 4% carbon. Ductile cast iron, however, has a higher tensile strength and is considerably more duetile. Which of the following is the major difference that accounts for the superior properties of the ductile iron? (A) The gray cast iron contains iron carbide, whereas the ductile iron contains graphite. (B) The gray cast iron contains flakes of graphite, whereas the ductile iron contains spheroids of graphite, (C) The ductile iron is tempered to give better properties (D) The ferrite grains in the gray east iron are excessively large. Gray cast iron contains flakes of graphite while ductile cast iron con- tains spheroids. The difference in the shape of the graphite gives, the ductile cast iron approximately twice the tensile strength and 20 times the ductility of the gray cast iron. The answer is (B). PROFESSIONAL PUBLICATIONS, INC. ‘MATERIALS SCIENCE O51 MATERIALS SCIENCE-98 In preparing a metallographic iron specimen, the grain boundaries are made ‘most: visible by which of the following steps? (A) grinding the sample with silicon carbide abrasive (B) polishing the sample with AlpOs (C) mounting the sample in an epoxy resin mold (D) etching the sample in a 2% solution of nitric acid in aleohol Etching the specimen with nitric acid in alcohol dissolves metal from ‘the surface and preferentially attacks the grain boundaries. It is the last step in the sample preparation process. ‘The answer is (D). MATERIALS SCIENCE-99 Which of the following statements is FALSE? (A) Low-alloy steels are a minor group and are rarely used. (B) Low-alloy steels are used in the heat-treated condition. (©) Low-alloy steels contain small amounts of nickel and chromium. (D) The addition of small amounts of molybdenum to low-alloy steels makes it possible to harden and strengthen thiek pieces of the metal by heat treatment. Low-alloy steels are one of the most commonly used classes of struc tural steels. The answer is (A) MATERIALS SCIENCE-100 ‘Which of the following statements is FALSE? (A) High-strength, low-alloy steels are not as strong as nonalloy, low-carbon steels. (B) Small amounts of copper increase the tensile strength of steels. (C) Small amounts of silicon in steels have little influence on toughness or fabricability. (D) Addition of small amounts of silicon to steel can cause a marked decrease in yield strength of the steel. PROFESSIONAL PUBLICATIONS, INC. 9-52 1001 SOLVED ENGINEERING FUNDAMENTALS PROBLEMS Addition of small amounts of silicon to steel increases both the yield strength and the tensile strength. ‘The answer is (D). MATERIALS SCIENCE-101 ‘Which of the following statements is FALSE? (A) Stainless steels contain large amounts of chromium. (C) The nonmagnetic stainless steels contain large amounts of nickel. (D) Stabilization of the faco-centered cubie crystal structure of stainless steels imparts a nonmagnetic characteristic to the alloy. There are only two basic types of stainless steels: magnetic (marten- sitic or ferritic) and nonmagnetic (austenitic). ‘The answer is (B). MATERIALS SCIENCE-102 For a completely corrosion-resistant: stainless steel, what minimum percentage of chromium in the alloy is required? (A) 11% (B) 3.2% (©) 83% (D) 11% For complete corrosion resistance, the chromium content must be at PROFESSIONAL PUBLICATIONS, INC. ‘MATERIALS SCIENCE 953 MATERIALS SCIENCE-103 ‘Which of the following would most likely require a steel containing 0.0% carbon that has been spheroidized, cold-drawn, and slightly tempered? (A) a bridge beam (B) a water pipe (C) a cutting tool (D) a ball bearing A hypoeutectoid steel that has been worked using the above process hhas good strength and excellent toughness. A cutting tool undergoes tremendous stress loads due to the relatively small contact area. It requires a stronger material than do the other objects. The answer is (C). PROFESSIONAL PUBLICATIONS. INC. MECHANICS 1 0 OF MATERIALS MECHANICS OF MATERIALS-1 Where do stress concentrations occur? I. near the points of application of concentrated loads II, along the entire length of high distributed loads IIL at discontinuities (A) Tand I (B)Tand Il = (C) Wand WE (D) 1, and ITT Stress concentrations occur under concentrated loads and at discon- tinuities, not under distributed loads. ‘The answer is (B). MECHANICS OF MATERIALS-2 ‘What is the definition of normal strain, ¢? (6 is elongation, and Lis the length of the specimen.) L+6 Lis 6 6 (a) =F we ent Me=t Strain is defined as elongation per unit length. ‘The answer is (D). PROFESSIONAL. PUBLICATIONS, INC. 102 1001 SOLVED ENGINEERING FUNDAMENTALS PROBLEMS MECHANICS OF MATERIALS-3 ‘The column shown has a cross-sectional area of 13 m?. What can the approxi- mate maximum load be if the compressive stress cannot exceed 9.6 kPa? F A (A) 120 kv (B) 122 &N (C) 125 kN (D) 130 kN ‘The equation for axial stress is = 1248 KN (125 KN) The answer is (C). MECHANICS OF MATERIALS~4 A copper column of annular cross section has an outer diameter, da, of 5 m, and is subjected to an axial loading of 200 kN. ‘The allowable compressive stress is 144 kPa. The wall thickness, ¢, should be most nearly \—t i! & (A) 0.5 m (B) 0.8 m (C)im (D) 2m PROFESSIONAL PUBLICATIONS, INC. MECHANICS OF MATERIALS 103 For axial stress, ‘Then, ‘Therefore, (im) "The answer is (C). MECHANICS OF MATERIALS-5 ‘What is most: nearly the stress at surface § of the cylindrical object shown? The specific weight of the material is 7 = 76.9 kN/m’. Fr 18kN, Lae | osm surface S “ B 03m 08m (A) 100 kPa (B) 150 kPa (C) 200 kPa (D) 250 kPa PROFESSIONAL PUBLICATIONS, INC. 10-4 1001 SOLVED ENGINEERING FUNDAMENTALS PROBLEMS ‘The stress at surface S is due to the weight of the material above it, in addition to the force F. The total lond is Fou =W4+F = W+F = (v9 5) (J) 3 m0 m) +15 4N = 1072 kN #” G)eanr 250.7 kN/m? (250 kPa) The answer is (D). MECHANICS OF MATERIALS-6 Considering the stress-strain diagram for aluminum, which point is the fracture (A) A (B) B (Qe @)d Point D is where fracture occurs. ‘The answer is (D). PROFESSIONAL PUBLICATIONS, INC. ‘MECHANICS OF MATERIALS 105 MECHANICS OF MATERIALS-7 In astreos-strain diagram, what isthe correct term for the stress level at ¢ = 0.2% offset? (A) the elastic limit (B) the plastic limit, (C) the offset rupture stress (D) the offset yield stress ‘This is known as the offset yield stress. ‘The answer is (D). MECHANICS OF MATERIALS-8 Consider this stress-strain diagram for a carbon steel in tension. Determine the region of perfect plasticity or yielding. { H | i i i c A 1 H { w (A) Oto A (B) AtoB (© BoC (D) CtoD ‘The plastic region is between points A and B. O to A is known as the linear region, B to C is where strain hardening occurs, and C to D is where reduction in area occurs. PROFESSIONAL PUBLICATIONS, INC 10-6 1001 SOLVED ENGINEERING FUNDAMENTALS PROBLEMS MECHANICS OF MATERIALS-9 Under which type of loading does fatigue occur? (A) static load {B) plane load (©) high load (D) repeated load Fatigue occurs under repeated loading cycles. MECHANICS OF MATERIALS-10 A specimen is subjected to a load. When the load is removed, the strain dis- appears. From this information, which of the following can be deduced about ‘this material? (A) It is elastic, {B) It is plastic. (C) Tt has a high modulus of elas (D) It does not obey Hooke’s law. By definition, elasticity is the property of a material by which it returns to its original dimensions during unloading. e answer is (A). MECHANICS OF MATERIALS-11 Which of the following may be tho Poisson ratio of a material? (A) 0.35 (B) 0.52 (©) 0.55 (D) 0.60 ‘The Poisson ratio must be in the range 0 < v < 0.5. Option (A) is, the only answer that satisfies this condition. ‘The answer is (A). PROFESSIONAL PUBLICATIONS, INC. ‘MECHANICS OF MATERIALS 10-7 MECHANICS OF MATERIALS-12 A2m long aluminum bar (modulus of elasticity = 70 GPa) is subjected to a tensile stress of 175 MPa. Find the elongation. (A) 35 mm (B) 5.0 mm (©) 75mm (D) 9.0 mm From Hooke’s law, oun) E°L (175% 10° 3) em 70x 109 = 0.005 m (5.0 mm) ‘The answer is (B). MECHANICS OF MATERIALS-~13 4.600 mm tall thin plate is placed in tension by a 6000 KN force as shown. What is the height (y direction) of the plate while tension is applied? ‘The modulus of elasticity, F, is 200 GPa, and Poisson's ratio, v, is 0.3. Assume the load is distributed uniformly across the plate and the yield streugth is not exoeeded. 25mm aI = | FE <4 a 600 mm 7 section AA. F= 5000 kN (A) 509.7 mm (B) 599.9 mm —(C) 600.2 mm —(D) 600.5 mm PROFESSIONAL PUBLICATIONS, INC. 10-8 1001 SOLVED ENGINEERING FUNDAMENTALS PROBLEMS ‘The Poisson ratio is defined as the negative ratio of lateral strain, ¢y, to axial strain, ¢,. Using this and the equation for axial stress and strain, a a) ‘Combining equations I and II, VE (0.3)(5000 kN) FA (200 x 108 =) (0.015 m2) 0005 ‘Therefore, the width while the plate is in tension is, ‘w= 600 mm — 4, 300 mm — (0.0005)(600 mm) = 599.7 mm The answer is (A). MECHANICS OF MATERIALS-14 What is most nearly the lateral strain, ¢,, of the steol spocimen shown if Fe = 3000 KN, E = 198 GPa, and v = 0.29? i f, ~ ro y A= 0.04 me (A) -4x10-# —(B) -1x10-# —(C) 1x10! (D) ax 10-4 PROFESSIONAL PUBLICATIONS, IN. ‘MECHANICS OF MATERIALS 10-9 From Hooke’s law and the equation for axial stress, oF 000s EEA” (igs 108 BY) (004 04) 3.89 x 10~* ‘Use Poisson's ratio. ‘The answer is (B). MECHANICS OF MATERIALS-15 A steel specimen is subjected to a tensile force, F, of 2000 KN. If Poisson’s ratio, v, is 0.29 and the modulus of elasticity, B, is 193 GPa, the dilatation, e, is most nearly 0.25 m (A) 65x 10- — (B) 88x 10- (CG) 88X10 (D) 6H x 04 Dilatation is defined as the sum of the strain in all three coordinate Gietions, Tn the axial = eto, Foe a ct PA Cosco 8) at PROFESSIONAL PUBLICATIONS, INC. 10-10 1001 SOLVED ENGINEERING FUNDAMENTALS PROBLEMS From Poisson’ ratio, Gaye = ~(0.29)(21 x 104) = -6.09x 10° ‘Therefore, ce bey tee (2.1 x 1074) + (2)(—6.09 x 10° 82K 10°F (8.8 x 10°5) [me answer is (B). MECHANICS OF MATERIALS-16 Given a shear stress of rey = 35000 kPa and a shear modulus of G = 75 GPa, the shear strain is most nearly (A) 25% 10-6 rad (B) 4.7.x 104 rad (©) 5.5 x 1074 rad (D) 8.3 x 10-4 rad Hooke’s law for shear gives ry 3800055 7 @ ie 75 x 108 m = 467x104 rad (4.7 x 107 rad) ‘Tho answer is (B) PROFESSIONAL PUBLICATIONS, INC. MECHANICS OF MATERIALS 10-11 MECHANICS OF MATERIALS~17 A 150 mm diameter rivet resists a shear force of V = 8 KN. Find the average shear stress in the rivet. pi (A) 230 kPa, (B) 370 kPa. (C) 430 kPa (D) 450 kPa. 8iN (0.150 m)? The answers ®)- | MECHANICS OF MATERIALS-18 A steel bar carrying a 3000 KN load, F, is attached to a support by a round pin 0.3 m in diameter. What is most nearly the average shear stress in the pin? 452.7 KN/m? (450 kPa) L Y (A) 10MPa (8) 12 MPa (©) 21 MPa (D) 25 MPa ‘PROFESSIONAL PUBLICATIONS, INC. 10-12 1001 SOLVED ENGINEERING FUNDAMENTALS PROBLEMS ‘The pin wil shear on two cross sections. F 3000 KN 2A ~ (a) (03 me ‘The answer is (C). wd = 21221 KN/m? (21 MPa) MECHANICS OF MATERIALS-19 What is most nearly the maximum allowable load, F 1.5 and the compressive yield stress, cys i8 20670 kPa? the factor of safety is F 125 mm | 125 mm (A) 220 kN (B) 240 kN (©) 300 kN (D) 420 kN - iets allonable = “EE = Sg = 13 780 e/a F=CutowasieA = (19700 5) (0.125 m)? = 215.3 KN (220 KN) ‘The answer is (A). PROFESSIONAL PUBLICATIONS, INC. MECHANICS OF MATERIALS 10-13, MECHANICS OF MATERIALS~20 The allowable tensile stross for a 6.25 mm diameter bolt with a thread length of 5.5 mm is 207 MPa, Tho allowable shear stress of the material is 103 MPa. Where and how will such a bolt be most likely to fail if placod in tension? (Assume threads are perfectly triangular and that the force is carried at the mean thread height.) (A) at the root diameter due to tension (B) at the threads due to shear (C) at the root diameter due to shear (D) at the threads due to tension ‘The bolt will most likely fail due to shearing of the threads or due to tensile failure of the bolt diameter. Foitowable,hread = Tatlowable (average shear area) Tattowabie (3¢h) (108 000 5) (4x) (0.00625 m)(0.0055 m) = 5.56 KN Fattowable,oot = Tellowable(root area) = (wore 8) (2) ootes = 6,35 KN ‘The shear stress in the threads will exceed the allowable stress before the tensile load becomes excessive. ‘The answer is (B). MECHANICS OF MATERIALS-21 Hexagonal nuts for 6.25 mm diameter bolts have a height of .5 mm. If the ultimate strength of the mut material in shear is 108 MPa, what is most nearly te maximum allowable shear force on the nut threads using a safety factor of 5? (A) 0.72 (B) 08 kN (C) 1.0 kN (D) 1.1 kN ‘PROFESSIONAL PUBLICATIONS, INC. 1014 1001 SOLVED ENGINEERING FUNDAMENTALS PROBLEMS Tattowable V = TatowabioA = Tatlowabie ($7dh) = (20600 5) (4x) (0.00625 m)(0.0055 m) 1KN (11 KN) MECHANICS OF MATERIALS-22 Determine the total length, L, of the fillet weld for the lap joint shown. The ‘weld has to resist a tension, F, of 500 KN. The effective throat for the weld, h, is 12 mm, and the allowable stress is 145 MPa, 500 kN 500 kN — — (A) 247 mm. (B) 252 mm. (©) 287 mm (D) 312 mm For a fillet weld, the average normal stress is F aL es = 0.287 m_ (287 mm) ‘The answer is (C). PROFESSIONAL PUSLICATIONS, INC. MECHANICS OF MATERIALS 10-15 MECHANICS OF MATERIALS-23 ‘What is most nearly the elongation of the aluminum bar (cross 3 cm) shown in the figure when loaded to its yield point? E = 69 GPa, and Gyala = 255 MPa. Neglect the weight of the bar. L=25m \ (A) 3.3 mm (B) 9.3 mm (©) 2mm (D) 15 mm From Hooke’s lar, the axial strain is o _ 285 x 10° Pa E~ 69x10" Pa ‘The total elongation is 6 = eL = (0.0087)(2.5 m) .0087 .00925 m_ (9.3 mm) ‘The answer is (B), MECHANICS OF MATERIALS-24 What is most nearly the total elongation of the rod shown if E = 69 GPa? Noglect bending. 1 Lyn 25m Dy = 300 mm —| |. Dy=150mm—e] 2 | [a= 178m fend view eoarei] side view (A) 001mm (8) 005mm —(C) 02 mm (D) 12mm PROFESSIONAL PUBLICATIONS, INC. 1016 1001 SOLVED ENGINEERING FUNDAMENTALS PROBLEMS Fly , Fla _F (Ia , la EA, ° BA, ~ E \A, ~ Ag, =F (5 28) aren) ( * (0 x 108 ®) « = 5.26 x10" m (0.05 mm) Srotal The answer is (B). MECHANICS OF MATERIALS-25 ‘What is most neatly the total elongation of this composite body under a force of 27 KN? Ey = 70 GPa, and E2 = 100 GPa. 05m 19m 1 Fa 17540 ©) a T D, = 0.06 m end view side view (A) 0.075 mm — (B) 073mm = (C) 1.2 mm (D) 3.0 mm ‘Total elongation is the elongation of section 1 plus the elongation of section 2. _ _Fli , Fla Sut = +8 = Fo + ae (176 eN)(0.5 m) (176 KN)(1.0 m) © (G) 0322 (1010) © (F) 00m (100 10 EY) 29x10 m (0.73 mm) PROFESSIONAL PUBLICATIONS, INC. MECHANICS OF MATERIALS 10-17 MECHANICS OF MATERIALS-26 ‘A 200 m cable is suspended vertically. At any point along the cable, the strain is proportional to the length of the cable below that point. If the strain at the ‘top of the cable is 0.001, determine the total elongation of the cable. (A) 0.050 m (B) 0.10 m (©) 0.15 m (D) 0.20 m Since the strain is proportional to the cable length, it varies from 0 at the end to the maximum value of 0.001 at the supports. The average strain is fae _ 0.001 ee = 0.0005 ‘The total elongation is 5 = eave = (0.0005)(200 m) = 0.10 m ‘The answer is (B) MECHANICS OF MATERIALS-27 ‘The figure shows a two-member truss with a load F = 50000 KN applied stat- ically. Given that Ly = 1.2m, Lz = 1.5 m, and each member's cross-sectional area, A, is 4000 mm?, what is most nearly the elongation of member AB after F is applied? Use 2 = 200 GPa. (A) 59mm = (B) 48mm (C) -36mm_—(D) -23 mm. PROFESSIONAL. PUBLICATIONS, INC. 10:18 1001 SOLVED ENGINEERING FUNDAMENTALS PROBLEMS AA free-body diagram of joint A gives A ai fa Frc ly Ran = Fos = F—2_ “ JiR + Ty (60000 be) (1.5 m) 5m) 39043 kN [AB isin compression Vaasa me? : eo Fay = -Rap = ~89009 kN 30 048 KN)(1.2 m) Geox 8) oot -0.0586 m (-59 mm) ‘The answer is (A) MECHANICS OF MATERIALS-28 The two bars shown are perfectly bonded to a common face to form an assembly. The bars have moduli of elasticity and areas as given. If force of F = 1300 kN compresses the assembly, what is mast neatly the reduetion in length? &,= 207 GPa = 2006Pe Ay = 8500 mm? ‘A= 7700 mm? uncompressed Hength = 3m. F (A) 12 mm (B) 14 mm (C) 15 mm (D) 1.6 mm ‘PROFESSIONAL PUBLICATIONS, INC. MECHANICS OF MATERIALS 10-19 From the principle of compatability, both bars are compressed the same length. a-Z Since «1 = &, i) [PROFESSIONAL PUBLICATIONS, INC. 10-20 1001 SOLVED ENGINEERING FUNDAMENTALS PROBLEMS Combining equations I and TI, 1300 KN — Fy = (42) Fy ia (6500 mun?) (207 GPa) = 606.2 KN 606.2 KN (0.0065 mt (207 x 108 EY en 451 x10 § = eL = (451x103 m) = 0.00135 m (1.4 mm) ‘The answer is (B). MECHANICS OF MATERIALS-29 A rigid weightless bar is suspended horizontally by cables 1 and 2 as shown The cross-sectional areas of the cables are given in the figure. The modulus of elasticity, B, is the same for both cables. Ifa concentrated load of F = 1500 kN is applied between points A and B, what is most nearly the distance, x, for the bar to remain horizontal? cable2 ll cable 1 fle ‘Ay = 1800 mm? ‘p= es0mm? =F Az | 1000 mm 600 mm Ay FP = 1500 kN eel x B L______.] ‘3000 mm (A) 1300mm —(B) 1600mm —(C) 1900mm_—(D) 2300 mm PROFESSIONAL PUBLICATIONS, ING. MECHANICS OF MATERIALS 10-21 A wok | Fe Fi a z 3 |__| ‘3000 mm From the free-body diagram, taking moments about point B gives SM (4500 kX) — (3000 mm). (1500 kN). = (3000 mm) Fi 2-2 R)R a ‘From a vertical force balance, Fy + Fh = 1500 &N om For the bar to remain horizontal, the deflection of cable 1 must equal the deflection of cable 2. bab Fily _ Fala EA, EAz ry = 2241 p, — (1000 mm) (1300 mm?) 1 > Tide (600 mm)(650 mm)” = 3.33P> i Solving equations II and III simultaneously, 3.33F, + Fy = 1500 kN 4.39F, = 1500 kN Fy = 346.4 LN Fy = 1158.6 KN PROFESSIONAL PUBLICATIONS, INC. 10-22 1001 SOLVED ENGINEERING FUNDAMENTALS PROBLEMS 2= (ay) - (2 my) (1153.6 kN) = 2307 mm (2300 mm) ‘Phe answer is (D). MECHANICS OF MATERIALS-30 A prismatic bar at 10°C is constrained in a rigid concrete wall at both ends. ‘The bar is 1000 mm long and has a cross-sectional area of 2600 mm?, What is ‘most nearly the axial force in the bar if the temperature is raised to 40°C? A= 2600 mm? © 7000 mm = modulus of elasticity 200 GPe ‘a = coofficiant of thermal exper 94x 10-!7C (A) 116 kN (B) 125 eN (©) 134 uN (D) M47 KN Elongation due to temperature change is given by 6 al(, ~T) = (04 x10-¢ %) (1000 mim)(40°C = 10°C) = 0.262 mm Elongation is sa _ (0.000282) (200 108 z Tm = 146.6 kN (147 KN) ‘The answer is (D) ‘PROFESSIONAL PUBLICATIONS, INC. F MECHANICS OF MATERIALS 10-23 MECHANICS OF MATERIALS-31 ‘What is most nearly the maximum axial load, F, that can be applied to the wood post shown without exceeding a maximum shear stress of 1650 kPa parallel to the grain? (a) 2k (B) 33 KN (©) 44 KN (D) 57 kN ‘The length of the diagonal parallel to the grain, d, (part of a 3-4-5 triangle) is a=08), @ @ d= (3) (100 mm) ‘The area of the inclined plane is 25 mm (0.125 m) A= (0.125 m)(0.100 m) = 0.0125 m? ‘The total shear on the plane is Vara (2030 5) (0.0125 m2) = 20.63 kN PROFESSIONAL PUBLICATIONS, INC. 10-24 1001 SOLVED ENGINEERING FUNDAMENTALS PROBLEMS ZK |e @ ‘The horizontal component of the shear is ve= (3) corsa ay) = 16510 ‘Draw the free-body diagram of the upper section. Include the normal compressive force F ZA Me N Balancing the x-components, Balancing the y-components, Yon =0=N,-F=0 $) ers wm nye awe(! = 22 kN ‘The answer is (A). PROFESSIONAL PUBLICATIONS, INC. MECHANICS OF MATERIALS 10-25 MECHANICS OF MATERIALS-32 ‘The shear strain, ¢, along a shaft is ris the radius from the shaft’s centerline, and dé/dz is the change of the angle of twist with respect to the axis of the shaft. Which condition is NOT necessary for the above equation to be valid! (A) The area of interest must be free of connections and other load applications. (B) The material must be isotropic and homogeneous. (C) The loading must result in the stress being a torsional couple acting along the axis, (D) rmust be the full radius of the shat. ‘The equation may be evaluated for any value of r, giving the stress distribution over the shaft cross section. ‘The answer is (D). MECHANICS OF MATERIALS-33 A 3 m diameter bar experiences a torque of 280 N-m. What is most nearly the ‘maximum shear stress in the bar? L —@ (A) 22 Pa (B) 31 Pa (©) 42 Pa (D) 53 Pa, Maximum shear stress occurs at the outer surface. "The equation for shear gives a 3m Tr 7(5) (280 Nem) (= FT uy = a at Gem) 52.8N/m? (53 Pa) ‘The answer is (D). PROFESSIONAL PUBLICATIONS, INC. 10-26 1001 SOLVED ENGINEERING FUNDAMENTALS PROBLEMS MECHANICS OF MATERIALS-34 ‘What is most nearly the angle of twist, ¢, for the aluminum bar shown? The shear modulus of elasticity, G, is 26 GPa. T= 1kNm d=06m (A) 0.00055° —(B) 0.0055" (©) 0.032 (D) 0.082 The angle of twist is given by TL (11. N-m)(17 m) °F Casio 8) (2) com 0.000565 rad) ( i ) rad, = 0.032" ‘The answer is (C). PROFESSIONAL PUBLICATIONS, INC, ‘MECHANICS OF MATERIALS 10-27 MECHANICS OF MATERIALS-35, ‘What torque, T, should be applied to the end of the steel shaft shown in order to produce a twist of 1.5°? Use @ = 80 GPa for the shear modulus. (A) 420 Nem (B) 560 N-m (©) 830 N-m (D) 1100 Nem Converting the twist angle to radians and calculating the polar moment of inertia J, oy (2m rad 0-05) (2228) - oon ma ry = 0.015 m 2 = 0.025 m x 4) (7 4 . J=5(s-rf) = G) ((0.025 m)* — (0.015 m)*) = 5.34 x 1077 mt Gl re Ze, (0 x10 3) (6.34 x 10-7 m*) = oo (0.026 rad) =1110 Nm (1100 Nem) ‘The answer is (D). PROFESSIONAL PUBLICATIONS, INC. 10-28 1001 SOLVED ENGINEERING FUNDAMENTALS PROBLEMS MECHANICS OF MATERIALS-36 Determine the maximum torque that can be applied to the shaft, given that the ‘maximum angle of twist is 0.0225 rad. Neglect bending, section 1 The angle of twist is J for a circular bar of diameter d is */z1r4 = 1/syrd*. ‘The total angle of twist, dtotat is equal to the sum of the angles of twist for the two different sections. The torque is the same for both sections. eotat = 01 + G2 7G (0.0225 rad) 36 PROFESSIONAL. PUBLICATIONS, INC. MECHANICS OF MATERIALS 10-29 MECHANICS OF MATERIALS-37 For the given shaft, what is most nearly the largest torque that can be applied if the shear stress is not to exceed 110 MPa? (A) 1700 Nm (B) 1900 N-m (©) 2400 Nm (D) 3400. Nem ‘Since the shear stress is largest at the outer diameter, the maximum torque is found using this radius. Tn = 7 For an annular region, T= Ford —rf) = (F) ((0.025 mt — (0.015 my = 5.34 x 1077 mt (6.34 x 10-7 m)* (110% 108 Ty mn 0.085 m = 2850 Nem (2400 N-m) ‘The answer is (C). PROFESSIONAL PUBLICATIONS, INC. 10-80 1001 SOLVED ENGINEERING FUNDAMENTALS PROBLEMS MECHANICS OF MATERIALS-38 A hollow circular bar has an inner radius r; and an outer radius rp. Ifry = ra/2, ‘most nearly what percentage of torque can the shaft carry in comparison with a solid shaft? Lo (3 (A) 25% (B) 55% (©) 75% (D) 95% The cquation for torsional stress is For the hollow shaft, For the solid shaft, PROFESSIONAL PUBLICATIONS, INC. MECHANICS OF MATERIALS 10-31 ‘Therefore, = 0.94 (95%) ‘The answer is (D). MECHANICS OF MATERIALS-39 ‘What is the minimum solid shaft diameter that can be used for the rotor of a 4.5 kW motor operating at 3500 rpm, if the maximum shear stress for the shaft is 60 MPa? (A) 12 mm (B) 2.1 mm (© 10mm (D) 20 mm ‘Th relationship between the power, P, transmitted by a shaft and the torque, T, is - 30 PROFESSIONAL PUBLICATIONS, INC. 10-32 1001 SOLVED ENGINEERING FUNDAMENTALS PROBLEMS ‘nis in rpm, T is in Num, and P is in W. Rearranging to solve for T, _ 30P __ (30)(4500 W) = OY = 12.28 Nem mn x (3500 =) imin, * Ta mas a _ Ta _ nds © Bmax 82 ‘Therefore, ys gar M8 _ | asyc (am) ® (wx 10° 3) =0.0101 m (10 mm) ‘The answer is (C). MECHANICS OF MATERIALS~40 A beam supports a distributed load, w, as shown. Find the shear force at = 2m from the left end, w= BkNIm. POUT (A) 1&N (B) 12 kN (©) 13 uN (P) 15 kN ‘The reactions at A and B are found by observation from symmetry to be Ra = Ry = 25 KN. PROFESSIONAL PUBLICATIONS, INC. MECHANICS OF MATERIALS 10-33 50kN Ry ie = 254N Sectioning the beam at x force is m, the free-body diagram with shear 10K v 25kN DA-0 25 KN — 10 KN-V kN V = 25 KN —10 KN = 15 kN. ‘The answer is (D). MECHANICS OF MATERIALS~41 For the beam shown, find the bending moment, M, at 2 = 3 m. w= 2 kN xe3m (A) 45kNm — (B) 60kNm —(C) 7.5kNm — (D) 9.0 KN PROFESSIONAL. PUBLICATIONS, INC. 10-34 1001 SOLVED ENGINEERING FUNDAMENTALS PROBLEMS By inspection from symmetry, Ry = Rp = 6 KN. Sectioning the beam at z= 3m gives F-(2 Jen kN ToMaa=0 (6 KN)(3 m) + (6 KN)(1.5 m) +M M18 WNm—9kNm Nn ‘The answer is (D). MECHANICS OF MATERIALS~42 Find the expression for the bending moment: as a function of distance from the left end, 2, for the following beam. m . POD j yay (B) Maz? 41 2 + On (D) Mw PROFESSIONAL PUBLICATIONS, INC. MECHANICS OF MATERIALS 10-35 SOM = 22 + (22) (}2) + M=0 M =~2" +22 ‘The answer is (C). MECHANICS OF MATERIALS~43, Which of the following is the shear force diagram for this beam? ein ry B 1m 2m kN (A) al 8 J-2kn SiN (B) al 8 -3kN PROFESSIONAL. PUBLICATIONS, INC. 10-36 1001 SOLVED ENGINEERING FUNDAMENTALS PROBLEMS: @) ! 5kN af ts By observation, the reactions at points A and B are Ra = 4 kN and Ry = 2 KN. Draw free-body diagrams of the left and right sections of, the beam. 2kN A P Pp B Fig= 4k Fy = 2kN Thus, V = +4 KN between points A and P, and V points P and B. 2 KN between ‘The answer is (A). ‘PROFESSIONAL PUBLICATIONS, INC. MECHANICS OF MATERIALS 10-37 MECHANICS OF MATERIALS-—44 Which of the following is the bending moment diagram for this beam? SKN Bs fe .5 KN. The shear force From the froo-body diagram, Ry = Rp = diagram is therefore, +15 kN} 45 kN PROFESSIONAL PUBLICATIONS, INC. 10-38 1001 SOLVED ENGINEERING FUNDAMENTALS PROBLEMS ‘The bending moment increases linearly to (1.5 kN)(2 m) 3 kN, then decreases linearly back to 0 kN-m. kim & The answer is (A). MECHANICS OF MATERIALS-45 ‘The cantilever beam shown is loaded by three concentrated forces. What is the ‘maximum shear force in the beam? p> kN 5kN Bkn inners reer rae (A) 1 kN (B) 2aN (© smn (D) 5 EN ‘Examining the shear force along the beam from left to right, for Om. ballet block (A) Both mechanical energy and momentum are conserved. (B) Mechanical energy is conserved; momentum is not conserved. (C) Momentum is conserved; mechanical energy is not conserved. (D) Neither mechanical energy nor momentum is conserved. Momentum is not conserved since an external force, gravity, acts on the bullet-block mass. Only mechanical energy is conserved. Ths anewer is) | PROFESSIONAL PUBLICATIONS, INC. DYNAMICS 11-29 DYNAMICS-34 ‘Three masses are attached by a weightless cord as shown. If mass ms is exactly halfway between the other masses and is located at the center of the flat sur- face when the masses are released, what is most neatly its initial acceleration? Assume there is no friction in the system and that the pulleys have no mass, Te oy surface m= 16k9 m= 12k9 TET. z (A) 1.0 m/s? (B) 1.2 m/s? (©) 98 m/s? (D) 12. m/s? Tmo ul Tag f = fi oe 20 Since my > mg, m; will move downward and ma will be displaced to the left. All masses contribute to the inertia of the system. Ty ~ Ts = azmotat = a(t + m2 +s) PROFESSIONAL PUBLICATIONS, INC. 11-30 1001 SOLVED ENGINEERING FUNDAMENTALS PROBLEMS. T, and Ty are the tensions in the cord due to the masses my and ms. T,=mg Ty = mag = (081 3) (pes) Ihe + hg +12 ke, = 1.23 m/s” (1.2 m/s) ‘The answer is (B), DYNAMICS-35 ‘The maximum capacity (occupant load) of an elevator is 1000 N. The elevator starts from rest, and its velocity varies with time as shown in the graph, What is most nearly the maximum additional tension in the elevator cable due to the occupants at full capacity? Neglect the mass of the elevator. v (ve) 2 4 6 time(s) (A) 960. (B) 1000. (©) 1200N (D) 400 N ‘The maximum tension occurs during the period of maximum accel ration. This occurs for 0.6 1m. On the surface of the sphere, 7 = 1 m. = pig =o th 7a ‘Gauss’ law states that the electric flux passing through a given closed surface is proportional to the charge enclosed by the surface. ‘There is no charge within the sphere. Therefore, the electric field is zero for r<1m. Only (D) is correct. ‘The answer is (D). PROFESSIONAL PUBLICATIONS, INC. 128 1001 SOLVED ENGINEERING FUNDAMENTALS PROBLEMS DC ELECTRICITY-10 Approximately how far away must an isolated positive point charge of 1 x 10-8 be in order for it to produce an electric potential of 100 V? The charge is in free space with ¢, = 8.85 x 10? C/Nea?. (A)090m = (B) 12m (©) 53m (©) 86m [Ata distance, r, fom a point charge, = = va fea a av & axw-#e ax (3.85 10-2 <—) 00 v) ; Na? 0.90 m ‘The answer is (A). DC ELECTRICITY-11 A point charge, q, in a vacuum creates a potential, V, at a distance, r. A reference voltage of zero is arbitrarily selected when r = a. If K = 1/dne,, which of the following is the correct, expression for V? a) Ke(4-3) ‘The total voltage is measured between the reference voltage, a, and r. - [ea--[ Ber l. ¥ -(!-2) ‘The answer is (C). ‘PROFESSIONAL PUBLICATIONS, INC. DC ELECTRICITY 12.9 DC ELECTRICITY-12 ‘What accelerating voltage is required to accelerate an electron to a kinetic energy of 5 x 10°! J? The charge of an electron is 1.6 x 10"? C. (A) 8 kV (B) 13 kV (© 19K (D) a1 kV For an electron, after the potential energy has been converted to Kinetic energy, kinetic energy is, Ex=a Fx 8x10 J gq 16x10 6 =31250V (31 kV) Ve The answer is (D). DC ELECTRICITY-13 A certain potential variation in the zy plane is given by the expression 1 ov = (—ehes) 45 (x 7) aa Which of the following gives the magnitude and direction (angle made with the r-uxis) of the electric field intensity at the point (2,1)? (A) -v9/4, © (B) 1/2, —n/A (©) VB, m/A— (D) 1/2, w/A Since this is equivalent to the expression given, +a) PROFESSIONAL PUBLICATIONS, INC. 1210 1001 SOLVED ENGINEERING FUNDAMENTALS PROBLEMS Evaluating at the point (2,1), aoa + @a? =1/2 E ‘The angle from horizontal that tho B field is directed is tand = ‘The answer is (D). DC ELECTRICITY-14 An electric dipole is placed in a uniform electric field of intensity, F, Given the information in the figure, what is most nearly the torque acting on the dipole? 2= 63,5 mm 8-30" (A) L7x 10-8 Nm (B) 3.3 x 10°? Nem (©) 48x 10-8 Nom (D) 9.5 x 10-? Nam ‘The torque is T = F(2a)sind F is tho force from the electric field. N F=Bq=009 5x17 4 = 0.09 5 x LTC =015N PROFESSIONAL PUBLICATIONS, INC. DC ELECTRICITY wad Solving for torque, T= (0.5 N)(2) | S22 ) sin g0° 41000 x10? Nn ‘The answer is (D). DC ELECTRICITY-15 Current: i is applied to a long N-turn solenoid with cross-section area A and length d. ‘The magnetic field intensity inside the solenoid is H = Vi/d when d is very large. What is the inductance of this long solenoid in ait? HoNA HoN?A HoNA HoNA wet et oF a @) ‘The magnetic flux passing through one turn of the solenoid is a= fa dS = oHA ‘The total flux enclosed by the NV turns is obtained by summing the contribution of all the turns. ®= 0 = WoNHA ‘The inductance is, ‘The answer is (B). PROFESSIONAL PUBLICATIONS, INC. 1212 1001 SOLVED ENGINEERING FUNDAMENTALS PROBLEMS DC ELECTRICITY~16 Which of the following is NOT a property of magnetic field lines? (A) ‘The field is stronger where the lines are closer together. (B) ‘The lines intersect surfaces of equal intensity at right angles. (C) Magnetic field lines have no beginnings and no ends. (D) The lines cross themselves only at right angles. Magnetic field lines do not cross. ‘Their direction at any given point is unique. ‘The answer is (D). DC ELECTRICITY-17 ‘The tesla is a unit of (A) permittivity (B) capacitance (C) inductance (D) mognetic flux density Tho tesla is a unit of magnotic lux density. The answer is (D). DC ELECTRICITY-18 ‘The south poles of two bar magnets are 7.5 cm apart in air. The magnets are of equal strength and repel each other with a force of 4.9 x 10~' N. What is most nearly the strength of each magnet? (A) 66x10-° Wb (B) 0.86 Wb (©) 11 Wb (D) 53 Wo ‘The force between two magnets of strength My and My is MM, eps? PROFESSIONAL PUBLICATIONS, INC, DC ELECTRICITY 12.13 My = Mz. Therefore, M= Vine (0.075 m)?(4.9 1 = 6.60 x 10“ Wb [unit poles} (A). DC ELECTRICITY-19 ‘A conductor has length of 1 m, electrical resistivity of 0.1 -m, and area of 0.01 m?. A uniform direct current having a density of 100 A/m? flows through this conductor. What is the power loss in the conductor? (ayow iW (© 10W () 100 W ‘The resistance of the conductor is pl _ (0.1 2-m)(1 m) ROA Ol me 102 ‘The current flows through the conductor is JA (100 4) (0.01 m?) <1 ‘Therefore, the power consumed in the conductor is P=PR=(1A)(109) =10W ‘The answer is (C). DC ELECTRICITY-20 For a field given by B = wH (Wb/m?), what is the energy storage per unit volume? au-2 ae -# he ~ Ht Ht 2 = Un oF BU a (Cu PROFESSIONAL PUBLICATIONS, INC. 1214 1001 SOLVED ENGINEERING FUNDAMENTALS PROBLEMS ‘The energy stored in a magnetic field H per unit volume is U 'ABH. Since B = wH, H = B/y. Therefore, ‘The answer is (A). DC ELECTRICITY-21 ‘The magnetic ux density, B, aud the magnetic field intensity, #, have the following relationship, B= po(H +M) Ho is the permeability of free space (in H/m), and M is the magnetic polarization of the material (in A/m). If B is increasing, which of the following may be true about the state of metal X at a value of H = 100 A/m? The B-H curve of the metal is as shown, 100 HiAim) (A) B=0.6 W/m; metal X is nonferrous (B) B=0.6 Wh/m; metal X is ferrous (©) B=1.0 W/m; metal X is nonferrous (D) B=1.0 Wo/m; metal X is ferrous Nonferrous metals do not exhibit hysteresis; hence, metal X is ferrous. ‘The hysteresis curve follows a counterclockwise path. Therefore, for B to be increasing at an H value of 100 A/m, B = 0.6 Wb/m. ‘The answer is (B) PROFESSIONAL PUBLICATIONS, INC. DC ELECTRICITY 12.15 DC ELECTRICITY-22 ‘Two identical coils of radius r are placed at a distance r apart as shown, Such @ configuration is called a Helmholtz coil. Which of the following describes the ‘magnetic field created by passing a uniform current through the assembly? (A) The magnetic field is negligible regardless of the magnitude of T. (B) The magnetic field is zero midway between the two coils. (C) The magnetic field is fairly uniform between the two coils. (D) The magnetic ficld is zero at the centers of the coils. The magnetic ficld between the two coils is the superposition of the field created by each coil. Since the currents in both coils are in the same direction, the direction of each individual magnetic field is also the same, using the right-hand rule. Therofore, the fields will not cancel each other. Since the two coils are circular with their centers aligned, the field between them will be fairly uniform. DC ELECTRICITY-23 Which statement is true? (A) Magnetic fiux lines have sources only. (B) Magnetic Aux lines have sinks only. (C) Magnetic flux lines have both sources and sinks. (D) Magnetie flux lines do not have sources or sinks. Magnetic flux lines are closed loops with no sources or sinks. No known particle produces lines of magnetisin ‘The answer is (D). PROFESSIONAL PUBLICATIONS, INC. 12:16 1001 SOLVED ENGINEERING FUNDAMENTALS PROBLEMS DC ELECTRICITY-24 A charge of 0.75 C passes through a wire every 15 s. What is most nearly the current in the wire? (A) 50mA (B) 9.4 mA (©) 20mA (D) 50 mA Current is the charge per unit time passing through the wire (0760) (10) 1s 0 mA ‘The answer is (D). DC ELECTRICITY-25 ‘What is most nearly the total resistance between points A and B? m=20 A eee nv Aya BV 5 (A) 09 @®) 22 (©) wa (D) 20 ‘The total resistance is the sum of the resistance between points A and C, plus the equivalent resistance of the resistors in parallel between points Cand B. Raorat = Ra + Res || (Ra + Ra) =R+4 : T Re eRe =204 - T sa+50 =204109 29 ‘The answer is (B) ‘PROFESSIONAL PUBLICATIONS, INC. DC ELECTRICITY 1247 DC ELECTRICITY-26 ‘What is the total resistance (as seen by the battery) of the following network? (A) 6.02 (B) 129 (C) 152 @) aan AB is a short circuit. Therefore, the rest of the circuit does not con- tribute to the resistance. The effective circuit is R=60 y= 120 Rectal = Ri + Ra || Ro 1 =hit+q—y Re =694—~+ mating =624+62 =na ‘The answer is (B). PROFESSIONAL PUBLICATIONS, INC. 12:18 1001 SOLVED ENGINEERING FUNDAMENTALS PROBLEMS DC ELECTRICITY-27 In the circuit shown, R= 10 Q, and the electromagnetic force, V, is 2 V. What is most nearly the current, J? a oR 4 lh 2Vv (A) 0.10 A (B) 030A (©) 067.4 (D) 33.4 Regat = B2R= pl ‘Uso Ohm's law. Current: is calculated as voltage divided by total resistance, ‘The answer is (B). PROFESSIONAL PUBLICATIONS, INC. DC ELECTRICITY 12:19 DC ELECTRICITY-28 Find the current passing through the 3.0 resistor. eo A 20 8 6n nv 120 (aoa (B) 03 A (1a (D) 4A Current from a battery will always follow a path of zero resistance in circuit. Instead of flowing through the 3.9 resistor and its neighbor- ing resistors, the current will follow the path BCDE, a short circuit. ‘There will be no current in the resistor. ‘The answer is (A) DC ELECTRICITY-29 What is most nearly the current passing through the 30 9 resistor? co-lk +h ° ¢ (A) 0.0.4 (B) 29 mA, (C) 50 mA. (D) 57 mA PROFESSIONAL PUBLICATIONS, INC. 12-20 1001 SOLVED ENGINEERING FUNDAMENTALS PROBLEMS wo. ‘The circuit is symmetrical, Therefore, a current, J;, flows through the resistors, Rj, and Rs. Another current, Iz, flows through resistor Fz. From Kirchhof’s current law at point A, Yi=0 h=2h Using Kirchhoff’s voltage law around the loop ABCDA. V=Rih+ Roh V = (80 2) fy + (10 2h V = (30 2)(2h) + (10 My 2 = (70h Ty = 0.0286 A Ip = 2h, = (2)(0.0286 A) ( =57 mA ‘The answer is ( PROFESSIONAL PUBLICATIONS, INC. DC ELECTRICITY 12.21 DC BLECTRICITY-30 What is most nearly the current through AB? Vv sa ¢ 0 like 10.0 100 50. 9 + 8 100 ehhh AW 1z0V (A) 054 (1A (3A (Dy 4a a eb; —M—_—] ay ae rea By redrawing the circuit and designating the currents as shown in loop, ACBA, the currents through the remaining loops can be expressed in terms of I;, Jz, and Zs. Since voltage equals resistance multiplied by current, for loop CDAC, 60 V = (5 Q) (Ta ~ Ig) ~ (10 2)(T, + Is) ~ (10 Dy 2h —In+8ly=—12A a For loop BEDAB, 120 V = (10. 9)(h + fr) + (10 9)(1 + J5) + (5 0= 25h, + 10f2 + 101s ~ 120 5h +2 +2 =24 A tu) PROFESSIONAL PUBLICATIONS, INC. 12-22 1001 SOLVED ENGINEERING FUNDAMENTALS PROBLEMS Around loop AGBA, OV=~-6 Mh + (10 Mh + (10 Mis Ty-2hy-2g=0A (mq Observe that adding Eqs. If and III can directly solve fy ‘The answer is (D). DC ELECTRICITY-31 In the circuit shown, what is the current through CD? A-050 B A=100 ¢ R=189 coll 2= 18 a ar en ee (A) 0.20 A (B) 0.60 A (©) 104 (194 A205 8 M=100 ¢ A050 eB m=100 ¢ Fy cell 2= 15 PROFESSIONAL PUBLICATIONS, INC. DC ELECTRICITY 12-28 ‘The method of superposition is used to find the current, I. Let 1 be the current from cell 1, and let Jz be the current from cell 2, Then, 1 =I +p. Short circuiting cell 2 to find J, as shown in illustration (.), the equivalent total resistance is Reotaisa = Pa + Ra || Ro RaRs Pa + Re (1.9) =R+ Short circuiting cell 1 to find Ja, as shown in illustration (b), the equivalent total resistance is, Reotai.2 = Ra || Re + Ra Ris R Ry +h (05 2)(1.5 9) 24 CSMESD) 315.9 15V b= TaBa A The total current is T=h+h=08Atlid = 192A (1.9 A) ‘The answer is (D). PROFESSIONAL PUBLICATIONS, IN. 1224 1001 SOLVED ENGINEERING FUNDAMENTALS PROBLEMS DC ELECTRICITY-32 For the network shown, find the voltage drop from C to D. AM=20, Fyn 5o > m= m0 A= 180 3 (a) 20 @ 25 © sov (©) sov ‘The total resietance ia Reowat = Ra + Ra || (Rs + Rs) 1 =R4+,— Rs t+ Ry 1 =204 5 waren =o v_wy foal = Roa em A ‘Use a current divider to find the current in section CDB. _ Ro Yeon tous (TREE) 202 =(1A) (33) O5 A TovpRs (0.5 A)(5 2) =25¥ ‘The answer is (B). PROFESSIONAL PUBLICATIONS, INC. DC ELECTRICITY 12-25, DC ELECTRICITY-33 Determine the voltage drop across the 4 0 resistor in the network shown. a0 20 an ev 2a(k 6a (A) 43 (8) 67V (ORV (D) 24 ‘The network is redrawn with the currents and circuit points labeled as shown. The current through BE is equal to the sum of currents from AB and CB. fp =h+h=2A+h Kirchhoff's voltage law around loop DCBE gives Veo = Rila + Raloo OV = (2 Mha+ (4 vw = (2M)fa + (412 A+ 1) Ja = -0.333 A [opposite to the direction that it was defined] Ion = 2 A—0.883 A= 1.67 A Vos = (1.67 A)(4 2) =668V (6.7 V) The answer ®B). PROFESSIONAL PUBLICATIONS, INC. 12.28 1001 SOLVED ENGINEERING FUNDAMENTALS PROBLEMS DC ELECTRICITY-34 ‘The voltage at point A in the network shown is most nearly 3a 30 ny (A) Lov (B) 23 (©) 30 () 45 ‘The circuit is redrawn, £ Mn60 yp mnsn 5 Superposition is used to find Ja. hh= ley ~ Inv Igy is the current through BA from the 6 V source, and Liz v is the current through BA from the 12 V source. The equivalent resistances are calculated by short circuit for each voltage source. Roy =Rs+Rj || Re PROFESSIONAL PUBLICATIONS, INC. DC ELECTRICITY 12.27 Riay = Ri + Ra || Rs =R+_ty Re * Te L Tot ale ont 30 Ra 3.0 12V ) (atm) ~ (8) Cer) =984 In = lov — hav =075 A054 Va=Vov~IaRs V—(0.25 A)(6 2) =45V ‘The answer is (D). DC ELECTRICITY-35 ‘What is the voltage drop across the 8 © resistor in the following circuit? ao aa a= wa (a) 8Vv (B) 12V (©) 20V (D) 2V PROFESSIONAL PUBLICATIONS, INC. 12-28 1001 SOLVED ENGINEERING FUNDAMENTALS PROBLEMS eal yy 10.0 Redrawing the circuit as shown, with Ic equal to the component of the current through the 8 { resistor due to the current source, and Jy equal to the component of the current through the resistor due to the voltage source, Isa =Io-ly But, Je =1 A, and Jy =0 A. Therefore, Ino=1A IR=(1A\(89) 8v Van ‘The answer is (A). DC ELECTRICITY-36 Determine the voltage drop across the 6 resistor. an an 20 aah ay ea (A) 60V. (8) 90 (©) 10v (D) Vv PROFESSIONAL PUBLICATIONS, IN. MATERIALS SCIENCE 9.27 ‘From the graphs, the modulus of elasticity of material B is greater ‘than that of material A. This means that material A is more duc~ tile, that is, it can undergo more strain before fracturing. However, ‘material B can withstand higher loads than material A. Only option (D) is correct. [Tae aner is (D). |] MATERIALS SCIENCE-49 Tf Uhe diagram below represents deformation of rigid bodies, what do zx, yy 1, and n refer to? (A) 2 strain, m = plastic deformation, n (B) x = strain, y = stress, m = plastic deformation, n = elastic deformation (C) x = stress, y = strain, m = clastic deformation, n = plastic deformation (D) z= strain, y = stress, m = clastic deformation, n = plastic deformation Option (C) is the only choice that fits the graph. he answer is (C). ‘PROFESSIONAL PUBLICATIONS, INC. 9.28 1001 SOLVED ENGINEERING FUNDAMENTALS PROBLEMS MATERIALS SCIENCE-50 Which of the following best describes the 0.2% offset yield stress? (A) It is the elastic limit after which a measurable plastic strain has occurred, (B) It is the stress at which the material plastically strains 0.2%. (C) It is the stress at which the material elastically strains 0.2%. (D) It is 0.2% below the fracture point of the material. By definition, the offset yield stress is where the material undergoes 0.2% plastic strain, ‘The answer is (B).. MATERIALS SCIENCE-51 Which of the following is true regarding the ductile-to-brittle transition temper- ature? L. It is important for structures used in cold environments. IL, It is the point at which the size of the shear lip or tearing rim goes to ILL, It is the temperature at which 20 J of energy causes failure in a Charpy venotch specimen of standard dimensions. (A) Tonly (B) Tand 1 (C) Lond = (D) Hand II Is the only choice that is false. A test piece broken at 20 J of energy usually has a small shear lip. swer is (C) MATERIALS SCIENCE-52 ‘Which of the following are true regarding creep? LL. Itis caused by the diffusion of vacancies to edge dislocations, permitting dislocation climb. IL It involves the plastic deformation of materials at loads below the yield stress. IIL It may involve whole grain sliding. (A) Tonly. (B) Monty (©) Tend TH == (D) 1,11, and PROFESSIONAL PUBLICATIONS, INC. MATERIALS SCIENCE 9:29 All are true, ‘The answer is (D). MATERIALS SCIENCE-53 Under conditions of very slow deformation and high temperature, it is possible to have plastic flow in a crystal at shear stresses lower than the critical shear stress. What is this phenomenon called? (A) slip (B) twinning ——(C) creep {D) bending Croep involves the flow of material MATERIALS SCIENCE-54 What does the Charpy impact test measure? I. the energy required to break a test sample IL the strength of a test: sample IIL. the ductile to brittle transition temperature of metals (A) Lonly, (B) Tonly (©) Monty () Tend 111 ‘The Charpy test measures toughness, the energy required to break a sample. By conducting the test at different temperatures, the brittle tran- sition temperature can be determined, ‘The answer is (D). PROFESSIONAL PUBLICATIONS, INC. 9-30 1001 SOLVED ENGINEERING FUNDAMENTALS PROBLEMS MATERIALS SCIENCE-65 A shaft made of good quality steel breaks in half due to fatigue. What would the surface of the fracture site look like? (A) like a cup and cone (B) quite smooth to the unaided eye, with ripples apparent under low-power magnification (C) smooth over most of the surface, with tearing at the location of fracture (D) very jagged and rough ‘Typically, the surface is mostly smooth. Where final fracture took place however, the surface is tom. ‘The answer is (C). MATERIALS SCIENCE-56 ‘To which of the following can the large discrepancy between the actual and theoretical strengths of metals mainly be attributed? (A) heat (B) dislocations (C) low density (DD) stress direction Although point defects do contribute to the discrepancy in strengths, ‘the major reason for the difference is the presence of dislocations. ‘The answer is (B). MATERIALS SCIENCE-57 ‘The ease with which dislocations are able to move through a crystal under stress accounts for which of the following? 1. ductility TL lower yield strength IIL. hardness (A) Lonly (B) Tonly (©) Monty (D) Tana 1 ‘The case with which dislocations move through a crystal accounts for its ductility and lower yield strength. ‘The answer is (D). PROFESSIONAL PUBLICATIONS, INC. MATERIALS SCIENCE 9-31 MATERIALS SCIENCE-58 As the amount of slip increases, additional deformation becomes more difficult ‘and decreases until the plastic flow finally stops. Slip may begin again only if a langer stress is applied. What is this phenomenon known as? (A) cooling (B) crowding (© strain hardening (D) twinning ‘This is known as strain hardening The answer is (C). MATERIALS SCTENCE-59 Which word combination best: completes the following sentence? “Plastic deformation of a single crystal occurs either by by , but: is the more common mettiod.” (A) bending; compression; bending, (B) shearing; compression; compression (C) stip; twinning; slip (D) twinning; stip; twinning or Bending, compression, and shear are elastic phenomen’ ‘more common method of plastic deformation than twinning. Slip is a ‘The answer is (C). MATERIALS SCIENCE-60 Which one of these statements is true for twinning? (A) Tt occurs at lower shear stresses than slip. (B) It is the most significant form of plastic deformation. (C) It cannot be caused by impact or thermal treatment. (D) It frequently occurs in hexagonal close-packed structures. Options (A), (B), and (C) are false. Twinning requires a relatively high shear stross, is much less common than slip, and can be caused by impact or thermal treatment. It occurs in hexagonal close-packed al structures, ‘The answer is (D). PROFESSIONAL PUBLICATIONS, INC. 9.32 1001 SOLVED ENGINEERING FUNDAMENTALS PROBLEMS MATERIALS SCIENCE-61 ‘Which of the following does NOT produce vacancies, interstitial defects, or im- purity defects in a material? (A) plastic deformation (B) slow equilibrium cooling (©) quenching (D) increasing the temperature (which increases atomic energy) Slow equilibrium cooling is used to reduce variations in the material. The answer is (B) MATERIALS SCIENCE-62 Which of the following are true statements about the modulus of elasticity, E? (A) It is the same as the rupture modulus, (B) Iti the slope of the stress-strain diagram in the linearly elastic region. (C) It is the ratio of stress to volumetric strain. (D) Its value depends only on the temperature of the material ‘The modulus of elasticity is equal to the ratio of stress to strain for a particular material. It is the slope of the stress-strain diagram in the linearly elastic region. ‘The answer is (B). PROFESSIONAL PUBLICATIONS, INC. MATERIALS SCIENCE 9-33 MATERIALS SCIENCE-63 ‘What is the modulus of elasticity, B, for a composite material in which the fibers take up 20% of the total volume and the load is applied parallel to the fibers as shown? 38 x 10" Pa 45 x 10° Pa Enver Evatt | _— matrix a (A) 2.76x10" Pa (B) 2.95x10"° Pa (C) 1.38x10" Pa (D) 3.45x10" Pa ‘Tho matrix and fibers experionco the same strain, . The total stress, 2, is the sum of the stresses carried by the fibers and the matrix. EyeVz + Eme(1 — V5) Vy is the fraction of the total volume taken up by the fibers. Thus, 2 = ByV5 + Em(l Vy) = (1.38 x 10" Pa)(0.2) + (8.45 x 10° Pa)(1 — 0.2) = 2.76 x 10! Pa ‘The answer is (A). MATERIALS SCIENCE-64 ‘What is the proper relationship between the modulus of elasticity, E, the Poisson ratio, v, and the bulk modulus of elasticity, K? (A) B=KQ-2) — (B) B= K(.-v) 3K Or () B=3Ka-2) PROFESSIONAL PUBLICATIONS, INC. 9-84 1001 SOLVED ENGINEERING FUNDAMENTALS PROBLEMS For an element in triaxial stress, the unit volume change can be ob- tained from Hooke’s law. Tho resultant equation is given by option ). The answer is (D). MATERIALS SCIENCE-65 A crystal is subjected to a tensile load acting along its axis. a is the angle between the tensile axis and the slip plane as shown. At what value of a will the shear stress in the slip plane be a maximum? Fe ieee (a) 0 (B) 30° (© 45° (D) 60° ‘The component of force along the shear surface is equal to F cosa. ‘The area of the shear surface, A,, is related to the cross-sectional area, A, by Ay PROFESSIONAL PUBLICATIONS, INC. MATERIALS SCIENCE 9.35 ‘Taking the first der ive and setting it equal to ero, 8 _(F\ lowe sin? $= (2) (eos? asin?) =0 cos? a — sin? =0 cosa = sina ana" The answer is (C). MATERIALS SCIENCE-66 An axial stress 0, = F/A is applied as shown. Calculate the resolved shear stress, 7,, along the slip plane. slip direction on ‘orm line to slip plane slip plane ssin 60° eos 30° (2) [mercer] PROFESSIONAL PUBLICATIONS, INC. 9.36 1001 SOLVED ENGINEERING FUNDAMENTALS PROBLEMS MATERIALS SCIENCE-67 1fG is the shear modulus, b is the magnitude of the Burgers vector, and ris half the distance between particles, what is the local stress, 7, required to bend dislocations around a particle? Gb or a = (B) Gir Og r @ ine tonsion is given by 7 = 2T/bl. T = Gl? and | = 2r. Therefore, Gor. the answer is (A). MATERIALS SCIENCE-68 Given that d is the distance between dislocations and 6 is the magnitude of the Burgers voctor, what is the expression for the misorientation angle 0 of a tile boundary? a (A) sina (B) tang = ale 6 By definition, tan ‘The answer is (B). MATERIALS SCIENCE-69 In general, what are the effects of cold working a metal? (A) increased strength and ductility (B) increased strength, decreased ductility (C) decreased strength and ductility (D) decreased strength, increased ductility ‘The strength of the metal will increase at the expense of a loss in ductility, ‘The answer is (B), PROFESSIONAL PUBLICATIONS, INC. ‘MATERIALS SCIENCE 9.37 MATERIALS SCIENCE-70 Which of the following does cold working a metal cause? (A) elongation of grains in the flow direction, an increase in dislocation density, and an overall increase in energy of the metal (B) clongation of grains in the flow direction, a decrease in dislocation density, and an overall decrease in energy of the metal (C) elongation of grains in the flow direction, a decrease in dislocation density, and an overall increase in energy of the metal (D) shortening of grains in the flow direction, a decrease in dislocation density, ‘nd an overall decrease in the energy of the metal Cold working @ metal produces elongations of grains coupled with increases in both dislocation density and energy. ‘The answer is (A) MATERIALS SCIENCE-71 Which of the following statements is FALSE? (A) The amount or percentage of cold work cannot be obtained from informa- tion about change in the area or thickness of a metal. (B) The process of applying force to a metal at temperatures below the tem- perature of crystallization in order to plastically deform the metal is called cold working. (©) Annealing eliminates most of the defects caused by the cold working of a metal. (D) Annealing reduces the hardness of the metal. ‘The percentage of cold work can be calculated directly from the ro- duction in thickness or area of the metal. ‘The answer is (A). PROFESSIONAL PUBLICATIONS, INC. 9:38 1001 SOLVED ENGINEERING FUNDAMENTALS PROBLEMS MATERIALS SCIENCE-72 Which of the following statements is FALSE? (A) There is a considerable increase in the hardness and the strength of a cold- worked metal. (B) Cold working a metal significantly reduces its ductility. (C) Cold working causes a slight decrease in the density and electrical conduc- tivity of a metal. (D) Cold work decreases the yield point of metal Cold working increases the y hardness of metal. eld point as well as the strength and ‘Lhe answer is (D). MATERIALS SCIENCE-73 Which of the following statements is FALSE? (A) Hot working can be regarded as the simultancous combination of cold work- ing and annealing. (B) Hot working increases the density of the metal. (C) One of the primary goals of hot working is to produce a fine-grained prod- uct. (D) Hot working causes much strain hardening of the metal. In hot working, the high temperature immediately releases any strain hardening that could occur in the deformation of the metal. ‘The answer is (D). PROFESSIONAL PUBLICATIONS, INC. MATERIALS SCIENCE 9-39 MATERIALS SCIENCE-74 Which of the following is FALSE? (A) Grain size is of minor importance in considering the properties of poly- crystalline materials. (B) Fine-grained materials usually exhibit greater yield stresses than coarse- grained materials at low temperatures, (©) At high temperatures, grain boundaries become weak, and sliding occurs. (D) Grain boundary sliding is the relative movement of two grains by a shear ‘movement parallel to the grain boundary between them. Grain size is an important factor to consider in understanding the properties of polycrystalline materials because it affects the area and length of the genin boundaries. ‘The answer is (A). MATERIALS SCIENCE-75 Which of the following correctly describes atoms located at grain boundaries? (A) They are subjected to the same type of interatomic forces that are present, in the interior atoms of the erystal. (B) They are located primarily in highly strained and distorted positions. (C) They have a higher free energy than atoms in the undisturbed part of the crystal lattice. (D) Allof the above are correct. ‘All are correct statements rogarding atoms at the grain boundary. ‘The answer is (D). MATERIALS SCIENCE-76 ‘What causes the vinyl interiors of automobiles to crack when subjected to pro- longed direct sunlight? (A) the volatilization (evaporation) of plasticizers {B) repetitive expansion and contraction of the plastic (C) oxidation of the plastic by sunlight and oxygen (D) all of the above All of the statements are true. The answer is (D). PROFESSIONAL PUBLICATIONS, INC 9-40 1001 SOLVED ENGINEERING FUNDAMENTALS PROBLEMS MATERIALS SCIENCE-77 Low-density polyethylene undergoes extensive (over 100%) elongation prior to rupture, while polystyrene undergoes only 1-2% elongation. What is the main reason for this difforonce? (A) The polyethylene is less dense. (B) The large styrene groups in the polystyrene prevent slippage. (C) More cross-linking occurs in the polystyrene. (D) Polyethylene is les crystalline. Polystyrene has large styrene groups on the side of its carbon chain. ‘These prevent slippage, making the polystyrene brittle. ‘The answer is (B) MATERIALS SCIENCE-78 ‘Which of the following describe(s) the modulus of elasticity of an elastomer? Lt I, Tts value increases directly proportional to the number of cross links in the elastomer. ith temperature. IIL, Itis directly proportional to the number of double bonds in the chemical structure, (A) Tonly (B) I only (©) Monty (D) and I Choice III is false, since a double bond prevents rotation along the bond, inhibiting elasticity. The answer is (D} MATERIALS SCIENCE-79 Which statement(s) describe(s) the glass transition temperature? I. It is the temperature at which the rate of volume contraction decreases abruptly, IL It is the temperature at which residual stresses in the glass can be relieved. IIL, It is the point where the material behaves more like a solid than a cous liquid. (A) Tonly (B) Tend = (©) Tand WE (D) Mana mir PROFESSIONAL PUBLICATIONS, INC MATERIALS SCIENCE 9.41 ‘Tho glass transition tomperature is the point at which the free move- iment of the glass molecules past each other becomes difficult. The glass begins to act like a solid, increasing in specific volume. ©. ‘The answer MATERIALS SCIENCE-80 If the following diagram represents the sintering of the ceramic MgO, what could the curves z and y refer to? Increase in property 1000 1200~—~1400"~—~—+1600~—~«T B00 sintering temperature °C) (A) 2 = grain size; y = porosity (B) 2 = grain size; y = strength (©) = = porosity; y = grain size (D) 2 = strength; y = grain s [As the sintering temperature increases, the strength of a ceramic will increase first and then drop abruptly. The grain size will increase linearly with rising temperature. The answer is (D). PROFESSIONAL PUBLICATIONS, INC. 9-42 1001 SOLVED ENGINEERING FUNDAMENTALS PROBLEMS MATERIALS SCIENCE-81 Of the following inorganic glasses, which have totrahedral lattice structures? S02, B2Os, BeF2, GeO, (A) SiO, and B20; (B) SiOz and BeF (C) SiO, BaOa, and BeF, (D) $i02, BeF2, and GeO SiO2, Bez, and GeO. have tetrahedral structures. BzOs has an almost triangular structure. MATERIALS SCIENCE-82 Which of the following is NOT an important criterion for forming a complete binary solid solution? (A) ‘The difference in radii should be less than 15% (B) ‘The constituent elements must have the same crystal structure. (C) The atoms should be close to one another in the periodic table. (D) The difference in atomic numbers should be small. All choices except option (D) are eriteria for a binary solid solution, ‘The answer is (D). MATERIALS SCIENCE-83 How can an ordered solid solution be distinguished from a compound? (A) In an ordered solid solution, the solute atoms occupy interstitial positions within the lattice. (B) The solute atoms in an ordered solid solution substitute for atoms in the parent lattice, (C) The atoms in an ordered solid solution form layers in the lattice structure. (D) When heated, an ordered solid solution becomes disordered before melting. Unlike a compound, an ordered solid solution becomes disordered when heated, ‘The answer is (D). PROFESSIONAL PUBLICATIONS, INC. MATERIALS SCIENCE, 943 MATERIALS SCIENCE-84 ‘What is transformed in a eutectoid reaction? (A) One liquid is transformed into two solids of different: composition. (B) A solid becomes a liquid at the eutectic temperature. (©) A liquid becomes a solid at the solidus temperature. (D) A solid becomes a liquid at the liquidus temperature. Inn eutectoid reaction, one liquid is transformed into two different solids. "The answer is (A) MATERIALS SCIENCE-85 Which of the following is the correct representation of a eutectic cooling reaction? (The subscripts denote different compositions.) (A) (liquid) —+ (solid): + (solid) (B) (solid): + (liquid) —+ (solid) (C) (solid); —+ (Golid)2 + (Golid)s (D) (solid): + (sotid)2 — (solid) A cutectic reaction is the transformation from one liquid phase to two solid phases. The answer is (A). MATERIALS SCTENCE-86 ‘Two pieces of copper are brazed together using a eutectic alloy of copper and silver. The braze material melts at 780°C. If a second braze is attempted in order to attach another piece of copper, which of the following is true? (A) The first braze will melt if the braze temperature is again 780°C. (B) The braze temperature must be lowered below 780°C. (C) The first braze will partially melt, causing the parts to slide. (D) The first braze will not melt at 780°C, but the second braze will. All compositions of copper and silver other than the eutectic will have ‘a melting point higher than the eutectic temperature. The alloy of the first braze will dissolve somewhat into the copper pieces, changing its composition. It will not melt again at the second braze temperature of 780°C. ‘The answer is (D). PROFESSIONAL PUBUCATIONS, INC. 44 1001 SOLVED ENGINEERING FUNDAMENTALS PROBLEMS MATERIALS SCIENCE-87 On an alloy phase diagram, what is the solidus temperature? (A) The point at which all solids completely reach the liquid stage. (B) The temperature of the liquid phase at which the first solid forms for a given overall composition. (C) The temperature of the solid phase at which the first liquid forms for a given overall composition. (D) The temperature at which the solid is at equilibrium. The solidus temperature is the temperature at which liquid first forms. ‘The answer is (C). MATERIALS SCIENCE-88 In this phase diagram, what can be said about the phases present in regions I, TI, and IN? temperature ® ‘composition 8 (A) a, 8, and 7 are present in rogion I. (B) and 7 are present in region II. (©) a A, and 7 are present in region 1. (D) a and 7 are present in region III. 8 and y are present in region II. + is not present in region III, nor is, B present in region I. ‘The answer is (B). PROFESSIONAL PUBLICATIONS, INC. MATERIALS SCIENCE 9-45 MATERIALS SCIENCE-89 Given the following phase diagram, determine the percentage of liquid remaining at 600°C that results from the equilibrium cooling of an alloy containing 5% silicon and 95% aluminum, ‘empereture (’C) (A) 0.0% (B) 47% (C) 53% {D) 67% Use the lever rule. At point A there is 1.4% Si and no liquid, while at point B there is 9.1% Si and all liquid. Therefore, 5% — 1.4% percent liquid = Spe x 100% = 47% ‘The answer is (B). PROFESSIONAL PUBLICATIONS, INC. 9.46 1001 SOLVED ENGINEERING FUNDAMENTALS PROBLEMS MATERIALS SCIENCE-90 Consider the Ag-Cu phase diagram given. Calculate the equilibrium amount of Gin an alloy of 30% Ag, 70% Cu at 850°C. ‘100 fast 1000 900 800 700 ‘emperature 0) 600 1 t H ' 1 500 | 400 t t ' t t 0 0 0 20 3) 40 60 60 70 a0 9% 4g cu cu iwe 8) (A) 0.0% (B) 22% (©) 49% (D) 52% At 70% Cu, A = 49% Cu and B = 92% Cu. % Cu in alloy ~ % Cu at point A Porcent = 5 Cy at point B= % Cu at point A x 100% 70% ~ 49% 2% — 49% ‘The answer is ( x 100% = 49% PROFESSIONAL PUBLICATIONS, INC. MATERIALS SCIENCE, 947 MATERIALS SCIENCE-91 ‘Using the given phase diagram, what are the relative weights of phases a1 and a for an alloy of 70% B at temperature T,? temperature o 1 20 30 40 80 60 70 80 90 100 8 Bit %) (A) 10% ay, 90% ag (B) 30% a, 70% a2 (C) 50% a, 50% a2 (D) 70% a1, 30% ae Let Wa, denote the weight fraction of a1 and Wa, denote the weight fraction of ag. From the diagram, Cq, = 25% and Cg, = 75%. Then, Was + Way =1 Wo: Car + WaxCax = Co Solving the two equations using Cy = 70%, Cay — Co _ 75% — 70% Wor = Gt Cas ~ 15% = 25% ‘The answer PROFESSIONAL PUBLICATIONS, INC. 9-48 1001 SOLVED ENGINEERING FUNDAMENTALS PROBLEMS MATERIALS SCIENCE-92 For 50% B at 1275°C as shown, what is the relative amount of each phase resent? 1600 1400 1300 1278 [. temperature (*0) i ‘100 ' ' 1 i 1 1 ! fa ' ' { ' ' ' 1 , tt 01020 30 4080-60 70-0 80 700 A 8 Ba) (A) 40% liquid, 60% solid (B) 45% liquid, 58% solid (C) 50% liquid, 50% solid (D) 60% liquid, 40% solid From the phase diagram, Ca = 65% and Cr, = 40%. With Co given as 50%, and denoting the weight fraction of liquid and solid by Wz and Wa, respectively, Wi +We=1 WiC1 + WaCa = Co Ca Cy _ 65% ~ 50% We CG, = 5% — 40% = 60% ‘The answer is (D). PROFESSIONAL PUBLICATIONS, INC. ‘MATERIALS SCIENCE 9.49 MATERIALS SCIENCE-93 Which of the following is NOT a structural class of steels? (A) carbon (B) high-strength, low-alloy (©) low-alloy (D) tool and die “Tool and die” steel is an application class, not a structural class. ‘The answer is (D). MATERIALS SCIENCE-94 Which of the following phases of steel has a faco-centered cuble structure? (A) ferrite (B) cementite (C) pearlite (D) austenite Only austenite has a face-centered cubic structure. ‘The answer is (D). MATERIALS SCIENCE-95 Low-carbon steels are generally used in the “as rolled” or “as fabricated” state. ‘What is the reason for this? (A) They come in many different shapes and thicknesses. (B) Their strongth generally cannot be increased by heat treatment. (C) They degrade severely under heat treatment. (D) Their chromium content is 50 low. Since their strength cannot be increased by heat treatment, low- carbon steels are used as fabricated. ‘The answer is (B). PROFESSIONAL PUBLICATIONS, ING. 9-50 1001 SOLVED ENGINEERING FUNDAMENTALS PROBLEMS MATERIALS SCIENCE-96 ‘The equilibrium cooling of a steel containing 0.8% carbon results in a product with little use because it is extremely brittle. Which of the following is the primary reason for this poor characteristic? (A) The material has not been cold worked. (B) The austenite grains are too small, and the carbide grains are too large. (O) Thick layers of iron carbide surround the coarse ferrite grains (D) The carbide forms thin plates that are brittle. ‘When hypereutectoid stocls aro slow cooled, brittle carbide plates are formed. ‘The answer is (D). MATERIALS SCIENCE-97 Ductile cast iron and gray cast iron both contain 4% carbon. Ductile cast iron, however, has a higher tensile strength and is considerably more ductile. Which of the following is the major difference that accounts for the superior properties of the ductile iron? (A) The gray cast iron contains iron carbide, whereas the ductile iron contains wraphite. (B) The gray cast iron contains flakes of graphite, whereas the ductile iron contains spheroids of graphite. (C) The ductile iron is tempered to give better properties. (D) The ferrite grains in the gray cast iron are excessively large. Gray cast iron contains flakes of graphite while duetile cast iron con- tains spheroids. ‘The difference in the shape of the graphite gives the ductile cast iron approximately twice the tensile strength and 20 times the ductility of the gray cast iron ‘The answer is (B) PROFESSIONAL PUBLICATIONS, INC. MATERIALS SCIENCE 9-51 MATERIALS SCIENCE-98 In preparing metallographic iron specimen, the grain boundaries are made ‘most visible by which of the following steps? (A) grinding the sample with silicon carbide abrasive (B) polishing the sample with AlzOs (C) mounting the sample in an epoxy resin mold (D) etching the sample in a 2% solution of nitric acid in aleohol Etching the specimen with nitric acid in alcohol dissolves metal from the surface and preferentially attacks the grain boundaries. It is the last step in the sample preparation process. The answer i (D) MATERIALS SCIENCE-99 ‘Which of the following statements is FALSE? (A) Low-alloy steels are a minor group and are rarely used. (B) Low-alloy steels are used in the heat-treated condition, (C) Low-alloy steels contain small amounts of nickel and chromaium. (D) The addition of small amounts of molybdenum to low-alloy steels makes it possible to harden and strengthen thick pieces of the metal by heat treatment. Low-alloy steels are one of the most commonly used classes of struc- tral steels. ‘The answer is (A} MATERIALS SCIENCE-100 Which of the following statements is FALSE? (A) High-strength, low-alloy steels are not as strong as nonally, low-carbon steels. (B) Small amounts of copper increase the tensile strength of stools. (©) Small amounts of silicon in steels have little influence on toughness or fabricability. (D) Addition of small amounts of in yield strength of the steel. PROFESSIONAL PUBLICATIONS, INC. 952 1001 SOLVED ENGINEERING FUNDAMENTALS PROBLEMS Addition of small amounts of silicon to steel increases both the yield strength and the tensile strength. ‘The answer is (D). MATERIALS SCIENCE-101 ‘Which of the following statements is FALSE? (A) Stainless steels contain large amounts of chromium. (B) ‘There are three basic types of stainless steels: martensitic, austenitic, and feeritic. (©) The nonmagnetic stainless steels contain large amounts of nickel (D) Stabilization of the face-centered cubic crystal structure of stainless steels imparts a nonmagnetic characteristic to the alloy. There are only two basic types of stainless stools: magnetic (marten- sitic or ferritic) and nonmagnetic (austenitic). (B) ‘The answer MATERIALS SCIENCE-102 For a completely corrosion-resistant stainless steel, what minimum percontage of chromium in the alloy is required? (A) 1.1% (B) 3.2% (©) 83% (D) 1% For complete corrosion resistance, the chromium content: must be at, least 11%. ‘The answer is (D). PROFESSIONAL PUBLICATIONS, INC. MATERIALS SCIENCE 9.53 MATERIALS SCIENCE-103 Which of the following would most likely require a steel containing 0.6% carbon that has been spheroidized, cold-drawn, and slightly tempered? (A) a bridge beam (B) a water pipe (©) cutting tool (D) a ball bearing A hypoeutectoid steel that has been worked using the above process has good strength and excellent toughness. A eutting tool undergoes tremendous stress loads due to the relatively small contact area. It requires a stronger material than do the other objects. The answer is (C). PROFESSIONAL PUBLICATIONS, INC. MECHANICS 1 0 OF MATERIALS MECHANICS OF MATERIALS-1 ‘Where do stress concentrations occur? I. near the points of application of concentrated loads IL. along the entire length of high distributed loads IIL. at discontinuities (A) Tand I (B) Tand 1M = (C) Wand (D) 1,11, and OT Stress concentrations occur under concentrated loads and at discon- tinuities, not under distributed loads. ‘The answer is (B). MECHANICS OF MATERIALS-2 What is the definition of normal strain, ¢? (6 is elongation, and Lis the length of the specimen.) (a) = Et8 @ «= 446 O« (D) « 6 LT r Strain is defined as elongation per u The answer is (D) PROFESSIONAL PUBLICATIONS, INC. 102 1001 SOLVED ENGINEERING FUNDAMENTALS PROBLEMS MECHANICS OF MATERIALS-3 ‘The column shown has a cross-sectional area of 13 m?, What can the approxi- ‘mate maximum load be if the compressive stress cannot exceed 9.6 kPa? F A (A) 120 kN (B) 122 kN (©) 125 kN (D) 130 eN ‘The equation for axial stress is F Cr" Food ~(00 38) 030 =124.8KN (125 kN) ‘The answer is (C). MECHANICS OF MATERIALS-4 A copper column of annular cross section hos an outer diameter, da, of 6 m, and is subjected to an axial loading of 200 KN. The allowable comprossive stross is 14.4 kPa, The wall thickness, t, should be most nearly e) ~~ @ (A) 05 m (B) 08 m © im (D) 2m PROFESSIONAL. PUBLICATIONS, INC. MECHANICS OF MATERIALS 10-3 oak a For axial stress, ‘Then, ~ (A200 NY = (us) ‘The answer is (C) MECHANICS OF MATERIALS-5 ‘What is most nearly the stress at surface § of the cylindrical object shown? The specific weight of the material is 7 = 76.9 kN/m®. Lam} | (A) 100 kPa (B) 150 kPa. (C) 200 kPa (D) 250 kPa. PROFESSIONAL PUBLICATIONS, INC. 10-4 1001 SOLVED ENGINEERING FUNDAMENTALS PROBLEMS ‘The stress at surface $ is due to the weight of the material above it in addition to the force F. The total load is Row =W+F=W+F = (29 8) (J) (03 my?(0.5 m) +15 aN = 17.72 kN rere G)oam 250.7 KN/m? (250 kPa) ‘The answer is (D). MECHANICS OF MATERIALS-6 Considering the stress-strain diagram for aluninum, which point is the fracture point? (AA ()B oc (D) D Point D is where fracture occurs. ‘The answer is (D). PROFESSIONAL PUBLICATIONS, INC. MECHANICS OF MATERIALS 105 MECHANICS OF MATERIALS-7 Inastress-strain di offset? (A) the elastic limit (B) the plastic limit, (C) the offset rupture stress (D) the offset yield stress sram, what is the correct term for the stress level at = 0.2% ‘This is known as the offset yield stress. ‘The answer is (D). MECHANICS OF MATERIALS-8 Consider this stress-strain diagram for a carbon steel in tension, Determine the rogion of perfect plasticity or yielding. 1 i { { i D (A) OtoA (B) AtwoB (C) Bor (D) CtoD The plastic region is between points A and B. O to A is known as the linear region, B to C is where strain hardening occurs, and C to D is, ‘where reduction in area occurs. ‘The answer is (B). PROFESSIONAL PUBLICATIONS, INC. 106 1001 SOLVED ENGINEERING FUNDAMENTALS PROBLEMS MECHANICS OF MATERIALS-9 Under which type of loading does fatigue occur? (A) static load (B) plane load (©) high toad (D) repeated load Fatigue occurs under repeated loading cycles. ‘The answer is (D).. MECHANICS OF MATERIALS-10 A specimen is subjected to a load. When the load is removed, the strain dis- appears. From this information, which of the following can be deduced about this material? (A) Te is clastic. (B) It is plastic. (C) It has a high modulus of elasticity. (D) It does not obey Hooke'’s law. By definition, elasticity is the property of a material by which it returns to its original dimensions during unloading. ‘The answer is (A). MECHANICS OF MATERIALS-11 ‘Which of the following may be the Poisson ratio of a material? (A) 0.35 (B) 0.52 (©) 0.55 (D) 0.60 ‘The Poisson ratio must be in the range 0 < v < 0.5. Option (A) is the only answer that satisfies this condition. ‘The answer is (A) PROFESSIONAL PUBLICATIONS, INC. MECHANICS OF MATERIALS 10-7 MECHANICS OF MATERIALS-12 ‘A.2m long aluminum bar (modulus of elasticity = 70 GPa) is subjected to a tensile stress of 175 MPa. Find the elongation. (A) 3.5 mm (B) 5.0mm (©) 7.5 mm (D) 9.0 mm From Hooke's law, ble L (x7 x 10° XS (2m) wx 0.005 m (5.0 mm) ‘The answer is (B). MECHANICS OF MATERIALS-13 A600 mm tall thin plate is placed in tension by a 5000 KN force as shown. What is the height (y direction) of the plate while tension is applied? ‘The modulus of clasticity, F, is 200 GPa, and Poisson's ratio, v, is 0.3. Assume the lond is distributed uniformly across the plate and the yield strength is not exceeded. 25mm ptr lhe F F — ee 600 mm Coa soction AA F= 5000 kN (A) 509.7 mm —(B) 599.9 mm —(C) 600.2 mm —(D) 600.5 mm PROFESSIONAL PUBLICATIONS, INC. 10-8 1001 SOLVED ENGINEERING FUNDAMENTALS PROBLEMS ‘The Poisson ratio is defined as the negative ratio of lateral strain, ¢y, to axial strain, ¢-. Using this and the equation for axial stross and strain, (a ener: (0.3)(5000 KN) "TEL eos 28) 082 ‘Therefore, the width while the plate is in tension is w= 600 mm ~ 4, = 600 mm ~ (0,0005)(600 mm) 599.7 mm ‘The answer is (A). MECHANICS OF MATERIALS-14 What is most nearly the lateral strain, ey, of the steol specimen shown if Fe = 3000 KN, E = 193 GPa, and v = 0.29? (A) -4x10-# —(B) -1x10-# (1x10! = (D) ax 10-4 ‘PROFESSIONAL. PUBLICATIONS, INC, MECHANICS OF MATERIALS 10-9 From Hooke’s Inw and the equation for axial stress, qeuuk 3000 kN EEA (190% 10 3 (0.04 m2) 3.89 x 10-4 Use Poisson's ratio. vex = ~(0.20)(3.89 x 10~*) 13x 10-* (-1x 10-4) a= ‘The answer is (B). MECHANICS OF MATERIALS-15 {A steel specimen is subjected to a tensile force, F, of 2000 KN. If Poisson’s ratio, V, is 0.29 and the modulus of elasticity, B, is 193 GPa, the dilatation, e, is most nearly 0.25 m (A) 65x10-$ (B) 88x10" — (©) BB 10-4 (D) 6.5 x 10-4 Dilatation is defined as the sum of the strain in all three coordinate directions. In the axial z direction, FE 2000 kN (0.049 m2) 241x104 PROFESSIONAL PUBLICATIONS, INC. 10-10 1001 SOLVED ENGINEERING FUNDAMENTALS PROBLEMS From Poisson's ratio, = —(0.29)(24 x 10-4) = 6.09 x 10-8 ‘Therefore, ete bee = (21x 10-4) + (2)¢ 8.82 10-* (8.8 x 10°) MECHANICS OF MATERIALS-16 an a shear stress of Tay = 35000 kPa and a shear modulus of G = 75 GPa, the shear strain is most nearly (A) 25x 10-6 rad (B) 4.7 10-4 rad (©) 5.5 x 10-4 rad (D) 8.3 x 10-4 rad .09 x 10~ Hooke’s Invr for shear gives Ny 35000 <5 im 75 x 108 ea B 67 x 10-4 rad (4.7 x 10~ rad) ‘The answer is (B). PROFESSIONAL PUBLICATIONS, INC. MECHANICS OF MATERIALS 10-11 MECHANICS OF MATERIALS-17 A 150 mm diameter rivet resists a shear force of V = 8 kN. Find the average shear stross in the rivet. (A) 280kPa—(B) S7DKPa——(C) 430 KPa (D) 450 kPa SN = 450.7 N/m? (450 KPa) (J) @.150 me ‘The answer is (D). MECHANICS OF MATERIALS-18 ‘A steel bar carrying a 3000 KN load, F,, is attached to a support by a round pin 0.3 m in diameter. What is most nearly the average shear stress in the pin? Lr d= 03m (A) 10 MPa (B) 12 MPa (©) 1 MPa (D) 25 MPa, PROFESSIONAL PUBLICATIONS, INC. 10:12 1001 SOLVED ENGINEERING FUNDAMENTALS PROBLEMS: ‘The pin will shear on two cross sections. F 3000 kN 24" a) osm? "1221 KN/m? (21 MPa) ‘The answer is (C) MECHANICS OF MATERIALS-19 What is most nearly the maximum allowable load, F, if the factor of safety is 1.5 and the compressive yield stress, oye, is 20670 kPa? F 125mm L__] 125 mm (A) 220 kN (B) 240 kN (©) 300 kN (D) 420 kN SF kN 20670 eemea tb = 13780 KN/m? F = CatowanteA (a7 38) (0150 = 215.9 KN (220 KN) ‘The answer is (A). PROFESSIONAL. PUBLICATIONS, INC. MECHANICS OF MATERIALS 10-13, MECHANICS OF MATERIALS-20 ‘The allowable tensile stress for a 6.25 mm diameter bolt with a thread length of 5.5 mm is 207 MPa, The allowable shear streee of the material is 103 MPa. Where and how will such a bolt be most likely to fail if placed in tension? (Assume threads are perfectly triangular and that the force is carried at the mean thread height.) (A) at the root diameter due to tension (B) at the threads due to shear (C) at the root diameter due to shear (D) at the threads due to tension ‘The bolt will most likely fail due to shearing of the threads or due to tensile failure of the bolt diameter. Fatowabla thro = Tallowabie(average shear area) Taowabte (34h) - (103000 8) }) (0.006 26 m)(0.0058 m) 5.56 KN Fanowabte,root = Tetlowabio(TOot area) = (207000 2) (F) 00625 my? = 6.35 kN ‘The shear stress in the thrends will exceed the allowable stress before the tensile load becomes excessive. ‘The answer is (B) MECHANICS OF MATERIALS-21 Hexagonal nuts for 6.25 mm diameter bolts have a height of 6.5 mm. If the ultimate strength of the nut material in shear is 103 MPa, what is most nearly the maximum allowable shear force on the nut threads using a safety factor of 5? (A) 0.72 kN (B) 08 kN (©) 10 kN (D) 1.1 kN ‘PROFESSIONAL PUBLICATIONS, INC. 10-14 1001 SOLVED ENGINEERING FUNDAMENTALS PROBLEMS Tatowable = 5 soon Hf 5 = 20600 kN/m? V = TationableA = Tattowabte (34h) = (200 =) (4x) (0.00625 m)(0.0055 m) = 111 KN (1.1 KN) ‘The answer is (D). MECHANICS OF MATERIALS-22 Determine the total length, L, of the fillet weld for the lap joint shown. ‘The weld has to resist. a tension, F, of 500 KN. The effective throat for the weld, h, is 12 mm, and the allowable stress is 145 MPa, 500 kN 500 kN = = (A) 247 oun. (B) 252 mm. (©) 287 mm () 312 mm Por a fille weld, the average normal stress is Ee P S00KN ( 45.000 5) (0.012 m) =0.287 m_ (287 mm) PROFESSIONAL PUBLICATIONS, INC. MECHANICS OF MATERIALS 10-15 MECHANICS OF MATERIALS-23 ‘What is most nearly the elongation of the aluminum bar (cross section of $ em x 3 cm) shown in the figure when loaded to its yield point? F = 69 GPa, and Oyied = 255 MPa, Noglect the weight of the bar. L=25m (A) 33mm (B) 9.3 mm (©) 12mm (D) 15mm From Hooke’ law, the axial strain is © _ 285 x 10° Pa 6" Bo Goxt0 Pa ~ O87 ‘The total elongation is (0.0087)(2.5 m) = 0.00925 m (9.3 mm) ‘The answer is (B). MECHANICS OF MATERIALS-24 What is most nearly the total elongation of the rod shown if E Neglect bending, 1 y= 25m © D, = 300 mm -+| | newemn|? [fae om = 27h, side view (A) 0.01 mm (B) 0.05 mm (©) 02mm (0) 12mm PROFESSIONAL PUBLICATIONS, INC. 10-16 1001 SOLVED ENGINEERING FUNDAMENTALS PROBLEMS F(t, fa Prorat E\a,* A; iy) ( =5.26x 10-5 m_ (0.05 mm) "The answer is (B). MECHANICS OF MATERIALS-25 ‘What is most nearly the total elongation of this composite body under a force of 27 KN? E; = 70 GPa, and E = 100 GPa. sm tom i i Fe se © jo T 1 0, =008m cvaviow Dy= 012m sow (A) 0.075 mm (B) 0.73 mm (C) 1.2mm (D) 3.0 mm ‘Total elongation the clongation of seton 1 pus the elongation of setin 2 Fly | Fly a s (175 kN)(0.5 m) a (175 KN)(1.0 m) (J) 12 mp (20x10 EY) © () (0.06 > (100 10° #3) =729% 10-4 m (0.73 mm) ‘The answer is (B). PROFESSIONAL. PUBLICATIONS, INC. ‘MECHANICS OF MATERIALS 10-17 MECHANICS OF MATERIALS-26 ‘A 200 m cable is suspended vertically. At any point along the cable, the strain is proportional to the length of the cable below that point. If the strain at the top of the cable is 0.001, determine the total elongation of the cable. (A) 0.050 m (B) 0.10 m (©) 0.15 m () 0.20 m Since the strain is proportional to the cable length, it varies from 0 at, the end to the maximum value of 0.001 at the supports. The average strain is = Emax _ 0.001 Cag = SE mw = 0.0005 ‘The total elongation is 5 = eave = (0.0005)(200 m) =0.10 m The answer is (B). MECHANICS OF MATERIALS-27 ‘The figure shows a two-member truss with a load F = 50000 KN applied stat- ically. Given that Ly = 1.2 m, [2 = 1.5 m, and each member's cross-sectional area, A, is 4000 mm?, what is most nearly the elongation of member AB after F is applied? Use £ — 200 GPa. (A) -59mm —(B) -48mm_ = (C) -36mm_—(D) -23 mm PROFESSIONAL PUBLICATIONS, INC. 10-18 1001 SOLVED ENGINEERING FUNDAMENTALS PROBLEMS A free-body dingram of joint A gives iF Fa Fae a VTE (60000 KN)(1.5 m) (GPmy + (15 my —Rap = -39043 kN (-39043 KN)(1.2 (os 0° ) 00 Ray = Feos0 = 19043 KN [AB is in compression} Fan Sap 0.0586 m (—59 mm) The answer is (A). MECHANICS OF MATERIALS-28 ‘The two bars shown are perfectly bonded to a common face to form an assembly. The bars have moduli of elasticity and areas as given. Ifa force of F = 1300 kN compresses the assembly, what is most nearly the reduction in length? E, = 207 GPa = 2006Pe ‘Ay = 8500 mm? Ay= 7700 mm? uncompressed length = 3m (A) 12mm (8) 14mm (© 15m (D) 1.6 mm PROFESSIONAL. PUBLICATIONS, INC. ‘MECHANICS OF MATERIALS 10-19 ‘From the principle of compatability, both bars are compressed the same length. a2 a-2 Since €1 = 2, From a force balance, Fy + Fy = 1300 kN g Fy = 1300 KN- Fy PROFESSIONAL PUBLICATIONS, INC. 10-20 1001 SOLVED ENGINEERING FUNDAMENTALS PROBLEMS Combining equations I and II, 1300 kN ~ Fy = (48) cy Ba 1300 KN = (+ aE) A AaB, 1300 KN ~ AE 149 1300 kN = (7700 mm")\(200 Pay 1+ 500 mm?)(207 GPa) = 606.2 kN 6 (0.0065 m?) (x x 108 5) kN a= =451 x10 L = (4.51 x 10-4)(3 m) = 0.00135 m (1.4 mm) 6 ‘The answer is (B), MECHANICS OF MATERIALS-29 A rigid weightless bar is suspended horizontally by cables 1 and 2 as shown, ‘The cross-sectional areas of the cables are given in the figure. The modulus of elasticity, H, is the same for both cables. If a concentrated load of F = 1500 KN is applied between points A and B, what is most nearly the distance, z, for the bar to remain horizontal? coble2 Mill (A) 1300 mm —(B) 1600 mm (©) 1900 mm (D) 2300 mm PROFESSIONAL PUBLICATIONS, INC. MECHANICS OF MATERIALS 10-21 "3000 mm From the free-body diagram, taking moments about point B gives = (1500 kN) — (8000 mm)F, (3000 mm)F, mm: =CR)A t From a vertical force balance, Fi + P= 1500 kN im For the bar to remain horizontal, the deflection of cable 1 must equal ‘the deflection of cable 2. (2000 mm) (1300 mm?! "(600 mm)(650 mm?) Fy au Solving equations II and ITT simultaneously, 3.33Fy + Fa = 1500 kN 4.33F, = 1500 kN Fy = 346.4 WN Fy = 1153.6 KN PROFESSIONAL PUBLICATIONS, INC. 10-22 1001 SOLVED ENGINEERING FUNDAMENTALS PROBLEMS 2=(Gn)F er) = 2307 mm (2300 mm) ‘Phe answer is (D). MECHANICS OF MATERIALS-30 A prismatic bar at 10°C is constrained in a rigid concrete wall at: both ends. ‘The bar is 1000 mm long and has a cross-sectional area of 2600 mm?. What is ‘most nearly the axial force in the bar if the temperature is raised to 40°C? y A= 2600 mm? 6 7000 mm 4 = modulus of elasticity = 200 Ge «= coofficiont of thermal expansion = 94x 10-*7C (A) use B) AN (CAND) TAN Elongation due to temperature change is given by S=al(t-Ti) SI (04 x10-% Fa) (1000 mm)(40°C — 10°C) = 0.282 mm Blongation is FL EA wy sa _ (2000282 ) (200 2° 5) L Tm = 146.6 KN (147 kN) The answer is (D) PROFESSIONAL PUBLICATIONS, IN. 6 126 m*) MECHANICS OF MATERIALS 10-28 MECHANICS OF MATERIALS-81 ‘What is most nearly the maximum axial load, F, that ean be applied to the wood post shown without exceeding a maximum shear stress of 1650 kPa parallel to ‘the grain? 1400 mm 100 mm (A) 2N (B) 39 kN (© 44aN (D) 87 kN ‘The length of the diagonal parallel to the grain, d, (part of a 34-5 triangle) is d=) @ @ d= (3) (100 mm) = 125 mm (0.125 m) ‘The area of the inclined plane is A= (0.125 m)(0.100 m) = 0.0125 m? ‘The total shenr on the plane is kN’ v= ra (1088) on = 206048 PROFESSIONAL PUBLICATIONS, INC. 10-24 1001 SOLVED ENGINEERING FUNDAMENTALS PROBLEMS Ze Nm @ ‘The horizontal component of the shear is ve (3) (20.63 KN) = 16.5 kN Draw the free-body diagram of the upper section. Include the normal compressive force. ue Fe Me v Balancing the z-components, N= (8) (16.5 kN) = 27.5 kN Balancing the y-components, DOR =0=N,-F=0 Pend (8) (27.5 kN) = 22 kN ‘The answer is (A). PROFESSIONAL PUBLICATIONS, INC. ‘MECHANICS OF MATERIALS 10-25 MECHANICS OF MATERIALS-32 ‘The shear strain, ¢, along a shaft is ris the radius from the shaft’s centerline, and dg/dat is the change of the angle of twist with respect to the axis of the shaft. Which condition is NOT necessary for the above equation to be valid? (A) The area of interest must be free of connections and other load applications, (B) The material must be isotropic and homogeneous. (©) The londing must result in the stress being a torsional couple acting along the axis, (D) rust be the full radius of the shaft. ‘The equation may be evaluated for any value of r, giving the stress distribution over the shaft cross section. ‘The answer is (D} MECHANICS OF MATERIALS-83 A 3:m diameter bar experiences a torque of 280 N-m, What is most nearly the ‘maximum shear stress in the bar? : -® (A) 2.2 Pa (B) 31 Pa (C) 42 Pa (D) 53 Pa ‘Maximum shear stress occurs at the outer surface. ‘The equation for shear gives 7 em Bt = 528.N/m? (63 Pa) PROFESSIONAL PUBLICATIONS, INC. 10-26 1001 SOLVED ENGINEERING FUNDAMENTALS PROBLEMS. MECHANICS OF MATERIALS-34 ‘What is most nearly the angle of twist, ¢, for the aluminum bar shown? The shear modulus of elasticity, G, is 26 GPa. (A) 0.00055" (B) 0.0055" (©) 0.032° (D) 0.082 ‘The angle of twist is given by TL a (11 KN-m)(17 m) (20% 10° £3) (5) 06 my sos way ( 18° = (0.000565 rad) ( =i) = 0.032" ‘The answer is (C). PROFESSIONAL PUBLICATIONS, INC. MECHANICS OF MATERIALS 10.27 MECHANICS OF MATERIALS-35 ‘What torque, 7’, should be applied to the end of the steel shaft shown in order to produce a twist of 1.5°7 Use G = 80 GPa for the shear modulus. L=10m outside 60mm (A) 420m (B) B60 Nem (C) 830Nem_—(D) 1100 Nam Converting the twist angle to radians and calculating the polar moment of inertia J, ‘Qn rad » (228) =m na = (1 ry = 0.015 m rp = 0.025 m J= 3 (i-r) = (3) ((0.025 m)* — (0.015 m4) =3.34x 1077 mt Gl T= Tr? (# x10? 3) (6:34 x 10-7 mé) a (0.026 rad) 1110 N-m (1100 Num) ‘The answer is (D). PROFESSIONAL PUBLICATIONS, INC. 10-28 1001 SOLVED ENGINEERING FUNDAMENTALS PROBLEMS MECHANICS OF MATERIALS-36 Determine the maximum torque that can be applied to the shaft, given that the ‘maximum angle of twist is 0.0225 rad. Noglect bending. section 1 T 1G a (A) 0.000 62552 (8) 0.05007 md *G (© 0.2507 (©) 0557 ‘The angle of twist is (oe GI J for a circular bar of diameter d is ¥/arr4 = ¥/aand!. The total angle of twist, dota, is equal to the sum of the angles of twist for the two different soctions. The torque is the same for both sections. rota = 01+ 2 TEL) | TL Gh” Gh - (Fe) Ga) G(0.0225 rad) 36L 1G = 0.000 6257 PROFESSIONAL PUBLICATIONS, INC. MECHANICS OF MATERIALS 10-29 MECHANICS OF MATERIALS-37 For the given shaft, what is most nearly the largest torque that can be applied if the shear stress is not to exceed 110 MPa? = f= 0015m (S = 0025 m (A) 1700.N-m —(B) 1900 Nm (C) 2400 Nem (D) 3400 Nem Since the shear stress is largest at the outer diameter, the maximum torque is found using this radius. wd Tova = For an annular region, Je ze -rf)= ) ((0.025 m)* — (0.015 my = 5.34 x 1077 m* (6x30 (0 xa S) oe 0.025 = 2850 Nem (2400 N-in) ‘The answer is (C) PROFESSIONAL PUBLICATIONS, INC. 10-30 1001 SOLVED ENGINEERING FUNDAMENTALS PROBLEMS MECHANICS OF MATERIALS-38 Ahollow circular bar has an inner radius r; and an outer radius ra. Ifr1 = 12/2, ‘most nearly what percentage of torque can the shaft carry in comparison with a solid shaft? Ne (3 (A) 25% (B) 55% (©) 75% (D) 98% ‘The equation for torsional stress is For the hollow shaft, For the solid shaft, PROFESSIONAL PUBLICATIONS, INC. MECHANICS OF MATERIALS 10-31 ‘Therefore, 4-3)" 4 ad ~Ssi6 _ iid cs 8 ~ 6 = 0.94 (95%) ‘The answer is (D). MECHANICS OF MATERIALS-39 ‘What ie the minimum eolid shaft diameter that can be used for the rotor of a 4.5 kW motor operating at 3500 rpm, if the maximum shear stress for the shaft is 60 MPa? (A) 12 mm (B) 21 mm (©) 10 mm (D) 20 mm ‘The relationship between the power, P, transmitted by a shaft and the torque, T, is an PD PROFESSIONAL PUBLICATIONS, INC. 10-32 1001 SOLVED ENGINEERING FUNDAMENTALS PROBLEMS ‘nis in rpm, T is in Nom, and P is in W. Rearranging to solve for T, 30P _ (30)(4500 W) SOP (80)(4500 W) _ 19.08 Nam m5 (s500 =) ini. eee Tmax = “7 = 37 Te nat Bmax ‘Therefore, vs or \u8 sor “-(m) > =0.0101 m (10 mm) (10)(22.28 Nea) x (w x 108 FS) ‘The answer is (C). MECHANICS OF MATERIALS~40 A beam supports a distributed load, w, as shown. Find the shear force at. = 2m from the left end. {ooo Be (B) 12kN (©) 13 kN (D) 15 kN (A) kN ‘The reactions at A and B are found by observation from symmetry 2 = 25 KN. to be Ra PROFESSIONAL PUBLICATIONS, INC. MECHANICS OF MATERIALS 10-33 50 kN Fa fe Fy Py = 25KN Sectioning the beam at x = 2m, the free-body diagram with shear force is 10KN v 25kN DR-0 25 KN -10kN-V =OkN V = 25 KN- 10 KN =15 KN The answer is (D). MECHANICS OF MATERIALS-41 For the beam shown, find the bending moment, M, at x = 3m. 218m £ 1. am om (A) 45kNm — (B) 6.0kKNm — (C) 7.5kKNm —(D) 9.0 KNm PROFESSIONAL PUBLICATIONS, INC 10-34 1001 SOLVED ENGINEERING FUNDAMENTALS PROBLEMS By inspection from symmetry, Ra = Ris at 2 =3 m gives 6 KN. Sectioning the beam #= (28) am om TMs = 0 (6 KN)(3 m) + (6 KN)(L5 m) +M=0 18 kN-m ~ 9 kN-m 9kNm ‘The answer is (D). MECHANICS OF MATERIALS~42 Find the expression for the bending moment as a function of distance from the left end, :, for the following beam. [> ain v : 1) Fa6in (A) M=-23 +22 (B) M=-274+1 (©) M=-2? 420 PROFESSIONAL PUBLICATIONS, INC. ‘MECHANICS OF MATERIALS WOM = -22 + (22) (fz) + M=0 M =a? 422 ‘Tho answer is (C). MECHANICS OF MATERIALS~43 ‘Which of the following is the shear force diagram for this beam? om a 3 poof : Tm Tm an (A) Al . Law si (B) Al 2 Louw PROFESSIONAL PUBLICATIONS, INC. 10-35 10-36 (Dd) 1001 SOLVED ENGINEERING FUNDAMENTALS PROBLEMS TaN, = al 8 SKN KN a 8 Is kn By observation, the reactions at points A and B are Ra = 4 kN and Rg =2 KN. Draw free-body diagrams of the left. and right sections of the beam. VesOkN Ve -2kN A Pp fe 8 Fgm AkN Fg = 2kN ‘Thus, V = +4 kN between points A and P, and V = -2 KN between points P and B. ‘The answer is (A) PROFESSIONAL PUBLICATIONS, INC. MECHANICS OF MATERIALS 10.37 MECHANICS OF MATERIALS-44 ‘Which of the following is the bending moment diagram for this beam? on 2m 2m Ls, 34m aim © @ (Dp) 2kNm B 8 « 8 3k fy % From the free-body diagram, Ra = Rp = 1.5 KN. The shear force diagram is therefore, 1B KN, 48 kN PROFESSIONAL PUBLICATIONS, INC. 10-38 1001 SOLVED ENGINEERING FUNDAMENTALS PROBLEMS ‘The bending moment increases linearly to ( KN)(2 m) = 3 KN, then decreases linearly back to 0 KN-m. 3kNm x ‘The answer is (A). MECHANICS OF MATERIALS-45 ‘The cantilever beam shown is loaded by three concentrated forces. What is the ‘maximum shear force in the beam? 7 SKN kN BkN 2m 2m 2m 2m (A) 1k (B) 2uN (©) 3kN (D) 5 kN Examining the shear force along the beam from left to right, for 0 m <2<2m, V=0KN For2m<2<4m, V=-3kN Fordm<2<6m, V=5mN For6m<2<8m, V=0KN PROFESSIONAL. PUBLICATIONS, INC. MECHANICS OF MATERIALS 10-39 45 KN 3kN ‘The maximum shear force is, therefore, 5 KN. ‘The answer is (D). MECHANICS OF MATERIALS-46 Which of the following bending moment diagrams corresponds to the simply supported beam shown? ‘The beam is subjected to a distributed load, w, between points B and C. a | PROFESSIONAL PUBLICATIONS, INC. 10-40 1001 SOLVED ENGINEERING FUNDAMENTALS PROBLEMS © 4 a aI Te. ' ! ©) 4 ! Ie a a For sections AB and CD, the beasn may be modeled as a) oN ie GY pay f) Ve Ve M(c) is parabolic, reaching a maximum near or at the center. ‘Tho answer is (C). PROFESSIONAL PUBLICATIONS, IN. MECHANICS OF MATERIALS 10-41 MECHANICS OF MATERIALS-47 What is most nearly the maximum allowable load, F', on the cantilever? ‘The maximum compressive stress is 7000 kPa, and the maximum tensile stress is 5600 kPa, The moment of inertia about the centroidal axis, Ia, is 20.6 x 10° mm. 101.6 mm ee 4 fp 635mm en } m- - A j._____| 101.6mm 3m Boa mm Jun = 20.8 X 108 mt cross section (A) 540. (B) 600N (©) 60N (D) GoN 1 00x10 mnt) ( 10.6 x 10-§ m* PROFESSIONAL PUBLICATIONS, INC. 10-42 1001 SOLVED ENGINEERING FUNDAMENTALS PROBLEMS For compression, Gsttowable = 7000 kPa, 01.6 mm + 50.8 mm — 63.5 mm = $8.9 mm (z000 5) (20.6 x 10-6 m4) sore = my OETA) F, 0.541 KN (541 N) For tension, Cynowable = 5500 kPa, and ¢ = 63.5 mm. (208) 0° m allowable = "C3 mn)(0.0656 m) F, = 0.595 kN (600 N) ‘The maximum allowable load is 540 N. ‘The answer is (A). MECHANICS OF MATERIALS-48 A simply supported beam with the cross section shown supports a concentrated load, F = 10 KN, at its center, C. What is most nearly the maximum bending stress in the beam? F=10kN A 8 € 2m 2m PROFESSIONAL PUBLICATIONS, INC. MECHANICS OF MATERIALS 10-43 "200 mm cr0a8 section (A) 2500 kPa (B) 3200 KPa (C) 8800kP2—(D) 4600 kPa ‘The reactions at A and Bare Ra = Rp = 5 KN by inspection from symmetry. Since the maximum bending moment occurs at C, Minnx = (5 KN)(2 m) = 10 kN-m ‘The moment of inertia about the neutral axis, NA, is the difference be- ‘sveen the moments of incrtia of an area measuring 20 mm x 500 mim and two areas measuring 85 mn x 450 mm. Is _ ()e~ mm)(500 mm)? — (35) ‘mm)(450 mm)* = 792 x 10° mm* Since ¢ = (3) (500 mm) = 250 mm, Me _ (10 kN-m) (0.25 m! Comes = 7 92 x 10-6 mt = 8157 KN/m? (8160 kPa) ‘The answer (B). PROFESSIONAL PUBLICATIONS, INC. 10-44 1001 SOLVED ENGINEERING FUNDAMENTALS PROBLEMS MECHANICS OF MATERIALS-49 For the cantilever beam shown, what is the maximum tensile bending stress? — w= 10 kW 100mm 2mm cross section (A) 230MPa —(B) 320MPa_-—(C) 480MPa_-——(D) 550 MPa. PROFESSIONAL PUBLICATIONS, ING. MECHANICS OF MATERIALS 10-45 ‘The maximum moment occurs at point A and is a result of the distributed load w. w is equivalent to a concentrated load, W (2/2)(6 m)(10 KN/m) = 30 KN, acting at a point (1/3)(6 m) = 2m from point. A. The equivalent loading diagram for the cantilever is as, follows. 30 kN —— Mg = (30 KN)(2 m) = 60 kNen ‘The upper part of the beam will be under tension, with c equal to the distance between the neutral axis, NA, and the top edge of the bean. (25 mm)(100 mm)(12.6 mm) + (25 mm)(75 mm)(62.5 mm) 2500 mm? + 1875 mm* = 38.9 mm (0.0839 em) ‘Use the parallel axis theorem to find J. : Ian ee (som mm)(25 mm)? + (100 mm)(25 mm)(34 mm ~ 12.5 mm)? - (3) (25 mm)(75 mm)* + (75 mm)(25 mm)(62.5 mm — 34 mm)? = 3.7.x 10 mm* Me ____ (60 kN-m)(0.0339 m) al 000 =) = 5.5% 10? kPa (550 MPa) (87% 10" mms ( ‘The answer is (D). PROFESSIONAL PUBLICATIONS, INC. 10-46 1001 SOLVED ENGINEERING FUNDAMENTALS PROBLEMS MECHANICS OF MATERIALS-50 ‘A composite beam made of steel and wood is subjected to a uniform distributed load, w. Determine the maximum compressive stress in the ste. be : : wm o wom cross section (A) 620MPa—(B) 850 MPa (C) 1100 MPa. (D) 1200 MPa ‘The maximum moment is atthe center ofthe beam, where x = 2 m. Ry = Ry =80KN by inspection kv ) (2 m)( m) Mraax = (60 RN)(2 m) — (= = = 50 kNem. PROFESSIONAL PUBLICATIONS, INC. ‘MECHANICS OF MATERIALS 10-47 Since Byood/Emesi = 4 MPa/200 MPa = 1/50, the wood is equivalent to a steel web 11mm thick, 100 mm A mm 10mm tT ‘50mm cross section ° 1= 98 = (J) (60 mm)(ao0 my? ~ (2) (7) (25 mmy(20 mn? = 2.076 x 108 mm* (60 kN-m)(0.05 ma) (2000 x) (2.076 x 10° mm*) ( = 1200 x 10% N/m? (1200 MPa) ‘Tho answer is (D) MECHANICS OF MATERIALS-51 For a rectangular beam under transverse (bending) loading, where is the location of maximum shear stress? (A) at the top edge (B) at the bottom edge (©) at the neutral axis (D) at a location between the top edge and the neutral axis PROFESSIONAL PUBLICATIONS, INC. 10-48 1001 SOLVED ENGINEERING FUNDAMENTALS PROBLEMS ‘The shear distribution is na-fonn ‘cross section shear distribution ‘The maximum chear stross ie at the neutral axis, The answer is (C). MECHANICS OF MATERIALS-52 ‘An I-beam is loaded as shown. What is most nearly the maximum shear stress, r, im the web at point C along the beam? rr —— (A) 160 kPa (B) 370 KPa (C) 400 KPa (D) 750 kPa ‘The zeaction at point A is found by taking the moment about point B. Mp =0 Ra Vo -Ra(3 m) + (2 KN)(2 m) + (1 KN)(1 m) .67 kN t= L067 KN PROFESSIONAL PUBLICATIONS, INC. MECHANICS OF MATERIALS 10-49 Tho shear stress is given by 7 = V@Q/It, where Q is the first moment of either the upper half or the lower half of the cross-sectional area with respect to the neutral axis. Q = A’g = (60 mom)(20 mm)(26 mm) + (100 mam)(20 man)(60 man) 45000 mm? ee (3) (100 mm)(140 mm)? — (2) (5) (40 mm)(100 mm)? = 16.2 x 10° mm* Finax OCCUTS at the nentral axis, Thus, VQ _ _(1670 N)(145.000 mm’) Tmax = Te = 62x 10° mm) (20 mr) = 0.747 N/mm? (750 kPa) ‘The answer is (D). MECHANICS OF MATERIALS-53 An I-beam is made of three planks, each 20 mm x 100 mm in cross section, lod together with a single row of nails on top and bottom as shown. If the longitudinal spacing between the nails is 25 mm, and the vertical shear force ‘acting on the cross section is 600 N, what is most nearly the load in shear per nail, F? 100 mm ti nen 20mm —e] 100mm ee (A) 56N (®) 76N (© 10N (0) 160 ‘The shear force per unit distance along the beam’s axis is given by vq TT PROFESSIONAL PUBLICATIONS, INC. 10-50 1001 SOLVED ENGINEERING FUNDAMENTALS PROBLEMS: For an I-beam, Q is the first moment of the upper flange area with respect to the 2-axis, @= Ag = (60 mm)(100 mm)(20 mm) = 120000 mm* re (3) (100 mm)(140 mm)* — (2) (3) (40 mm)(100 men)* = 16.2 10° mm" (600 N)(120000 mm* 16.2 x 10° mmé f = 4.44 N/mm The load capacity of the nails per unit length is F/D. Therefore, P= Lf = (2mm) (« a =m1N (10N) The answer is (C). MECHANICS OF MATERIALS~54 Considering the orientation of shear force 7,» in the illustration, find the direction of the shear stress on the other three sides of the stress element av —S av —— — — — =< — (a) (B) © () PROFESSIONAL PUBLICATIONS, INC. ‘MECHANICS OF MATERIALS 10-51 For static equilibrium, the shear stresses on opposite faces of an ele- ment must be equal in magnitude and opposite in direction. Also, ‘the shear stresses on adjoining faces must not produce rotation of the element. " es Jar ‘The answer is (D). MECHANICS OF MATERIALS-55 If the principal stresses on a body are oy = 400 kPa, a2 = —700 kPa, and og = 600 kPa, what is the maximum shear stress? (A) 150 kPa (B) 250 kPa (C) 550 kPa (D) 650 kPa ‘The maximum shear stress is equal to one-half of the difference bo- tween the principal stresses. Comparing the three combinations, the ‘maximum shear stress is given by the difference between oz and a3, le ~os |=teouPac 0 uP 2 ‘The answer is (D). PROFESSIONAL PUBLICATIONS, INC. 10-62 1001 SOLVED ENGINEERING FUNDAMENTALS PROBLEMS MECHANICS OF MATERIALS-56 For the element of plane stress shown, find the principal stresses. 10MPa — Yo ttre Hi. — (D) oma = 85 MPa, dnin = 15 MPa ‘The stresses on the element are oy=-10MPa tz = 40 MPa +4, 50 MPa 10 MPa (2 MPa +10 MI 2 = 20 MPa +50 MPa = 70 MPa or ~30 MPa + (40 MPa)? ‘The answer is (C). PROFESSIONAL PUBLICATIONS, INC. ‘MECHANICS OF MATERIALS 10-53 MECHANICS OF MATERIALS-57 What are the principal (maximum and minimum) stresses of the stress element shown? si e Ye aio a Kom ‘ (A) @max = 1.16 kPa, Omin = (B) omax = 2.00 kPa, Omin (C) omax = 3.24 KPa, omin (D) ones = 8.24 KPa, omy = —3.24 kPa ‘The siresos on tho clement are Ge~HDkPa oy = V4KPA Tuy = 9 KPO 4 Geb oy, maxmin = EH kPa 44 kPa pote ( = 5.24 kPa or ~3.24 kPa ‘The answer is (D). PROFESSIONAL PUBLICATIONS, INC. 10-54 1001 SOLVED ENGINEERING FUNDAMENTALS PROBLEMS MECHANICS OF MATERIALS-58 ‘What is most nearly the maximum principal stress of the element shown? ; {is _l 40MPa 40 MPa —___ 10 MPa 20 MPa (A) 80 MPa. (B) 34 MPa (C) 40 MPa (D) 44 MPa ‘The stresses on the element are o2=40MPa o,=20MPa ty = -10 MPa oma = St | 40 MPa +20 MP. 2 PROFESSIONAL. PUBLICATIONS, INC. MECHANICS OF MATERIALS 10-55 MECHANICS OF MATERIALS-59 For the following stress clement, what is most nearly the maximum shear stress? (A) 10 MPa (8) 1 MPa (©) 4 MPa () 27 MPa ‘The maximum shear stress is = 26.9 MPa ‘The answer is (D). ‘PROFESSIONAL PUBLICATIONS, INC. 10-56 1001 SOLVED ENGINEERING FUNDAMENTALS PROBLEMS. MECHANICS OF MATERIALS-60 For the state of plano stress shown, what azo the i cipal planes? ion angles of the prin- ” Pe 5 Y__somra 40 MPa (A) 32.5° and 122° (B) 25.5° and 115° (©) -26.5° and -117° (D) ~11.5° and ~102° ‘The stresses on the element are o2=40MPa 9, tan 24, = 27 = ao =-1.33 20) = -53.0° or ~233° 26.5" or -117° PROFESSIONAL PUBLICATIONS, INC. ‘MECHANICS OF MATERIALS 10-57 MECHANICS OF MATERTALS-61 A steel (oyieia = 200 MPa) pressure tank is designed to hold pressures up to ‘MPa, The tank is cylindrical with a diameter of 1m. If the longitudinal stress rust be less than 20% of the yield stress of the steel, what is the necessary wall thickness, ¢? (A) 2 mm (B) 4mm (©) 88 mm (D) 120 mm For a thin-walled cylinder of diameter d containing a pressure p, one (1 -0.2)ei0a wa. (7x0 OBeraa (og) (20 x10 3) =0.044 m (44 mm) a ‘The answer is (B). MECHANICS OF MATERIALS-62 {In designing a cylindrical pressure tank 1 m in diameter, a factor of safety of 2.5 is used. The cylinder is made of steel (2yisaa = 200 MPa), and will contain pressures up to 7 MPa. What is the required Wall thickness, ¢, based on circumferential stress considerations? (A) 22 mm (B) 44mm (©) 88 om (D) 120 mm. For a thin-walled cylinder of diameter d containing a pressure p, Pd _ Oyicta m2 (1.28) (7x108 5) am) L.25pd m)O™) pata 200 x 10° 5 = 0.044 m (44 mm) The answer is (B). ‘PROFESSIONAL PUBLICATIONS, INC 10-58 1001 SOLVED ENGINEERING FUNDAMENTALS PROBLEMS MECHANICS OF MATERIALS-63 What is most nearly the maximum principal strain at a point where ¢- = 1500 bm, €y = ~750 yam, and éxy = 1000 pn? (A) 1160 ym = (B) 1490 um (C) 1610 m= (D) 1830 pm ‘Phe equation for principal strain gives conte ta (59) +(BY = 1500 wm = 750 pam (mens Som)", (sompumy" 2 875 jm + 1231 jam Snax = 1606 pn (1610 pm) ‘The answer is (C). MECHANICS OF MATERIALS-64 A beam of length Z carries a concentrated load, F, at point. C. Determine the deflection at point C in terms of F, L, E, and I, where E is the modulus of clasticity, and I is the moment of inertia. PROFESSIONAL. PUBLICATIONS, INC. ‘MECHANICS OF MATERIALS 10-59 ‘Computing M for the different: beam sections, aft FH ossco C5 Por ge tFe-9) (a = 18.75 KN (19 KN) ‘The answer is (D). DYNAMICS-26 ‘A rope is used to tow an 800 kg ear with free-rolling wheels over a smooth, level road. The rope will break if the tension exceeds 2000 N. What is the greatest acceleration that the ear can reach without breaking the rope? (A) 12m? —(B) 25 m/s? (C) 3.8 m/s? (D) 4.5 m/s?” Frnax = Tama 2000 N = (800 kg) trax PROFESSIONAL PUBLICATIONS, INC. 11-22 1001 SOLVED ENGINEERING FUNDAMENTALS PROBLEMS Rearranging to solve for @imaxs 2000 N mex = 309 kg 7 25 1/8 ‘The answer is (B). DYNAMICS-27 A force of 15 N acts on a 16 kg body for 2 s. If the body is initially at rest, how far is it displaced by the force? (A) Lim (B) 15m (19m (D) 21m ‘The acceleration is found using Newton’s second law. F_15N a io kg OOS For a body undergoing constant acceleration, with vp = 0 m/s® and t=2s, 1 m r= dat? = (2 3)? Ar = hat! (3) (0 8) @ i = 1.88 m ‘The answer is (C). PROFESSIONAL PUBLICATIONS, INC. DYNAMICS 11-23 DYNAMICS-28 A car of mass m = 150 kg accelerates in 10 s from rest at a constant rate to a speed of v = 6 m/s. What is the resultant force on the car due to this ‘acceleration? (A) 75N (B) 90N (©) 95N (D) 98N For constant acceleration, vevotat 6 105 =0.6 m/s? F = ma = (150 kg) (06 2) 90.N The answer is (B) DYNAMICS-29 A man weighs himself twice in an elevator. When the elevator is at rest, he ‘weighs 824 N; when the elevator starts moving upward, he weighs 932 N. Most nearly how fast is the elevator accelerating, assuming constant acceleration? (A) 0.64 m/s? (B) 11. m/s?@ = (C) 1.3. m/s®*——(D) 9.8 m/s? ‘The mass of the man can be determined from his weight at rest. W=mg W _ sun n= 9 = 84.0 kg ‘At constant acceleration, Pama F _ 932N-824N m B40 kg = 1.29 m/s? (1.8 m/s") ‘The answer is (C). PROFESSIONAL PUBLICATIONS, INC. 1-24 1001 SOLVED ENGINEERING FUNDAMENTALS PROBLEMS DYNAMICS-30 ‘A truck weighing 1.4 KN moves up a slope of 15°. What is the force generated by the engine if the truck is accelerating at a rate of 3 m/s"? Assume the coefficient of friction is = 0.1. 1 J (A) 876N (B) 926 (©) 930 (D) 958 In the direction parallel to the slope, a force balance gives Themen ~ (Wein + Fy) Fy is the friction force, which is equal to uN = WV cos0. F = W(sin8 + 20084) + ma = (1400 N)(sin 15° + 0.1 00815°) + ( = 925.7 N (926 N) The answor ®). PROFESSIONAL PUBLICATIONS, INC. DYNAMICS 11-25 DYNAMICS-31 In the illustration, the two pulleys and the horizontal surface are frictionless. ‘The cord connecting the masses ma and mp is weightless. What is the ratio of ‘the acceleration of mass A to the acceleration of mass B? Assume the system is released from rest. ro] oar | youn (ay a2 @1 ©2 (D) ma/m Assuming the accelerations of both masses are constant, their respec- tive displacement equations are ta = jaqt? 2p = apt? mtr pulley B PROFESSIONAL PUBLICATIONS, INC. 11-26 1001 SOLVED ENGINEERING FUNDAMENTALS PROBLEMS ‘Taking the ratio of za to zp, ae mop Since mass B is supported by a two-segment rope section and mass A is pulled by only one rope, the displacement: of mass B is half the displacement of mass A, Therefore, a = 2ep aa _ 2zB an te 2 ‘The answer is (O)- DYNAMICS-32 A simplified model of a carousel is illustrated. The 8 m long arms AB and AC ‘attach the soats B and C, each with a mass of 200 kg, to a vertical rotating shaft. ‘What is the maximum angle of tilt, 8, for the seats, if the carousel operates at 12 rpm? (A) 39° (B) 40° (C) 45° (D) 51° PROFESSIONAL PUBLICATIONS, INC. DYNAMICS 1.27 ‘The free-body diagram is ‘The angular velocity, «, of the carousel is = 1.257 rad/s ‘The rotational force, F, expressed in terms of 8 is F=ma=mrw* = mlsin Ou” 2 = (200 kg)(8 m) sin (1287 at) = (2528 N) sind From the ftee-body diagram, F __ (2528 N)sind W (20 ke) (9.81 1.288: af oo (Fi) = 39,07° (39°) ‘The answer is (A) ‘PROFESSIONAL PUBLICATIONS, INC. 11-28 1001 SOLVED ENGINEERING FUNDAMENTALS PROBLEMS | DYNAMICS-33 In the ballistic pendulum shown, a bullet of mass m is fired into a block of mass -M that can swing freely. Which of the following is true for the system during the swing motion after impact? pivot rigid arm. mw billet block (A) Both mechanical energy and momentum are conserved. (B) Mechanical energy is conserved; momentum is not. conserved. (C) Momentum is conserved; mechanical energy is not conserved. (D) Neither mechanical energy nor momentum is conserved. ‘Momentum is not conserved since an external force, gravity, acts on the bullet-block mass. Only mechanical energy is conserved. ‘The answer is (B). PROFESSIONAL PUBLICATIONS, INC. DYNAMICS 11.29 DYNAMICS-34 ‘Three masses are attached by a weightless cord as shown. If mass ma is exactly halfway between the other masses and is located at the center of the flat sur- face when the masses are released, what is most nearly its initial acceleration? Assume there is no frietion in the system and that the pulleys have no mass. m= kg frictionless h te pulley ey surfece m= 16k9 my = 12kg (A) Lo m/s? (B) 1.2 m/s* (©) 98 m/s? (D) 12. m/s? heme Ty= ma TTT ‘ Since m1 > mg, m1 will move downward and mp will be displaced to the left, All masses contribute to the inertia of the system. Ty ~ Ts = agrees = 214 + mg + ms) PROFESSIONAL PUBLICATIONS, INC. 11.30 1001 SOLVED ENGINEERING FUNDAMENTALS PROBLEMS 1, and Ty ate the tensions in the cord due to the masses m1 and ms. T=mg m kg = 12 ke = (28 3) (we eae Tet sng) .23 m/s* (1.2 m/s*) ‘The answer is (B). DYNAMICS-35 ‘The maximum capacity (occupant load) of an elevator is 1000 N. The elevator starts from rest, and its velocity varies with time as shown in the graph. What is most nearly the maximum additional tension in the elevator cable due to the ‘occupants at full capacity? Neglect the mass of the elevator. vim) 2 4 6 ime) (A) 960. (B) 1000N {C) 1200 N (D) 1400 N The maximum tension occurs during the period of maximum acoel- eration. This occurs for 01s <¢ <2, with acceleration, a, equal to v/t = 4 m/s/2's = 2 m/s. The mass of the occupants is m = 1000 N/9.81 m/s®. During this time, yr T=W+ma=1000N+ ma=T-W wwe) 2) =1204N (1200 N) The answer is (C). PROFESSIONAL PUBLICATIONS, INC. DYNAMICS 11-31 DYNAMICS-36 What is most nearly the kinetic energy of a 3924 N motorcycle traveling at 40 Jem /h? (A) 11100 J (B) 12300 J (©) 236003 (D) 246009 w ‘The kinetic energy is Ex = dmv? (3) oo xe) (aa 2)’ 4640 J (24600 3) ‘The answer is (D). DYNAMICS-37 A lead hammer weighs 45 N. In one swing of the hammer, a nail is driven 1.5 ‘em into a wood block. The velocity of the hammer’s head at impact is 4.5 m/s. What is most nearly the average resistance of the wood block? 45N hammer (A) 8000.) (B) 3100 N (©) 3920 N (D) 4090.) PROFESSIONAL PUBLICATIONS, INC. 11-32 1001 SOLVED ENGINEERING FUNDAMENTALS PROBLEMS Because energy is conserved, the kinetic energy of the hammer before impact is equal to the work done by the resistance force of the wood block. m = 45 N/9.81 m/s? = 4.59 kg, v = 4.5 m/s, and aan = 0.015 m, ) 9) (4.5 2 0.015 m =3098N- (3100 N) The answer is (B). DYNAMICS-38 ‘An automobile uses 74.6 KW to maintain a uniform speed of 96 km/h. What is the thrust force provided by the engine? (A) 0.87 1a (B) 2.8 kN (C) 82 kN (D) 5.6 kN Power is defined as work done per unit time, which, for a linear sys- ‘tem, is equivalent to force times velocity. Therefore, P=Fv (74.6 kW) (201 am) (6 ®) (1000 2) (saa =2797.5N (2.8 kN) ‘The answer is (B), PROFESSIONAL PUBLICATIONS, INC. DYNAMICS 11-33 DYNAMICS-39 ‘A 580 N man is standing on the top of a building 40 m above the ground, What is his potential energy relative to the ground? (A) 10% (B) DET (©) 43 (D) 2343 Ww; The answer is (D). (580 N)(40 m) = 23 200 J DYNAMICS~40 ‘A 900 N object is initially suspended on a 180 m long cable. ‘The object is then raised 100 m. Ifthe eable weighs 16 N/m, how much work is done? We |e0m % |190m Sow ‘900 N (A) 1000003 (B) 2980003 (©) 3200003 + (D) 398000 7 ‘The weight of the extended cable for the two situations is v7 = 080m (16%) = a0 w= (60m (168) = 180 ‘PROFESSIONAL PUBLICATIONS, INC. 11-34 1001 SOLVED ENGINEERING FUNDAMENTALS PROBLEMS ‘These weights may be considered to be concentrated at the midpoints of the extended cables. Choosing the datum to be at the top of the cable, and using the work-energy principle, the work done is equal to ‘the difference in potential energies of the two situations. E=( 1)(distance) + Wil Ey1 = (900 N)(—180 m) + (2880 N) ( 421200 J Epa = (900 N)(—80 m) + (1280 N) (=) 123200 J W = Byx ~ Ep, = ~123200 J — (—421 200 J) = 298000 7 Tho answer is (B). DYNAMICS-~41 A.0.05 kg mass attached to a spring (spring constant, k = 0.5IN/m) is accelerated to a velocity of 0.4 m/s, What is the total energy for the body in the following diagram? Neglect the spring mass. state equilibrium, position voams spring m= 0.05 kg | k= 05Nim orm (A) 0.00253 (B) 0.00403 (C) 0.00655 (D) 0.0092 3 The total energy is the sum of the kinetic and potential energies. E=Ey+E, = rv? + Bea? (5) (0.05 ke) (0.4 By +( ‘The answer is (C). PROFESSIONAL PUBLICATIONS, INC. DYNAMICS 11-35 DYNAMICS~42 A 1000 kg car is traveling down a 25° elope. At the instant that the speed is 13 m/s, the driver applies the brakes. What constant force parallel to the road must be generated by the brakes if the car is to stop in 90 m? (A) 1290 N (B) 2900.N (©) 5080.N (D) 8630.N ‘The change in energy is equal to the work done by the brakes. The change in velocity squared is, 2_y2 my? 2 -vz=0- (13 3) 169 m2 /s' ‘The change in elevation of the car is hoho= — (90 m) sin25° = -38 m AE, +E, = Fa }m(v? — v8) + mg(h— ho) = Fa (3) (1000 ke) (10 =) + (1000 kg) (9.81 27) (38 m) = F(00 m) F —5080 N ‘The answer is (C).. ‘PROFESSIONAL PUBLICATIONS, INC. 11-36 1001 SOLVED ENGINEERING FUNDAMENTALS PROBLEMS DYNAMICS~43 A bullet of mass 100 g is fired at a wooden block resting on @ horizontal surface. A spring with stiffness k = 53 kKN/cm resists the motion of the block. If the maximum displacement of the block produced by the impact of the bullet is 3.4 ‘om, what is most nearly the velocity of the bullet at impact? Assume there are no losses at impact, and the spring has no mass. ‘ood block aE (A) 250 km/h (B) 450 km/s (C) 630 km/h (D) 890 km/h Due to the conservation of energy, the kinetic energy of the bullet before impact is equal to the potential energy of the spring-mass- bullet system at maximum compression. Eroutier = Epayetem Jrmounev? = pea? ka? Mpaiet ‘58000 9) (3-4 cm)? (ae 2) Oke (00 1000 = = 891 km/h (890 km/h) he answer is (D). PROFESSIONAL PUBLICATIONS, INC. DYNAMICS 1-37 DYNAMICS~44 A simple pendulum consists of a 100 g mass attached to a weightless cord. If the mass is moved laterally such that h = 5 em and then released, what is the ‘maximum tension in the cord, T'? pivot (A) 1.08.) (B) 112N (© 118 (D) 1.25N ‘The maximum tension will occur when the pendulum is at its lowest point, position 2 in the figure, ‘The force balance in the vertical y direction gives ma, =T—mg T= ma, +mg = amg a From the conservation of energy, By: = Bao mgh = }mv? v= igh a) ‘Equations I and II give > Tag = TV 2GRY ve mg ~ 00 (081 3) (PERE) (as) LAT kgm/s? (1.18 N) ‘The answer is (C). PROFESSIONAL PUBLICATIONS, INC. 11-38, 1001 SOLVED ENGINEERING FUNDAMENTALS PROBLEMS DYNAMICS~45 AA stationary passenger ear ofa train is set into motion by the impact of a moving locomotive. What is the impulse delivered to the car if it has a velocity of 11 1m/s immediately after the collision? ‘The weight of the car is 56.8 kN. (A) 45.51N8 — (B) 575KNs (0) GBTENS —(D) 64.1 KN From the impulse-momentum principle, Imp = Amy mv1 + Imp = mva Imp = m(v2 ~ vi) ‘The answer is (C). DYNAMICS-~46 Which of the following statements is FALSE? (A) The time rate of change of the angular momentum about a fixed point is equal to the total moment of the external forces acting on the system about the point. (B) Tho coefficient of restitution can be less than zero. (C) The frictional force always acts to resist motion. (D) Momentum is conserved during elastic collisions. ‘The coefficient of restitution is defined as the ratio of the impulses cor- responding to the period of restitution and to the period of deformation of a body, respectively. Its value is always between O and 1, The answer is (B). PROFESSIONAL PUBLICATIONS, INC. DYNAMICS 11-39 DYNAMICS~47 ‘Two identical balls hit head-on in a perfectly elastic collision. Given that the initial velocity of one ball is 0.85 m/s and the initial velocity of the other is =0.53 m/s, what is the relative velocity of each ball after the collision? (A) 0.85 m/s and ~0.53 m/s (B) 1.2 m/s and -0.72 m/s (©) 1.2 m/s and ~6.1 m/s (D) 1.8 m/s and -0.98 m/s Let v; and v2 be the velocities of balls 1 and 2, respectively, after the n of momentum equation is mons ts 0a 2 + (-059 8) =v +¥0 witty = 038 m/s | Since kinetic energy is conserved, vy + dy = frnv} + rod (085 ®)" + (088 ®)* =vt+v3 vee v} <1 m/e? fo Combining Eqs. 1 and Il, J ~ 0.82v2 ~ 0.4488 = 0 = 0.85 m/s or ~0.53 m/s_ [negative value not used) ao-v 022-045 ® 53 m/s ‘The answer is (A). PROFESSIONAL PUBLICATIONS, INC. 11-40 1001 SOLVED ENGINEERING FUNDAMENTALS PROBLEMS DYNAMICS~48 A steel ball weighing 490 N strikes a stationary wooden ball weighing 490 N. If the steel ball has a velocity of 5.1 m/s at impact, what is its velocity immediately after impact? Assume the collision is central and porfectly elastic. “y (A) -5 m/s (B) -2 m/s (© Om/s (D) 5 m/s ‘Since the balls have the camo woight, they have equal mass. Denoting the instances before and after the collision by the subscripts 1 and 2, respectively, vy; = 5.1 m/s and vy: = 0. Conservation of momentum gives MVar + MyeVeot Vea + Vw tmgVoa + Masa 5.1 m/s 0 Conservation of energy gives }mqvi, + fmuvd, = }mav3, + Imyv2y my? Vit vie =vh = (51 2) = 26.01 m?/s? uy Solving Eqs. I and IT simultaneously, 2 via (2001 S ~ 102% + vB) 2v}, — 10.2¥92 = If v2 = 5.1 m/s, then vy2 = 0 m/s, and no change has occurred uring the collision. This is physically impossible, so vy2 = 0 m/s. PROFESSIONAL PUBLICATIONS, INC. DYNAMICS 1-41 DYNAMICS~49 ‘Two masses collide in a perfectly inelastic collision. Given the data in the il- lustration, find the velocity and direction of motion of the resulting combined vy = 10 rs ms (A) The mass is stationary. (B) 4 m/s to the right (©) 5 m/s to the left (D) 10 m/s to the right Let the positive direction of motion be to the right. Let ms be the resultant combined mass moving at velocity vs after the collision. Since momentum is conserved, mv + mayo = mava However, mg 1 + m2 = Amy + my = Sma. Therefore, tons (10 2) +m (20 ©) = navy 40a ~ 20m = Sas va = 4 m/s to the ight ‘The answer is (B). DYNAMICS-50 AA ball is dropped onto a solid floor from an initial height, ho. If the coefficient of restitution, e, is 0.90, how high will the ball rebound? 31 position (A) 0.45h; (B) 081k; (©) 0.85h1 (D) 0.90hy PROFESSIONAL PUBLICATIONS, INC. 11-42 1001 SOLVED ENGINEERING FUNDAMENTALS PROBLEMS ‘The subscripts 1, 2, and 3 denote the positions shown. 2 a ky 2 TTT LITO, Conservation of energy gives, before impact, Ex sorat = Exsctat Since the kinetic energy at position 1 and the potential energy at position 2 are zero, mght = dmv} va = Vig After impact, the kinetic energy at position 3 is zero Amv} = mghy v2 = V2gha By definition, the coefficient of restitution is Yall — Vitoor ‘The answer is (B). PROFESSIONAL PUBLICATIONS, INC. DYNAMICS DYNAMICS-51 ‘A mass suspended in space explodes into three picces whose masses, initial ve- locities, and directions are given in the illustration. All motion is within a single plane. Find the velocity of ms. m= 20.mis (A) 20 m/s (B) 23 m/s (©) 35 m/s (D) 40 m/s Defining the x and y axes as shown, conservation of momentum for the x direction gives mzv2 60830" + mgvs cos = 0 22 60830" + Avs cos = 0 (2) (40 ©) c0830° = ~Avs 0080 20 © cos 30” = —v5 e080 nat Na C088 a PROFESSIONAL PUBLICATIONS, INC. 14 1001 SOLVED ENGINEERING FUNDAMENTALS PROBLEMS For the y direction, mvy + maya sin 30° + mgvgsind = 0 sm (20 3) +21, (40) sinao® = smvpsino dvs si () Equations I and II give ao tan = -——_§._ (rm *)e os" ara® N38 = = eo8 40.0" = 22.9 m/s ‘The answer is (B) DYNAMICS-52 ‘Which of the following statements is FALSE? (A) Kinematics is the study of the effects of motion, while kinetics is the study of the causes of motion. (B) The radius of gyration for a mass of uniform thickness radius of gyration for a planar area of the same shape. (©) Angular momentum for rigid bodies may be regarded as the product of angular velocity and inertia. (D) The acceleration of any point within a homogenous body rotating with a constant angular velocity is proportional to the distance of that point to the center of mass. identical to the A body rotating at a constant angular velocity has no angular accel- eration. The answer is (D) PROFESSIONAL PUBLICATIONS, INC. DYNAMICS 1145 DYNAMICS-53 A uniform beam of weight W is supported by a pin joint and a wire. What will be the angular acceleration, cy, at the instant that the wire is cut? ie L a -—+ ———as ws co) # @ % 48 ‘The only force on the beam is its weight acting at a distance of L/2 from point O. Taking the moment about O, w(@) For a slender beam rotating about its end, Toa ‘The answer is (B). [PROFESSIONAL PUBLICATIONS, INC. 11-48 1001 SOLVED ENGINEERING FUNDAMENTALS PROBLEMS DYNAMICS-54 A thin circular disk of mass 25 kg and radius 1.5 m is spinning about its axis with an angular velocity of w = 1800 rpm. It takes 2.5 min to stop the motion by applying a constant force, F, to the edge of the disk. The force required is most nearly F (B) 16N (© aN (D) 2N The relationship between the retarding moment, Fr, and the decel- eration is Fr=~Ioa ‘Designating the positive rotational direction as counterclockwise, «0 = =1800 rpm. Therefore, = (-}) gone). m (eo =236N (24N) The answer is (C), ‘PROFESSIONAL PUBLICATIONS, INC. DYNAMICS u7 DYNAMICS-55 ‘A mass, m, of 0.025 kg is hanging from a spring whose spring constant, k, is 0.44 N/m. If the mase is pulled down and released, what is the period of oscillation? (A) 0505 (B) 128 (©) 158 (D) 218 By definition, the period Tis im 0.025 kg T= 2ny/™ = ap | 2028 _ ous X =156 DYNAMICS-56 A body hangs from an ideal spring. What is the frequency of oscillation of the body if its mass, m, is 0.015 kg, and k is 0.5 N/m? (A) 051 He (B) 0.66 Hz (©) 0.78 Hz (D) 0.92 He By definition, the frequency, f, is N ped fE-3 | QnVim Qe \ 0.015 kg = 0.92 He ‘The answer is (D). DYNAMICS-57 What is the natural frequency, w, of an oscillating body whose period of oscilla- tion is 1.8 «? (A) 18 rad/s (B) 27 rad/s (C) 85 rad/s (D) 4.2 rad/s ‘The answer is (C). PROFESSIONAL PUBLICATIONS, ING. 14g. 1001 SOLVED ENGINEERING FUNDAMENTALS PROBLEMS DYNAMICS-58 A one-story frame is subjected to a sinusoidal forcing function q(t) = Qsinwt at the transom. What is most. nearly the frequency of q(t), in hertz, if the frame is in resonance with the force? t= Qsinet man t36k9 lateral stiffness k= SkNem| (A) 2.6 He (B) 2.9 He (©) 36 He (D) 9.7 He Resonance occurs when the forced frequency, w, equals the natural frequency, tn m= 135 kg 000 ®) (100 £2 i= (5000 %.) (100 22) - 500000110 1 fe _ 1] Sooo % f"a5Vim = ae \ Take = 9.69 Hz (9.7 Hz) The answer (). DYNAMICS-59 In the mass-spring system shown, the mass, m, is displaced 0.09 m to the right of the equilibrium position and then released. Find the maximum velocity of m. ky =17 kN i= 17 Nim }-—— JIM m= 15kg WL frictionless ‘surface (A) 0.3 m/s (B) 5 m/s (© 8 m/s (D) 14 m/s PROFESSIONAL PUBLICATIONS, INC. DYNAMICS 1149 ‘The Kinetic energy before the mass is released is zero. ‘The maxi- ur when the mass returns to the point of static ‘equilibrium, where the deflection is zero and, hence, the potential energy equals zero. Therefore, since the total energy of the system is, constant, Epa Phat + bhox} Bea mv? splacement of each spring is 2-000 m = 135 m/s (1d m/s) ‘The answer is (D). DYNAMICS-60 ‘A cantilever beam with an end mass, m = 7000 kg, deflects 5 cm when a force of 5 KN is applied at the end. The beam is subsequently mounted on a spring of stiffness, k, = 1.5 kN/em. What is most nearly the natural frequency of the mass-beam-spring system? i; x B= 5om PROFESSIONAL. PUBLICATIONS, INC. 11-60 1001 SOLVED ENGINEERING FUNDAMENTALS PROBLEMS (A) 15 rad/s (B) 31 rad/s (©) G0 rad/s —(D) 6.3 rad/s A cantilever with an end mass m can be modeled as follows. Wilt, z. Zs OTA, 5000 N Bem For this model, both springs undergo the san ks = 1000 N/em deflection. Hence, = 2500 N/em The natural frequency is, therefore, (2:00 2)(m 7000 ke 5.98 rad/s. (6.0 rad/s) ‘The answer is (C). PROFESSIONAL PUBLICATIONS, ING. 12 DC ELECTRICITY DC ELECTRICITY-1 Which statement about a charge placed on a dielectric material is true? (A) The charge diffuses across the material's surface. (B) The charge diffuses through the interior of the material. (C) The charge is confined to the region in which the charge was placed. (D) The charge increases the conductivity of the material. Ina diele« ric, all charges are attached to specific atoms or molecules. ‘The answer is (C} DC ELECTRICITY-2 ‘The coulomb force, F, acts on two charges a distance, r, apart. What is F proportional to? 1 1 ©) 5 (Ayr (B) 7 (©) The coulomb force is Gre? 4 and g2 are the charges, and e is the permittivity of the surrounding, medium, Hence, F is proportional to the inverse of r2, ‘The answer is (C). PROFESSIONAL. PUBLICATIONS, INC. 122 1001 SOLVED ENGINEERING FUNDAMENTALS PROBLEMS DC ELECTRICITY-3 ‘The force between two electrons in a vacuum is 1 x 10-25 N. Approximately how far apart are the electrons? (A) 1410" mB) 5.1.x 107! m (©) 48x 10-7 m (D) 1.710" m Coulomb's law is 102 Gnegr? ¢o = 8.85 x 10" C9/N.m®, Also, for an electron, g Solving for r, 1.6 x 10-19 C) 6 x 10-9 C. eee ee 4 (885 x 10-2? &,) (1 x 10-1 N) Nom 48 x 10-7m. ‘The answer is (C). DC ELECTRICITY-4 ‘Two solid spheres have charies of 1 C and —8 C, respectively. ‘The permittivity, oy is 8.85 x 10-1? C?/N-m?, and the distance between the sphere centers, r, is 0.8 m, Determine the force on the spheres. (A) -1x10.N (B) -8x10"N (0) ON (D) 8x 108 N Because of their symmetry, charged spheres may be treated as point charges. Use Coulomb's law. qe (1.C)(-8 C) " eae __ Fee (esnca0-» 5) oom? =-8 x10 N The answer is (B). PROFESSIONAL PUBLICATIONS, INC. DC ELECTRICITY 12:3 DC ELECTRICITY-5 A parallel plate capacitor with plates of area A that are separated a distance d by air is initially charged with charge qe. ‘The energy stored in the capacitor initially is B. ‘The plates are then separated by 2d. What is the new energy stored in the capacitor? (a) o (B) 05z OE (D) 28 ‘The energy initially stored in the capacitor is, -£ P= 5G Cis the initial capacitance. After the increase in plate separation, the capacitance, C’, is a w wHGG~* ‘The increased energy is added into the system when force is used to separate the plates against the electrostatic force between them. The answer is (D). DC ELECTRICITY-6 A 0.001 C charge is separated from a 0.003 C charge by 10 m. If P denotes the point of zero electric field between the charges, determine the distance, 1, etween the 0.001 C charge and point P. 1x wre - axiote (A) 22m (B) 3.7m (©) 63m () 4m PROFESSIONAL PUBLICATIONS, INC. 124 1001 SOLVED ENGINEERING FUNDAMENTALS PROBLEMS Electric field intensity E at point 2 due to a point charge, Q, at point lis + is the distance betwoen points 1 and 2. At the point where E is zero, the electric fleld due to the 0.001 C charge equals the field due to the 0.003 C charge in magnitude. 0.001. C 0.003 C Gmc ~ Tre(10m— a (10 m — 2)? = 32? 2? 410n—50=0 Solving for the positive x value, b+ VR dae 2a =10m+ JG0my = WG m\—80 m) 3.66 m (3.7 m) ‘The answer is (B). DC ELECTRICITY-7 A.3.C charge and a 5 C charge are 10 m apart. A 7 C charge is placed on a line connecting the two charges, x meters away from the $C charge. If the 7 C charge is in equilibrium, find the value of x. me Oo © PE _—_y tom (A) 89m (B) 44m (©) 50m (D) 5.7m PROFESSIONAL. PUBLICATIONS, INC. DC ELECTRICITY 125 At equilibrium, Fgr = Fis. Using Coulomb's law, BOC) __ (7OVGC) Area? ~ Fneg(10 m— 2) (21 C7)(10 m~ 2)? = 3507 2? + 30x — 150 =0 Solving for a positive value of 2, b+ Vera Tae er) -30 m+ /@Omp— CI m)(—150 ma) ae 36m (44 m) ‘The answer is (B). DC ELECTRICITY-8 ‘Two charges, A and B, of equal and opposite value are separated by a distance, d. ris the distance from a charge to any point, P, lying on the normal plane that bisects the length d. What is the electric field at point P if K is a constant equal to 1/4re? a =F PROFESSIONAL PUBLICATIONS, INC. 126 1001 SOLVED ENGINEERING FUNDAMENTALS PROBLEMS ‘The total electric field will be in the z direction only, since the y components of the charges cancel each other out. By definition, with a, denoting the naa (2a unit radial veetor, Therefore, Bias = Ea + Bp = (SE) cos + (EB) cose cay) [Mie aneweris (A). | PROFESSIONAL PUBLICATIONS, ING. DC ELECTRICITY 127 DC ELECTRICITY-9 ‘A hollow metallic spherical shell has a charge of 0.001 C. The shell is 2 m in diameter. Which of the following correetly shows the variation of electric field with respect to the distance, r, from the conter of the sphere? (A) (C) ° ° 8 a 5 37 Se er Ze 5 7 @) 88 7 54 ae = 2 Fi 1 ab Wo 1234567 1234667 rim) Fim) ° 2 a 3 37 i Sot! 1 ge = a + 5| aaa © ash “ari a3 2b 2 if i 1224667 1294567 ro rim) Outside the sphere, Coulomb's law can be used to find the electric field. Thus, the electric field varies as 1/r? for r > 1m. On the surface of the sphere, r= 1 m. HF Tease oar ee 4x (8.85 x 10-7 5) (1m)? Gauss’ law states that the eleetrie flux passing through a given closed. surface is proportional to the charge enclosed by the surface. ‘There is no charge within the sphere, ‘Therefore, the electric field is zero for r<1m, Only (D) is correct. ‘The answer is (D). PROFESSIONAL PUBLICATIONS, INC. 128 1001 SOLVED ENGINEERING FUNDAMENTALS PROBLEMS DC ELECTRICITY-10 Approximately how far away must an isolated positive point charge of 1 x 10~® C be in order for it to produce an electric potential of 100 V? The charge is in free space with € = 8.85 x 10-!? C?/Nom?, (A)090m (8) 12m (© 53m (0) 86m Ata distance, r, from a point charge, v feos a nv _ 1x10 an (8.85 x 10-12 © ) cava v) u Nat? =0.90 m ‘The answer is (A). DC ELECTRICITY-11 A point charge, g, in a vacuum creates a potential, V, at a distance, r. A. reference voltage of zero is arbitrarily selected when r = a. If K = 1/4re,, which of the following is the correct: expression for V? 14 tea () Ke(3-3) wat (©) a(2-2) ©) ««(3-3) PROFESSIONAL PUBLICATIONS, INC. DC ELECTRICITY 29 DC ELECTRICITY-12 ‘What accelerating voltage is required to accelerate an electron to a kinetic energy of 5 x 10"! J? ‘The charge of an electron is 1.6 x 10-19 C. (A) 8KV (B) 13kV (©) 19kV (D) a1 kV For an electron, after the potential energy has been converted to Kinetic energy, kinetic energy is Ee=W Eb x10-8 ven i ¢ = 91250 (81 kV) ‘The answer is (D). DC ELECTRICITY-13 A certain potential variation in the zy plane is 1 aA w= ( ) Gy Which of the following gives the magnitude and direction (angle made with the z-axis) of the electric field intensity at the point (2,1)? (A) -v3/4, x (B) 1/2, -w/4 — (C) 2/2, m/4 — (D) 1/2, 0/4 given by the expression Since this is equivalent to the expression given, a) PROFESSIONAL PUBLICATIONS, INC. 12:10 1001 SOLVED ENGINEERING FUNDAMENTALS PROBLEMS Evaluating at the point (2,1), 2 Be freon: =1/2 The angle from horizontal that the E field is directed is ‘The answer is (D). DC ELECTRICITY-14 An electric dipole is placed in a uniform electric field of intensity, E. Given the information in the figure, what is most nearly the torque acting on the dipole? 0.03. Ne we 635mm (A) L7x10-° Nem (B) 3.3 10-8 Nam. (©) 48x 10-8 Nem (D) 9.5 x 10" ‘The torque is Nun T = F(2a)sin8 P is the force from the electric field. N P= Eq = 0.09 5 x17C =0.15N PROFESSIONAL. PUBLICATIONS, INC. DC ELECTRICITY wat Solving for torque, 7 = (0.5 N)(2) (sta 1000 = =9.5 x 10-7 Nm ‘The answer is (D). DC ELECTRICITY-15 Current é is applied to a long NV-turn solenoid with cross-section area A and length d. The magnetic field intensity inside the solenoid is H = Ni/d when d is very large. What is the inductance of this long solenoid in air? boNA oN? A HoNA boNA (a) a ®) d a id © @) ‘The magnetic flux passing through one turn of the solenoid is 0, = [B-a8= nit ‘The total flux enclosed by the NV’ turns is obtained by summing the contribution of all the turns. = 0% =mNHA The inductance is Ni ® _woNHA_ won (*) A _ woNPA 7 be i i a The answer is (B). PROFESSIONAL PUBLICATIONS, INC. 12.12 1001 SOLVED ENGINEERING FUNDAMENTALS PROBLEMS DC ELECTRICITY-16 Which of the following is NOT a property of magnetic field lines? (A) The field is stronger where the lines are closer together. ‘The lines interseet surfaces of equal intensity at right angles. ‘B) (C) Magnetic field lines have no beginnings and no ends. (D) The lines cross themselves only at right angles. is unique. ‘The answer is (D). DC ELECTRICITY-17 ‘The tesla is a unit of (A) permittivity (B) capacitance (C) inductance (D) magnetic flux density ‘The tesla is a unit of magnetic flux density. ‘The answer is (D). DC ELECTRICITY-18 The south poles of two bar magnets aro 7.5 em apart in air. The magnets are of equal strength and repel each other with a force of 4.9 x 10~* N, What is most nearly the strength of each magnet?” (A) 66x 10-° Wb (B) 0.86 Wo (©) 1 Wb (D) 53 wo ‘The force between two magnets of strength My and Mz MM; apr? PROFESSIONAL PUBLICATIONS, INC. DC ELECTRICITY 1213 My = Mp. Therefore, M = Jane =y/4n (7x1 (0.075 m)2(4.9 x 1 = 6.60 x 10°© Wb [unit poles) DC ELECTRICITY-19 A conductor has length of 1 m, electrical resistivity of 0.1 9-m, and area of 0.01 m?. A uniform direct current having a density of 100 A/m? flows through this ‘conductor. What is the power loss in the conductor? (a) ow j@)iw (c) 10 W (D) 100 W ‘The resistance of the conductor is pL _ (0.1 O-m)(1 m) SATs ‘The current flows through the conductor is P)=1A A A= (100 -4,) (001» J (00 3) (00 ‘Therefore, the power consumed in the conductor is P= AR = (1 A)(10 9) = 10 W ‘The answer is (C). DC ELECTRICITY-20 For a field given by B volume? MH (Wb/m?), what is the energy storage per unit B we a H (a) U=5 (B) Ua Ou=5; ©) U=53 PROFESSIONAL PUBLICATIONS, INC. 1214 1001 SOLVED ENGINEERING FUNDAMENTALS PROBLEMS ‘The energy stored in a magnetic field H per unit volume is U = ‘ABH. Since B= ui, H = B/y. Therefore, BB U=4B- = a ‘The answer is (A). DC ELECTRICITY-21 ‘The magnetic flux density, B, and the magnetic field intensity, HZ, have Ue following relationship. B= pol +M) is the permeability of free space (in H/m), and M is the magnetic polarization of the material (in A/mm). If B is increasing, which of the following may be true about the state of metal X at a value of = 100 A/m? The B-IT curve of the metal is as shown. (wey) 10 os, (A) B=0.6 Wb/m; metal X is nonferrous (B) B=0.6 Wb/m; metal X is ferrous (C) B=1.0 Wb/m; metal X is nonferrous (D) B=1.0 W/m; metal X is ferrous Nonferrous metals do not exhibit hysteresis; hence, metal X is ferrous. ‘The hysteresis curve follows a counterclockwise path. Therefore, for B to be increasing at. an H value of 100 A/m, B = 0.6 Wb/m. ‘The answer is (B). 'PROFESSIONA\, PUBLICATIONS, INC. DC ELECTRICITY 12.15 DC ELECTRICITY-22 ‘Two identical coils of radius r are placed at a distance r apart as shown. Such a configuration is called a Helmholtz. coil. Which of the following describes the magnetic field created by passing a uniform current through the assembly? (A) The magnetic field is negligible regardless of the magnitude of I. (B) The magnetic field is zero midway between the two coils. (©) The magnetic field is fairly uniform between the two coils. (D) The magnetic field is zero at the centers of the coils. ‘The magnetic ficld between the two coils is the superposition of the field created by each coil. Since the currents in both coils are in the same direction, the direction of each individual magnetic field is also the same, using the right-hand rule, Therefore, the fields will not cancel each other. Since the two coils are circular with their centers aligned, the field between them will be fairly uniform. ‘The answer is (C). DC ELECTRICITY-23 Which statement is true? (A) Magnetic flux lines have sources only. (B) Magnetic flux lines have sinks only. (C) Magnetic flux lines have both sources and sinks. (D) Magnetie flux lines do not have sources or sinks. ‘Magnetic flux lines are closed loops with no sources or sinks. No known particle produces lines of magnetism. [arte | PROFESSIONAL PUBLICATIONS, INC. 1216 1001 SOLVED ENGINEERING FUNDAMENTALS PROBLEMS DC ELECTRICITY-24 A charge of 0.75 C passes through a wire every 15 s. What is most nearly the current in the wire? (A) 50mA (8) 94mA (©) 20 mA (D) 50 mA. Current is the charge per unit time passing through the wire. mA _ oo) (000 = iss DC ELECTRICITY-25 ‘What is most nearly the total resistance between points A and B? a M=20 6 a5 pb w= yn Bv 5 (ay on (B) 129 (©) 160 (D) 20 ‘The total resistance is the sum of the resistance between points A and C, plus the equivalent resistance of the resistors in parallel between points Cand B, Reotat = Ry + Re || (Ro + Rs) =29+100 22 ‘The answer is (B). PROFESSIONAL PUBLICATIONS, INC. DC ELECTRICITY 12.17 DC ELECTRICITY-26 ‘What is the total resistance (as seen by the battery) of the following network? sa 20 sa a a 8 20 120. wa 20 (A) 609 (B) 12.0 (C) 159 (D) M2 ‘AB is a short circuit. Therefore, the rest of the circuit, does not con tribute to the resistance. The effective circuit is Ay=60 Ryn 120 Raotat = Ra + Ra || Rs =mtaty ay Ta Be 1 =60+ 54 patpa =60460 =Ra The answer is). | PROFESSIONAL PUBLICATIONS. INC. 12.18 1001 SOLVED ENGINEERING FUNDAMENTALS PROBLEMS DC ELECTRICITY-27 In the circuit shown, R= 10 0, and the electromagnetic force, V, is 2-V. What is most nearly the current, I? a oR Lik 2Vv (A) 0.0 A (B) 030A (©) 067A (D) 33.4 Rural = Ri] 2R= TT RoR (200.2) 3 1.67 2 Use Ohm's law. Current is calculated as voltage divided by total resistance. ‘The answer is (B).. PROFESSIONAL PUBLICATIONS, INC. DC ELECTRICITY 12.19 DC ELECTRICITY-28 Find the current passing through the 3 0 resistor. sn A 120 B 6a ma > ce 120 120, 30 = = (Ay oa @) 03.4 (1a (4A Current from a battery will always follow a path of zero resistance in circuit. Instead of flowing through the 3 9 resistor and its neighbor- ing resistors, the current will follow the path BCDB, a short: circuit. There will be no current: in the resistor. ‘The answer is (A). DC ELECTRICITY-29 What is most nearly the current passing through the 30 9 resistor? ze gg lie +) wok CY nol) Sun 8 © (A) 00 A (B) 29 mA (©) 50 mA (D) 57 mA PROFESSIONAL PUBLICATIONS, INC. 12-20 1001 SOLVED ENGINEERING FUNDAMENTALS PROBLEMS R= 100 ‘The circuit is symmetrical. Thorefore, a current, I, flows through the resistors, Ry, and Rs. Another current, Iz, flows through resistor Rg, From Kirchhoff’s current law at point A, Yr=0 h=2h Using Kirchhoff’s voltage law around the loop ABCDA. V=Rht+ Rh 2V = (30 Oo +(10 MK, 2V = (30 (2h) + (10 ON, 2V = (709) Ty = 0.0286 A Jn = 2h = (2)(0.0286 A) (2000 =) 57 mA ‘The answer is (D). PROFESSIONAL PUBLICATIONS, INC. DC ELECTRICITY 1221 DC ELECTRICITY-30 ‘What is most nearly the current through AB? Vv 60 100 th ¢ 100, 100, 5a o + 8 ron, chalile WW 10 (A) 054. (B)1A (3A (D) 4a won eb ——wy——_1 foo eee mh By redrawing the circuit and designating the currents as shown in loop ACBA, the currents through the remaining loops can be expressed in terms of 11, In, and Jp. Since voltage equals resistance multiplied by current, for loop CDAC, cov 20, — In + 5Is (5 9) (In ~ Is) ~ (10 9) (Ii + Ts) ~ (10 Og -12A 0 For loop BEDAB, 120 V = (10 2)(Tr + 2) + (10 2)(Kr + Is) + (5 DH 0= 251, + 10% + 10% — 120 Bh +2 + 2g = 24 A im PROFESSIONAL PUBLICATIONS, INC. 1222 1001 SOLVED ENGINEERING FUNDAMENTALS PROBLEMS Around loop ACBA, OV =—-(65 Yh + (10 Nis + (10 MIs 1)-2h-2h=0A a Observe that adding Eqs. II and III can directly solve Jy. The answer is (D), DC ELECTRICITY-31 In the circuit shown, what is the current through CD? M=050 B h=10N C Ae1sa cell 1= 1.5 1s ke ® € D (A) 0.20 A (B) 0.60.4 (©) Lo (D) 194 A950 Bp Mm=10n © r Rajon cel 1 = 1.5 PROFESSIONAL PUBLICATIONS, INC. DC ELECTRICITY 12.23, ‘The method of superposition is used to find the current, I. Let Ty be the current from cell 1, and let J be the current from cell 2. Then, T= h + Ip. Short circuiting cell 2 to find J, as shown in illustration (2), the equivalent total resistance is Resta = Ri + Ra || Rs Bay Math =Ri+ Short circuiting cell 1 to find Z2, as shown in illustration (b), the ‘equivalent total resistance is Roouata = Ri || Ra + Ra RR ~ Ri+Re (15.9) oan +R The total current is T=h+h=08@A+11A =192A (19) The answer is (D} PROFESSIONAL PUBLICATIONS, INC. 12-24 1001 SOLVED ENGINEERING FUNDAMENTALS PROBLEMS DC ELECTRICITY-32 For the network shown, find the voltage drop from C to D. Am=20, R=50 , A= 200 av A= 160 (A) 20V (B) 25v (©) 30 (P) 8ov The total resistance is Reoral = Ri + Re |i (Rs + Ra) 1 preaioelereme ee Trot Use a current divider to find the current in section CDB. Ro -a(22) 05 A ‘The answer is (B).. PROFESSIONAL PUBLICATIONS, INC. DC ELECTRICITY 12.25 DC ELECTRICITY-33 Determine the voltage drop across the 4 9 resistor in the network shown. Bo 20 40 = ev 2a(h 6a (A) 43V (8) 6rVv (Rv (D) 4Vv ‘The network is redrawn with the currents and circuit points labeled ‘as shown. The current through BE is equal to the sum of currents from AB and CB. Ing =hh+h=2A+h Kirchhoff’s voltage law around loop DBE gives Ven = Rife + Raloc GV = (2 No + (4 D)Iow = (2 Mh + (4.2)(2 A+ h) I, = 0.383 A. [opposite to the direction that it was defined) Tog =2 A 0.383 A= 1.67 A Vee = (1.67 A)(4 9) 6.08 (6.7 V) The answer is (B), PROFESSIONAL PUBLICATIONS, INC. 12:26 1001 SOLVED ENGINEERING FUNDAMENTALS PROBLEMS { DC ELECTRICITY-34 ‘The voltage at point A in the network shown is most nearly 30 30 Vv (A) Lov (B) 23. (©) 30 (D) 45V ‘The circuit is redrawn, Superposition is used to find Ja. ta= lov hav Jey is the current through BA from the 6 V source, and Iya is the current through BA from the 12 V source. The equivalent resistances are calculated by short circuit for each voltage source. Roy = Rot Ri || Re PROFESSIONAL PUBLICATIONS, ING. DC ELECTRICITY 12.97 Riay = Ri + Ra || Re =80 tav= (H2*) (ge) Rav) \Ra+ Be, Jn= ley ~ hav = 0708-054 0.25 A Va = Vov - bRs V— (0.25 A)(6.0) LV 05 A (ox) (Sx) ‘The answer is (D). DC ELECTRICITY-35 What is the voltage drop across the 8 © resistor in the following circuit? 8a rah S100 (A) Vv (B) 12V (©) 20 (D) 22 PROFESSIONAL. PUBLICATIONS, INC. 12-28 1001 SOLVED ENGINEERING FUNDAMENTALS PROBLEMS yy ao 100 Redrawing the circuit as shown, with Ic equal to the component of the current through the 8 @ resistor due to the current source, and Jy equal to the component of the current through the resistor due to the voltage souice, Isa =lo—ly But, Io = 1A, and Jy =0 A. Therefore, ha=la Ven = IR = (1 A)(8 2) =8Vv ‘The answer is (A). DC ELECTRICITY-36 Determine the voltage drop across the 6 2 resistor. Bn 4a 20 at av 6a (A) 60V (8) 9.0 (c) ov (D) 18v PROFESSIONAL PUBLICATIONS, INC. DC ELECTRICITY 1229 c Mask gp aan From superposition, with Zc designating the current through the resistor from the current source, and Iy designating the current through the resistor from the voltage source, Isa=Ie+lv Jo and Ty are " (gem=R) ao -@%(zar5a0n) .667 A h nv VO" Rat Rat Re wv v 40420462 Bt Is = 0.667 A+1 A= Voa= Ie oR = (1.67 A)(6.) =10V 67 A Phe answer is (C) PROFESSIONAL PUBLICATIONS, INC. 1230 1001 SOLVED ENGINEERING FUNDAMENTALS PROBLEMS DC ELECTRICITY-37 ‘The rated voltage drop across a device is 50 V, and the current drawn is 30 A. ‘What is most nearly the power rating of this device? (A) 0.66 hp (B) 1.0 hp (© 15 hp (D) 2.0 hp Power is calculated as current multiplied by voltage. pa ry ~ (S060 Vv) 746 55 =2.01 hp (2.0 bp) DC ELECTRICITY-38 In the circuit shown, V = 6 V, and the internal resistance of the source, r, is 19. For a specific value of R, the power output is the maximum possible, What is most nearly this maximum output power? a (A) 45W (B) 60 Ww (©) 99W (D) 18 W Output power is given by the following expression. P=PR v ae Rer Ry? PROFESSIONAL PUBLICATIONS, INC. DC ELECTRICITY 12-31 Maximum power occurs when dP/dR = ap ah (R+ry? = AR+ rR R+ro2R R= ?(-2(R+r)*R+(R+r)~) =12 ‘Therefore, the maximum power is Ry? ry = £M6VY “Garo Pros = ‘The answer is (C). DC ELECTRICITY-39 ‘What is the time constant of the network? 1a ——————_ (A) 0.001 s (B) 108 (©) 100 (D) 1000 s ‘The time constant, 1, is 7 = RC = (1000 9)(1 x 10-* F) 0.001 s PROFESSIONAL PUBLICATIONS, INC. 12-82 1001 SOLVED ENGINEERING FUNDAMENTALS PROBLEMS DC ELECTRICITY-40 For the two capacitors shown, O; = 1 uF, and Cz = 3 uP. What is the equivalent capacitance between A and B? G & if tt tt A 8 =F (A) 0.75 uF (B) 1.0 uF (©) 20 pF (D) 40 uF tar tae =aji+0 =ay0 i tt GM CMe A B =F By dofinition, q = CV. For capacitors in series, the charge, q, is the same on each capacitor. Therefore, VeNth 4l4,4 cata 14,4 Carta C= iCe_ _ (LEYS uP) O1+C, 14 uF = 0.75 uF ‘The answer is (A). PROFESSIONAL PUBLICATIONS, ING. DC ELECTRICITY 12:33 DC ELECTRICITY-41 ‘The equivalent capacitance of capacitors C, and Cy connected in series is 7.3 UB. If the capacitance of C, = 9.6 uF, what is most nearly the capacitance of Cx? (A) 23 oF (B) 31 uF (©) 35 uF (D) 49 uF For capacitors in series, the equivalent capacitance, C, is ‘The answer is (B). DC ELECTRICITY~42 Find the voltage at point A at the instant the switch is closed. 1ka rv a 1H (A) 00V (B) Lov (©) 30 (0) 2Vv ‘The initial current at ¢ = 0* is zero, so there is no voltage drop across, the resistor. The full 12 V potential appears across the inductor. Vani2V ‘The answer is (D). PROFESSIONAL PUBLICATIONS, INC. 12-34 1001 SOLVED ENGINEERING FUNDAMENTALS PROBLEMS DC ELECTRICITY-43 Which of the following statements regarding the motion of a conductor through. a changing magnetic field is FALSE? (A) The lines of magnetic flux pass from the north pole to the south pole of the magnet. (B) When a conductor is “open circuited,” no current flows despite its motion through the field. (©) The conductor must move at constant velocity in order to generate a cur- rent. (D) A current flowing counter to the direction of the conduetor’s motion will create a torque. A varying amount of flux give rise to a current. Flux through a conducting loop can be varied either by changing the magnetic field or by changing the speed of a conductor through the field. Thus, a conductor accelerating through a magnetic field will generate current. The answer is ( DC ELECTRICITY-44 Modern locomotives are powered by DC motors driven by DC generators, These are, in turn, driven by diesel engines. Which of the following is the reason for using such a configuration instead of AC generator-motor sets (A) The DC configuration provides high torque and good incremental power at low spoeds, and performs equally well at high speeds. (B) The DC equipment is significantly less expensive. (C) Historically, the DC engine configuration has been used, There is no reason to change this. (D) By using the DC equipment, the power factor problems associated with AC equipment are avoided. At low speeds, the DC system is best because it gives high torque and excellent control. DC motors or generators generally cost: more than AC units because they have windings on the armature, which the AC units lack. ‘The power factor problem has nothing to do with the decision not to use AC systems. At low speeds, the torque delivered by AC units is poor. @). ‘The answer PROFESSIONAL PUBLICATIONS, ING. DC ELECTRICITY 12:35 DC ELECTRICITY-45 In a DC motor, what is the definition of “field resistance”? (A) Its the load resistance seen by @ generator without considering inductance. (B) It is the resistance of the excitation circuit. (C) It is the resistance of the armature windings plus the load resistance. (D) It is the static resistance of the motor. ‘The ficld cireuit is the circuit that excites the pole pieces, thereby producing the flux cut by the armature windings. The field resistance is the resistance of this circuit. The answer is (B) | DC ELECTRICITY-46 Ina DC motor, which of the following does NOT cause sparking at a commuta- tor? (A) no load on the output leads (B) frozen armature (C) high brush contact resistance (D) graphite brushes with good contact pressure Sparking at the commutators results from having very low resistance at that point. It is normally associated with the use of copper or mainly copper brushes because the resistance of the brushes is non- linear and drops as current increases. The answer is (C). DC ELECTRICITY-47 "The armature in a DC generator has one or more pairs of conductors or coils in which current is produced. In general, which of the following is true about the amount of power produced’? (A) No gain in power is achieved beyond four pairs of coils. (B) More coils give more power. (C) Power is related only to the number of poles. (D) Power is a function only of the output voltage and current. PROFESSIONAL. PUBLICATIONS, INC. 12.36 1001 SOLVED ENGINEERING FUNDAMENTALS PROBLEMS Every coil that cuts the flux lines gives more power. It is desirable to place as many coils on the armature as possible. ‘The answer is (B). DC ELECTRICITY~-48 Which of the following limits the number of coils that may be placed on the armature of a DC motor or generator? (A) the type of winding used (B) the number of poles (C) the size of the load on the motor (D) coil to coi! arcing due to the broakdown of insulation Coils must be well insulated from each other when spaced close to- gether, otherwise failure will occur due to arcing. There is also a physical limit on the number of coils that will fit on an armature. However, this is due to the volume of insulation material needed to prevent shorts. ‘The answer is (D). DC ELECTRICITY-49 the overall torque of a DC motor is T= Kr6laz Kr is a constant for the particular machine, ¢ is the total magnetic flux per pole, Tq is the armature current, and 2 is the number of conductors on the surface of the armature. Which of the following is FALSE regarding the above equation? (A) It applies to both motors and generators. (B) It applies only to a machine having, an even number of pairs of poles. (C) It applies since torque is indirectly dependent upon the pole winding current. (D) It applies whether or not there is a load on the system. ‘The equation is valid for generators or motors having any number of poles. Option (B) is false. ‘The answer is (B). PROFESSIONAL PUBLICATIONS, INC. DC ELECTRICITY 12.37 DC ELECTRICITY-50 A DC system can be protected from lightning by including a thyrite tube in the cireuit that connects the high-voltage line with the ground. Which of the following is FALSE regarding a thyrite tube? (A) It maintains a very high resistance at or below the system operating voltage. (B) Its resistance becomes low at very high voltages. (C) It has a very fast recovery time with regard to voltage change. {D) Its fusable link melts at high voltages. ‘The resistance of thyrite tubes drops significantly near the operating ‘voltage, allowing large currents to be discharged. They do not melt at high voltages, but instead recover immediately after passing large currents to the ground. DC ELECTRICITY-51 ‘The magnetic saturation curve limits the voltage at which a generator or motor can operate. Which of the following statements regarding saturation curves is FALSI (A) As field current increases, the hysteresis effect limits the increase in the flux produced, (B) Poles that allow the production of more flux permit higher operating volt- ages. (C) More flux at a coustant field voltage can be produced by increasing the umber of poles. (D) Saturation does not depend upon the type of steel used in the poles. ‘The magnetic permeabilities of steels vary greatly. ‘The answer is (D). PROFESSIONAL PUBLICATIONS, INC. 12.38 1001 SOLVED ENGINEERING FUNDAMENTALS PROBLEMS DC ELECTRICITY-52 Series and shunt motors are connected liko series and shunt generators, respec- tively, The terms refer to the manner in which the self-excitation of the poles is connected to the unit. Which of the following statements is FALSE? (A). The torque curve of a shunt motor is linear. (B) Field coils of the shunt motor or generator are in parallel with the armature ‘windings. (©) Field coils of the series motor or generator aro in series with the armature windings. (D) ‘The torque curves of both shunt and series motors are not affected by the value of the armature current. ‘The torque for these motors is directly related to the armature cur- rent. The answer is (D) DC ELECTRICITY-53 In terms of efficiency, shunt and series motors or generators have similar charac- teristics. Which of the following statements is FALSE? (A) Series motors have low torque at low speeds. (B) Shunt and series motors have approximately 80% efficiency above one-third of the rated load. (C) Efficiency decreases with lower speeds for both types of motors. (D) The two types of motors have very similar efficiency curves. Although the efficiency curves for the two types of motors both drop at lower operating speeds, they are still quite different, ‘The answer is (D). PROFESSIONAL PUBLICATIONS, INC. DC ELECTRICITY 12-39 DC ELECTRICITY-54 What is pole pitch? (A) the mica used to insulate the poles from each other (B) the space on the stator allocated to one pole (C) the space on the stator allocated to two poles (D) the angle at which the pole windings are wound ole pitch is defined as the periphery of the armature divided by the ‘number of poles. Thus, it is the space on the stator allocated to one pole. The answer ie (B). DC ELECTRICITY-55 Which of the following statements regarding a compound motor is FALSE? (A) It has a shunt winding. (B) It has a series winding. (C) It has commutators, armature windings, and field windings. {D) Its speed remains fairly constant when subjected to sudden loads. ‘The advantage of a compound motor is its ability to respond to sud- den heavy loads. The motor’s speed is reduced quickly when the load is applied, transferring the kinetic energy of the system to the work ‘The answer is (D). DC ELECTRICITY-56 Which of the following does NOT contribute to core losses in DC motors? (A) eddy currents in the armature (B) hysteresis losses in the armature (C) commutator losses (D) eddy currents and hysteresis losses in the armature PROFESSIONAL PUBLICATIONS, INC. 12-40 1001 SOLVED ENGINEERING FUNDAMENTALS PROBLEMS ‘The induced eddy currents together with the hysteresis losses consti- tute the core losses. Commutator losses do not contribute to the core losses. ‘The answer is (C). DC ELECTRICITY-57 Which of the following are power losses in a DC motor? (A) PR tosses (B) gear aud frictional losses (C) hysteresis losses (D) all of the above All of the choices are types of power losses in a DC motor. Note that core losses include hysteresis losses, The aver ie (D) | DC ELECTRICITY-58 Which of the following statements is FALSE regarding large DC motors? (A) To avoid flashover, the voltage difference between adjacent commutators should not exceed 15 V DC. (B) ‘Two, four, six, or eight coils may be laid in a slot in the armature to make ‘maximum use of the flux and to generate as much power as possible. (C) The pitch of the winding of a given coil is the number of slots spanned by the coil. (D) Wave winding does not pass under all poles at one time. ‘The lap winding loops several times under the same poles, while the wave winding lies under all poles at the same time. ‘The answer is (D). PROFESSIONAL PUBLICATIONS, INC. DC ELECTRICITY 1241 DC ELECTRICITY-59 Which of the following statemonts is FALSE about the operation of parallel shunt generators? (A) The drooping load or decreasing terminal voltage characteristic of shunt generators makes two or more units operating in parallel more stable. (B) The use of several units makes maintenance and repair easier. (C) The parallel configuration makes it possible to add units as needed, and to shut off unnecessary units at low load demands. (D) One large unit: would be more expensive than the use of several smaller ones, even if it ran at full load at all times. Although it is rare to be able to load a unit fully at all times duo to ‘maintenance considerations, one large unit; operating as such would be more economical. ‘The answer is (D). DC ELECTRICITY-60 How many commutators does a DC machine require to produce the waveform shown? current (A) two (B) three (©) four (D) five Vea Ne Ni on fae ae ee TAA AA YS ome VY VVYYY / / / / PROFESSIONAL PUBLICATIONS, INC. 12-42 1001 SOLVED ENGINEERING FUNDAMENTALS PROBLEMS ‘Two commutators are needed for each coil. Each coil produces a single sine wave. If the alternate peaks in the figure are reversed, two full sine waves are described. ‘Therefore, sineo there are two coils, four commutators are needed. wer is (C), DC ELECTRICITY-61 A generator proviees power to a lond dissipating 160 kW. Current flows over 760 m of two-conductor copper feeder having a resistance of 0.026 © per 100 m. The generator voltage is constant at 800 V. What is most nearly the voltage delivered at the specified load? (A) 612 (B) 652 V (©) 72V (0) lov Since it is a two-conductor feeder, the total resistance of the feeder is R=2Drotak 7 1.026 2 = eran my ($5222) = 0400 By definition, V = IR. Visas = 800 V — 1(0.40 2) 800 V — Via ~~ Oda Using this equation for I, the expression P = VI gives 160000 W = Viena __ (800. = Viona = (Maem) tn 64.000 = 800Vioed — Vidas 0 = Vidug — 800Viona + 64.000 PROFESSIONAL PUBLICATIONS, INC. DC ELECTRICITY 12-43 Solving for Voads Visa 800 V (800 V)? — (4)(1)(64000) 2 = 90.2 V or 710 V Of the choices, the only reasonable voltage is 710 V. ‘The answer is (D). DC ELECTRICITY-62 In a DC machine, if the field current Iy increases to 27, what will the new torque be as a function of the initial torque, Tm? Noglect satu (A) Tn (B) Tn (C) AT @) Tn® ‘The mechanical torque produced is, 60 Tn = 5pK ale ‘The magnetic flux, &, generated by the field is = Kyl Therefore, the new torque is Ton = SK aKa =m ‘The answer is (B) PROFESSIONAL PUBLICATIONS, INC. 13 AC ELECTRICITY AC ELECTRICITY-1 ‘An alternating current with a frequency of 60 Hz is paseed through a moving eoil galvanometer that measures DC current. What will the galvanometer reading be equal to? (A) the peak value of the AC current (B) the average value of the AC current (C) the rms value (D) a negligible amount If the galvanometer is designed to measure DC current, it will not be able to respond quickly enough to measure an alternating current of 60 Hz. ‘The reading will be negligible. ‘The answer is (D). AC ELECTRICITY-2 Which of the following effects are generally less for an alternating current than for a direct current? (A) heating effects (B) chemical effects (C) magnetic effects (D) impedance Chemical effects are generally less for an AC current than for a DC current. Heating and magnetic effects are generally greater for an AC ‘current than for a DC current. Impedance for an AC current is either larger than or the same as a DC current. ‘The answer is (B) PROFESSIONAL PUBLICATIONS, INC. 132 1001 SOLVED ENGINEERING FUNDAMENTALS PROBLEMS AC ELECTRICITY-3 The following two sine waves, K and M, are plotted as phasors. Determine which of the numbered vectors—v1, v2, Vs, or vs—corresponds to K. y K= Keo (ut + 0) M= My cos (wt) (A) Kev (®) K=v. (C) K=vs (D) K=v% ‘The magnitude of the vector corresponds to the amplitude of the wave. Thus, the vector, K, is longer than M. All angles are measured from the positive z-axis, with a leading angle measured counterclock- wise by convention. Therefore, since the peak of K leads that of M by less than 90°, the A vector lies in the first quadrant. The only choice satisfying these conditions is option (D). The answer is (D). PROFESSIONAL PUBLICATIONS, INC. AC ELECTRICITY 133 AC ELRCTRICITY-4 A wire carries an AC current of 3cos100xt A. What is the average current aver 6s? (AOA (B) 4/64 (C) 15a (D) 6/r A IET is the total period of time and J(t) is the current as a function of time, the average current is ip med [108 ‘Therefore, for the particular AC current, ip 5 ff seostoostae = pf costoozeae 240 ‘The answer is (A). AC ELECTRICITY-5 What is the Jims value for the waveform shown? If T is the period and J is the current, the rms (effective) value is, aa P(oae PROFESSIONAL PUBLICATIONS, INC. Tams = 134 1001 SOLVED ENGINEERING FUNDAMENTALS PROBLEMS For the square wave shown, ‘Therefore, AC ELECTRICITY-6 A sinusoidal AC voltage with an rms value of 60 V is applied to a purely resistive circuit as shown. What steady voltage most nearly generates the same power as the alternating voltage? cov (ems) 7 (A) 38.V (B) av (©) Vv (D) 85 PROFESSIONAL PUBLICATIONS, INC. AC ELECTRICITY 13.5 By definition of average power, Fos . nee Exma ov Pave ‘The answer is (C). AC ELECTRICITY-7 ‘What is most nearly the average current through the resistor, R, in the rectifier shown? Assume ideal diodes. 120sinwtV (a) 004 (076A (BAA (0) 438A ‘The type of rectifier shown ise “fll wave” rectifier, with an average current of ee Baye for 0 full-wave rectifier is 1 2 [ (120 V)sinwt de T PROFESSIONAL PUBLICATIONS, INC. 13-6 1001 SOLVED ENGINEERING FUNDAMENTALS PROBLEMS In the preceding equation, T = 2r/w. 2 pr , Baw =F [020 V)sinutat -C) (89 “28 _ 20 tme= (#°) (wa) 06. A (3.1 A) 12 ‘Therefore, AC ELECTRICITY-8 For the circuit shown, /; = Jy before the switch is closed. If the switch is closed at time t = 0, what is the behavior of Ty at t = 0? switch closed atr=0 (A) 1: is discontinuous and decreasing. (B) 1; is discontinuous and increasing. (C) 1; is continuous and decreasing. (D) Fis continuous and increasing. Fort <0, the current travels through L; and La. After the switeh is closed, J; = Ip + Ip, with Ip slowly decaying through the short. As + goes to infinity, the current will travel around the outer loop with PROFESSIONAL PUBLICATIONS, ING. AC ELECTRICITY 13-7 I= In = B/Py and Ip = 0. At <0, fh = by = B/(Ry + Ra), but for t2 0, I(t) = FyQe I (&) (ern) a (aem)* ne (&) (- = (age) em" Ri \R +R) ) Ree Therefore, [1 is continuous and increasing at t= 0. ‘The answer is (D). AC ELECTRICITY-9 ‘A2pP capacitor in the circuit shown has a reactance of Xc = 1500 9. What is ‘most nearly the frequency of the AC source? A EO im m (A) 3.0 He, (B) 53 Ha (©) 60 Ha (D) 120 Hx ‘The reactance is PROFESSIONAL PUBLICATIONS, INC. 13-8 1001 SOLVED ENGINEERING FUNDAMENTALS PROBLEMS ‘Therefore, ae CxXe ‘The frequency is, The answer is (B). AC ELECTRICITY-10 If the capacitor and the inductor in the circuit shown have the same reactance, what is most nearly the frequency of the AC source? (A) 27 He, (B) 180 He (©) 210 Ha (D) 1200 He If the inductor and capacitor have the same reactance, then PROFESSIONAL. PUBLICATIONS, INC. AC ELECTRICITY 13-9 The frequency, f, ia pete * OR ~ VOL 7 1 © Ba T5 x 10-8 F)(50 x 108 H) = 184 Hx (180 Hz) ‘The answer is (B). AC ELECTRICITY-11 An alternating voltage of E = 10sinwt V is applied to the RCL circuit shown. What is the effective current, Irma if the circuit is in resonance with the driving voltage? t per rosin ot (~ son mM tt c (A) 0.141 A. (B) 0.200 A. (©) 107A (D) 114A ‘At resonance, the impedance of the circuit is equal to the impedance of the resistor. Therefore, PROFESSIONAL PUBLICATIONS, INC. 13:10 1001 SOLVED ENGINEERING FUNDAMENTALS PROBLEMS Additionally, "Phe answer is (A). AC ELECTRICITY-12 In the RCL circuit shown, R = 10.2, @ = 30 uF, and L = 0.5 H. At ap- proximately what frequency will the rms current be one-third of the maximum Possible rms current? ‘The magnitude of the current is aa I-E + (ot-3.) toa Ensinat (~ law te (A) 37 Ha (B) 41 He (C) 46 Hz (D) 160 Hz ‘The maximum rms current occurs at resonance. That is, Fi Jaman = i PROFESSIONAL PUBLICATIONS, INC, AC ELECTRICITY 13-11 For the rms current to be one-third of the maximum, 1 Ems aR ( @-2))" a wus) Solving for the positive w value, w bart PIC — (v2)(10 2) as (05 Hy? = 288s) * ot 288 _ vir, fae “OE TS HO x 10-* FY f ln = 45.8 He (46 Hz) ‘The answer ©) PROFESSIONAL PUBLICATIONS, INC. 13.12 1001 SOLVED ENGINEERING FUNDAMENTALS PROBLEMS AC ELECTRICITY-13 Which of the following statements regarding transformers is FALSE? (A). The copper losses (JR) in the primary and secondary coils are equal. (B) ‘Transformer power losses are generally low, approximately 1-3%. (C) Power losses in transformers are converted to heat, which is then dissipated. (D) One three-phase transformer weighs more than three equivalent single- phase transformers. Power conversion using a three-phase transformer is more efficient than conversion using three separate single-phase units. Reduced weight and space requirements are obtained for the three-phase transformer, ‘The answer is (D). AC ELECTRICITY-14 An ideal step-up transformer with a power factor of 1.0 is used in the circuit shown. The turns ratio is 70, and the primary rms voltage is 120 V. What is ‘most nearly the average power dissipated due to the resistance, R? v ln= 70] 120 Vine IIE R= 100 (A) Ww (8) 29 (©) 84x10 Ww (D) 71x 10° W For an ideal transformer, the turns ratio is carne ad Verns,2 = T0Vems,1 = (70)(120 V) PROFESSIONAL PUBLICATIONS, INC. AC ELECTRICITY 13:13, Since power is given by P = IR = V2/R, Vero: ane (8400 V)? 102 7.06 x 10 W (7.1.x 10° W) ‘There is no power loss in the primary cireuit. Paves ‘The answer is (D). AC BLECRICITY-15 In a transformer, the total voltage induced in each winding is proportional to the number of turns in that winding. ALM FE, Na Dinegarding ll loose, determine Bs mls 3 [6 gE ls T=200 mm 8O M100 m= 800 (A) 45. (B) 65 (©) 75V (D) Vv From the ratio given, m= 2H Es = na )e) (2)(B)ev =v The answer is (D). PROFESSIONAL PUBLICATIONS, INC. 13.14 1001 SOLVED ENGINEERING FUNDAMENTALS PROBLEMS AC ELECTRICITY-16 Determine the resonant frequency, w, of the circuit shown. Or Oy off wz Resonance occurs when X¢ = Xz. Since Xe = 1/jwC and Xp, Jol, aha = jul jot ‘The answer is (A). AC ELECTRICITY-17 Determine most nearly the power angle, 4, in the AC circuit if R = 25 2, 1 =0.2 H, V = 200 V, and f = 30 Hz. | GY (A) 36° (B) 40° (©) 52° () sr PROFESSIONAL PUBLICATIONS, INC. AC ELECTRICITY 13.15 ‘The power angle is the impedance angle, The impedance for the circuit is Z=R+5Xt = 25 2+ j2n(80 Ha)(0.2 H) = 25 04 597.79 AC ELECTRICITY-18 ‘Approximate the impedance of the circuit. ‘The line current is I, and the line voltage lies along the real axis (ie., has a zero phase angle). e=120V (a) 120 (B) 132 ua (D) 152 newt) RHE agers Ry +jXz _p (fr 5Xe & ONT XE . 102-5120 = (120) (a OFF C2 a) = 4.92 - 76.9 A ‘PROFESSIONAL PUBLICATIONS, INC. 18:16 1001 SOLVED ENGINEERING FUNDAMENTALS PROBLEMS n=. (xox) (BEE Te-ike~ ike) at iXe. _p (% 1X0) E+ XG a 20 2+ 20.0 = (020 V) (a OE+ 0 oF) =34j30A ‘The total current is I=hth = (4.92 — 75.90 A) + (8 + 38.0 A) = 7.92 52.90 A ‘The impedance is (sa + (90 MF 14.39 (142) ‘The answer is (C) AC ELECTRICITY-19 ‘What is most nearly the power factor for the following circuit? The total impedance of the circuit is Z= 13.4 + j4.9 0. E=120V (A) 74% (B) 79% (©) 84% (D) 94% PROFESSIONAL PUBLICATIONS, INC. AC ELECTRICITY 13.17 ‘The power factor is Preat R eer Since R = 13.4 2 and Z = 14.3 2, 13.4.9 cos = TT * 100% = 93.7% (94%) AC ELECTRICITY-20 ‘What is the input impedance of the ideal op amp shown? fe sont L cutee Py aR (B) Of Rm © gs ithe ‘To find the input impedace, a test voltage, Vr, is applied to the input. The resistance seen by the test voltage will be equal to the impedance: Riy = Vr/Ir. The circuit can be replaced with its equiv- alent. PROFESSIONAL PUBLICATIONS, IN. 13.18 1001 SOLVED ENGINEERING FUNDAMENTALS PROBLEMS By Kirchoff’s law, Ir = Io + Ii) In an ideal op amp, there is no current drawn by the positive and negative terminals. ‘Therefore, Ty) = 0 and Ip = Ig. Around that loop, ‘The answer is (B). AC ELECTRICITY-21 What is the input impedance of the following ideal op amp? f a input — (A) Ri @) & oF ©) R+h ‘To find the input impedance or resistance, the circuit is examined using a test voltage, Vir, and a test current, Ip. The circuit diagram becomes fe ‘PROFESSIONAL PUBLICATIONS, INC. AC ELECTRICITY, 13.19 ‘The input resistance will be Vr Ra= For an ideal op amp, the voltage at the (-+) terminal equals the voltage of the (—) terminal. Therefore, V = 0 and AC ELECTRICITY-22 ‘What is most nearly the output impedance, Rout, of the circuit shown? Assume no current is drawn at the (-+) and (—) inputs, and that the op amp has a small internal resistance, Rig, at the output. (A) Raw (0 PROFESSIONAL PUBLICATIONS, INC. 13-20 1001 SOLVED ENGINEERING FUNDAMENTALS PROBLEMS In terms of its operation, the op amp diagram is like the solid part in ‘the following illustration. Ay is very large. To find the output resistance, a test voltage, Vr, is attached to the output and the input is grounded. Then, Bout = Tt The test current, Ir, is equal to the internal current, Tin, plus the forced current, I. Since the inputs draw no current, Iy = 0. ‘There fore, Mie Rise WV) Tr = Jin = Rane Since no current is drawn at the inputs, the (+) input is at 0 V, and the (—) input is at Vp, so that V = Vp. ‘Thus, fee sea) Ring Ve. Pin Te 144 ave Rw Fou = Fe = TFA Since Ay is very large and Rigs is very small, Rous #0 ‘The answer is (D). PROFESSIONAL PUBLICATIONS, INC. AC ELECTRICITY 13.21 AC ELECTRICITY-23 ‘The 700 Hz signal shown is injected into the circuit shown. What will be the ‘output signal? nap +15V in rt out a ~ 40k ww \ f t =v 2040 wh input = 700 He 10K a) ray By Vout (roo ti} 9V ov oa (B) +8, Vout (2100) °V av © You OV} 0) wv Vase (700 Hah OY av ‘The op amp part of the circuit isa simple noninverting amplifier with a gain of Vou _ 10V+20V Ve 7 lov = $Y PROFESSIONAL PUBLICATIONS, INC. 13-22 ‘The answer is (D). 1001 SOLVED ENGINEERING FUNDAMENTALS PROBLEMS ‘The input into the amplifier is a high-pass filter with a cutoff fre- quency of 1 1 4°> FENG ~ Ded XIO HOVER TOF) = 200 Hz Thus, the AC component of the signal will pass through and be amplified three times, while the DC component will be cut out, result ing in a 9 V amplitude sinusoid centered about 0 V. This is known as an active high-pass filter. Thus, the correct output is shown in option (D). AC ELECTRICITY-24 ‘The signal shown is the input to the ideal op amp. Which of the choices is the output signal? (B) oun 15V “BV PROFESSIONAL PUBLICATIONS, INC. AC ELECTRICITY 13-23 (©) av << (D) & bob Since the amplifier is an inverting, amplifier, 9 ko Vout = = (38) Vin = —3Vin Both the DC and the AC components will be amplified. The DC component is (—3)(3 V) = -9 V, so the new waveform is centered at V =—9 V. The AC component is (3)(8 V) = 24 V peak-to-peak. Since the amplifier has only a 15 V source, though, the voltage will be clipped at +15 V. Since it never goes to +15 V, the upper half of the output signal will be intact, and the lower half will be clipped at 15 V. ‘The answer is (D). AC ELECTRICITY-25 ‘Two AC signals, V; and V3, are to be combined such that Vour = $¥2— §V1 shown is used. What must be the values of Fy, ‘The subtracting amplifier cir Ra, Rg, and Ra? fy PROFESSIONAL PUBLICATIONS, INC. 13-24 1001 SOLVED ENGINEERING FUNDAMENTALS PROBLEMS (A) Ri =240, (B) Ry =240, (C) Ry =440, 10, Ro = 10 kM, Ry =2 KA @) R=5409, KO, Ro = 440, Ry = 2k The output for this op amp configuration is Additionally, Therefore, (Ruths) (_Re _ Re. Vas ( Ri ) (atm) ‘The ratio R3:R, must be 5:2. Therefore, the initial values Ry = 2 kQ 5 k@ are chosen. Thus, the coefficient of V2 is (B28) ( Ry ) = (2s) ( Re J =a Ry Rat Ra) ~ 2k Pa +a 1) (gm) 2) \Ra+ Re ue Rah TRy = 38, +3R AR, = 3K Ra= jhe 3/2 =3/7 ‘Try the values of Ry and Ry in the four answer choices to see if they satisfy the relation. PROFESSIONAL PUBLICATIONS, INC. AC ELECTRICITY 13:25 K9 and Ry = 8 kM are chosen, Checking the results, a (2489) ( Bk 3 Vy Ten) \axosanm) M34 m ‘The answer is (B). AC ELECTRICITY-26 {A sinusoidal signal with maximum voltage, Vo = 80 mV, is to be amplified without inversion to at least 0.3 V. Which of the following operational amplifier Configurations will best achieve this? Ascume ideal op amp. s0Kn wv ate (eae Ls Va roKa ov s0Ka ®) PROFESSIONAL PUBLICATIONS, INC. 1001 SOLVED ENGINEERING FUNDAMENTALS PROBLEMS 13.26 soxn ve By Vin R RA © Vo av Sua ron apy ee o > Vie eons -5v A noninverting topology is required. The resistances and voltages are labeled in the following illustration. a Ry er Min WWW liv, fe “Mee For this topology, _ (Rite Vout = ( te) Yn ‘The voltage has to be amplified by a factor of 10 in order to get 30 mV up to 03 V. M+R _4y Ro” Ry =9R PROFESSIONAL PUBLICATIONS, INC. AC ELECTRICITY 13.27 Rwy is not important, since very little current is drawn through it. Of ‘the answer choices, options (A), (B), and (D) are the correct topolo- ties, and options (A) and (B) have the correct Ry to Re ratio. The next criterion is that the supply voltage, Voc, must be greater in magnitude than the output voltage, or clipping will occur. Since op- tion (A) has a negative input supply of 0 V, it will clip the output. Only option (B) will satisfy all requirements. ‘The answ (8). AC ELECTRICITY-27 In the ideal op-amp configuration shown, Vou = 12 V sinusoidal as shown in the waveform, Ry = 60 kO, Ry = 30 Kk, and Mz = 10 kX. Nothing is known about the inputs except that V, = 5V2, and that they are 180° out of phase with the output, From this information, what are the maximum voltages of the inputs? Ry fy (a) @) () (D) YW =3.75 V, = 0.75 V PROFESSIONAL PUBLICATIONS, INC. 13-28 1001 SOLVED ENGINEERING FUNDAMENTALS PROBLEMS ‘This is an adding amplifier with an output of Rey, Re You = = (Fev + eM 5Va. Substituting this into the equation for Voyy and evaluating with the given R values and Vout, GORD) 5 | (Goo a=—((Shin) + (itin) 4) Vp =-0.75 Vi = (6)(-0.75 V) =-3.75V Since the output is a sinusoid, the inputs must also be sinusoids. It is ‘known that they are 180° out of phaso, which is confirmed by the negative sign of the voltage. Thus, the maximum voltages are Vi = 3.75 V and Vy = 0.75 V. ‘Tho answer is (D). AC ELECTRICITY-28 A zero-crossing detector is needed for the noisy circuit shown. The phase of the detector output is not important. (That is, the detector can show a time lag.) Which of the following op amp configurations would be best? 10V pa PROFESSIONAL PUBLICATIONS, INC. () AC ELECTRICITY 13-29 @) +gV Y——e > Yon ov ay 7" —— $$ Vay 10 “5v +15 () +ev % 2 + Yot 2S sv abv 2k, P >———+ Ve wnS agy 1k Because of the noise, a simple comparator such as in option (A) will not work. There will be false zero crossings where the noise crosses ero at the signal crossing. ‘The configuration shown in option (C) will not work for the same reason as the comparator. The device shown in option (B) is a nonin- verting amplifier, which will amplify the entire signal. A device with hysteresis, such as a Schmitt trigger, is needed. Such a device will not. change until a threshold is reached, and will not change again until the negative threshold is reached. The configuration is, +¥ec — 7 You Vee Ry Fa PROFESSIONAL PUBLICATIONS, INC. 13-30 1001 SOLVED ENGINEERING FUNDAMENTALS PROBLEMS Ra Vesreshold = (ee) Voo ‘The only Schmitt trigger circuit is given in option (D). With the Ry ‘and Ry resistances shown, it will trigger at V = (1 kS/2 k-+1 k®) (15 V) =5 V, which will work. ‘There will be some delay, and the signal will be inverted. ‘The answer is (D). AC ELECTRICITY-29 A 30 mY sinusoidal signal must be inverted, amplified to 6 V, and chopped at 4 V. If the following circuit is used, what aro the values of Ry, Ia, and the avalanche voltage of the zener diodes, Z? There is a forward voltage drop of 0.7 V for the diodes. 30mv Input ov -30 mv ev av output ov nay av wn is Ry + Vo ‘PROFESSIONAL PUBLICATIONS, INC. (a) (B) () (D) AC ELECTRICITY 13-81 Ry = 1k, Rp =20kN, Z=4.0V Ry =1kM, Rp = 200K, 7=40V Ry =2KM, Ry = 400K, Z=3.3-V Ry =2kO, Ra = 800k, Z ‘The amplification is similar to that found in the normal inverting amplifier, Ba Vou __6V Ry Vin 30x11 v = 200 ‘This leaves options (B) and (C) to be the possible choices. When Vou Z, one of the diodes is forward biased with a voltage drop of 0.7 V, while the other diode is avalanched at the zener voltage. This keeps Voxe constant. Thus, Via = 4 V is equal to the zener voltage plus the 0.7 V voltage drop of the other diode. Vou = Z+0.7V Z=4V-07V=33V So, Re 200K; and Z=3.3 V. ‘The answer is (C). AC ELECTRICITY-30 An AC alternator operated as a motor is called a synchronous motor. Which of the following statements regarding synchronous motors is FALSE? (A) @) © ‘The average speed, regardless of load, does not decrease, since the motor rust operate at a constant speed. When a load is increased, the increased torque is a result of the shift in the relative positions of the ficlds on the rotor and stator. ‘The relationship between speed, frequency, and numberof poles is the seme for the rotating field of the induction motor and for the alternator. The poles of a synchronous motor must be salient. PROFESSIONAL PUBLICATIONS, INC. 13-32 1001 SOLVED ENGINEERING FUNDAMENTALS PROBLEMS Salient poles have laminated pole pieces. Although salient poles are generally used, either salient or nonsalient poles can be used in a synchronous motor. The answer is ( AC ELECTRICITY-31 Which of the following statements about induction motors is FALSE? (A) ‘They are used to increase the line power factor. (B) They have no slip rings, no brushes, and no excited field current, (C) They have no commutators and no windings on the armature. (D) Squirrel-cage induction motors operate at essentially constant speeds. Induction motors degrade the power factor. All the other answer choices are true, ‘The answer is (A) AC ELECTRICITY-82 A single-phase induction motor is not self-starting. Instead, auxiliary methods must be used, such as varying inductance, resistance, and capacitance. Which of the following is FALSE regarding this situation? (A) A capacitor motor uses capacitance to split the phase, resulting in two phases almost 90° apart. (B) Capacitor motors have lower starting torque than comparably sized single- phase induction motors. (C) To obtain a higher reactance, a capacitor can be used when starting and then be switched out of the cireuit by mechanical means. (D) If the capacitor remains in the circuit, the power factor will have a value close to unity. Due to the favorable phase relationship, the torque is higher for a capacitor motor than for other types of single-phase motors, For example, a capacitive phase split motor gives better torque than & resistively split motor. ‘Therefore, option (B) is false ‘The answor is (B). PROFESSIONAL. PUBLICATIONS, INC. AC ELECTRICITY 13.33 AC ELECTRICITY-33 ‘A squirrel-cage motor has euch low resistance that it draws excessive currents ‘when starting. Which of the following actions will NOT reduce this problem? (A) connecting the windings as in a three-phase, wye, transformer, taking 58% of the normal line voltage; then, at sufficient motor speed, switching to a delta connection (B) using an in-line rheostat (©) using an autotransformer to reduce line voltage (D) using a class A motor ‘A class A motor draws a heavy starting current, usually 200-300% of ‘the normal load. The other alternatives reduce the effective voltage across the windings, thus reducing the problom of excessive currents. AC ELECTRICITY-34 Which of the following statements about AC generators is FALSE? (A). The poles of an AC generator are located on the rotor. (B) The three main types of AC generator are direct-connect engine driven, water driven, and turbine driven. (©) An AC generator uses commutators. (D) Large turbine driven generators usually have two pole rotors to accommo date the high speed of the turbine, Commutators are not used in AC machines. It is the relative motion between the rotor and the stationary armature located on the stator that generates the power. ‘The answer is (C). PROFESSIONAL PUBLICATIONS, INC. 13-34 1001 SOLVED ENGINEERING FUNDAMENTALS PROBLEMS AC ELECTRICITY-35 Which of the following is FALSE regarding rotating machinery? (A) The avoidance of harmonies in the production of a sine wave can be achieved by using a coil having multiple loops passing through adjacent slots rather than using only one pair of slots. (B) Uniformity in the production of flux on a pole can be obtained by using distributed field windings over a portion of the rotor surface. (C) AC generator ratings are usually given in units of kVA (kilovolt amps). (D) At zero power factor, the generator delivers real power to a lond. A power factor of 1 delivers only real power and no reactive power to a resistive load. A zero power factor is associated with a nonresistive load or with no-load conditions. ‘The answer is (D) AC ELECTRICITY-36 In the following transistor circuit, fis 100, and the DC base-to-emitter voltage is 0.6 V. What is the output voltage, Vous? (A) 03V (B)2V (av (D) 10 R= 08k0 yeav Vi-Vg _ 3V-06V y= patty 7900012 A (0.12 mA) Io = (1+ Bly = (1+ 100)(0.12 mA) = 12.1 mA, Veo ~ Tea = 10 V (12.1 mA) (wana) (08 9) (Fz) Ip Vout 03 ‘The answor is (A). ‘PROFESSIONAL. PUBLICATIONS, INC. AC ELECTRICITY 1335, AC ELECTRICITY-87 ‘The circuit chown represents a matched lossless transmission line. What is the maximum load voltage, Vz? 2,~ Hoon } Lt 0 costat)V vi] | z=s000 (A) 200 (B) 30 (©) 400 (D) 500 V For a matched lossless transmission line, the characteristic impedance, Zo, equals the load impedance, Z, with Vj = Vz and y= Ir, a8 shown in the following illustration = oon he = La ¥,~ 600 costat) V % 4 %= 39000 3009 = 00% (|srme sara) ~ PROFESSIONAL PUBLICATIONS, INC. 13.36 1001 SOLVED ENGINEERING FUNDAMENTALS PROBLEMS AC ELECTRICITY-38 A.10-pole synchronous motor operates on a 60 cycle voltage. What is the speed of the motor? (A) 520 rpm (B) 620rpm_ = (C) 660 xpm_——(D) 720 rpm The synchronous speed for AC motors is 120f _ (120)(60 Hz) ? 10 = 720 rpm ‘The answer is (D) AC ELECTRICITY-39 ‘The core of a 400 Hz. aircraft transformer has a net cross-sectional area of 13 cm?, The maximum flux density is 0.9T, and there are 70 turns in the secondary coil. What is most nearly the rms voltage inducod in the secondary coil? (A) 130. V (B) 150 (© 10V (D) 1500 v ‘The induced voltage is Tn the preceding equation, NV is the number of turns, and B is the fx density in Teslas. B= 0.9sinwt, where w = 2xf = 2n(400 rad/s) Therefore, a V=-NaBA In the preceding formula, A is the cross-sectional area of the core. ‘Therefore, NwBA _ (70)(2n)(400 Hz)(0.9 T)(13 x 10-4 m?) va v2 =M6V (150) ‘The answer is (B) PROFESSIONAL PUBLICATIONS, INC. Ver AC ELECTRICITY 13-37 AC ELECTRICITY~40 ‘A 150 kVA, 1000 V single-phase alternator has an open circuit emf of 750 V. When the alternator is short circuited, the armature current is 460 A. What is ‘most nearly the synchronous impedance? (A) 169 (B) 229 (©) 269 (p) 329 Synchronous impedance, Z, is Voc 2o Voc is the open-circuit voltage, and I, is the armature current when the alternator is short circuited. ‘Therefore, 750 V 460.4 63.2 (1.69) ‘The answer AC ELECTRICITY-41 In a balanced three-phase system with a power factor of unity, the line voltage, Ej, and the line current, J, deliver normal AC power. What is the expression for the power, P? (A) P= Bh (B) P= 3B (© P ype (D) P=V8Eh ‘The power developed by a three-phase generator is three times the coil voltage, B, multiplied by the coil current, Ie. P=3Ele ‘The line voltage has the following relationship with the coil voltage. B= VBE. PROFESSIONAL PUBLICATIONS, ING. 13.38 1001 SOLVED ENGINEERING FUNDAMENTALS PROBLEMS ‘Therefore, since I. = It for a power factor of 1, EE v5 V3Eh P Eyl Tho answer is (D). AC ELECTRICITY-42 A three-phase alternator has three armature coils, each rated at 1200 V and 120 A. What is most nearly the kVA rating of this unit? (A) 430 kVA (B) 440 kVA (C) 520 kVA (D) S40 KVA The kVA rating is equal to the power output. KVA = SEI (3)(1200 V)(120 A) = 482000 VA (430 KVA) ‘The answer is (A) AC ELECTRICITY-43 ‘What is the relationship between the line current, J,, and the coil current, I, in a balanced delta system? Ie I i @) nee I D) = Vir. (A) h Va Bh Be (© hak (D) i= v3) ‘The three-phase line-phase relations for a balanced three-phase delta iysieu avo h=V3l, = V3. ‘The answer is (D). PROFESSIONAL PUBLICATIONS, INC. 1 4 PHYSICS PHYSICS-1 ‘The figure shown is used to indicate combinations of color primaries for subtrac- tive mixing of colors. Which one of the following is true? (A) 1 = green, (B) 1 = yellow, 4 (C) 1 = cyan, 5 = green, 7 = white (D) 1 = magenta, 5 = red, 7 = black ‘This figure could be rotated so that 1 = magenta, yellow, or cyan. However, the only choice that has all three colors in the proper places is option (D). ‘The answer is (D). PROFESSIONAL PUBLICATIONS, INC. 142 1001 SOLVED ENGINEERING FUNDAMENTALS PROBLEMS PHYSICS-2 ‘Which of the following statements is FALSE? (A) wavelength of visible light > wavelength of microwaves (B) frequency of radio waves < frequency of infrared waves (C) wavelength of x-rays > wavelength of gamma rays (D) frequency of ultraviolet > frequency of infrared ‘The electromagnetic spectrum is owe | waves | tntared | visiblo | utravitet | xrays | samme Increasing froquency —> <— increasing wavelength wavelength of microwaves is greater than the wavelength of ible light. ‘Therefore, option (A) is false, ‘The answer is (A) PHYSICS-3 A light source emits a total luminous flux of 1000 Im distributed uniformly over a quarter of a sphere. What is most nearly the luminous intensity 2.5 m from the source? (A) 42 Im/m? —(B) 61 tm/m? (©) $8 1m/m?——(D) 62 Im/m? ‘Luminous intensity, 1, is In the preceding equation, Z, is the luminous flux and A is the surface area of sphere around the light source. The area of a quarter of a sphere is 50.94 Im/m? (51 Im/m?) PROFESSIONAL PUBLICATIONS, INC. PHYSICS. 143 PHYSICS—4 A 100 W lightbulb emits a total luminous flux of 1500 Im, distributed uniformly over @ hemisphere. What is most nearly the illuminance at a distance of 2 m? (A) 11 im/m? —(B) 21 lm/m?—(C) 34 1m/m? (1D) 60 Lm/m? The illumination, B, is E In the preceding equation, @ is the luminous flux and A is the area. A Therefore, 59.7 Im/m? (60 Imm?) ‘The answer is (D). PHYSICS-5 A lightbulb is used to light a stage 2.5 m below. A chair site on the stage 1.0 im from a spot directly below the bulb. Ifthe bulb has a luminous intensity of 150 lm, what is most nearly the illumination on the floor around the chair? (A) 7.7 Im/m? — (B) 19 1m/m? —(C) 21 lm/m?—(D) 51 Lm/m* The illumination, B, is given by the following formula, PROFESSIONAL PUBLICATIONS, INC. ded 1001 SOLVED ENGINEERING FUNDAMENTALS PROBLEMS In the preceding equation, I is the luminous intensity of the source, @ is the angle from the normal to the surface the light stsikes, and is the distance from the light source. Aightoult om 2.5 m 0080 = orm JOB wa me = VI mE (aso im) (757) 7.25 m? = 19.2 Im/m? (19 im/m?) E The answer is (B), PHYSICS-6 Light of wavelength and intensity Jy passes through a 0.05 m thick slab of glass whose absorption coefficient for that wavelength is 15 m~!, What is most nearly the intensity, J, of the light after passing through the slab? (A) 0.3% (B) 0.5% (C) 0.6% (D) 0.8% I ‘The absorption coefficient, @, is 15 m=". Therefore, Tm Tye~(08/¥(0.05 =047Ig (0.5T) ‘The answer is (B). PROFESSIONAL PUBLICATIONS, INC. PHYSICS 145 PHYsICcs-7 ‘A light ray passing through air (n = 1) strikes a glass surface (Ngius = 1.52) at an angle of a = 60° from the normal to the surface. What is the angle, , between the reflocted light and the surface? (A) 7.5° (B) 15° (C) 30° (D) 45° < ‘The reflection law states that the angle of incidence is equal to the angle of reflection (a =a). Therefore, a/ = 60° and f = 30°. ‘The answer is (C). PHYSICS-8 ‘Which material type usually has « higher index of refraction? (A) lighter materials (B) heavier mat (C) denser materials (D) less-dense materials In general, denser materials have higher indices of refraction. ‘The answer is (C). PROFESSIONAL PUBLICATIONS, INC. 46 1001 SOLVED ENGINEERING FUNDAMENTALS PROBLEMS PHYSICS-9 ‘What is the path of the refracted ray in the following illustration? (A) (B) ‘The light ray is refracted at interface 1. It is not normal to the surface of the glass, so its path changes direction. This eliminates options (A) and (D). Since nga > Mair, the ray is bent toward the normal to the surface of the glass at interface 1, and away from the normal to the surface at interface 2. Thus, option (B) is the correct path of the ray. The anwer is) | PROFESSIONAL PUBLICATIONS, INC. PHYSICS 147 PHYSICS-10 ‘How can the index of refraction of material be defined? (©) n= See Indices of refraction are defined such that ng sings = nz sin, where ‘nq and nz are the indices of refraction of materials a and x, and da and @p are the angles betwoen the light ray and the normal to the Interface between the two materials. The reference index for air, nq, is 1. Therefore, sings be ‘From the illustration, G2 = be sings singe ‘The answer is (B). PROFESSIONAL PUBLICATIONS, INC.

You might also like